You are on page 1of 219

Maja CvitkoviC KOMBINATORIKA (Zbirka zadataka)

CIP - Katalogizacija u publikaciji Nacionalna i sveuEiliena biblioteka, Zagreb UDK 519.1 CVITKOVIC, Maja Kombinatorika / Maja Cvitkovif. - Zagreb : ELEMENT, 1994. - VI, 224 str. : ilustr. ; 24 cm Bibliografija: str. 223, . ISBN 953-6098-1S-4,: 3% c&x5
A

941031149

vP

?
i

4 -

*,: -

C .

""S'f~*,t
Y.

..* -.a.

ISBN 953-6098-16-4

MAJA

CVITKOVIC

KOMBINATORIKA
ZBIRKA ZADATAKA

K N J NICA I~
FUnOLTETA I3imsmmHNlKE I ~CUNR~TVA ZAGREB Unska 3

Zagreb
1994

Urednik
Prof. dr. sc. NEVEN E L E Z O V I ~

Recenzenti

JELENA G U S I prof. ~, NOGO Mr. sc. GORANKA

Nakladnik ELEMENT, Zagreb

Design ovitka

BOJEVREMENA, Zagreb

Tisak

SPIRIDION BRUSINA, Donja Lomnica

q J I f N I C A JLIETA ~LEKTRQTEII~WU?
\ W A

ZAGREB

Nijedan dio ove knjige ne smije se umnaiati, fotokopirati niti reproducirati na bilo koji nazin, bez pismene dozvole nahladnika

Upute za uporabu . .
. . . . . . . . . . 3 . . . . . . 11 . . . . . . . . . . . . . . . . . 25 . . . . . . . . . . . . . . . . . . . . . . 35 . . . . . . . . . . . . . . . . . . . . . . 35 Brojanje klasa ekvivalencije . . . . . . . . . . . . . . . . 52 Formula ukljuzivanja-iskljuzivanja . ' . . . 72 . . 85 5 . Rekurzivne relacije . . . . . . . . . . . . . . . . . . . . Primjeri rekurzivnih relacija . . . . . . . . . . . . . . . . . . 8 5 Tehnika rjeiavanja . . . . . . . . . . . . . . . . . . . . . . . 91 . ,101 6 . Funkcije izvodnice . . . . . . . . . . . . . . . . . . . . 7 . Kombinatorno dokazivanje identiteta . . . . . . . . . . . ,116 8. Zbrka zadataka . . . . . . . . . . . . . . . . . . . . . . .I31 Zadaci . . . . . . . . . . . . . . . . . . . . 131 . . . . . . . . . . . . . 144 Rjeienja . . . . . . . . . . . . Z . . . . . . . . . . . . . . . . . . . . . . . . . . . . . .199 9 . Programi Podskupovi n -5lanog skupa . . . . . . . . . . . . . . . . I PZO1' ; Uredene n-torke s elementima iz zadanog skupa . . . . . . . ,. . 207 . Permutacije s elementima iz zadanog skupa . . . . . . . . . . 209 RaEunanje Elanova niza pomoCu rekurzivnih relacija . . . . . . 213 Biblioteka . . . . . . . . . . . . . . . . . . . . . . . . . . . .215 ZakljuEni zadaci i napomene . . . . . . . . . . . . . . . . . . 216 . ,218 10. Posljednji zadatak . . . . . . . . . . . . . . . .

1. 2. 3. 4.

Uvodne vjeZbe . . . . . MatematiEka indukcija Dirichletov princip . . Prebrojavanje . . . . . . Elementarni zadaci .

. . . . . . . . . .

. . . . . . . .

.
+'

.f

Definicije pojmova

Upute za uporabu
Kombinatorika na elementarnom nivou karakterizirana je vrlo malim brojem novih pojmova i Eesto vrlo teSkim zadacima. Zato je ovdje dobra prilika da se osvrnemo na tehniku rjeSavanja zadataka. Da bi se uopCe moglo govoriti o rjeSavanju zadataka, potrebne su Ielja i odluEnost da ih se rijeSi. Gitanje rjeSenja nema nikakve veze s rjeSavanjem zadataka. Ono poEinje uzimanjem papira i olovke. Prvi problem koji se javlja pri rjeSavanju zadataka je razumijevanje zadatka. $to je zadano? Sto se traIi; kako su traIeni objekti uvjetovani zadanim objektima? Lako je prepoznati je li zadatak jasan. Onaj tko je razumio zadatak, znat C e ga prepriEati vlastitim rijeEima, nacrtati skicu, navesti primjere. Kad znamo Sto trebamo Einiti, moramo odluEiti kako demo to ostvariti. Ideje sarno rijetko padaju s neba - treba aktivno krenuti u upoznavanje zadatka i pitati se na primjer: Znam li neki sliEni zadatak? Ako da, koje su razlike u odnosu na rjeSavani zadatak? Jesu li premostive? Kako bi trebalo izgledati rjeSenje? Znam li neka moguCa rjeSenja, rjeSenja u posebnim sluEajevima? U ovoj fazi zgodno je napraviti skicu zadanih i traIenih objekata. Rezultat ove faze je plan rjeSavanja. Ukoliko se on kasnije pokaIe neispravnim, vratit Cemo se u ovu fazu ili Eak sasvim na poEetak. Nakon Sto je plan razraden, zadatak treba poEeti "rjeSavatiV. Niti ova faza nije sasvim mehaniEka: pri provodenju zacrtanog plana moramo paziti pribli6avamo li se rjeSenju ili smo na putu da odlutamo ili se zavrtimo u krugu. Osim toga, treba paziti dodajemo li zadatku nesvjesno nove uvjete koji olakSavaju rjeSavanje (dodavanje novih uvjeta nije lose, ali ga treba imati pod kontrolom, jer moramo biti svjesni da u tom sluEaju viSe ne rjeSavamo originalni zadatak). Na kraju, nakon Sto zadatak proglasimo rijeSenim, treba prvo provjeriti rjeSenje. U svakom sluEaju treba se pitati jesmo li dobili oEekivani rezultat. Niti usporedba rezultata s rezultatima sliEnih zadataka nije naodmet. Kad utvrdimo da je rezultat ispravan, moIemo se prepustiti najinteresantnijoj, ali Eesto zanemarivanoj fazi. NeoptereCeni od pritiska da moramo rijeSiti zadatak moZemo prodiskutirati zadatak i njegovo rjegenje.

0. UPUTE ZA UPORABU

Mofemo li rezultat objasniti ili do njega doCi jog na koji naEin? Jesu li svi uvjeti zadatka nuBni? $to se dogada ako neki od uvjeta ispustimo ? a Koje su moguCe generdizacije? Koje su posljedice zadatka? Uvijek se moZemo pitati zagto se zaustaviti na prvom toCnom rje'ienju. ZGto matematika ne bi imala svoje stilske vjefbe? Zbirka zadataka namijenjena je uEenju. Zadatke je pofeljno rjegavati redom, pokugavajuCi svaki zadatak rijegiti. Mnogi zadaci imaju nakon rjeSenja napomene koje sadrfavaju smjernice za diskusiju: ukazuju na zanimljive korake u rjegenju, postavljaju sliCne zadatke i moguka poopkenja. Nije ih pogeljno preskakivati - one omoguCavaju bolje upoznavanje objekata s kojima se radi. Pogotovo ako neki zadatak ne znate rijegiti i proEitate rjegenje, napomena je dobra prilika da ipak neSto od zadatka rijesite samostalno. Tek tako zadaci daze pod kofu. Zbrku zadataka moBe se poEeti listati paralelno s rjegavanjem zadataka iz zbirke. Pokusajte za svaki zadatak vidjeti Eto se traBi, pa usporedite to s repertoarom koji vam nudi do tog trenutka svladana tehnika. Ako mislite da znate rijeiiti zadatak, pokugajte ge rijesiti. Ako mislite da zadatak ne znate rijegiti, potraZite neito drugo ili dalje rjegavajte zadatke iz zbirke. TreCi dio sadrgava osnovne algoritme za prebrojavanje raznih objekata. Nakon Sto se upoznate s prebrojavanjem, moBete ga poEeti Citati. To moBe biti na primjer kad se umorite od "obiEnog" prebrojavanja. Ali isto tako, ako nemate ideju Sto poEeti s nekim zadatkom, mozete ga prvo pokuSati istrafiti pomoku raEunala, a zatim dobivene rezultate iskoristiti u rjegavanju zadataka.

Uvodne vjeiibe
Bavljenje kombinatorikom podrazumijeva poznavanje skupova i operacija sa skupovima, te relacija i funkcija. Pri rjegavanju zadataka, Eesto zadanih priEom o objektima koje treba prebrojati, u prvom koraku treba prepoznati skupove i operacije koji dobro opisuju zadane objekte. KaBemo jog da treba napraviti matemati* model. To je najEeSCe i najteBi korak pri rjegavanju zadataka. Ponovimo zato skupovne operacije i osnovna svojstva relacija i funkcija kroz zadatke u kojima treba sloZiti matematiEke modele raznih objekata. Ovi zadaci nisu rijegeni. Rjegavajte ih zato u paru ili manjoj skupini - to omoguCava kontrolu rjesenja i razmjenu ideja. U ovom se poglavlju nalazi i nekoliko rijegenih vjeZbi koje se kasnije kao Einjenice koriste u zadacima vezanim uz prebrojavanje.

AUB

AnB

A\B

AAB

1. Neka je G skup svih gramofonskih ploCa neke osobe, C skup svih njenih kompakt diskova. Opigite rijeEima nekome tko ne vlada pojmovima "skup" i "element" skupove G U C , G n C .

1. UVODNE V J E ~ B E

2. Neka je K skup svih gramofonskih ploEa s klasiEnom glazbom, a P skup svih gramofonskih ploEa na kojima netko pjeva. RijeSite zadatak analogan prethodnom za ova dva skupa.

3.

Nadite tri skupa disjunktna sa skupom P iz prethodnog zadatka.

4. Neka je ovdje P skup svih vaSih gramofonskih ploEa na kojima netko pjeva. Nadite nekoliko moguCih skupova X za koje je P U X podskup skupa svih vaSih grarnofonskih ploEa. Zatim P U X treba biti jednak skupu svih vaSih gramofonskih ploEa. Presjek tih skupova ne mora biti prazan. Zadajte nekoliko skupova X takvih da P U X nije podskup skupa svih vas'ih gramofonskih ploEa.

5. DokaIite ili opovrgnite jednakost slijedeCih parova skupova. Jednakost skupova ( X = Y ) dokazuje se obiEno tako, da se prvo dokaIe da je svaki element skupa X ujedno i element skupa Y ( X C_ Y ) . Zatim se d o h i e da je svaki element skupa Y element skupa X i te dvije Einjenice zajedno daju Ieljenu tvrdnju. Jednakost se opovrgava nalcenjem protuprimjera - primjera za koji nije zadovoljena jednakost. Ako skupovi nisu jednaki, nadite i dodatne uvjete na skupove A , B i C kojima oni postaju jednaki. U svakom sluEaju nacrtajte Vennove dijagrame tih skupova.

6.

Podsjetite se Sto znaEe pojmovi komutativnosti i asocijativnosti.

1 zloEina. Za svaki od 7. Vodi se sudski proces protiv Eovjeka optuienog za 1 njih postoje i drugi mogu6 poEinitelji. Neka je univerzalni skup jednak skupu svih moguCih zloEinaca - poEinitelja barem jednog od tih zloEina. Definirajte i ostale skupove potrebne za ovaj sudski proces i odgovorite na pitanje: kako pomoCu skupova moIemo izreCi optuibu? Jedan sudski vje6tak rekao je da je svih 11 zloEina poEinila ista osoba. Izrazite to skupovno.

8. Neka je E skup svih uEenika jedne Skole koji uEe engleski jezik, N skup svih onih koji uEe njemaEki. OpiSite rijeEima skupove E \ N , N \ E , E A N .
9. Neka je jog T skup svih uEenika koji uEe talijanski jezik. Izrazite pomoCu skupova E , N i T , te do sada definiranih skupovnih operacija,skup svih uEenika koji uEe samo talijanski. UEinite to na dva naEina. Izrazite zatim skup svih uEenika koji uEe toEno jedan, te skup uEenih koji uEe toEno dva od navedenih jezika. OznaEite sve dobivene skupove na Vennovom dijagramu.

10. Je li skupovna razlika komutativna operacija? Asocijativna? OznaEite potrebne skupove na Vennovim dijagrarnima. Ako mislite da je odgovor na

1. UVODNE V J E ~ B E

pojedino pitanje pozitivan, dokafite odgovarajuCu tvrdnju. Ako mislite da je odgovor negativan, nadite protuprimjer.
11. Na maturi je bilo po jedno pitanje iz hrvatskog jezika, biologije, fizike, matematike, latinskog i povijesti umjetnosti. Svaki je uEenik poneSto napisao kao odgovor na svako od pitanja, ali je potpuni odgovor dao samo iz nekih. Oni koji su potpuno odgovorili na tri pitanja polozili su maturu s dovoljnim uspjehom, oni koji su odgovorili na Eetiri s dobrim. Za pet toEnih odgovora ocjena je bila vrlo dobar, a za Sest izvrstan.

Ako je kao univerzalni skup zadan skup svih uEenika Skole koji su u tom ispitnom roku polagali maturu, definirajte ostale skupove potrebne za opisivanje ovog problema. Skupovi trebaju, biti takvi; da se iz njih do sada definiranim skupP~vnim operacijama mogu rekonstruirati svi podaci. Primjer loSe definiranih skupova su skupovi uEenika slozenih prema dobivenoj ocjeni: 01, 0 2 , 03, 04, O5 (zagto?). 12. Neka je U skup uEenika jednog razreda. Nadite komplemente skupa svih djevojEica, zatim skupa svih odliEnih, te skupa svih uEenika koji iz tjelesnog imaju ocjenu 1 ili 5. Skupa svih djevojEica koje iz tjelesnog imaju 5. Koji su sve skupovi ovdje bili u igri? Skicirajte ih.
13. Neka je S neki skup Skolskih svjedodfbi. S C e u ovoj vjeZbi biti univerzalni skup. Neka je 0 1 skup svjedodzbi s barem jednom ocjenom izvrstan, 0, skup svjedofbi sa svim izvrsnim ocjenama. Prikafite te skupove Vennovim dijagramima. OpiSite rijeEima skupove O i l 0," , O1 \ 0,.

Neka je N1 skup svjedodzbi s barem jednom jedinicom. Ucrtajte i taj skup 1 \ 0 5 \ Nl , 0 1 \ (05 U N I ) , na Vennov dijagram. OpiSite rijeEima skupove 0 Nl\Ol, 0 1\ N l , 0 1nN1, 0 5 nN1 Neka je 0 2 skup svjedodfbi s barem dvije izvrsne ocjene. Nacrtajte Vennov dijagram sa skupovima S,01, 0 2 i 0, . Opigite rijeEima skupove 0," , O1 \ 0 2 , 0 2 \ O8
14. Jeste li prethodnu vjefbu proSli na brzinu? Ponovite je jog jednom. Definirajte svoje podskupove skupa S i pogledajte koje se zanimljive unije, presjeci i razlike mogu pomoCu njega definirati.

15. Je li razlika skupova asocijativna operacija? Komutativna? Sto moHete reCi o skupovima (A \ B ) \ C i (A \ C ) \ B ?
16. Kako mofemo izraziti (A \ B) (A\B)\C s B n C ?

\C

pomoCu B U C ? U kakvoj je vezi

17. Izrazite barem na dva razliEita naEina ( A \ B) \ C kao uniju disjunktnih skupova. I jednom kao skupovnu razliku nekih dvaju skupova.

1. UVODNE V J E ~ B E

U ormaru su hlaEe, suknje, koSulje i puloveri koji se mogu dobro kombinirati u okviru uobiEajenih naEina oblaEenja. OpiSite skup svih moguCih oblaEenja tih predmeta, ako treba obuti hlaEe i pretoplo je za pulover. Koji skup odgovara oblaEenju suknje, ako za naveEer treba ponijeti i pulover? Skupu svih oblaEenja? $to se dogada ako je neki od ovih skupova prazan?
18. 19. OpiSite barem na dva naEina Kartezijev produkt skupa koSulja iz prethodnog zadatka sa samim sobom. Zatim jog Kartezijev produkt skupa hlaEa sa skupom koSulja i jog jednom sa skupom koSulja. 20. Kako se mogu sve mogudnosti ispunjavanja sportske prognoze prikazati pomoCu Kartezijevog produkta? 21. Ako je S skup svih vrsti sladoleda u jednoj slastiEarnici, kako mofemo interpretirati P ( S ) ,partitivni skup skupa S ? cemu odgovara skup svih naruEivanja sladoleda?

. . . , Sn skupovi uEenika 22. Ako je R skup uEenika nekog razreda, a Sl,S2, koji sudjeluju na 1.,2.,.. . , n -toj slobodnoj aktivnosti, je li skup {S1, S2,. . . ,Sn) particija skupa R ?
23. Neka su za razred iz prethodne vjeibe i za prirodne brojeve izmedu 1 i 12 skupovi Ri zadani kao skupovi svih uEenika rodenih u i-tom mjesecu. Je li {R1, R2,. . . , R12} particija skupa R ? 24. Nadite jednu relaciju izmedu skupa svih raEuna izdanih na jedan dan u nekoj trgovini i skupa svih artikala koji se prodaju u toj trgovini.

25. Nadite barem dvije relacije na skupu H x K , gdje je H skup hlaEa, a K skup kosulja u neEijem ormaru.
26.

Definirajte barem dvije relacije na skupu svih uEenika jednog razreda.

27. "Znati neEije ime" je obiEno refleksivna relacija. Nadite jog jednu relaciju koja je refleksivna i jednu koja nije refleksivna.
28. Mofe se oEekivati da je "biti poznanik" simetriEna relacija. Nadite jog jednu simetriEnu relaciju. 29. "Prepisao domadu zadadu iz nekog predmeta" je relacija koja nije simetriEna (premda je moida od neke dvojice prijatelja jedan zaduien za domaCe zadaCe iz engleskog, a drugi za matematiku). Nadite jog jednu relaciju koja nije simetriEna.

30. "Biti potomak" je tranzitivna relacija. Nadite jog jednu tranzitivnu relaciju.

31. %ti prijatelj" nije tranzitivna relacija (premda bi to netko mogao oEekivati). Nadite jog jednu relaciju koja nije tranzitivna. 32. Nadite primjer relacije koja je refleksivna, ali nije niti simetricna niti tranzitivna. U ovom i slijededim zadacima pokugajte nadi "Iive" primjere. Tek ako vam to ne uspije, uzmite skup nekih prirodnih brojeva i konstruirajte relaciju nabrajanjem elemenata. 33. Nadite primjer relacije koja je simetriEna, ali nije niti refleksivna niti tranzitivna. 34. Nadite primjer relacije koja je tranzitivna, ali nije niti refleksivna niti simetriEna. 35. 36. 37. 38.

Nadite primjer relacije koja je refleksivna i simetriEna, ali nije tranzitivna. Nadite primjer relacije koja je refleksivna i tranzitivna, ali nije simetriEna. $to bi bilo s relacijama koje su simetriEne i tranzitivne? Nadite primjer relacije koja je i refleksivna i simetrizna i tranzitivna.

39. Koja od navedenih svojstava ima relacija lLigli/iduu isti razred" na skupu svih ljudi? Koja svojstva mofemo ozekivati na skupu svih prvoSkolaca? Koja svojstva ima relacija "biti isto godigte"? 40. Relaciju izmedu dvaju skupova mofemo prikazati tako da Kartezijev produkt prikafemo kao tablicu, te da za svaki par u relaciji na odgovarajuie mjesto u tablici upiSemo krifik. Drugi prikaz je pomodu grafova: nacrtamo skupove na kojima je definirana relacija, te za svaki par (a, b ) iz relacije nacrtamo strelicu od a prema b . Ako je relacija zadana na jednom skupu, taj skup crtamo samo jednom. Prikaiite neke od relacija iz prethodnih primjera tablicno i grafovski. Kako se u tabliEnom zapisu relacije oEituju svojstva refleksivnosti, simetriEnosti i tranzitivnosti? Kako se t a svojstva oEituju u grafovskom prikazu? gto moIete redi o relacijama ekvivalencije? 41.

Nadite klase ekvivalencije za relaciju ekvivalencije "biti isto godigten.

42. Privatne telefonske brojeve u nekom gradu mofemo svrstati u blokove prema njihovim prvim dvjema znamenkama. Uvjerite se da je to particija skupa svih privatnih telefonskih brojeva. Kako moSe glasiti odgovarajuda relacija ekvivalencije? Nadite barem dva rjegenja. 43. TipiEni primjeri injekcija su razna pridrufivanja matiEnih brojeva. Unutar jednog sustava (npr. jedne knjifnice) ne smiju postojati dva korisnika s istim matiEnim brojem. S druge strane moraju postojati "slobodni" matiEni brojevi - oni koji de biti pridrugeni bududim korisnicima.

1. UVODNE V J E ~ B E

Nadite jog dvije funkcije Eija se injektivnost koristi u svakodnevnorn Eivotu. Neka barem jedna od njih ne bude injektivna.
44.

Objasnite z S t o ima smisla ostavljati otisak prsta na policiji.

45. Neka je V skup svih odraslih voCaka u jednom voCnjaku. Nadite skup Yl i preslikavanje fi : V -t Yl koje je injekcija. Nadite zatim skup Y2 i preslikavanje f2 : V 4 Y2 koje za neke voCnjake nije injekcija. Izaberite zatim dvije

domene i dva preslikavanja u skup tih voCaka, od kojih jedna jest, a druga nije injekcija.
46. TipiEni primjeri surjekcija su razna preslikavanja sa skupa nekih ljudi na skupove poslova koje treba obaviti. Svaka osoba smije raditi samo jedan posao, svaki posao treba biti obavljen. Pritom nije bitno je li neki posao obavljalo vise ljudi.

Nadite joS dvije funkcije Eija se surjektivnost koristi u svakodnevnom Eivotu. Neka barem jedna od njih ne bude injektivna.
47. Sa skupa svih fiziEkih i pravnih osoba koje posjeduju stan postoji jedna prirodna surjekcija na skup svih stanova. Izrecite je! SliCno naEalost ne vrijedi za skup svih vlasnika pasa i skup svih pasa.
48. Sa skupa svih ljudi u skup svih boja ne postoji surjekcija "imati oEi odredene boje". Redefinirajte kodomenu tako da navedeno preslikavanje bude bijekcija. Generalizirajte. 49. TipiEni primjeri bijekcija su razna kodiranja. Ako neki predmet zamijenimo nekom oznakom, onda dva predmeta ne smiju imati istu oznaku. S druge strane, svaki kod u nekom sistemu odgovara nekom realnom predmetu. To omoguCava da podatke o predmetima drZimo u raEunalu, a ne u skladistu pokraj predmeta i da unatoE tome za svaki predmet mofemo doCi do podataka o njemu i da za svaki podatak moBemo reCi o kojem se predmetu radi.

Nadite jog dva primjera bijekcija. Potrudite se da budu bitno razliEita od ovog primjera.
50. Neka slijedeCi primjer pokaEe kako ponekad nesvjesno koristimo preslikavanja. Mali Mirko studira matematiku, jednako kao njegov stariji brat Veljko. Razlika je jedino u tome Sto Mirko ne pohada redovito predavanja. Jednog dana zamoli Mirko Veljka da mu da svoje zabiljeske s predavanja prof. MijeniCa BeleCi se spremiti za ispit iz linearne algebre. Nakon toga Veljko osvane s biljegnicom iz kombinatorike. Objasnite gto se dogodilo? $to bi se dogodilo da je prof. MijeniC Veljku predavao dva predmeta? Koliko je predmeta prof. MijeniC predavao Mirku? 51.

Nadite nekoliko permutacija skupa uEenika jednog razreda.

1. UVODNE V J E ~ B E

52. Neka je funkcija p sa skupa svih ljudi starijih od godinu dana na skup svih zubara: svakom Eovjeku je pridruZen "njegov" zubar (ovdje je na djelu bila idealizacija - Sto smo sve pretpostavili da bi p bila funkcija?). Kojem objektu u ordinaciji zubara z odgovara p-'(2) ?
53. Svakom preslikavanju f : X -t Y moZemo pridrugiti skup svih praslika po tom preslikavanju: kerf := {f-'(y)ly E f ( X ) ) . Taj skup zovemo jezgrom preslzlcavanja f . DokaZite da je jezgra preslikavanja particij a domene.

Posljedica te Einjenice je da svaka funkcija prirodno inducira jednu relaciju ekvivalencije na svojoj domeni: dva elementa domene su u relaciji ako i samo ako im se slike pri zadanoj funkciji podudaraju. To svojstvo Eesto nesvjesno koristimo pri zadavanju relacija ekvivalencije: kaZemo da su x i y u relaciji ako i samo ako imaju istu vrijednost svojstva S , pri Eemu je svojstvo S takvo, da je jednoznaEno odredeno elementom za kojeg se odreduje. Na primjer, relaciju "imati jednak broj godina" moCi Cemo uzeti kao primjer relacije ekvivalencije, dok relacija "imati rasnog psa iste pasmine" nije relacija ekvivalencije na skupu svih vlasnika rasnih pasa. Analizirajte svoje primjere relacija ekvivalencije!
54.

Sto moZete reCi o jezgri za preslikavanje koje je injekcija, surjekcija, bijek-

cija?
55. U jednoj (dalekoj) zemlji studenti voze bicikle. Svaki student ima toEno jedan bicikl. Poznat je tip bicikla svakog studenta. Jedna tvornica guma Zeli nagraditi dobre studente tako da im pokloni komplet guma za njihove bicikle. Koji su skupovi i koja preslikavanja ovdje u igri?
56. Ako je f preslikavanje koje korisnicima neke knjiZnice pridruguje njihove njihove matiEne brojeve, moZete li zamisliti kad se koristi njeno inverzno preslikavanje?

57. Neka je f : X + Y i g : Y ilustriraju slijedeCe tvrdnje:

-t

2 . DokaZite, odnosno nadite primjere koji

(a) ako je g o f surjekcija, onda je g surjekcija; f nije nuZno surjekcija (b) ako je g o f injekcija, onda je f injekcija; g nije nuZno injekcija
58. Neka je f : X + Y . f je surjekcija ako i samo ako postoji funkcija g : Y + X takvadaje f o g = l y .
Rjeienje.
z EX

-+. Ovaj smjer trebali ste dokazati u prethodnoj vjeibi ( ly


:= f

je surjekcija). za svako y E Y . Tako

+. Definiramo funkciju g : Y -+ X tako da je g(y) E f . To moiemo napraviti jer je f surjekcija, pa je f -l(y)


o

' ( { y ) )

# 0 za svako

definirana funkcija g zadovoljava f

g = ly (provjerite to!). g sigurno injekcija.

4' Prema prethodnoj vjeibi znamo da je

10

1. UVODNE V J E ~ B E

% Pokuiajte izraEunati koliko postoji dobrih g-ova za zadanu surjekciju f. Kad je g jedinstven?
+fJ U svakoj je Hkoli funkcija f koja uEenicima pridruiuje njihove razrede surjekcija. ZnaEenje desnog inverza moiemo opisati ovako: neka je g funkcija koja iz svakog razreda izabire njegovog predstavnika u nastavniEkom vijetu. Tada C e moCi na sjednice biti pozvani predstavnici razreda koji C e ujedno biti i uEenici tog razreda. To ne bi moglo vrijediti da funkcija f nije surjekcija, tj. da postoje razredi bez uEenika.

Uvjerite se na ovom primjeru da g nije nuino i lijevi inverz funkcije f trebali izgledati razredi, odnosno funkcija f da bi g bio i lijevi inverz?

Kako bi

59. Neka je f : X + Y . f je injekcija ako i samo ako postoji funkcija g : Y - t X takvadaje g o f = 1 x .


Rjerienje.

-+. Ovaj smjer trebali ste dokazati u pretprethodnoj vjeibi ( l x

je injek-

cija).

J . Definiramo funkciju g : Y + X tako d a je {g(y)) = f (y) za svako z E f (X) , a bilo Hto inaEe. To moiemo napraviti jer je f injekcija, pa je f (y) jednoElan za svako y E f (X) . Tako definirana funkcija g zadovoljava g o f = lx (provjerite to!).

-' -'

tP

Prema pretprethodnoj vjeibi znamo d a je g sigurno surjekcija.

PokuEiajte izraEunati koliko postoji dobrih g-ova za zadanu injekciju f. Kad je g jedinstven?

+fJ Funkcija f koja svakom gradaninu neke zemlje pridruiuje njegov matiEni broj je injekcija. U Hrvatskoj joj je kodomena skup svih trinaestoznamenkastih brojeva. Neka je g preslikavanje sa skupa svih trinaestoznamenkastih brojeva u skup svih gradana Republike Hrvatske koja svakom broju koji je neEiji JMBG pridruiuje upravo t u osobu, a ostalima bilo koga (npr. prvu osobu s veCim JMBG). Da je g lijevi inverz od f znaEi da moZemo na dokumente pisati svoj matiEni broj umjesto imena i prezimena, a d a se pri tome neCe izgubiti informacija o osobi (u najgorem sluEaju C e trebati nazvati policiju i pitati o kome se radi). To ne bismo mogli napraviti ako bismo "dijelili" matiEni broj s jog nekom osobom.
OpiSite rijeEima Hto bi u ovom primjeru znaEilo da je g i desni inverz od f g bio i desni inverz?

. Kad bi

+-f' Ovaj i prethodni zadatak Eesto se u dokazima koriste zajedno. Dokaiemo li d a funkcija ima obostrani inverz, ona je bijekcija.

MatematiEka indukcija
Mnoge tvrdnje koje treba dokazati sadrfe u sebi kao parametar neki prirodni broj. Neke od njih mogu se dokazati direktno (npr. za prirodni broj n i realni = (qn+' - l)/(q - 1) moBe se dokazati mnofenjem broj q # 1 tvrdnja Cy=,qqi relacije s q i oduzimanjem od polazne relacije). Kod kompliciranijih tvrdnji Eesto poseBemo prvo za provjerom tvrdnje na nekim manjim vrijednostima parametra, te nam ti primjeri mogu dati ideju kako dokazati tvrdnju za proizvoljnu vrijednost parametra. S druge strane, mofe se dogoditi da iz primjera vidimo naEin kako se npr. konstruiraju tra5eni objekti za neku vrijednost parametra ako nam je poznat izgled tih objekata za njegove manje vrijednosti. To se dogada npr. pri popisivanju svih n-torki sastavljenih od znamenaka 0 i 1. Ispred svake od n - 1-torki sa znamenkama iz istog skupa stavimo prvo znamenku 0, a zatim isto to ponovimo sa znamenkom 1ispred starih n - 1 -torki. Tako odmah vidimo da traBenih n-torki ima dvostruko vige nego n- 1-torki. Lako se vidi da postoje dva niza duljine 1, a zatim se izvede da nizova duljine 2 ima 2 . 2 = 4 , 4 2 = 8 nizova duljine 3. Nasludujemo da nizova duljine n ima toEno 2 n . Dodatno znaEenje ovoj metodi daje princip matemati5ke indulccije: Ako za neki skup S vrijedi: 1ESi akoje n E S , o n d a j e i n + l E S , ondaje S = N . Da matematickom indukcijom dokafemo prethodni primjer, trebamo promatrati skup svih prirodnih brojeva n za koje je broj n-8anih nizova sastavljenih od znamenaka iz skupa { O , l ) jednak 2n. Provjera tvrdnje za prvi element skupa S zove se baza indukcije, a dokaz da je n 1 E S Eim je n E S nazivamo lcoralcom indukcije.

Princip matematiEke indukcije Eesto se javlja i u nekim drugim oblicima, npr.:

2.

MATEMATICKA INDUKCIJA

kao baza indukcije provjerava se tvrdnja n o E S , a korak indukcije ostaje nepromijenjen - dobivamo S = {no, no 1,n o 2,. . .) , korak indukcije se provodi ovako: ako za svaki i E {1,2,. . . ,n ) vrijedi i E S , onda je n 1 E S - zakljuEak ostaje nepromijenjen, ako je N E N neki prirodni broj, a korak indukcije provodimo sarno za one brojeve n za koje je n 1 E NN , onda je zakljuEak S = NN . Nadalje, ponekad se korak indukcije zapisuje kao prijelaz s n - 1 na n . Dok su prethodno nabrojani oblici matematieke indukcije nuBno potrebni jer primjena pojedinog od njih ovisi o tvrdnji koju treba dokazati, odluka da li 6e se pri koraku indukcije krenuti od n ili n - 1 stvar je ukusa i elegancije zapisa.

KlasiEne tvrdnje koje se dokazuju matematiEkom indukcijom ukljuEuju razne sume, nejednakosti i sliEne aritmetiCke tvrdnje. Ovdje 6emo se baviti primjenom matematieke indukcije na kombinatorne probleme. Na kraju jog jedno upozorenje: nakon Sto se dobro uvjeBba dokazivanje metodom matematiEke indukcije, moZe se dogoditi da se ona provodi mehaniEki. Pritom se ne upoznaje dobro struktura objekata s kojima se radi i od dokazivanja tada ima malo koristi. Zgodno je svaku tvrdnju pokuSati prvo dokazati direktno, a tek ako se ne vidi direktni dokaz, krenuti indukcijom, te pri dokazivanju pokuSati Sto je viSe moguCe doznati o objektima s kojima radimo.

1. D o u i t e da za proizvoljni n E N vrijedi: ako je zadano n pravaca u ravnini, takvih da se nikoja tri ne sijeku u istoj toEki i da nikoja dva nisu pardelna, oni dijele ravninu na 1 n(n 1)/2 dijelova.

Rjes'enje. Baza indukcije. n = 1. Ako je zadan jedan pravac u ravini, oiiito je d a je on dijeli u dva dijela. I stvarno, uvrStavanjem n = 1 u formulu koju ielimo dokazati, dobivamo

Pretpostavka indukcije. Pretpostavimo da ova tvrdnja vrijedi za neki n E N . Dakle za bilo koji skup n pravaca u ravnini koji zadovoljava uvjete zadatka, broj podruiija na koja oni dijele ravninu dan je izrazom n(n

+ 1)

Korak indukcije. Ako sada imamo skup od n+ 1 pravca, onda znamo na koliko podruiija dijeli ravninu nekih n od njih. Uoiiimo skup od npr. prvih n pravaca i pogledajmo kako se poveCava broj podruiija u ravnini kad dodamo jog jedan ("zadnji") pravac. On je razliiiit od svih prethodnih pravaca, pa dijeli svako podruiije kojim prolazi na dva dijela. Dolaze8 "iz beskonaiinosti", t a j pravac prolazi prvo jednim neograniiienim podruiijem, sve do presjeka s prvim pravcem koji mu se nade na putu. Zatim ide u drugo podruiije, do drugog pravca, itd. Nakon Sto naide na zadnji pravac kojeg presijeca, ulazi ponovno u jedno neograniiieno podruiije. Dakle, broj podruiija kojima prolazi n 1-vi pravac je za jedan veti od broja presjeka sa starim pravcima.

2.

MATEMATICKA

INDUKCIJA

13

Dakle, zasada znamo novi pravac poveCava broj podruEja za broj presjeka sa starim pravcima + l . BuduCi d a se nikoja tri pravca ne sijeku u jednoj tozki i da se svaka dva pravca sijeku, broj presjeka sa starim pravcima jednak je broju starih pravaca, tj, n . BuduCi da smo sakupili sve podatke, moiemo izraEunati na koliko podruEja dijeli ravninu n 1 pravaca:

i t 0 odgovara poEetnoj formuli s uvritenim n

+ 1 umjesto

n.

% Ako ste veC svladali rekurzivne relacije, onda moiete i izvesti ovaj rezultat. Trebate samo oponagati prethodni dokaz piiudi rekurzivnu relaciju reda 1.

+ I ' Za pravce od kojih se nikoja tri ne sijeku u toEki i nikoja dva nisu medusobno paralelna, kaZe se d a su u opc'em polo2aju. + I ' Citatelji spretni u analitiEkoj geometriji moCi Ce Einjenicu d a je broj novih podruEja za jedan vedi od broj presjeka sa starim pravcima izvesti i precizno.
% Ako je za neki n zadano n pravaca, nadite naEin za numeriranje podruEja pri kojem se uzastopni brojevi nalaze na susjednih podruzjima.

9-t Formulirajte sami tvrdnju zadatka ako iz zadatka trebamo izbaciti uvjet da pravci moraju biti u opCem poloiaju.
9+ Formulirajte zadatak sliEan ovom, s kruinicama umjesto pravaca. Kako glasi uvjet o presjecima kruinica? Rezultat treba glasiti n2 - n 2 .

2. Dokaiite da se ravnina presjeEena s n pravaca moie obojiti dvama bojama - plavom i bijelom, tako da svako od dobivenih podruiija ravnine bude jednobojno, a da podruiija koja imaju istu duiinu kao granicu budu raznobojna.
Rjes'enje. Baza indukcije. Ako je n = 1 , ravnina je podijeljena na dva dijela obojimo jedan od njih plavo, a drugi bijelo. Pretpostavka indukcije. Pretpostavimo d a znamo obojiti ravninu presjeEenu s n pravaca tako d a zadovoljimo uvjete zadatka. Korak indukcije. Dodamo li sad jog jedan pravac, uvjet zadatka naruien je jedino na podruiijima koja kao granicu imaju neki podskup tog pravca. Naime, svi parovi podruEja koji su upravo nastali dodavanjem novog pravca su medusobno susjedni i u parovima su iste boje. Odaberimo jednu poluravninu odredenu novim pravcem i promijenimo boju novonastalim podruzjima u toj poluravnini. Time smo rijeiili problem s bojom koji je nastao oko novog pravca - podruiija oko novog pravca sada su u parovima raznobojna, ali su svi susjedi podruqa kojima smo promijenili boju, a nalaze se u promatranoj poluravnini, pogregne boje. Trebamo dakle promijeniti boju i svim njihovim susjedima kojima jog nismo promijenili boju, zatim svim njihovim susjedima itd. ToEnije, boju trebamo promijeniti svim podruiijima u promatranoj poluravnini. Dobiveno bojanje zadovoljava uvjet zadatka, jer ga zadovoljava unutar svake poluravnine i na granici poluravnina.

14

2. MATEMATI~KAINDUKCIJA

9+ I za neke druge krivulje koje dijele ravninu na dva dijela (kruinice, elipse, parabole, sinusoide . . . ) mogli bismo analogno zakljuEiti d a moiemo obojiti ravninu u dvije boje tako da nikoja dva susjedna podruEja ne budu obojena istobojno.
Stovi'se, kad god su granice podruEja takve da u bilo koju toEku ulazi paran broj linija koje odgovaraju granicama, ravnina se moie obojiti u dvije boje. "Zemljopisnu kartu" moiemo preslikati na kuglu (globus) i tako dobiti i dvobojno bojanje kugle. Dokaz poEiva na crtanju tih granica u jednom potezu i nije pretjerano teiak. Nacrtajte nekoliko primjera i poku'sajte ga izvesti.

4-t Nadite neke krivulje koje dijele ravninu u po dva dijela, ali proizvoljni skup tih krivulja ne zadovoljava svojstvo da u svaku toEku ulazi paran broj linija.

4 ' OpCenito, ako je ravnina bilo kako podijeljena na podruEja, dokazano je d a se ona moie obojiti u Eetiri boje uz iste uvjete kao u zadatku. To je poznati problem Eetiri boje. Lako je vidjeti d a je za bojanje potrebno barem Eetiri boje. (pokuHajte sami nat i krivulje u ravnini - "granice driava" tako da dobivenu zemljopisnu kartu ne moiete obojiti u manje od Eetiri boje). Teie je dokazati d a su Eetiri boje i dovoljne. Problem Eetiri boje prvi put se spominje polovinom 19. stoljeCa i od tada privlaEi painju mnogih matematiEara. UobiEajeno mu je pristupiti sa stanovista teorije grafova. Lako je dokazati da je dovoljno pet boja. Do nedavno dano je nekoliko "dokaza" problema Eetiri boje od kojih su se neki uspjeli odraati i po vise godina dok im se nisu otkrile pogre'ske. Problem Eetiri boje rije'sen je 1976. Dokaz je sveden na niz provjera koje su provedene na raEunalu, tako da jog uvijek mnogi matematiEari sumnjaju u njegovu ispravnost.
3. DokaEite da za proizvoljni n E N konveksni n-terokut ima n(n - 3)/2 dijagonala.
Rjegenje. Trokut nema niti jednu dijagonalu, a i predloiena formula za n = 3 daje rezultat 0.

Pretpostavimo d a tvrdnja vrijedi za proizvoljni n-terokut. Tada zadanom n 1-terokutu moiemo "otpiliti" jedan trokut, npr. po dijagonali izmedu prvog i n-tog vrha. Time smo dobili n-terokut Eije su sve dijagonale ujedno i dijagonale polaznog n 1-terokuta, a njihov broj nam je poznat iz pretpostavke indukcije. Osim tih dijagonala, promatrani n 1-terokut ima joH neke dijagonale: onu koja je bila pretvorena u stranicu i jog n - 2 dijagonale iz n 1 -vog vrha. Broj njegovih dijagonala zato iznosi

Hto smo i trebali dobiti.

4-t Ovaj se rezultat moie dobiti i direktnim prebrojavanjem. Pokugajte ga izvesti. 4-t Gdje se u ovom dokazu koristila konveksnost?
+ I ' SliEno se moie dokazati i d a je zbroj unutra'snjih kuteva svakog n-terokuta jednak (n - 2)180 .

2. MATEMATICKA INDUKCIJA

15

4. DokaZite da je za proizvoljni n E N i skupove A1, A2, . . . ,A, njihova simetriEna diferencija A1 A A2 A . . . A A, sastavljena od svih x E U:="=lAi koji se nalaze u toEno neparno Ai -ova.
Rjebenje. Primijetimo prvo d a je AlAA2A.. . AA, (bez zagrada, tj. toEno naznaEenog redoslijeda izvodenja operacija) dobro definirano zbog asocijativnosti simetriEne diferencije.

Za n = 1 tvrdnja je oEigledna. Nacrtajte primjere i provjerite Hto se dogada za n = 2 i n = 3 . Pretpostavimo d a tvrdnja vrijedi za neki n i pokuHajmo vidjeti koje elemente sadrii A l A A 2 A . . . AAnAAn+1. A l A A 2 A . . . AAnAAn+l = ( A l A A 2 A . . . AAn)AAn+l (prema definiciji simetriEne diferencije) = ((AlAA2A .. . AAn) \ &+I) U (&+I \ ( A l A A 2 A . . . AAn)) (koristeti pretpostavku indukcije)
=

({ x E U ~ = ~ : A;
x se nalazi u neparno Ai -ova, i = 1 , . . . ,n ) \ An+1) U (&+I \ { 2 E 'Jr=lAi : x se nalazi u neparno A; -ova, i = 1 , . . . ,n ))

{XEU~Z~~A;: x se nalazi u neparno Ai -ova, i = 1,.. . ,n i ne u A,+l

)U

{ x E U;!?~~A;
=

x se nalazi u parno A; -ova, i = 1,.. . ,n i u A,+l

)=

{ x E u ~ ~ ~ A ~ : x se nalazi u neparno Ai -ova, i = 1,.. . ,n

+ 1 ).

5. Ako ste razbili Salicu u n dijelova, dokazite da je za sastavljanje galice potrebno n - l lijepljenje. Pretpostavimo da pri svakom lijepljenju spajamo samo dva dijela - bilo krhotine bilo veC zalijepljene dijelove Salice.
Rjegenje. Ako je n = 2 , dovoljno je jedno lijepljenje.

Pretpostavimo d a je za slaganje Halice razbijene u n komadita potrebno toEno n - 1 lijepljenje. Ako vam se Halica razbila u n 1 komadita, nakon prvog lijepljenja, ostat te n komadita koje trebamo spojiti. Prema pretpostavci indukcije njih moiemo zalijepiti u n - 1 koraka (ova tvrdnja se u stvari malo razlikuje od pretpostavke indukcije - izrecite t u razliku). Dakle za sastavljanje Halice razbijene u n 1 dijelova potrebno je n koraka, bez obzira na redoslijed slaganja i naEin grupiranja komadita.

tP Citatelji upoznati s teorijom grafova u ovom bi zadatku trebali prepoznati teorem koji kaBe d a svako binarno stablo s n listova ima jog n - 1 (unutragnji) vrh.
'%) Pokugajte izraEunati broj potrebnih lijepljenja piguti rekurzivnu relaciju za broj potrebnih lijepljenja.

Svatko tko je pokuSao lijepiti razbijenu ialicu zna da je Eesto potrebno lijepiti i po vise od dva dijela istovremeno. U tom sluEaju moiemo samo redi da C e posao lijepljenja biti zavrien u najviSe n - 1 koraka.

% Ako grupa od barem [n/2] ljudi lijepi razbijenu Salicu, tako d a Sto je moguCe viSe ljudi radi istovremeno, te svaki Eovjek u svakom koraku lijepi po dva dijela, u koliko C e koraka biti zavrieno lijepljenje? Napravite primjere za nekoliko malih n . PokuSajte naCi pravilo i dokazati ga. Uputa: osobito su bitni brojevi oblika n = 2 k .
6. Gdje je pogregka u sljedeCoj Stefovoj izjavi: "Ja sam trekinom Dalmatinac. Naime, moja majka je bila trekinom Dalmatinka i moj otac je treCinom Dalmatinac."
Rjes'enje. Da neito u ovoj izjavi nije u redu lako je vidjeti - bilo koja osoba moie biti npr.

(a) napola Dalmatinac - npr, majka Dalmatinka i otac Zagorac ili oboje roditelja napola Dalmatinci ili sliEno . . . (b) Eetvrtinu Dalmatinac - npr. majka napola Dalmatinka i otac Zagorac ili neSto sliEno . . . (c) tri Eetvrtine Dalmatinac ili sliEno . . . itd. OpCenito, ako se sloiimo sa Einjenicom d a se svojstvo "biti Dalmatinac" nasljeauje pola-pola od svakog od roditelja, Eovjek moie biti svojim k/2' -tim dijelom Dalmatinac, gdje je k nenegativan cijeli broj manji ili jednak 2i (dokaiite to promatrajuti rodoslovno stablo do nivoa na kojem su svi preci "Eisti"). 113 nije broj takvog oblika.

- npr. majka Dalmatinka i otac napola Dalmatinac ili

U Eemu je pogregka? Stefovo razmigljanje (tj. korak indukcije) bilo je ispravno. PogreEIka je onda mogla biti samo u tome Hto je tvrdio da su mu roditelji treCinom Dalmatinci. Dakle, nedostajala mu je (toEije njemu ili veC njegovim roditeljima ili nekim daljim precima koji su pokrenuli t u neistinu) toEna baza indukcije.

7 . Pretpostavimo da imamo n jednakih kvadratnih ploEica jediniEne duljine brida. PosloZimo ih u ravninu tako da se parovi ploEica ili ne dodiruju ili se dodiruju u vrhu ili u cijeloj stranici. DokaZite da je opseg dobivenog lika (u opseg dodajemo i duljinu granica svih rupa, ukoliko ih ima) paran broj.
Rjes'enje. Ako je n = 1, imamo jednu ploEicu. Njen opseg iznosi 4 jedinice, i t o je paran broj.

Sada nam ostaje samo dokazati d a se dodavanjem nove ploEice opseg mijenja za paran broj. Iznos za koji C e se promijeniti opseg ovisi o poloiaju u koji Cemo staviti ploEicu, toEnije o broju bridova starog lika na koje C e se nasloniti nova ploEica. Ako se nova ploEica naslanja na
0 starih bridova, opseg se poveCava za 4,

1 stari brid, opseg se povedava za 2,

2.

MATEMATICKA

INDUKCIJA

17

2 stara brida, opseg ostaje nepromijenjen, 3 stara brida, opseg se smanjuje za 2, 4 stara brida, opseg se smanjuje za 4.

U svim sluEajevima promjene su parne. NapiHite precizno korak indukcije sami.

9+ Da smo imali pravokutne ploEice razliEitih cjelobrojnih dimenzija morali bismo odustati od uvjeta d a se ploEice smiju dodirivati samo u cijelom bridu. Kako treba formulirati zadatak za t a j sluEaj?

9+

Kako bi se morale dodirivati ploEice da bi opseg bio cjelobrojan?

8. Pretpostavimo da moIemo kupiti pogtanske marke samo u vrijednosti od po 3 i 5 kuna (u proizvoljnim koliEinama). DokaBite da i u tom sluEaju moZemo pomoCu maraka koje su nam na raspolaganju platiti svaku poStarinu veCu od 8 kuna, zaokruzenu na cijeli broj kuna.
Rjes'enje 1. Baza indukcije - poitarina iznosi 8 kuna. Kupimo jednu marku od 3 i jednu od 5 kuna. Pretpostavka indukcije. Pretpostavimo d a znamo izabrati marke ako je iznos poZtarine k kuna, gdje je k 2 8 . Korak indukcije. Pogledajmo kako Cemo, ako je poznat jedan moguCi izbor maraka za platanje pogtarine od k kuna, izabrati marke kojima Lemo platiti za kunu skuplju po'starinu. Krasno bi bilo kad bismo imali na raspolaganju marku od jedne kune - samo bismo je prikljuEili veC izabranim markama. No, marka s najmanjim iznosom vrijedi 3 kune, 'st0 nas navodi na misao da Cemo neke od maraka koje smo pripremili za plaCanje poHtarine od k kuna morati ukloniti. SlijedeCi opisanu strategiju, lako moiemo vidjeti da moguCe rjeHenje glasi: imamo li u veC pripremljenoj skupini maraka marku od 5 kuna, uklonimo je i stavimo umjesto nje dvije marke od po 3 kuna (i time se ukupni iznos uveCao za jednu kunu) ako u pripremljenoj skupini nemamo marku od 5 kuna, onda sigurno imamo barem tri marke od po 3 kuna (prisjetite se, plaCamo iznose veCe ili jednake 8 kuna, a u skupini maraka kojom plaCamo poHtarinu od toEno 8 kuna postoji marka od 5 kuna). Tada uklonimo te tri marke i umjesto njih stavimo dvije marke od po 5 kuna. Rjes'enje 2. Da bi indukcija izgledala "normalno", tj. da imamo bazu indukcije na kakvu smo navikli i korak indukcije kojim prelazimo s tvrdnje za k na tvrdnju za k 1 , napravili smo malo kompliciraniji korak indukcije. Evo i prirodnijeg rje'senja! Ideja je u tome d a poveCanje iznosa poHtarine realiziramo uvijek samo lijepljenjem marke od po 3 kune (bez uklanjanja pripremljenih maraka).

Baza indukcije. BuduCi d a ielimo svaki iznos poHtarine doseLi dodavanjem marke od tri kuna, u bazi indukcije trebaju stajati izbori maraka za postarine od 8, 9 i 10 kuna. Napravite to sami! Pretpostavka i korak indukcije. Kao Hto je ranije opisano, korak je u stvari prijelaz s pogtarine iznosa k na pogtarinu iznosa k 3 .

PokuHajte ovaj dokaz modificirati tako d a se zaista vidi d a se radi o "klasiEnojn matematiEkoj indukciji (u koraku indukcije dokazujemo da ako tvrdnja vrijedi za neki n , da

vrijedi i za n od njih.

+ 1). Uputa: rastavite tvrdnju zadatka na tri tvrdnje i uoEite dokaze svake

9+ Drugo rjegenje dokazuje Eak i "jaEuVtvrdnju: za plaCanje pogtarine dovoljne su dvije marke od po 5 i preostale marke od po 3 kune. Ako ielimo ograniEiti broj maraka od e biti potrebno maraka po 3 kune, uz neograniEenu koliEinu maraka vrijednosti 5, koliko C vrijednosti 3 kune? 9+ Ovaj zadatak nalikuje na jednu drugu, dosta poznatu tvrdnju: ako su m, n E N , relativno prosti, onda postoje u, v E Z takvi d a je
m u + n v = 1, pa onda i za svaki i E Z postoje w, z E Z takvi d a je Tvrdnja se lako dokazuje koriitenjem Euklidova algoritma. Razmislite o nekim pooptenjima zadatka! Usporedite ovu tvrdnju s tvrdnjom iz zadatka.

% Za ulazak na neku priredbu Elanovi drugtva-organizatora pladaju 15, a ostali 25 kuna. Doprinosi se bacaju u gkrabicu. Koliki je minimalni iznos koji se mora naCi u ikrabici, a da se viie ne moie odrediti koliko je ljudi bilo prisutno? Nadalje, ako svi ubacuju priloge u iznosima koji su vigekratnici od 5 , od kojeg ukupnog iznosa nadalje se neCe moCi otkriti prijevara? 9. Dano je 2n 1 utega teZine cjelobrojne u gramima. Koji god od tih utega uklonili, preostali utezi se mogu grupirati u dvije skupine od po n utega, tako da teZine tih skupina budu jednake. DokaZite da je svih 2n 1 utega jednake teZine.

Rjes'enje. Parametar koji nam se nudi za provodenje matematieke indukcije je ukupni broj utega ili njegova polovina ( n ) . Pailjivo EitajuCi zadatak vidi se i d a je npr. teiina najteieg utega takoder jedan od moguCih parametara. Pokuiajte s tim parametrima provesti matematiEku indukciju direktno, kao Hto smo to Einili do sada. Radite prvo s malim vrijednostima pararnetra - pet utega ili maksimalna teiina jednaka 3 ili 5 sasvim su dovoljni. Problem koji se pritom javlja je d a pri smanjivanju parametra ne moiemo viSe provjeriti pretpostavku zadatka za manji, odnosno lakSi skup utega. Upoznajmo zato pobliie zadani skup utega. Moida iz nekog svojstva uspijemo vidjeti kako moiemo smanjiti neki od uoEenih parametara. Koji god uteg izvadili, ostale moiemo podijeliti u dvije skupine, tako d a je zbroj teiina u jednoj jednak zbroju teiina u drugoj. Zato znamo d a je zbroj teiina svih utega bez izdvojenog paran broj (ako je a - b paran, onda je i a b paran). OznaEimo li teiinu pojedinog utega s u; , a ukupnu teiinu sa S , onda imamo 21.9 - ui za svaki i E N2n+l .

Pretpostavimo da je S paran. Tada zbog upravo otkrivenog svojstva i svi utezi moraju biti parne teiine, pa moiemo svaki od njih prepoloviti. Uzmimo po jednu polovinu svakog od utega. Novi, manji utezi zadovoljavaju uvjete zadatka (cjelobrojne su teiine i zadovoljavaju uvjet o vaganju), pa za paran S imamo osiguran prijelaz na manju ukupnu teiinu.

2.

MATEMATICKA

INDUKCIJA

Ako je S neparan, onda i svi utezi moraju biti neparne teiine. Uklonimo svakom utegu po jedan gram (otpilimo ruEku) i ostatak prepolovimo. Uzmimo po jednu polovinu svakog od utega. U dobivenom skupu utega svi su utezi cjelobrojne teiine u gramima. Da li se saEuvao uvjet o vaganju? U svakom od starih poloiaja ravnoteie uklonili smo po n grama sa svake strane (obje skupine su bile velizine n ) i ostatak u svakoj skupini prepolovili. Time je ravnoteia ostala saEuvana, a mi smo ponovno smanjili ukupnu teiinu promatranog skupa utega zadriavajudi uvjet zadatka. Time je zavrgena skica smanjivanja ukupne teiine skupa utega. Vidimo da postoji moguknost tvrdnju zadatka dokazati indukcijom po ukupnoj teiini svih utega. UoEimo da se pri ovom postupku nije mijenjao broj utega. Zato uzmimo proizvoljni n 6. N i provedimo matematiEku indukciju po S . Skicirajmo dokaz.
2n

+ 1 utega teiine 1 i tvrdnja vrijedi.

Baza indukcije. Najmanja ukupna teiina je S = 2n

+1.

U tom sluEaju imamo

Pretpostavka indukcije. Pretpostavimo da svaki skup utega ukupne teiine manje od

S koji zadovoljava uvjete zadatka sadrii utege jednake teiine. Korak indukcije. Ako su utezi ukupne teiine S , onda znamo t u teiinu smanjiti, a
da skup utega ostane iste veliEine i da ostane zadovoljen uvjet o vaganju. Postupci su ranije opisani i razlikuju se za paran i za neparan S. U lakLem skupu utega su prema pretpostavci indukcije svi utezi jednake te8ine. Prema tome su i u polaznim skupovima svi utezi jednake teiine (jer je smanjivanje teiine bilo injektivno).

9+ Naaite jedan skup utega (:to manji to bolji) koji zadovoljava tvrdnju zadatka do na Einjenicu da je uvijek u obje skupine po n utega, ali za kojeg ne vrijedi zakljuEak kao u zadatku. 9+
Moie li se indukcija provesti jog po kojem parametru?

10. Kralj je pozvao najbolje matematieare svoga kraljevstva u dvorac, da bi odredio koji je od njih najmudriji. Kad su se okupili, rekao im je: "Stavio sam bijele SeSiriCe na glave nekih od vas. Oni koji nisu dobili bijeli SeSiriC, dobili su crni. Smijete se promatrati i razmiSljati, ali ne i razgovarati. Sada C u otiCi i vraCati se svakog punog sata. Svaki put kad se vratim, Belim da mi onaj koji je u proteklom satu otkrio da nosi bijeli SegiriC, to kaIe." Kralj je otiSao. U njegovom n-tom dolasku svih n matematieara koji su imali na glavi bijele SeSiriCe je ustalo i obavijestilo kralja da zna da nosi bijeli SeSiriC. ZaSto? Dodatni uvjeti: kralj nikada ne laZe, matematieari su mudri i svjesni su toga, u prostoriji nema ogledala, SeSiriCi su mali (nitko ne vidi obod svog GeSira).
Rjes'enje. Ako je kralj stavio samo jednom matematiEaru na glavu bijeli Le'siriC, onda on razmiglja ovako: "Svi ostali matematiEari imaju na glavi crne lieSiriCe. BuduCi da kralj nikad ne laie, onda Sam ja t a j koji ima bijeli LeSiriC." Ostali matematiEari vide po jedan bijeli ZeSirit, pa znaju: ako njegov vlasnik vidi same crne, javit C e se odmah po prvom kraljevom dolasku. Zato moraju Eekati prvi kraljev dolazak i vidjeti d a li C e se osoba s bijelim HegiriCem javiti. Ako se on ne javi, znaEi da vidi jog jedan bijeli. Primjer - n = 2 . Svaki od matematizara koji vidi n - 2 crna ZiegiriCa i jedan bijeli razmiglja ovako: "Da ja imam na glavi crni 'SeLirid, onda bi onaj na Eijoj glavi vidim bijeli

2.

MATEMATICKA

INDUKCIJA

vidio na svim ostalim glavama crne, pa bi (upravo pokazano) znao d a na glavi nosi bijeli SeSir i javio se kralju u njegovom prvom dolasku. BuduCi da on to nije napravio, to znaEi da i ja nosim bijeli SeSiriC." Tada Eeka do sljedeCeg (ukupno drugog) kraljevog dolaska i javi se.

S druge strane, oni koji vide po n - 3 crna SeSiriCa znaju d a C e se vlasnici bijelih javiti u drugom dolasku ako oni vide po n - 2 crna. Tek ako se oni ne jave, znaEi da oni koji vide po n - 3 crna SeSiriCa, nose bijele.
Iskoristimo ovaj primjer da napravimo korak indukcije. Pretpostavimo prvo d a se, ako je kralj stavio bijele SeLiriCe na glave k matematiEara, oni javljaju u k-tom kraljevom dolasku. Ako je kralj podijelio Ic 1 bijeli SeSirit, onda svaki od matematiEara koji je dobio ja imam na glavi crni SeSiriC, bijeli (oni koji vide n - k - 1 crnih) razmiSlja ovako: LLDa onda bi to znaEilo d a je kralj podijelio Ic bijelih - upravo ove koje vidim na glavama svojih kolega. Ali znam (prema pretpostavci indukcije) d a bi se oni tada javili u k -tom kraljevom dolasku." Zatim svaki od tih matematiEara Eeka k-ti kraljev dolazak i tada, nakon Sto vidi da se nije javio niti jedan matematiEar, zna da i on nosi bijeli SeSir i javlja se kralju u njegovom sljededem dolasku. S druge strane, oni koji vide p? n - k crnih i svih k 1 bijelih HeLiriCa znaju da se do ukljuEivo k 1-vog kraljevog dolaska ne trebaju brinuti (upravo pokazano). U k 1-vom dolasku se matematiEari s bijelim SeLiriCima jave, pa vlasnici crnih na t a j naEin doznaju da nose crne SeLiriCe.

Ovo nije kodiranje, tj. tajni dogovor izmedu matematiEara kao Lto na prvi pogled izgleda. ProuEite dokaz joS jednom i uvjerite se d a se matematiEari zaista ne mogu javiti ranije.

11. U neka polja n x n tablice upisane su zvjezdice. Poznato je da nakon brisanja proizvoljnog broja redaka (0 ili 1 ili . . . ili n - 1) ostaje barem jedan stupac s toEno jednom zvjezdicom. Dokagite da nakon' brisanja proizvoljnog broja stupaca (0 ili 1 ili . . . ili n - 1) ostaje barem jedan redak s toEno jednom zvjezdicom.
Rjeiienje. Za n = 1 tablica ima jedno polje i u njega mora biti upisana zvjezdica (da bi nakon brisanja 0 redaka ostao stupac s jednom zvjezdicom). Nakon brisanja 0 stupaca ostaje redak s jednom zvjezdicom. Primjer - n = 2 . Pogledajmo prvo kako uopCe izgleda tablica. Postoji stupac s toEno jednom zvjezdicom, da bi nakon brisanja 0 redaka ostao stupac s toEno jednom zvjezdicom. Bez smanjenja opCenitosti pretpostavimo d a je to drugi stupac i d a se zvjezdica nalazi u drugom retku. Pogledajmo Sto se dogada u sluEaju brisanja jednog retka. Ako je t a j redak prvi, sve je u redu. No, Zelimo li obrisati drugi red, mora ostati jedna zvjezdica u nekom od stupaca preostale tablice. Ona sigurno nije u drugom stupcu (jer je u poEetnoj tablici u drugom stupcu bila samo jedna zvjezdica i to u drugom redu). Dakle, ona mora biti u prvom retku. OznaEimo li kriZiCem poziciju na kojoj moZe stajati bilo Lto, tablica izgleda ovako:

2.

MATEMATICKA

INDUKCIJA

21

ili se moie svesti na ovakav oblik permutiranjem redaka, odnosno stupaca (sjetite se, radi lakSeg rada fiksirali smo poloiaj prve zvjezdice). Tablica ovakvog oblika oEito zadovoljava zadanu tvrdnju. Preformulirajmo zadatak i dokaiimo d a se reci i stupci tablice koja zadovoljava uvjete zadatka uvijek mogu permutirati tako da matrica poprimi oblik

Ako to dokaiemo, zadatak je rijegen, jer za svaki n E N matrica ovakvog oblika zadovoljava tvrdnju zadatka. Za n = 1 dokaz veC imamo, pa pretpostavimo onda da L'nova" tvrdnja vrijedi za n=k-1.

U k x k matrici u jednom stupcu (pretpostavimo: zadnjem) mora stajati toEno jedna zvjezdica (pretpostavimo: na zadnjoj poziciji) - zbog eventualnog brisanja 0 redaka.

No, brigemo li vise od 0 redaka, moie nam se dogoditi da obrigemo i zadnji redak. Tada ne moZemo raEunati na zvjezdice iz zadnjeg retka. Stoga tablica koja se dobiva brisanjem zadnjeg retka i zadnjeg stupca (njega moiemo slobodno obrisati nakon brisanja zadnjeg retka jer je ostao prazan) je (k - 1) x (k - 1) tablica koja mora zadovoljavati poEetne pretpostavke. Prema pretpostavci indukcije ona je trokutasta, pa je i cijela k x k tablica takoder trokutasta.

12. U gradu Zurbaganu sve su ulice bile dvosmjerne. Kad ih je trebalo popravljati, gradske vlasti su dio ulica proglasile jednosmjernima. Nakon Sto je prvi dio popravaka zavrSen, ulice koje su bile dvosmjerne postale su jednosmjerne, a one koje su bile jednosmjerne postale su dvosmjerne. Za vrijeme popravaka u gradu se moglo putovati izmedu bilo koja dva mjesta. DokaEite da se u Zurbaganu moEe organizirati promet tako da sve ulice budu jednosmjerne, a da se ipak moEe stiCi od bilo kojeg mjesia u gradu do bilo kojeg drugog mjesta u gradu.
Rjes'enje. Dovoljno je promatrati putovanja izmedu raskrZCa umjesto putovanja izmedu bilo koje dvije toEke u gradu. Nadalje, jasno je da Cemo, promatrajuCi plan grada Zurbagana, bitnima smatrati samo ona raskrgCa u kojima se susreCu barem tri ulice.
BuduCi d a za Zurbagan ne pomajemo niti broj niti raspored ulica, morat demo dokazati d a se u svakom gradu promet moie organizirati u jednosmjernim ulicama Eim se popravak ulica moie organizirati kao u Zurbaganu. Iz uvjeta zadatka znamo da u takvim

gradovima nema slijepih ulica. Nadalje, dodavanje nove ulice nekom od tih gradova olakLava promet za vrijeme popravka, ali s druge strane ne utjeEe niti na plan popravaka niti na kasnije uvoetene jednosmjernog prometa. PokuHajmo zato dokazati tvrdnju indukcijom po drugom zanimljivom parametru u zadatku - broju "bitnih" raskrLCa. Ako je t a j broj 0, onda se grad sastoji samo od jedne kruine ulice (ako uopCe i ima koju ulicu). Ulicu takvog grada lako je proglasiti jednosmjernom. Pretpostavimo d a znamo organizirati jednosmjerni promet u svim gradovima u kojima je moguCe popravak ulica organizirati kao u Zurbaganu, koja nemaju viHe od n "bitnih" raskrLCa.

U zadanom (proizvoljnom) gradu s n 1 bitnih raskrHCa uoEimo jedno od tih raskrLCa. Neka se ono zove A . BuduCi da i u tom gradu moiemo organizirati popravak kao u Zurbaganu, znamo d a od raskrLCa A vode barem dvije ulice prema skupu ostalih bitnih raskrgCa (inaEe bi A bilo ulazak u slijepu ulicu, pa se popravak ne bi mogao organizirati na opisani naEin). Ako jedna od tih ulica vodi prema npr. raskrSCu B , onda nakon uvodenja jednosmjernog prometa u ulici AB (od A prema B ) moramo moCi doCi i iz B u A . Jasno je d a iz B u A ne mora voditi ulica, jer se moiemo kretati i zaobilaznim putem. Spajanjem ulice AB i puta iz B u A dobivamo "kruinu turd'. Uvedimo u nju jednosmjerni promet u bilo kojoj orijentaciji. Ostalo nam je jog "usmjeriti" ostale ulice.
Prije njihovog usmjeravanja uoEimo d a moiemo stiCi od bilo kojeg do bilo kojeg raskrLCa uoEene 'lkruine ture" koristeCi samo njene bridove. Dakle, kretanje po njoj je nezanimljivo, pa moiemo sva njena raskrLCa smatrati jednim raskrSCem (tj, jednim velikim prstenastim trgom). Tako dobiveni grad ima manje od n raskrHta i u njemu je moguCe organizirati popravke kao u Zurbaganu (Eak s istim rasporedom popravaka, osim :to sc po novom trgu moiemo slobodno kretati). Prema pretpostavci indukcije u nano' vo- ;rgani=iiranom gradu rnoiemo uvesti jednosmjerni promet u svim ulicama, tako d a se h ~ ~ ~ e ~ & f 'bilo & b kojeg d do bilo kojeg mjesta u gradu. Zajedno s veC usmjerenom ulicom 4 7;" c f -9. -3 pov&f6g~lanj*~rometa Eini dobru regulaciju prometa u cijelom gradu uz koriHtenje samo n<&hfig: s#n$$&ih:ulica. ' ->, . i-J+%Z.& c* 4+ Ako ste se veC upoznali s osnovama teorije grafova, pokuLajte "prevesti" ovaj zadatak na jezik teorije grafova.

4 '

13. Niz balansiranih zagrada je konaEni nix sastavljen od znakova iz skupa {(, )) koji zadovoljava predodLbu "dobro ugnijeidenih zagrada". Skup svih nizova balansiranih zagrada definira se trima svojstvima: (1a)Prazni niz (niz koji ne sadrii niti jedan znak) niz je balansiranih zagrada. (1b)Ako je x niz balansiranih zagrada, onda je i (x) niz balansiranih zagrada. (1c)Ako su x i y nizovi balansiranih zagrada, onda je i xy niz balansiranih zagrada (xy je niz koji Eitan slijeva prvo sadrii niz x , a na njega je nalijepljen niz y ) .

S druge strane, postoji definicija koja kaZe da je niz w s elementima iz skupa {(, ) } niz balansiranih zagrada ako zadovoljava sljededa svojstva:
(2a) w ima isti broj otvorenih i zatvorenih zagrada.

2.

MATEMATICKA

INDUKCIJA

23

(2b)Svaki poEetak x niza w = xy sadr2i vise ili jednako otvorenih nego zatvorenih zagrada. Dokagite da su ove dvije definicije ekvivalentne: ako se neki niz moBe definirati pravilima (I), onda zadovoljava svojstvo (2) i obratno.
RjeBenje. (1) =+- (2). Da niz generiran prvom definicijom zadovoljava svojstvo (2a) je oeigledno, jer se u definiciji (1) zagrade pojavljuju uvijek u paru. Svojstvo (2b) je ispunjeno zato, jer se u (1) pojavljuje uvijek otvorena ispred zatvorene zagrade. Izrecimo to preciznije.

Promatrajmo razliku izmedu broja 2tvorenih i broja zatvorenih zagrada za svaku poziciju niza. To je dubina ugnjeidenja. Zelimo d a ona bude uvijek veCa ili jednaka n d i . Ako je zadnji korak pri definiranju niza bio (lb), onda su razlike na mjestima Clanova novog niza za jedan veCe od razlika na mjestima odgovarajukih Elanova starog niza. Ako je zadnji korak bio (lc), razlike odgovaraju razlikama u nizovima x i y .

KNJI~NICA
PAIWLTETA

ELEKTROTEHNIN
U svakom sluEaju nizovi x i y su kraki od polaznog niza w i definirani su pravilima (1).Dovoljno je dakle za njih znati d a zadovoljavaju definiciju (2).
Dogli smo do razloga za primjenu matematieke indukcije po duljini niza. Niz duljine
0, definiran pomoku (I), zadovoljava i (2). Pretpostavimo da svaki niz duljine n definiran

r RA~UWWA ZAGREB - Unsko :

prema (1) zadovoljava i definiciju (2), te dokaiimo da onda i nizovi duljine n 2 definirani pravilima (1) zadovoljavaju (2) (zaZto smo preskoEili n 1?). Prema prethodnom razmatranju dovoljno je dokazati d a "sastavni dijelovi" niza duljine n 2 ( x , odnosno x i y ) zadovoljavaju definiciju (2). No to vrijedi prema pretpostavci indukcije i time je dokaz zavrgen.

(2) + (1). Ponovno znamo d a je niz balansiranih zagrada parne duljine (ovaj put je argument drkueiji - izrecite ga). PokuZajmo i ovaj dokaz provesti indukcijom po duljini niza. Ako je niz duljine 0, onda zadovoljava svojstvo (la) definicije (1). Pretpostavimo da svaki niz duljine n koji zadovoljava definiciju (1) zadovoljava i definiciju (2). Analizirajmo niz duljine n + 2 . Na prvoj poziciji u nizu je prema (2b) sigurno otvorena zagrada i odgovarajuka dubina ugnjeidenja je 1. Prema (2a) je dubina zadnje pozicije jednaka 0. Do krakih nizova doCi Eemo tako, da potraiimo prvu poziciju dubine 0. Gdje

god ona bila, znamo d a je odgovarajuti znak zatvorena zagrada (zagto?), pa polazni niz znakova moZemo zapisati kao w = (wl)wl'. Nizovi w' i wl1 mogu biti i prazni.

Dubine u nizu w' su za jedan manje od dubina na odgovarajutim pozicijama niza w i zbog naEina izbora w' pozitivne. Dubine niza w" su jednake odgovarajutim dubinama niza w i takoder pozitivne. Dakle w' i wl' su nizovi duljine ne vete od n koji zadovoljavaju definiciju (2). Prema pretpostavci indukcije oni zadovoljavaju i (I), a to znaEi d a je niz w dobiven iz dvaju nizova, w' i wl1 , koji zadovoljavaju pravila (1) transformacijama sadrianim u (1): w' -+ (w') , (w'), wl' + (wl)w" . Napisite program koji uEitava neki niz zagrada i provjerava d a li je on niz ba9 ) lansiranih zagrada. NapiSite program koji uEitava neki prirodnih broj i generira sve nizove balansiranih zagrada duljine manje ili jednake tom broju. Objasnite kakva je korist od ekvivalentnih definicija. nizove balansiranih zagrada 9 Razmislite o definicijama i programima za LLprave" koji sadrZavaju okrugle, uglate i vitiEaste zagrade.

Dirichlet ov princip
cesto se neki vrlo tegki zadaci mogu rijeSti primjenom oEigledne tvrdnje, zvane Dirichletov princip: Ako n+ 1 zeEeva treba raspodijeliti u n pretinaca, onda C e postojati barem jedan pretinac s barem dva zeca u njemu. Primijetite da Dirichletov princip ne moZe reCi o kojem se pretincu radi, niti koliko C e u njemu biti zeEeva. Dirichletov pricip moBe se izreCi i opCenitije: Ako k n r zeEeva, r 2 1, treba raspodijeliti u n pretinaca, onda C e postojati barem jedan pretinac s barem k 1 zeEeva u njemu.

Ocjena C e biti najbolja ako je broj zeEeva tako prikazan pomoCu k i r , da je k maksimalan, tj. 1 r n . Zadaci u kojima se moBe jednostavno primijeniti Dirichletov princip su redovito zadaci u kojima se traZi dokaz egzistencije nekog objekta. Primjenom Dirichletovog principa naBalost se ne moBe odgovoriti na pitanja u vezi s konstruiranjem traZenih objekata, niti dati toEan odgovor o broju traBenih objekata. Stoga su zadaci tipa L'dokaZida postoji. . . " prikladni za pokuSaj rjeSavanja Dirichletovim principom, dok zadaci tipa "koliko ima objekata" ili "koji objekti . . . " najEeSCe nemaju direktno rjegenje primjenom Dirichletovog principa. Gesto se zadaci umjesto Dirichletovim principom mogu jednostavno rijeSiti tako da se krene od pretpostavke da tvrdnja nije istinita, pa se dode do kontradikcije (tj. dokaBe se obrat po kontrapoziciji tvrdnje iz zadatka). Uostalom tako se dokazuje i sam Dirichletov princip.

< <

3. DIRICHLETOV PRINCIP

'U uEionici irna 15 raEunala, a u razredu ima 35 utenika. Dokaiite da postoji raEunalo za kojim C e sjediti barem troje uEenika.
RjeHenje 1 . SmjeStavamo uEenike za raEunala kao zeEeve u pretince, pa moZemo primijeniti Dirichletov princip za n = 15. Imamo 35 = 2 . 15 5 raEunala ( Ic = 2), pa slijedi da postoji raEunalo za kojim sjedi barem troje uEenika.

Rjebenje 2. Prirodniji naEin rjeeavanja bio bi ovaj: kad bi za svakim raEunalom sjedilo najviee dvoje ljudi, u razredu bi moglo biti najvige 2 . 15 = 30 uEenika. No, uEenika je vige, pa zakljuEujemo d a mora postojati raEunalo za kojim sjedi vige od dvoje uEenika.

tP U promatranom razredu je petero uEenika "viSak" u odnosu na broj raEunala. No unatoE tome ne moZemo reCi da C e za toEno pet raEunala sjediti barem troje uEenika, jer moZe svih petero uEenika sjediti za jednim raEunalom, ili da Eak cijeli razred sjedi oko jednog raEunala!

:t"

Tablica 5 x 5 popuni se brojevima iz skupa {-1,0,1). IzraEunaju se sume pojedinim recima, stupcima i na obje glavne dijagonale. Dokaiite da, kako god bila tablica popunjena, medu tim sumama postoje dvije jednake!
Rjebenje. Nema bag puno moguCnosti za sume - od 5 . (-1) do 5 . 1 . To je ukupno 11 moguknosti. IzraEunatih suma po stupcima ima 5, jog ih je 5 po recima i dvije na dijagonalama. Dakle, ukupno 12 suma koje poprimaju vrijednosti iz 11-Elanog skupa. Medu njima mora biti jednakih.

'9

U vreCi se nalaze Earape 13 razliEitih boja. Koliko najmanje Earapa treba izvuCi iz vreCe da bi medu njima sigurno bile dvije Earape iste boje?
Rjes'enje. Dok izvlaEimo Earape, odmah Cemo ih slagati prema boji. 13 je mogutnosti za boju izvuEene Earape, pa ako Ielimo da sigurno budu izvuEene dvije istobojne Earape, moramo izvudi najmanje 1 3 . 1 1 = 14 Earapa.

4+ Da sluEajno imamo Eetiri noge, t a j rezultat bi bio 1 3 . 3 + 1 = 40. ObrazloZite!

DokaZite da u proizvoljnom skupu ljudi postoje barem dva Eovjeka koji du prisutnim ljudima imaju jednak broj poznanika.
Rjebenje. Ako je prisutno n ljudi, onda svaki od njih medu prisutnim ljudima moZe imati O , 1 , . . ,n - 1 poznanika. Odavdje ne moZemo niSta direktno zakljuEiti jer mogutnosti za broj poznanika ima jednako kao i ljudi u promatranom druStvu. No,

ako postoji osoba s 0 poznanika, ne postoji osoba s n - 1 poznanika ako postoji osoba s n - 1 poznanika, ne postoji osoba s 0 poznanika. Dakle, promatrani skup ljudi moBe biti takav d a svatko moZe poznavati ili O , 1 , . . . ,n - 2 ili 1,2,. . . ,n - 1 ljudi. U oba sluEaja skup ljudi se prema broju poznanika raspada na n - 1 klasa. BuduCi da je u skupu n Ijudi, prema Dirichletovom principu postoji klasa u kojoj je barern dvoje ljudi.

3. DIRICHLETOV PRINCIP

27

9 Relacija "biti poznanik" je simetriEna i to je bilo bitno u rjeSavanju ovog zadatka. Istaknite mjesto na kojem smo koristili t u Einjenicu.
5. DokaBite da u konveksnom poliedru postoje dvije plohe s jednakim brojem bridova.
Rjes'enje. Treba d_okazati da konveksni poliedar ima viSe ploha nego mogudih izbora broja bridova ploha. Cinjenica d a je poliedar konveksan znaEi da svake dvije susjedne plohe imaju toEno jedan zajedniFki brid - uvjerite se u to i nadite protuprimjer za neki nekonveksni poliedar! Dakle, svaka ploha ima onoliko susjeda koliko ima bridova. Neka ploha s maksimalnim brojem bridova ima toEno S susjeda. Tada sve ostale plohe imaju 3 , 4 , . . . , S bridova. Medu njima su i susjedi promatrane plohe. Vidimo da ved medu susjedima ima S ploha, a samo S - 2 mogudnosti za broj bridova. Dakle i medu njima sigurno postoje dvije plohe s jednakim brojem bridova.

tP UoEite da je broj susjeda promatrane plohe za dva vedi od broja mogudnosti za broj bridova. To znaEi da C e postojati
ili barem tri plohe s jednakim brojem bridova ili barem dva para ploha s jednakim brojem bridova (npr. dva trokuta i dva Eetverokuta)

tP U stvari, ocjena bi mogla biti jog bolja. Naime, ako su sve plohe poliedra susjedi plohe s maksimalnim brojem bridova, onda imamo piramidu, tj. sve ostale plohe su trokuti. InaEe imamo i vise od S "preostalih" ploha.
6. U ravnini je zadano 5 cjelobrojnih toCaka. DokaBite da je poloviSte barem {edne od duIina odredenih tim toEkama cjelobrojna toEka.
Rjes'enje. Koordinate polovigta dutine s krajevima (xl, yl) i (x2,y2) dane su formulom ((21 x2)/2, (yl y2)/2). Dakle poloviSte de biti cjelobrojno ako i samo ako su prve koordinate oba kraja duiine iste parnosti, te ako isto vrijedi i za druge koordinate. Parnosti prve i druge koordinate mogu biti (par,par) , (par, nepar) , (nepar,par) ili (nepar, nepar) . Vidimo d a postoje samo Eetiri mogudnosti za parnost koordinata tih pet toEaka, pa zakljuEujemo da postoje dvije toEke koje imaju prve i druge koordinate iste parnosti.

9 Koliko bi trebalo biti zadano toEaka u prostoru da bi se mogao izvesti analogni zakljuEak?

9-t Koliko bi trebalo zadati cjelobrojnih toraka u ravnini, od kojih nikoje tri nisu kolinearne, da bismo sa sigurnogdu mogli redi da barem jedan od trokuta odredenih tim toEkama ima cjelobrojno tetigte?
Koji bi nas argument mogao dovesti do rjegenja 2. 32 toEaka, 9? Razmislite o poopdenjima ovog problema na prostor!
i

+ 1,a koji do najmanjeg broja

$d3Koliko

treba i m d i karata iz snopa od 52 karte d a bi se izvukle:

3. DIRICHLETOV PRINCIP

u drugoj najmanje jedna, u tredoj najmanje dvije kuglice, u zadnjoj, m-toj najmanje m - 1 kuglica. To bi bilo najmanje 0 1 2 . . . (m - 1) = m ( m - 1)/2 kuglica, a na raspolaganju nam je n komada, n < m(m - 1)/2. Dakle, nije moguCe rasporediti n kuglica u m kutija, tako d a niti u jednom paru kutija ne bude jednaki broj kuglica.

+ + + +

9+ Ovo rjeBenje ne koristi "Eisti" Dirichletov princip, ali je metoda sliEna. Usporedite d a s drugim rjesenjem zadatka o raEunalima. Izrecite koje su sliEnosti i razlike u odnosu na Eisti Dirichletov princip.
Dokafite da medu proizvoljnih n + 1 brojeva postoje dva Eija je razlika @. jiva s n .
Rjes'enje. Svojstvo razlika djeljiva s n ne djeluje odmah kao da svrstava zadane brojeve u neke klase. No, razlika dvaju brojeva je djeljiva nekim brojem ako i samo ako ti brojevi imaju isti ostatak pri dijeljenju tim brojem. Tako dobivamo prirodno svrstavanje brojeva u klase, a polazna tvrdnja glasi: l1DokaZite da me& proizvoljnih n 1 brojeva postoje dva koja daju isti ostatak pri dijeljenju s n " .

- 1. Dirichletov princip
, ,

To je praktiEki oEigledno, jer postoji samo n ostataka pri dijeljenju s n : O , 1 , . odmah zakljuEuje dokaz.

.. ,

U svakorn skupu od 7 brojeva postoje dva Eiji je zbroj ili razlika djeljiva

s 10.

Rjes'enje. Ako razlika nije djeljiva s 10, onda polaznih 7 brojeva imaju medusobno razliEite ostatke pri dijeljenju s 10 (vidi rjesenje prethodnog zadatka). Pogledajmo sada kad je zbroj dvaju brojeva djeljiv s 10. NuHni uvjet za to je da je odgovarajudi zbroj ostataka djeljiv s 10. Ako ne ielimo da suma bude djeljiva s 10, onda pri dijeljenju s 10 ako jedan od brojeva daje ostatak
1, 7, 1, 1, 71

0 1 2 3 4 5

drugi ne smije davati

,,
9,

0, 9,
8,

9,

,,
I1

7, 6, 5.

Sve u svemu, dobili smo prostor za smjestiti 6 brojeva. A zadano ih je 7.. . . Poopdite ovaj zadatak: l1usvakom skupu od . . . brojeva postoje dva Eiji je zbroj Cl-) ili razlika djeljiva s 2n". Koliko brojeva treba uzeti d a bi neki zbroj ili razlika bili djeljivi s 2n+1?

12.

Za svaki n E N postoji visekratnik od n oblika 11... l o o . . . O .

Rjes'enje. Pokugamo li svesti ovaj zadatak na neki od prethodnih, dobivamo formulaciju: l1zasvaki n N neki od brojeva oblika 11. . .loo. . . 0 je djeljiv s n u . Ili, nastavimo

30

3. DIRICHLETOV PRINCIP

li dalje: s n".

. . .) je djeljiva svaki n E N neka od razlika brojeva iz skupa {1,11,111,1111,

BuduCi da je u tom skupu viie od n + 1 brojeva, tvrdnja koju ielimo dokazati slijedi iz jednog od prethodnih zadataka.

tP Zadatak nalik ovome koji medu neuputenima moie izazvati vige zbrke od ovog, je sljedeti: Dokaiite d a postoji broj koji poEinje znamenkama 01041994 a djeljiv je s 1994.
1 3 ; ) UEenik rjesava zadatke, svaki dan barem jedan, a tjedno najvise 12. DokaZite da postoji nekoliko uzastopnih dana u godini u kojima rijesi toEno 20 zadataka.
RjeHenje. Uvedemo li kao osnovne jedinice koje Cemo promatrati brojeve zadataka rijeSene u jednom danu, morali bismo promatrati sve njihove sume: od po jednog, dva, tri,. . . dana. To djeluje nepraktizno, pa je bolje odluziti se da s, npr. a l , az, . . . ,a365 oznaEimo koliEinu rijeLenih zadataka do prvog, drugog, . . . , 365-tog dana. Tada moramo promatrati razlike parova brojeva. ToEnije, zanima nas d a li postoje i, j ( i ne bi postojali, onda bi svi brojevi

< j , jasno)

takvi da je a j

- ai = 20.

Kad

bili razliEiti. Pokugat Cemo dokazati da medu brojevima od 1 do moguc'eg broja rjes'enja +20 nema mjesta za sve ove brojeve. UEenik nikad ne rijegi vise od 12 zadataka tjedno, te vige od Lest dnevno. BuduCi da godina ima 52 tjedna i jedan dan, svi su ovi brojevi manji od 12 .52 6 20 = 650. Brojeva u tablici ima 2 . 365 = 730 > 650, pa postoje dva jednaka. To ne mogu biti dva broja iz istog reda, jer uEenik svaki dan rijeSi barem po jedan zadatak ( a j < a j + l ) . Dakle, postoje i, j ( i < j , jasno) takvi da je a j - a j = 20.

+ +

4+ Koji bi brojevi mogli u zadatku stajati umjesto konstanti 12 i 20? Koje bi konstante dogle u obzir ako uEenik svaki dan rijeLi barem dva zadatka?
14. Medu n 1 razliEitih prirodnih brojeva manjih od 2 n postoje tri, takva da je jedan od njih jednak zbroju ostala dva.
Rjes'enje. Zadatak kaie d a ako izaberemo vilie od pola brojeva iz skupa Nzn = {1,2,. . . ,2n) , onda oni neCe nikada moCi biti dovoljno razbacani da bi se sprijeEilo d a suma nekih dvaju brojeva bude jednaka nekom trekem. Da bismo mogli Ymati red" medu sumama, uredimo dani niz prirodnih brojeva. Dajmo im imena tako da bude 0

< a1 < a2 < . . . < an+l < 2n.

PouEeni prethodnim zadatkom promotrimo sve sume a j a j za i < j - moida je neka od njih jednaka nekom od ak -ova, gdje je k # i, j . Odmah moiemo uoEiti d a sume nisu nuino medusobno razlitite, pa ne moiemo zakljuEiti kao u prethodnom zadatku.

3. DIRICHLETOV PRINCIP

31

Ovaj se nedostatsk lako moie ispraviti - promatrajmo samo sume za npr, i = 1 ( a i je najmanji element niza, pa od svih mogudih i-ova te sume imaju opdenito najveCu Sansu d a 'pogode' neki od ak -ova). Tada imamo n razliEitih suma. Sume mogu lekati izmedu 1 2 = 3 i (n - 1) (2n - 1) = 3n - 2 , pa ponovno ne mokemo zakljuEiti kao ranije.

Kad smo se uvjerili d a sa sumama ne mokemo (lako) pun0 postiCi pokugajmo s razlikama, zapisujudi tvrdnju koji kelimo dokazati u obliku ak - ai = a j , gdje su i, j i k medusobno razliEiti. Razlike imaju prednost d a "padaju" u manji skup: {1,2, . . . ,2n-2). Kao u prethodnom pokuSaju, i ovdje demo promatrati samo razlike oblika ak - a1 , za k > 1 . Imamo sada n medusobno razliEitih razlika i n - 1 medusobno razliEitih brojeva:

- a1 a3 - a1 . . . a n - a1 an+l - a1 a2 a3 ... an, (a1 u listu nismo uvrstili zato jer felimo izabrati tri razlic'ita broja; zagto nismo uvrstili an+l ?). Svi napisani brojevi su iz skupa {1,2,. . . , 2 n - 2). Sada prema Dirichletovom principu mokemo zakljuEiti d a postoji barem jedan par jednakih brojeva u ovoj listi, a prema naEinu kako smo je slofili znamo d a je jedan od tih brojeva u prvom, a drugi u drugom redu. Takoder je oEito d a niti jedan od brojeva od kojih radimo razliku ne moie biti jednak razlici (tj. nije niti ak - a1 = ak niti ak - a1 = a1 ).
a2

%+ Kako izabrati n brojeva manjih od 2n takvih da niti jedan od njih ne bude jednak sumi nekih drugih dvaju?

gto se dogada ako dozvolimo i sume jednakih brojeva?

15. Zadano je 20 razliEitih prirodnih brojeva manjih ili jednakih 70. DokaHite da medu razlikama koje od njih moHemo napraviti postoje barem Eetiri jednake.
Rjeienje. OznaEimo zadane brojeve tako d a nam bude olakSano stvaranje razlika
0 < m l < m2 < . . . < m20 < 70. Promatramo li sve medusobne razlike, bit C e ih 20 . 1912 = 190, a mogu poprimiti vrijednosti iz skupa Ne9. BuduCi d a je 6 9 . 3 = 207 > 190, to izgleda kao d a ima dovoljno mjesta d a smjestimo sve razlike u skup N69, a da se niti jedna ne ponovi Eetiri puta. Lako se vidi d a niti trik upotrijebljen u'rjezenju prethodnog zadatka (oduzimanje m l od svakog od ostalih brojeva) ne daje rjegenje ovog zadatka. OEito je d a problem leii u tome :to smo do sada razlike promatrali kao medusobno nezavisne. Pogledajmo sada "najmanje" razlike - one Eijim se zbrajanjem dobivaju sve ostale m2 - ml,mg -m2,...,m20 - mlg. Pretpostavimo d a medu.tih 19 razlika ne postoje Eetiri jednake. Dokazat Cemo d a se tada od njih ne moie napraviti najveCa razlika: m 2~ ml

< 69.
+. . . + (m2 - m l )

Promatrani skup radika sadrzi najviSe 3 jedinice, najviSe 3 dvojke, itd, najvige 3 Sestice i barem jedan broj koji nije manji od 7. Tada je najmanja moguCa razlika najveCeg i najmanjeg elementa m20

-ml

=
=

(m20 - m19) (m19 - m18) 3.(1+2+3+4+5+6)+7 70,

32

3. DIRICHLETOV PRINCIP

Sto daje kontradikciju s pretpostavkom da su svi brojevi manji ili jednaki 70.

tP Ovo rjeienje ima zanimljivu geometrijsku interpretaciju: zamislimo da pokria] , a 70 koristeCi toEno 19 ploEica cjelobrojne duljine, medu kojima vamo segment [l, ne postoje Eetiri ploEice jednake duljine. Zadatak kaZe d a C e uvijek barem jedna ploEica "viriti" iz segmenta.

<

16. Covjek je autostopirao 10 sati i preSao 45 kilometara. U prvom satu preSao je 6 , a u zadnjem samo 3 kilometra. DokaZite da postoje dva uzastopna sata u kojima je preSao barem 9 kilometara.
Rjeienje. Pretpostavimo d a tvrdnja ne vrijedi. Ako s ki oznaEimo broj prijedenih kilometara u i-tom satu ( kl = 6 , k10 = 3), onda imamo:
ki JoH znamo d a je
10

+ k;+l

< 9, i = 1 , . . . ,9.

ki = 4 5 . Zbrojimo sada sve nejednakosti za ki -6

+ 2(kl + k2 + . . . + kg + klo) - 3 < 9 . 9

+ ki+1 , dobivamo:

ili, nakon uvritavanja: 2 . 4 5 - 6 - 3 < 81. Kontradikcija.

4-t Usporedite ovaj zadatak sa zadatkom o uEeniku koji rjegava zadatke. UoEite sliEnosti i razlike!

17. DokaZite da je niz zadnjih 4 znamenaka brojeva niza 6 , 6 2 ,6 3 , . . . periodiEan.


Rjeienje. Dovoljno je pokazati da postoje k, m E N takvi d a 6k i 6k+m imaju jednake zadnje Eetiri znamenke, tj, d a postoji n E N takav d a je:
6k+m - 6k = 10000n. Tada je za svaki p E N : 6k+m+p - 6k+p = 10000. (n6p), pa je niz zadnjih 4 znamenaka brojeva niza 6,6', 63,. . . perioditan. Prva tvrdnja trivijalno vrijedi, jer postoji samo konaEno moguCnosti za izbor zadnje Eetiri znamenke, a beskonaEno brojeva oblika 6 k . Prema Dirichletovom principu sigurno e zavrLavati jednakim skupinama Eetiriju C e se naCi dvije potencije od 6, 6k i 6k+m , koje C znamenaka.

18.

DokaZite da postoji potencija broja 3 koja zavrSava znamenkama 0001.

Rjeienje. Dirichletov princip ne moie dati odgovor tipa "postoji element koji ulazi u toEno odredenu klasu". Pogledajmo ipak Sto se moie reCi u vezi sa zadnje Eetiri znamenke.
Kao u prethodnom zadatku moie se reCi d a postoje k, m, n E N takvi d a je
3k+m

- 3k = 10000n.

Mi bismo htjeli dobiti

3, y

E N takve da vrijedi

3" = 1ooooy

+ 1,

3. DIRICHLETOV PRINCIP

33

ili, napisano sliEnije tvrdnji koju imamo: 3"

- 1 = 1ooooy.

To je upravo prva jednakost, jer je n sigurno djeljiv s 3k (3 ne dijeli 10000). OznaEimo li n' := n 1 3 ~ ,dijeljenjem prve jednakosti s 3k dobivamo upravo oblik kojeg smo traiili: 3k

- 1= 10000n'.

19. DokaZite da je svaki racionalni broj p l q periodiEan decimalni broj. Koliki je maksimalni moguCi period? Prisjetite se, racionalni brojevi s konaEnim decimalnim prikazom imaju iza "zadnje" decimale jog beskonaEno mnogo nula.
Rjes'enje. Dijelimo li dva cijela broja "pjeSice", radimo to tako, da uzimamo po jednu znamenku broja kojeg dijelimo, prikljuEujemo prethodno dobivenom ostatku i onda dijelimo t a j broj tako d a jednoznamenkasti kvocijent pripisujemo zdesna prethodno dobivenom kvocijentu. S ostatkom postupak ponavljamo sve dok ne dodemo do znamenke jedinica. Ako pri dijeljenju nakon prikljueivanja znamenke jedinica dobijemo ostatak 0, to znaZ d a je kvocijent cjelobrojan, pa onda i periodiEan decimalni broj (s beskonaEnim brojem nula). InaEe piSemo decimalnu toEku iza napisanog kvocijenta, te nastavljamo: ostatku dopisujemo 0, dijelimo, dopisujemo kvocijent, ostatku ponovno dopisujemo 0 itd. Ako u tom postupku dodemo do ostatka 0 - dobili smo konaEan decimalni broj (koji jest periodican jer ima beskonaCno mnogo nula ' h a repu"). Tvrdimo da i inaEe dobivamo beskonaEni periodiEni decimalni broj.

MoguCi skup ostataka je {O,1, 2 , . . . ,q - 1). Prema Dirichletovom principu prije ili kasnije de se jedan od njih ponoviti. BuduCi d a Cemo i u prvom i u drugom pojavljivanju tog ostatka s njim napraviti istu operaciju (dopisivanje nule i dijeljenje), oEito je d a Cemo e se ponovno obavljati iste operacidobiti i isti rezultat - kvocijent i ostatak, nad kojima C je koje C e davati iste rezultate. Dakle niz znamenki kvocijenta (cijele operacije dijeljenja) nakon kvocijentika koji odgovara prvom pojavljivanju promatranog ostatka jednak je nizu znamenki kvocijenta nakon kvocijentiCa koji odgovara drugom pojavljivanju tog istog ostatka. Jasno je da sliEna tvrdnja vrijedi i za drugo i treCe i, optenito, za bilo koja dva pojavljivanja promatranog ostatka. Iz toga slijedi d a je niz znamenki iza odgovarajukeg kvocijenta beskonaEan i periodiEan.

9-+ PokuHajte ovu priEu zapisati precizno. PoFeti moiete npr. na sljededi naEin: neka su r l , ~ 2ra, , . . . ostaci koje dobivamo nakon prelaska decimalne totke. . . .
Sto mislite - d a li je izbor baze u kojoj ielimo prikazati racionalni broj bitan? Za koje q Cemo u pojedinoj bazi dobiti konaEan prikaz?

9 9

Koliki najvige moie biti period? Kad period moie biti manji?

Neka je zadan skup od 10 proizvoljnih dvoznamenkastih brojeva. Dokazite da postoje dva neprazna disjunktna podskupa zadanog skupa, takva da je suma brojeva u jednom jednaka sumi brojeva u drugom.

Rjes'enje. Uvjerimo se d a moguCih podskupova ima viHe nego raspoloiivih suma. To Hto podskupovi za koje C e izaCi da imaju jednake sume mo6da neCe biti disjunktni, ne

3. DIRICHLETOV PRINCIP

smeta - ako se oni zovu A i B zamijenit Cemo ih skupovima A \ B i B \ A . Ti skupovi C e oEito biti disjunktni. Neprazni C e biti zato jer je A # B , a imaju jednake sume elemenata. Slijedi raEun: ukupni broj nepraznih podskupova je 2'' - 1 = 1023. Suma elemenata podskupa moBe najvige iznositi: 99 98 . . . 90 = 945. 945 < 1023 i tvrdnja je dokazana.

+ + +

21. U jednoj (dalekoj) zemlji postoji n dvoraca, povezanih cestama tako da je svaki dvorac raskriije triju cesta. Ujedno, dvorci su i jedina raskriija. Vitez je krenuo u Betnju jednom cestom koja vodi iz njegova dvorca. Kod prvog dvorca skrenuo je desno. Kod svakog sljedeteg dvorca kretao se ovako: ako je kod prethodnog dvorca skrenuo lijevo, skrenuo je desno, a ako je kod prethodnog skrenuo desno, skrenuo je lijevo. Dokaiite da C e se vitez vratiti kuCi.
Rjeienje. Ako C e se dovoljno dug0 Hetati, sigurno C e neki od dvoraca posjetiti barem dvaput. To kaie Dirichletov princip. No, to nam nije dovoljno da dokaiemo d a C e se vratiti kuCi, jer vitez moie tako kruBiti nekim skupom "tudih" dvoraca. Primijenimo li Dirichetov princip na parove (dvorac, izlazna cesta) moBemo ustvrditi da de vitez sigurno barem iz jednog dvorca barem dvaput izaCi istom cestom. No, to j o H uvijek nije dovoljno! MoBemo j o H malo promijeniti objekte na kojima primjenjujemo Dirichletov princip i reCi da C e vitez sigurno barem dvaput proti na isti naEin kraj istog dvorca. To znaEi d a C e vitez i u prvom i u drugom prolazu kraj tog dvorca izaCi istom cestom skrenuvgi npr. lijevo. Ali to znaEi da C e na raskrSCu ispred, u oba prolaza skrenuti desno, na pretposljednjem raskrSCu lijevo itd. Dakle (do na konaEnost cijele getnje) Hetnja prije drugog posjeta promatranom dvorcu je jednaka getnji prije prvog posjeta. Posebno, ako je prvi put posjetio t a j dvorac u k -tom koraku, onda je sigurno k koraka prije druge posjete bio kod kuCe. To oEito nije bio poEetak ture, pa je tvrdnja dokazana.

Prebrojavanje
U ovom poglavlju pokuSat Cemo odgovoriti na niz pitanja u vezi s prebrojavanjem raznih rasporeda. Pritom C e dodatne poteSkoCe stvarati razna ograniEenja, odnosno zathjevi na njihova posebna svojstva.

Elementarni zadaci
Velik broj zadataka vezanih uz prebrojavanje mofe se rijesiti upotrebom triju oEiglednih principa za konaEne skupove: princip s u m : ako je skup A veliEine a , a skup B veliEine b , te ako su A i B medusobno disjunktni, onda je njihova unija A U B veliEine a b .

Na primjer, ako u razredu ima d djevojaka i m momaka, onda u tom razredu ima d m uEenika (ili kafemo da jednog uEenika moBemo izabrati na d m nacina).

princip produlcta (teorem o uzastopnom prebrojavanju): ako je skup A veliEine a , a skup B veliEine b , onda je njihov Kartezijev produkt veliEine ab .

To znaEi da za razred iz proslog primjera mofemo na dm naEina zamisliti jedan par koji se sastoji od jednog momka i jedne djevojke (ili kaiiemo da na dm naEina mofemo izabrati jedan "par" iz tog razreda).
principjednakosti: Ako postoji bijekcija izmedu skupa A i skupa B , onda su A i B jednakobrojni, tj. IAl = IBI. Princip jednakosti koristimo onda, kad je umjesto zadanih objekata jednostavnije prebrojati neke druge objekte koji su sa zadanima u bijekciji. Da

4. PREBROJAVANJE

bi ga mogli efikasno primjenjivati, moramo imati "zalihu" objekata koje poznajemo i koje znamo prebrojati. Na primjer, za razne Skolske izvjestaje radimo samo s imenima uEenika (ili s njihovim matiEnim brojevima!) - rezultat ostaje isti kao da brojimo npr, izostanke stvarnih uCenika, a informacija koja se time gubi (osoba umjesto imena) za statistiku nije bitna. Princip sume i princip produkta Eesto se koriste i za'vige skupova odjednom ( d o h matematiEkom indukcijom iz ranije navedenih principa). Ako su Ai , za i = 1,2, . . . , n medusobno disjunktni skupovi (tj. Ai n Aj = 0 , zasve i,j E N n , i # j), ondaje

( A l U A 2 U. . .U A , ( = I A l ( + ( A 2 ( + . . . + ( An(.
Za proizvoljne skupove

Ai, i = 1 , 2 , . . . , n vrijedi

/Al x A2 x

... x An/ = / A l l . l A 2 ( . . . ..]An(.

U zadacima neCemo uvijek naglaSavati primjenu tih principa.

j " )Jedno dijete ima tri zelene drvene bojice razlieitih nijansi, dvije plave, dvije
smede i jednu Butu. Na koliko naEina to dijete moBe izabrati jednu olovku?
Rjes'enje. Dijete moie izabrati ili zelenu ili plavu ili smedu ili t u t u olovku. Dakle skup bojica iz kojeg on0 moBe izabrati olovku jednak je unijl skupa zelenih, skupa plavih, skupa smedih i skupa iutih bojica. Ti skupovi su medusobno disjunktni, pa je prema principu sume veliEina tog skupa jednaka 3 2 2 1 = 8 .

+ + +

@ Na koliko naEina moie dijete iz prethodnog zadatka isabrati bojice za ertanje stabla sa smedim deblom i zelenom kroSnjom?
Rjes'enje. Dijete C e prvo posegnuti za jednom smedom bojicom, nacrtati deblo, a zatim posegnuti za zelenom i nacrtati kroSnju. Tako je svaki izbor bojica jedan uredeni par: na prvoj komponenti je jedna smeda, a na drugoj jedna zelena bojica. Prema principu produkta takvih je parova 2 3 = 6 komada.

9-t Vidite li gdje smo koristili princip jednakosti?

tP Sve crteie moiemo smjestiti u tablicu koja kao retke ima razliEite smecte, a kao stupce razliEite zelene boje.

4. PREBROJAVANJE

kroHnja

deblo
I,
\

svjetlozelena
L

zelena kao trava


I

tamnozelena

svjetlosmede

tamnosmede

Popunite ovu tablicu crteiima u odgovarajuCim bojama.

(3 Na koliko naEina C e dijete iz prethodnih zadataka moCi nactrati stablo sa smedim deblom, zelenom kro5njom i plavim cvjetovima iste nijanse?
Rjegenje 1. Za svako od 6 stabala iz prethodnog zadatka moiemo na 2 naEina izabrati boju za cvjetove. Prema principu produkta moguCih bojanja ima 6 . 2 = 1 2 . ~ j & e n j e 2. Izaberimo jednu smedu, jednu zelenu i jednu plavu bojicu umjesto da veC nactranom stablu dodajemo cvjetove. Broj naEina izbora je prema principu produkta za tri skupa jednak 2 . 3 . 2 = 1 2 .

+P Prvo rjegenje opisuje uredene parove (stablo, cvjetovi) ili, uvrgtavanjem rjesenja prethodnog zadatka, ((deblo, krositja), cvjetovi), a drugo uredene trojke (deblo, krosizja, cvjetovi). Medu tim uredenim parovima i uredenim trojkama postoji oEigledna bijekcija, pa i to opravdava Einjenicu d a smo u oba rjeBenja dobili isti rezultat.

9 Nacrtajte tablicu koja u stupcima ima svih 6 naEina crtanja stabla, a u recima mogu6e boje cvjetova i popunite je odgovarajuCim crteiima.
4. 'Na koliko naEina rnoBe dijete iz prethodnih zadataka izabrati bojice za Idje stabla s deblom i krosnjom razliEite boje ili nijanse?
R j e k n j e 1. VeC znamo d a svako bojanje stabla odgovara uredenom paru bojica. Ovdje je za razliku od prethodnih zadataka potrebno samo d a se bojice u paru razlikuju.
Svih uredenih parova bojica ima 8 8 . Neka uredenih parova s razliEitim komponentama ima p . Uredenih parova s jednakim komponentama ima onoliko koliko i bojica 8 komada. Princip sume sada daje tj. p = 56.

RjeZenje 2. Uredene parove bojica koji predstavljaju bojanje stabla moiemo brojati i ovako: prvu komponentu biramo na 8 naEina. Bez obzira na to koju prvu komponentu

4. PREBROJAVANJE

smo izabrali, drugu komponentu biramo na 7 naEina - izabiremo bilo koju od preostalih bojica (prvu, drugu . . .ili sedmu od preostalih, npr. onim redoslijedom kojim su sloiene u kutiji). Rezultat, prema teoremu o uzastopnom prebrojavanju, glasi: 8 . 7 = 56.

tP Kod jednostavnih zadataka EeiCe Cemo koristiti drugi naEin prebrojavanja. Samo tam0 gdje je zadatak prekompliciran i gdje bismo se lako mogli zabuniti, koristit Cemo prvi - sigurniji ali sporiji naEin.
Dijete je odluEilo nacrtati i posljednji crtei stabla: deblo, krolnju, cvjetove i p(icu u krosnji, sve razliEitih boja ili nijansi. Na koliko naEina C e moCi izabrati boje za taj crtei?
Rjezenje. Za deblo bira bojicu kao jednu izmedu 8, za krognju kao jednu od preostalih 7, za cvjetove kao jednu izmedu preostalih 6, a za pticu kao jednu izmedu preostalih 5 bojica. Rezultat glasi: 8 . 7 . 6 5 .
a

tP BuduCi da su cvjetovi; ptica u krognji, za njih se moie upotrijebiti i boja debla. To znaEi da, kad nacrtamo krognju, trebamo vratiti u kutiju olovku kojom smo nacrtali deblo. Rezultat: 8 . 7 . 7 . 6
U jednoj Skoli postoje tri prva razreda. U Ia je 34 ubnika, u Ib 37, a Ic je najmanji od t a tri razreda s 32 uEenika.

&) Na koliko naEina moBemo izabrati dva predstavnika prvog razreda te Skole?
,(b) Na koliko naEina mogemo izabrati dva predstavnika ako oni moraju biti iz razliEitih razreda?

(a) UEenika prvih razreda te gkole ima ukupno 34 37 32 = 103 i iz tog skupa biramo dva razliEita predstavnika. Prvog moiemo izabrati na 103 naEina, recimo d a je izabran Marko. Drugog mnoiemo potom izabrati na 102 naEina, recimo da je izabran Jbalko. Prema tome dva uEenika moiemo izabrati na 103.102 naEina ako pazimo na poredak iaabranih. Kako nam je svejedno da li smo izabrali Marka pa Janka ili pak obratno, to se dva uEenika mogu izabrati na 103.102/2 = 5253 naEina. (b) Predstavnici mogu biti iz Ia i Ib, iz Ia i Ic ili iz Ib i Ic. Ta tri skupa razliEitih izbora su disjunktna. Prvi je veliEine 34 . 3 7 , drugi 34 . 3 2 , a treCi 37 $ 3 2 . Ukupni broj naEina izbora predstavnika dvaju prvih razreda je jednak 34.37 34.32 37.32 = 3530.

+ +

tP
(34+","32)

(Moiete preskoEiti pri prvom Eitanju). Isti rezultat smo mogli zapisati i u obliku

- (y)- ("2') - ("2").

4-t Sve naEine izbora dvaju uEenika prvog razreda moiemo prikazati tablicom u kojoj i reci i stupci odgovaraju uEenicima tih razreda. Izbori brojani u prvom dijelu zadatka su sadriani u dijelu tablice iznad glavne dijagonale. OznaEite na toj tablici izbore iz drugog dijela zadatka. Svaki redoslijed uEenika u stupcima i recima tablice ne daje lijepo rjelienje!

4.

PREBROJAVANJE

.,?
7

U jednoj zemlji, koja obiluje dobrim matematiEkim knjigama, imaju lest razlicitih knjiga s opisanim Dirichletovim principom, tri s rekurzivnim relacijama, Eetiri s matematiEkom indukcijom. Nadalje, u pet su razlicitih knjiga obradene teme matematiEka indukcija i rekurzivne relacije, a u sedam matematitka indukcija i Dirichletov princip. Na koliko se naEina moZemo opskrbiti literaturom iz toEno dvaju od ovih podruEja?
Rjes'enje. MoZemo kupiti ili jednu ili dvij? knjige. OdluEimo li se za kupnju jedne knjige, izbor moZemo obaviti na 5 7 naEina. Zelimo li pak kupiti dvije knjige, izbor, u ovisnosti o izabranim podruEjima, moiemo obaviti na 6 - 3 6 . 4 3 . 4 naEina.

Ukupno imamo (5

+ 7) + ( 6 . 3 + 6 . 4 + 3 . 4 ) = 66 mogutnosti izbora.

9 Vet se u ovom (relativno jednostavnom) zadatku vidi da konazno rjegenje (u nagem sluEaju 66) ne govori previge. Stoga Eesto neCemo pisati konaEno rjegenje, nego Cemo se zadovoljiti srectenim oblikom rjegenja - najEegCe onim koji omogudava jednostavnu generalizaciju na neki drugi broj objekata. Ako ste rje5avajuCi zadatak doEli do rjegenja u nekom drugom obliku, pokugajte svesti svoje i predloZeno rjegenje na isti oblik. Ako i ne uspijete, ne znaEi d a je vage rjegenje netoEno - Eesto je transformacija izraza koju treba provesti vrlo tegka. Provjerite u tom sluEaju jog jednom put kojim ste dogli do rjegenja.
i.

4iliUjabuku kolari se nalazi 12 jabuka i 10 krulaka. Dunja C e uzeti jedan komad voCa ili krugku. U kojem od tih sluEajeva ostaje veCi izbor za Vilnju,
-

koja namjerava uzeti po jedan komad svake vrste?


Rjes'enje. Ako Dunja uzme jabuku, ViLnja moZe jabuku izabrati na 11, a krugku na 10 naEina. Ukupan broj ViHnjinih izbora je tada 11.10 = 110. Ako Dunja uzima krugku, broj ViHnjinih izbora je 12 . 9 = 108.

Vidimo d a Vignja moZe vige birati u sluEaju da Dunja uzima jabuku.

9 Ako imamo zadan a b , tada je a . b najveCi ako je a = b . To slijedi direktno iz nejednakosti izmectu aritmetiEke i geometrijske sredine ili iz izraza (a+b12 = (a-b)2+4ab. Kako se geometrijski moie interpretirati t a Einjenica?

4-t Pokugajte rijegiti ovaj zadatak ako imamo j jabuka i k krugaka, ako Dunja uzima d komada voCa za npr. j 2 k (sluEaj j k je simetriEan)

<

Na koliko se naEina moZe prognozirati ishod nogometnog prvenstva? Igra 8 momtadi, a zanima nas:

@"'redoslijed svih momEadi, /(b) redoslijed prvih tetiriju momtadi. Pretpostavljamo da ne postoji l'dijeljenje" jednog mjesta na vise momcadi, nego je redoslijed jedinstveno odreden rezultatima odigranih utakmica.
(a) Ishod nogometnog prvenstva je uredena l&torka momEadi u kojoj se svaka momEad pojavljuje toEno jednom. Prvoplasiranu momEad biramo na 18 naEina, drugoplasiranu

4.

PREBROJAVANJE

medu preostalih 17, trekeplasiranu medu preostalih 16 itd. Izborom prvih 17 momEadi jedinstveno odredena posljednja momEad. RazliEitih ishoda dakle ima 18 . 1 7 . . . . . 2 . 1.

tP OpCenito je broj svih razmjegtaja n elemenata jednak n . ( n - 1) . . . . . 1, Bto se oznaEava s n! i Eita n faktorzjel.

tP Ako na poEetku prvenstva odluEujemo tko Ce biti pobjednik, a ostale pustimo d a igraju, dobivamo rezultat u obliku n . (n - I)! i tome identitet n! = n . (n - l)! . Njega se moie i vrlo jednostavno raEunski izvesti.
(b) Ishod nogometnog prvenstva u ovom je sluEaju uredena Eetvorka momcadi u kojoj se svaka momEad pojavljuje najvige jednom. Takvih Eetvorki ima 18 . 17 . 16 . 15. tP BuduCi da se Eesto broje razne k-torke sastavljene od elemenata n-Elanog skupa u kojima se niti jedan element ne ponavlja, Eesto se pojavljuju rezultati nalik rezultatu ovog zadatka. Stoga se uvodi oznaka n . (n - 1) . (n - 2) . . . . (n - k 1) =: (n)k i t a j se broj naziva padaju6m faktorzjelom. Uz t u oznaku rezultat ovog dijela zadatka je (6)4, tP Pretpostavimo da nas zanima realni ishod - redoslijed prvih Eetiriju momEadi, ali nije bitno tko je treti, a tko Eetvrti (zbog ulaska u medunarodne kupove).

Tada bi ona dva ishoda u smislu prethodnog dijela zadatka koja se razlikuju samo u redoslijedu treCe- i Eetvrtoplasirane momEadi Einila jedan ishod u smislu ovog zadatka. I obratno, svaki ishod u smislu ovog zadatka generira dva ishoda u smislu prethodnog dijela zadatka. Ako broj ishoda koji trebamo izraEunati oznaEimo s i , onda je 2i = 18.17.16.15. Dobivamo rezultat 1 8 . 1 7 . 1 6 . 1 5 / 2 .

;10

Na nekom natjecanju po kup-sistemu poznati su parovi osmine finala i shema natjecanja, tj. otisnuta je tablica kao na slici. Nakon svakog odigranog kola, na crtu koja izlazi iz pojedinog susreta napiSemo pobjednika tog susreta. Na koliko se naEina mo2e prognozirati popunjena tablica (tj. svi ishodi kupa)?

2
~9

I &

$
MI
M,

Rjegenje. Ishod svakog od susreta osmine finala (prvo kolo) moiemo prognozirati na 2 naEina - pobjeduje ore ili dolje napisani klub. Dakle ishod cijele osmine finala monaEina. SliEno za svaki od susreta Eetvrtfinala mmoiemo refi d a iemo prognozirati na li C e pobijediti momEad koja dolazi iz "gornjeg" ili "donjeg" dvobo'a. Ishod Eetvrtfinala rnoiemo dakle prognozirati na 24 naEina. Za polufiaale imamo 2' mogufih ishoda, za finale 2' = 2 .

Tako smo za svaki od susreta odredili da li pobjeduje gornji ili donji klub. Ishod kupa je odreden ishodom osmine finala, Eetvrtfinala, polufinala i finala, pa cijelu tablicu dakle moZemo popuniti na 28 - 2 4 . 22 . 2 l = 215

% Koliko ima razliEitih ishoda za one kojima je bitno tko je igrao u finalu i tko je
pobijedio?

4. PREBROJAVANJE

Ako imamo Sest razliEitih kuverti i Eetiri razliEite postanske marke iste nominalne vrijednosti, na koliko naEina mo5emo (a) izabrati kuvertu s markom (b) polijepiti sve marke na kuverte (nije bitan iznos maraka na svakoj pojedinoj kuverti) (c) polijepiti sve marke na kuverte, tako da na svaku kuvertu bude nalijepljena najviSe jedna marka?
(a) Biramo uredeni par (luverta, marla). Njih irna 6 . 4 = 2 4 . (b) Svakoj marki pridruiit demo jednu kuvertu (moida C e biti viZe maraka kojima C e biti pridrusena ista kuverta). Bez smanjenja opkenitosti marke moiemo numerirati (poredati). Tada de lijepljenja odgovarati uredenim Eetvorkama kuverata - prvu marku nalijepiti Cemo na bilo koju od Zest kuverata, drugu takoder itd. I Takvih Eetvorki ima 6 . 6 . 6 . 6 .

26 5

q r

9-t

, .

'v'" ,. i

, <,

. ,

"/

Zagto nismo pridruiivali kuvertama d r k e ?

(c) Ova C e lijepljenja odgovarati uredenim Eetvorkama kuverata s razliEitim komponentama. Prebrojimo onda sve takve Eetvorke! Prvu kuvertu biramo na 6 naEina. Nakon toga drugu biramo kao jednu od preostalih 5, pa tredu kao jednu od preostalih 4 i na kraju Eetvrtu kao jednu od preostalih 3. Traienih Eetvorki stoga ima 6 . 5 . 4 . 3 = 360.

9+ Isti rezultat dobili bismo i da smo htjeli polijepiti Zest maraka na Eetiri kuverte, ako ponovno ne smije biti viZe od jedne marke na pojedinoj kuverti. Objasnite zaZto!

9-t Ako s K oznatimo skup kuverata a s M skup maraka, onda vidimo da smo u (b) i (c) dijelu zadatka promatrali neke funkcije izmedu tih skupova. 0 kakvim se funkcijama radilo? Generalizirajte zadatak na k kuverata i m maraka (pri Eemu k moie biti veCi, jednak ili manji od m ).
'" '?

12. Na koliko naEina mo5emo iz skupine od 6 rajEica i 8 paprika staviti nekoliko komada povrCa u kosaru? Kosara smije ostati i prazna, a bitno je samo koliko je komada kojeg povrCa u koSari!
Rjes'enje. MoguCa stavljanja povrCa u koZaru moiemo prikazati pomoCu uredenih parova brojeva: prva komponenta kaie koliko je u koiari rajEica, a druga broji paprike. Tada je prva komponenta iz skupa N6 U (0) , a druga iz N8 U ( 0 ) . Ukupno mogudih naEina uzimanja povrda irna (6 1) . (8 1) = 6 3 .

Na koliko naEina moiemo iz skupine od 6 rajEica, 8 papika i 2 komada luxa staviti nekoliko komada povrCa u koSaru, ako je bitno da u koSari budu barem dvije rajEice, barem jedna paprika i barem jedan luk? Kao i u prethodnom zadatku i ovdje je bitno samo koliko je komada kojeg povrCa u kogari. Koliko ima izbora ako zahtijevamo i da barem dvije rajEice, barem jedna paprika i barem jedan luk trebaju ostati izvan kogare?

42

4. PREBROJAVANJE

Rjes'enje. Bududi d a nam je bitan samo broj komada pojedine vrste povrda, a ne i koji su komadi stavljeni u ko'saru, moZemo odmah u kogaru staviti onoliko povrCa koliko nam treba za salatu (dvije rajEice, jednu paprika i jedan luk). Nakon toga uzmemo jog povrda iz preostale skupine od 4 rajEice, 7 paprika i jednog luka. Rezultat je tada (vidi prethodni zadatak): (4 1) . (7 1) . (1 1) = 80.

Rezultat moiemo interpretirati i ovako: broj rajEica u ko'sari biramo iz skupa {2,3,4,5,6), broj paprika iz skupa {1,2,. . . , 8 ) , a broj glavica luka iz skupa {1,2). Da bismo rije'sili drugi dio zadatka, odvojimo prvo skupinu povrCa koju moramo staviti u koSaru i skupinu povrCa koju moramo ostaviti izvan ko'sare. Nakon toga ostaju nam jog 2 rajEice, 6 paprika i niti jedan luk od kojih moiemo proizvoljnu koliEinu povrCa staviti u kogaru (ili, Bto je ekvivalentno, ostaviti izvan kogare). Rezultat: (2+1).(6+1).(0+1) = 21 .

+P

Dvojica gusara na6li su na jednom pustom otoku u Toplom moru sanduk zlatnicima, srebrnjacima i amuletima od slonove kosti. BuduCi da se nisu mogli nagoditi kako da raspodijele blago, razgledavali su sve moguCe raspodjele. Svaku raspodjelu gledali su deset minuta. Ako znamo da su se na taj naEin neprekidno 8 dana i 3 i pol sata bavili raspodjelom blaga, te ako znamo da je u sanduku bilo najviSe srebrnjaka, a najmanje amuleta, izraEunajte broj zlatnika, srebrnjaka i amuleta od slonove kosti! Svi zlatnici medusobno, svi srebrnjaci medusobno i svi amuleti medusobno su jednaki.
Rjes'enje. Gusari su radili ukupno 11730 minuta, 'sto znaEi da je bilo ukupno 1173 moguCih raspodjela blaga. Ako je bilo z zlatnika, s srebrnjaka i a amuleta od slonove kosti, onda vet znamo da treba biti
1173 = (z

+ l)(s + l)(a + 1).

No, buduCi da je jedini rastav broja 1173 u faktore (do na raspored faktora) 1173 = 2 3 . 1 7 . 3 , to je zbog uvjeta da najvige ima srebrnjaka, a najmanje predmeta od slonove kosti: s = 22, z=16 i a = 2 .

mf

Dvoje ljudi raspolaie sa 6 novEanica od po 50 i sa 4 od po 100 kuna

(a) Na koliko ih naEina mogu medu sobom podijeliti? (b) Na koliko ih naEina mogu podijeliti tako da svaki od njih dobije jednak broj novEanica? (c) Na koliko ih naeina mogu podijeliti tako da svaki od njih dobije isti iznos novaca?
Rjeienje.
(a) Ovakav zadatak smo vet rijegili. Rezultat: (6

+ 1) . (4 + 1) = 35.

e biti (b) Prvi Eovjek treba uzeti 5 novEanica - moZe samo birati koliko od njih C vrijednosti 100 kuna: 0,1,2,3,4. Rezultat: 4 1 = 5 .

4-t Za'sto nije mogao slobodno birati koliko Ce uzeti novEanica vrijednosti 50 kuna?

4. PREBROJAVANJE

43

tP Ovaj dio zadatka bi se jako zakomplicirao d a je bilo viSe vrsta novEanica. Takav zadatak bit C e rijeSen kasnije.
(c) Svaki od njih C e dobiti (6.50 4.100)/2 = 350 kuna. Ako prvi Eovjek uzme p novEanica od po 50 kuna i s novcanica od po 100 kuna, onda mora biti: ili, nakon krakenja: Ukupni broj nenegativnih cijelih rjegenja ova jednadibe je mali, a potragu nam olakgava i Einjenica d a p mora biti neparan (pogledajte joS jednom jednadibu!). MoguCa rjeSenja su (P, s) E {(1,3), (3,2), (5,111. tP Ovaj zadatak ne spada u prebrojavanja kakva smo imali do sada, ali mu je formulacija toliko sliEna prethodnom zadatku, da ga je vrijedilo rijeSiti.

9+ Kako biste rijegili svaki od ovih zadataka ako bi bilo dozvoljeno da dio novca odnesu u banku na zajedniEki raEun?

@ \ 7

16. U ovom zadatku brojimo prirodne brojeve s raznim svojstvima. a) Koliko irna prirodnih brojeva manjih od 10000 s razliCitim znamenkama? (b) Koliko irna Eetveroznamenkastih brojeva s razliEitim znamenkama? ( c ) Koliko irna prirodnih brojeva vekih ili jednakih 100, a manjih od 10000 s razliEitim znamenkama? (d) Koliko irna neparnih brojeva vekih ili jednakih 100, a manjih od 10000 s razliEitim znamenkama? (e) Koliko irna prirodnih brojeva manjih od miliona koji ne sadrfe znamenku 3 u svom zapisu? (f) Koliko irna prirodnih brojeva manjih od tri miliona koji ne sadrfe znamenku 3 u svom zapisu? (g) Koliko irna prirodnih brojeva manjih od miliona koji sadriie znamenku 3 u svom zapisu?

(a) Prirodni brojevi manji od 10000 su prirodni brojevi s najviSe Eetiri znamenke. BuduCi d a imamo na raspolaganju i nulu, moiemo brojati uredene Eetvorke znamenaka.
- dakle na 9 naEina, treCu na 8 Zetvrtu na 7. Rezultat: 10 9 . 8 7 .

Prvu znamenku biramo na 10 naEina, drugu biramo medu preostdim znamenkama

9-t UoEite sa su svi brojevi koje smo brojali veCi od nule.


(b) Znamenke biramo kao u prethodnom dijelu zadatka, ali ne smijemo dozvoliti da na prvo mjesto dode nula. Dakle, prvu znamenku biramo izmedu 9 mogudih, drugu izmedu 9 preostalih, treCu izmedu 8, Eetvrtu izmedu 7 preostalih. Fkzultat: 9 . 9 . 8 . 7 .

tP Mogli smo i od broja svih brojeva s razliEitim znamenkama manjih od 10000 oduzeti broj brojeva s razliEitim znamenkama manjih od 1000. Rezultat: 10.9.8.7-9.8.7 = 9.9.8.7.
(c) Biramo Eetiri znamenke. Prvu znamenku moiemo izabrati na 10, drugu na 9 naEina. Pritom se ne smije dogoditi d a obje "vodeCeV znamenke budu jednake nuli, jer

4. PREBROJAVANJE

bismo tako dobili dvoznamenkast broj (broj manji od 100). No, to se niti ne moie dogoditi jer biramo razliEite znamenke. Nastavimo li birati, vidimo d a treCu znamenku moiemo izabrati na 8, a Eetvrtu na 7 naEina. Rezultat: 10 . 9 . 8 . 7 . Usporedbom sa zadatkom (a) vidimo da su svi prirodni brojevi manji od 10000 +f' koji odgovaraju uredenim Eetvorkama s razliEitim komponentama ujedno i veCi od 100. (d) Ako i u ovom dijelu zadatka nastavimo birati kao i prije, mogli bismo zapasti u problem. Naime zadnja (Eetvrta) znamenka mora biti neparna. Broj izbora nepotrogenih neparnih brojeva ovisi o tome da li smo, odnosno koliko, "troSilin neparne brojeve za prve tri znamenke. Tu nam teorem o uzastopnom prebrojavanju ne daje rezultat, jer on broji situacije u kojima za svaki izbor npr. prve koordinate imamo neki fiksni broj naEina izbora druge koordinate. PokuBajmo sada brojati unazad, tj. tako da se prvo rijeBimo zadnje znamenke one koja je zadavala ograniEenja. Nju moiemo izabrati na 5 naEina. SljedeCu znamenku - npr. znamenku desetica - moiemo izabrati na 9 naEina, znamenku stotica na 8, znamenku tisudica na 7 naEina. Rezultat: 5 . 9 . 8 . 7 .

9+ Za sve one koji ne iele odustati od brojanja slijeva nadesno, evo upute kako to napraviti: klasificirajte sve brojeve prema broju neparnih znamenaka medu prve tri znamenke. Taj broj moie biti 0,1,2 ili 3. Rezultat glasi: 5 . 4 . 3 . 5 3 . (5 . 5 . 4 . 4) 3 . (5.5.4.3)+5.4.3.2=2520.

(e) Znamenke mogu biti iz skupa {0,1,2,4,5,6,7,8,9) . Biramo Zest znamenki. Svi izbori Lest znamenki iz tog skupa osim Sestorke 000000 su dobri. Traienih brojeva dakle ima g6 - 1 komada.

(f) Svaki od izabranih brojeva iz prethodnog dijela zadatka je dobar. No, dobri su i svi oni brojevi koji kao prvu znamenku imaju 1 ili 2 a iza toga proizvoljnu Hestorku znamenaka razliEitih od 3. Rezultat: (g6 - 1) g6 g6 = 3 . g6 - 1.

+ +

(g) Kad vet imamo izraEunat broj svih brojeva manjih od miliona koji ne sadrie tri u svom zapisu, ostaje nam jog samo oduzeti t a j broj od broja svih prirodnih brojeva manjih od miliona. Rezultat: (lo6 - 1) - (g6 - 1) = lo6 - g6.

+f NapiSemo li rezultat kao:


dobivamo sve brojeve manje od miliona grupirane prema poziciji na kojoj se zadnji put pojavljuje trojka. Tako 10i95-i broji sve brojeve koji na poziciji i 1 imaju trojku, ispred te pozicije bilo Bto, a iza te pozicije znamenke razliEite od tri.

U kontekstu poglavlja u kojem de biti rijeEi o kombinatornom dokazivanju identiteta bi se Einjenica d a se npr. k-torke brojeva koje sadrfe znamenku 3 mogu grupirati prema zadnjem pojavljivanju trojke zajedno s direktnim prebrojavanjem upotrijebila za dokazivanje jechakosti

Promatrajmo iifre koje se sastoje od niza slova iza kojeg slijedi niz znamenaka. Koliko ima Sifara koje se sastoje od tri slova i Eetiri znamenke? Koliko ima iifara koje se sastoje od najviSe tri slova i Eetiri znamenke?

4. PREBROJAVANJE

45

Rjeienje. Prvo slovo moZemo birati na 30 naEina, isto tako i drugo i trete. Prvu znamenku moiemo birati na 10 naEina, a isto tako i ostale. Rezultat: 3 0 ~ 3 0 ~ 3 0 ~ 1 0 ~ 1 0 ~ = 10~1 270000000.

Ako Zelimo iifre s najviie tri slova iEetiri znamenke, to moZe biti iifra ili s niti jednim, ili s toEno jednim, ili s toEno dva ili s toEno tri slova. Rezultat:

10.10 10 10 3 0 . 1 0 . 1 0 10.10 30~30~10~1. 010 ~10 3 0 . 3 0 . 3 0 . 1 0 . 1 0 * 10 . l o =

+ + +
279310000.

Ck gto mislite, zagto ne moZemo jednostavno proHiriti abecedu praznim znakom i rezultat napisati kao 31 . 3 1 . 3 1 . 1 0 . 1 0 . 1 0 . l o ?
UoEite razliku izmedu praznog znaka i nule iz prethodnog zadatka - prazni znak na poEetku rijeEi skraCuje rijeE jednako kao i nula na poEetku broja, ali razlika ipak postoji!

4-t (preskoEiti pri prvom Eitanju) Da li se rezultat zapisan kao

moZe direktno interpretirati?

U nekoj zemlji htjeli su poveCati broj moguCih registarskih tablica, ne eCavajuCi broj znakova na tablici. Umjesto najviSe Sesteroznamenkastih brop jeva, na tablicama C e biti tri slova i najviSe troznamenkasti broj. Koliko puta C e biti viSe tablica? Niti u jednom sluCaju nije dozvoljena nula kao registarski broj.
Rjes'enje. Prije je bilo lo6 - 1 razliEitih tablica. Sada C e ih biti 303 . (lo3 - 1 ) . Omjer broja novih i starih tablica iznosi 303/(103 - 1) m 27. Dakle broj tablica poveCat C e se otprilike 27 puta.

e na tablicama biti najmanje jedno, a najviHe tri slo4-, U stvari, odluEeno je da C va, te jedno- do troznamenkasti broj. Kako glasi rje'senje u tom sluEaju? Koliko C e posto tablica imati oznake koje se lako Eitaju, tj. ili jedno slovo ili samoglasnike i suglasnike koji alterniraju?

Ako felimo svako od 30 slova abecede i svaku od 10 znamenaka prikazati 9 ka niz sastavljen od nula i jedinica duljine k , koliki najmanje mora biti k ? Koliki mora biti k ako felimo prikazivati i skup od izabrana 32 specijalna znaka (interpunkcije, zagrade, itd.)?
r,

Rjes'enje. Broj nizova duljine k sastavljenih od dvaju znakova je 2 k . Ako ielimo kodirati 30 10 znakova pomoCu takvih nizova, mora biti

2k 2 40.
Slijedi k 2 6 . Ako ielimo kodirati 30

+ 10 + 32 znaka sliEno dobivamo k 2 7 .

4.

PREBROJAVANJE

+ I ' U prvom sluEaju ostaju nam joi 24 neiskoriitena niza, u u drugom 56. Dakle, u drugom sluEaju moZemo nekih 30 od preostalih nizova namijeniti za kodiranje malih slova.

tP Postoji i druga strategija u odnosu na problem Sto uEiniti s viikom nizova dakle s nizovima koji ne sluZe za kodiranje niti jednog znaka. Oni mogu sluiiti tome d a bi se po prijemu kodirane poruke u sluEaju da se primi upravo njih, znalo d a je doilo do pogreike u prijenosu podataka. Da bi to bilo efikasno, treba na posebni naEin izabrati nizove koji C e odgovarati znakovima. Time se bavi grana diskretne matematike koja se zove teorija kodiranja.
Morseovi znakovi su naEin kodiranja znakova pomoCu tockica i crtica u kojem je dozvoljena promjenjiva duljina znakova. Kolika najmanje mora biti duljina najduljeg niza u skupu Morseovih znakova, da bi se moglo kodirati 30 slova i 10 brojeva?
Rjes'enje. Ukupni broj poruka ponovno mora biti veCi ili jednak 30 maksimalna duljina niza jednaka k , onda mora biti: 2 + 2 2 + . . . + 2 k 240. Zbrajanjem izraza na lijevoj strani slijedi 2 " ' te na kraju k
-2

+ 10.

Ako je

2 40,

2 5.

9 , I oEekivaii smo rezultat manji ili jednak rezultatu prethodnog zadatka. gtoviie, netko bi mogao reCi d a smo mogli oEekivati i da je rezultat toEno za jedan manji. Odgovorite na pitanje: za koje koliEine znakova koje treba kodirati rezultat ovog zadatka nije za jedan manji od rezultata prethodnog.

Profesor je, Eim je stigao kuCi, ustanovio da je zaboravio kigobran ili u nci ili u poSti ili u apoteci ili u duCanu. Odmah je izasao iz kuCe i krenuo traiiiti kigobran ponovno posjeCujuCi mjesta na kojima se zadriiavao tijekom tog dana. Ako pretpostavimo da C e se vratiti kuCi Eim nade kigobran, koliko ima moguCih obilazaka tih mjesta?

Rjes'enje 1 . Ako je kiHobran zaboravio u banci, prije banke je mogao obiti jog niti jedno, jedno, dva ili tri mjesta. Broj obilazaka nula mjesta iznosi 1, broj obilazaka jednog mjesta 3, dvaju mjesta 3 . 2 , triju 3 . 2 .1. Ukupni broj obilazaka koji zavriavaju u banci je jednak 1 3 3 2 3 . 2 1. Jednak broj obilazaka bi bio i u sluEaju d a j e kiiobran zaboravio u poiti, apoteci ili u dutanu. Rezultat zadatka:

+ +

4.(1+3+3.2+3.2.1)=4.16=64. Rjes'enje 2. MoguCe ture moiemo klasificirati i prema broju mjesta koja je profesor posjetio prije nego i t o je naiao kiSobran. Broj tura koje se sastoje od jednog mjesta je jednak 4, tura koje se sastoje od dva mjesta ima 4 . 3 , od tri mjesta 4 . 3 2 , od Eetiri 4 . 3 3 2 . 1 . Ukupnibroj turatadajejednak 4 + 4 . 3 + 4 . 3 . 2 + 4 . 3 . 2 . 1 = 6 4 .
+

4.

PREBROJAVANJE

47

22. Student se vraCa kuCi s predavanja. Ako je dobre volje, vrati se direktno. InaEe ode u jedan od svojih omiljenih kafiCa X , Y ili Z i tam0 popije CocaColu ili Fantu. Ako mu to podigne moral, ode kuCi, a inaEe svrati u jog jedan od svojih omiljenih kafiCa i popije Coca-Colu ili Fantu, te nakon toga ode kuCi. Koliko ima mogudnosti za put tog studenta kuCi ako je bitno i Sto je sve popio? Koliko je tih mogudnosti ako je bitno da je student bio u kafiCu X ? U koliko sluEajeva student posjeCuje kafiC X , te pije jednu Coca-Colu i jednu Fantu?
Rjes'enje. Student dolazi kuCi ili direktno ili nakon jedne stanke u kafiCu ili nakon dvije stanke u razliEitim kafiCima. U svakom kafiCu u kojem je stao, popio je jedno od dva moguCa piCa. Rezultat: 1 + 3 . 2 + ( 3 . 2 ) - ( 2 . 2 ) =31.
Ako brojimo puteve s predavanja do kuCe u kojima student posjeCuje kafiC X , dobivamo rezultat (prvi pribrojnik odgovara posjeCivanju jednog, a drugi posjeCivanju dvaju kafida) : 1. 2 ((1 - 2 ) . (2 . 2) ( 2 . 2) . (1 .2)) = 18.

Ako je popio dva pita, student je bio u dva kafita. Mogao je prvo biti u kafiCu X , a zatim u nekom od preostala dva ili obratno. Mogao je popiti Coca-Colu u prvom, a Fantu u drugom kafiCu ili obratno. Rezultat: 2.2+2.2=8.

'23. Na putu izmedu mjesta A i mjesta B ima n semafora koji nisu medusobno sinkronizirani i ne mogu se zaobiCi. Svaki od semafora ima samo crveno i zeleno svjetlo. Na koliko naEina vas mogu ti semafori zaustaviti na putu? Da li ima viSe moguCih prolaza kroz raskrSCa pri kojima na viSe od polovine raskrSCa moramo stati ili prolaza pri kojima kroz vise od polovine raskrSCa prolazimo bez stajanja?
Rjes'enje. Na svakom od n semafora moHe biti ili crveno ili zeleno svjetlo, pa je ukupni broj stanja promatranog skupa semafora 2 n .
Za svako stanje skupa semafora u kojem na viLe od polovine semafora gori crveno svjetlo postoji jedno stanje skupa semafora u kojem na viHe od polovine semafora gori zeleno svjetlo i obratno (na svakom od semafolia promijenimo svjetlo). Opisano preslikavanje je bijekcija izmedu skupa svih stanja semafora u kojima nas viHe od polovine semafora zustavlja i skupa svih stanja semafora u kojima prolazimo kroz vise od polovine raskrHta. Dakle, broj razliEitih stanja tih semafora pri kojima se zaustavljamo na viHe od polovine raskrHCa je jednak broju svih stanja semafora pri kojima se zaustavljamo na manje od polovine raskrHCa.

+f Zaustavljao nas je svaki pojedini semafor, pa je interpretacija tog dogadaja kao niza elementarnih dogadaja crveno, zeleno prirodna. IspriEavajuCi se zbog zakaSnjenja opisat Cemo podskup skupa semafora koji nas je zaustavio. ZakljuEujemo da podskupova n-Elanog skupa (npr. skupa semafora) ima 2 n .

4. PREBROJAVANJE

'a

Putern u cjelobrojnoj koordinatnoj rnre.5 nazivamo svaki povezani skup toEaka u ravnini koji je sadriian u uniji pravaca x = k , y = k , gdje je k cijeli broj. Svaki put je jedinstveno odreden svojim koracima duljine 1. Obratno, svaki skup koraka uz neka ograniEenja zadaje neki put.

IzraCunajte broj puteva koji poEinju u ishodis'tu, a sadrBe ukupno n koraka udesno ili prema gore.
Rjegenje. Za svaki od koraka odluEujemo u kojem C e od dva smjera voditi. Zato imamo ukupno 2n mogutnosti izbora.

tP Ponovno postoji bijekcija izmedu objekata brojanih u ovom zadatku i podskupova nekog n-Zlanog skupa. MoZemo na primjer r e 6 da izbor jednog podskupa skupa svih koraka znaEi da upravo u tom koracima idemo udesno.
4+ Gdje Cemo se nalaziti nakon n koraka? Nacrtajte nekoliko primjera i pokuiajte zakljuEiti sami. Ako smo napravili x koraka udesno, napravili smo n - x koraka prema gore i stigli u toEku (x, n - x) . Broj horizontalnih koraka mogli smo birati proizvoljno, pa to znaEi da se on nalazio izmedu 0 i n . Zavrine toEke su zato cjelobrojne toEke pravca x y = n koje se nalaze u prvom kvadrantu.

9+

Moiete li izreCi i rijesiti trodimenzionalni analogon ovog zadatka?

25. Neka su k, n E N o . Promatramo najkrate puteve izmedu zadanih poEetaka i krajeva. (a) IzraEunajte broj puteva od neke toeke na pravcu x y = -k do neke toeke na pravcu x + y = n koji prolaze kroz toEku (0,O). (b) Ako je k < n izraEunajte broj puteva od pravca x y = lc do pravca x + y = n , koji stalno borave u prvom kvadrantu.

(a) Svaki od tih puteva moiemo razloiiti na dva puta: od pravca x y = - k do ishodiHta i od ishodista do pravca x y = n . Ta dva dijela puta su medusobno nezavisna, pa rezultat glasi

2k . 2n.

(b) Moiemo krenuti od bilo koje toEke na pravcu x y = k . Od koje god toEke krenuli, do pravca x y = n ima n - k koraka. Ti koraci mogu se po volji birati iz

4. PREBROJAVANJE

49

zadanog skupa koraka {gore, desno) . Nacrtajte skicu! Na pravcu x toEka u prvom kvadrantu, pa rezultat glasi: (k

+y = k

irna k

+1

+ 1) . 2n-k.

9+ Usporedite rjeSenja ovih zadataka za k = 0 . Provjera ovakvog rubnog uvjeta je jedan od uobicajenih naEina kontrole rjegenja.
26

Rjes'enje 1. Biramo koliko ielimo parnih brojeva iz Ng, i barem jedan neparan broj. Zato svaki podskup skupa Ng, koji sadrii barem jedan neparni broj moiemo prikazati kao uniju P U N ,gdje je skup P sastavljen od parnih, a N je neprazni skup sastavljen samo od neparnih brojeva. Izbore moiemo provesti tako da P i N biramo nezavisno, a onda ih kombiniramo u traiene skupove. Parnih brojeva manjih ili jednakih 2n irna n , pa svih podskupova skupa parnih brojeva manjih ili jednakih 2n irna 2,. Neparnih brojeva u promatranom skupu irna takoder n , a buduCi d a traEimo bilo koji podskup skupa neparnih brojeva manjih od 2n osim praznog, moguCih izbora neparnih brojeva irna 2n - 1. Ukupni broj izbora je 2,. (2, - 1 ) . Rjes'enje 2. Od broja svih podskupova skupa N2, oduzmemo broj onih koji sadrie samo parne brojeve. Rezultat: 22n - 2,.
n L

' I

Koliko irna podskupova skupa

Ng,

koji sadrBe barem jedan neparan broj?

9+
27.

Prebrojite joS skupove koji sadrZe barem dva neparna broja.

. (//

,, -

IzloZbeni prostor svakog poduzeCa na sajmu veliEine je 10 x 10 metara. Svakome je dano 10 paravana tlocrta oblika slova L duljina krakova 1,metar. Na koliko naEina se na jednom izloBbenom prostoru mogu sloZiti paravani na izlozbeni prostor, ako se zahtijeva da se, prolazeii bilo kojim rubom izloZbenog prostora ne vidi njemu paralelni rub?
Rjesenje. Na prvi pogled izgleda kao d a ovaj zadatak ima beskonaEno mnogo rje'senja. No, uvjet na broj i oblik paravana, te Einjenica da se "ne smije vidjeti druga strana" bitno suiavaju broj moguCih postavljanja paravana. Promatramo li izloibeni prostor u smjeru sjever-jug, vidimo d a na svakom od 10 metara duljine istok-zapad mora biti po jedan paravan. Isto vrijedi i za smjer istok-zapad. Dakle, nacrtamo li na podu zadanog kvadrata koordinatnu mreEu s razmakom linija po 1 metar, na dobivenoj L'Eiahovskoj ploEi" Cemo u svaki stupac i svaki redak morati postaviti po jedan paravan. On de trebati stajati tako d a pokriva rubove svog polja, na bilo koji od Eetiri moguCa natina. Zadatak smo sveli na dva problema: prvo treba izabrati polja na kojima C e stajati paravani, a onda za svaki od paravana izabrati jedan od Eetiri poloEaja. Polja na kojima de stajati paravani izaberimo tako d a prvo postavimo po jedan paravan ispred svakog stupca (jer u svakom stupcu mora biti po jedan paravan). U svakom retku mora biti takocter po jedan paravan, pa to znaEi da C e za paravan u prvom stupcu broj izbora redaka biti 10, za drugi stupac 9, za treCi 8, itd. Paravan u zadnjem stupcu smjestit Cemo u posljednji slobodni redak. Broj izbora svih 10 polja je lo!. Broj postavljanja paravana je 10!41 .

4 . PREBROJAVANJE

@ Ako "Gahovsku ploEu" promatramo kao 10 x 10 matricu Eiji su elementi jednaki nuli ako je odgovarajute mjesto prazno, a jednaki jedinici ako na tom mjestu stoji paravan, onda svaki raspored iz zadatka odgovara jednoj permutacijskoj matrici, n x n permutacijska matrica P = [ p i j ] se iz permutacije u skupa Nn dobiva na sljedeCi naEin: Pij =

1 ako je u(i) = j, 0 inaFe.

9 Gdje nastaju problemi za 12 paravana na istom prostoru, a gdje na 10 x 12 metara velikom pravokutniku?

tP Postavljanje paravana moie se tehniEki rijeziti na sljedeCi naEin: na svakom od presjeka linija nactrane koordinatne mreie nalazi se rupa u koji pristaje Siljak koji se nalazi u vrhu paravana. Prebrojite rnoguCe poloiaje izabiruCi rupe za uCvr&5ivanjeparavana i njihove poloiaje.
/--%

28. UEenik treba napisati domaCe zadaCe iz z predmeta i nazvati p prijatelja. Ako se uEenik izmedu svake dvije napisane zadaCe treba "odmoriti" (tj. porazgovarati s prijateljem), te ako niti u jednom trenutku dok nisu jog napisane sve zadaCe broj nazvanih prijatelja ne smije biti veCi od broja napisanih zadaCa, na koliko naEina taj ucenik moie organizirati svoje poslove?
Rjes'enje. Shema u koju uEenik mora poslagati pisanja zadaCa i razgovore s prijateljima mora izgledati ovako: Z P Z P . . . Z P Z P P . . . P,

gdje slovom Z oznaEavamo pisanje jedne zadate, a s P telefonski razgovor s jednim prijateljem. Mora biti p 2 z da bi se uEenik izmedu svake dvije zadade mogao odmoriti. Dakle, shema u koju stavljamo zadaCe i razgovore je samo jedna. Svako njeno popunjavanje je odabir jednog redoslijeda pisanja domaCih zadaCa i jednog redoslijeda telefoniranja. ZadaCe moiemo na z pozicija razmjestiti na z ! naEina. Isto tako p razgovora moiemo sloiiti na p mjesta na p! naEina. BuduCi da su izbori redoslijeda pisanje zadaCa i redoslijeda telefoniranja nezavisni (tj. svako rjesenje zadatka je uredeni par koji se sastoji od jednog redoslijeda pisanja zadaia i jednog redoslijeda telefoniranja), rezultat za p 2 z glasi z!p! . Za p < z veC smo vidjeli da je rezultat jednak 0.

@ Pretpostavimo da je npr. z = p i da zahtijevamo samo d a izmedu svaka dva telefonska razgovora uEenik mora napisati jednu domaCu zadatu, te izmedu svake dvije domaCe zadaCe mora nazvati jednog prijatelja (osim za "rubne" poslove - prvi i zadnji). U tom sluEaju bile bi moguCe dvije sheme:
Z P Z P . . .Z P P Z P Z . . .P Z , pa bi rezultat bio 2z!p! = 2 ~ ! ~ .

+f Zadatak je bitno teii ako izbacimo uvjet da se uEenik mora odmarati!


29. n kandidata za neki posao predstavlja se pred troElanim povjerenstvom. Svaki od Elanova povjerenstva rangira kandidate prema svom migljenju. Pravilo

4. PREBROJAVANJE

je da C e neki kandidat biti prihvaden ako su ga barem dvojica Elanova povjerenstva stavili na prvo mjesto. IzraEunajte u koliko C e se posto sluEajeva (od svih moguCih lista) izabrati neki kandidat.
Rjes'enje. Ukupno ima (n!)3 moguCih skupova lista - od svakog Elana komisije po n! . IzraEunajmo jog iz koliko od tih lista smijemo zakljuEiti d a imamo pravog kandidata za posao. Na n naEina moiemo odluEiti koji C e kandidat biti izabran. Zatim, on moie biti izabran zato jer su 1. i 2., 2. i 3., 1. i 3, ili sva tri Elana komisije glasali za njega. Oni Elanovi komisije koji su glasali za kandidata koji C e biti izabran mogu ostatak liste popuniti na (n - I)! naEina, a oni koji ga nisu stavili na prvo mjesto, na prvo mjesto su morali staviti nekog drugog, pa je broj njihovih izbora (n - 1) . ( n - I)! Dakle, za svaki od prva tri sluEaja imamo (n (n - 1) . (n - I)! moguCih lista. U zadnjem sluEaju broj moguCih lista iznosi (n - I ) ! ~ . Ukupni broj naEina izbora jednog, unaprijed odredenog kandidata tako iznosi:

Broj naEina izbora nekog kandidata je n puta veCi, a traieni postotak je:

Kad je ovaj postotak jednak loo%, kad je veCi od 50%?

+f' Zadatak se mogao rijeLiti i tako d a se prvo izraEunao broj lista u kojima je kandidat na prvom mjestu kod barem dvojice Elanova komisije, a kod treCeg na bilo kojem mjestu. U tom sluEaju se od trostrukog tog broja trebalo oduzeti dvostruki broj lista u kojima je kandidat na prvom mjestu kod sva tri Elana komisije. Objasnite to! 0 metodi rjeiavanja nalik na predloienu bit C e rijeEi u poglavlju o formuli ukljuEivanja-iskljucivanja.

9 Interpretirajte broj sluEajeva u kojima de biti izabran jedan odredeni kandidat, 3n - 2 geometrijski: prikaiite sve moguCe izbore prvog kandidata na trima listama u trodimenzionalnom prostoru. Izbori C e onda biti predstavljeni pomoCu n3 toEaka poslaganih u obliku kocke. $to onda predstavlja broj 3n - 2 ?
9 Ako je odluEeno da Ce biti primljeni svi kandidati koji se nalaze medu prvih k kandidata na listi kod barem dvojice Elanova komisije, koliki treba biti k da bi sigurnw e netko biti primljen u barem 50% sluEajeva? bio primljen neki kandidat? Za koje Ic C

4. PREBROJAVANJE

Brojanje klasa ekvivalencije


Nekog pastira su upitali kako broji svoje ovce. Nato je on odgovorio: "Vrlo jednostavno. Legnem na travu, prebrojim noge i rezultat podijelim sa Eetiri."

Petorica klinaca tajne grupe trebaju uskladiti milljenja o vainom problemu g k a k o obrati susjedovu tresnju. Da bi bolje upoznali milljenja drugih, a da ne budu previSe uoEljivi, sastaju se u parovima. Ako svaki par na kraju svog susreta napiSe saietak razgovora, koliko C e na kraju biti papiriCa koje C e trebati pregledati?
Rjeienje 1. Broj napisanih saietaka bit C e jednak broju parova klinaca. Do sada smo rjeliavali samo zadatke koji bi nam mogli pomoCi pri brojanju uredenih parova. Prebrojimo uredene parove i pogledajmo moie li nam rezultat pomoCi pri rjeliavanju postavljenog zadatka.
Uredenih parova klinaca s razliEitim koordinatama ima 5 . 4 . Oni broje razgovore, ali i na primjer par (Marko, Luka) kaZe i da je Marko prvi stigao na mjesto razgovora, a ne Luka kao u paru (Luka, Marko) . BuduCi da za svaki par postoje dva izbora prvog pristiglog klinca, broj razgovora je upola manji od broja svih uredenih parova s razliEitim koordinatama i iznosi 5 . 412.
%
t ' r

C?

I
a
%

I ~ u k a , ~ a r k o~ v a n , ~ a r k o~ u r a , ~ a r k o~ a t i j a ; ~ a r i o
. !
Luka,Ivan Ivan,Luka Luka, Jura Jura,Luka Ivan. Jura Luka,Matija Matija,Luka 1van.Matiia

Marko.Luka

I Marko.1van I Marko.Jura I Marko.Matiia I

+P Skup svih uredenih parova podijelili smo u klase prema sastavu uredenih parova: dva para su u istoj klasi (su u relaciji) ako se sastoje od istih elemenata (vidi sliku). Svaka klasa odgovara jednom dvoElanom skupu. VeliEina skupa uredenih parova je 5 . 4 , svaka klasa sadrii po dva elementa, pa je broj klasa (broj skupova) jednak 5 . 4 / 2 . +P DrukEije napisano, napravili smo sljedede: prvo smo prebrojali uredene parove kao lit0 smo to radili i do sada. Nakon toga, svaki uredeni par klinaca interpretirali smo kao uredeni par (razgovor, stigao prvi) i prebrojali nove objekte. Zbog oEigledne bijekcije izmedu novih i starih uredenih parova, prema principu jednakosti ima ih jednako. Razgovora neka ima r , za svaki razgovor biramo na dva naEina prvog pristig!og klinca, pa novih uredenih parova sada ima r . 2 . Rezultati brojanja uredenih parova iz rjeLenja i ovih uredenih parova moraju biti jednaki: 5 4 = 2 r . Iz toga slijedi r = 5 . 4 / 2 .

4.

PREBROJAVANJE

Rjes'enje 2. Sada kad smo dobro prouEili rjeienje koje C e se koristiti u kompliciranijim zadacima, moZemo predoEiti joL jedno rjegenje. Kako bi sve teklo po planu, klinci su nacrtali tablicu u koju su biljeiili odriane susrete.
Ivan Jura Luka Marko Matija

[ Ivan
Luka Marko

Polja u kojima se ne nalaze minusi predstavljaju susrete i u njih se za svaki odrZanih susret treba upisati po jedan plus. Objasnite zagto se minusi nalaze na dijagonali, a zagto u donjoj polovini tablice. Susreta prema ovom ima (5.5-5)/2.
% Zadatak za odmor. Da ne budu previLe uoEljivi, svaki se klinac svakoga dana sastaje samo s po jednim Elanom grupe. Koliko C e najmanje dana trajati ti susreti? Nadite nekoliko rasporeda koji se medusobno razlikuju ne samo u rasporedu dana.
Nacrtajte djecu kao toEke, a susrete svakog dana kao duiine jedne boje izmedu njih i predoEite svoje primjere. OpiLite rijeEima dobiveni graf. Razmislite o broju svih rasporeda.

f l Koliko dijagonala ima konveksni n -terokut?


Rjes'enje. Dijagonale konveksnog n-terokuta odgovaraju dvoElanim skupovima toEaka koje nisu spojene bridom. Izbrojimo ponovno uredene parove sastavljene od toEaka koje mogu biti krajevi dijagonala. Time brojimo vektore umjesto duiina izmedu odgovarajudih vrhova.
Za svaki od vrhova, postoje joi n - 3 vrha tog n-terokuta koja su razliEita od njega samog i nisu s njim spojene bridom. Dakle, uredenih parova razliEitih nesusjednih vrhova ima n ( n - 3) . Medu uredenim parovima vrhova koje smo upravo prebrojali nema niti jednog para - s jednakim koordinatama. Nadalje, dijagonale AB i B A su jednake za bilo koji izbor vrhova A i B . Zato dvoElanih skupova s traienim svojstvom ima upola manje nego odgovarajutih uredenih parova. Rezultat: n(n - 3)/2.

-3

Koliko ima razliEitih ishoda pri bacanju dviju jednakih igraCih kocaka?

Rjes'enje. Zadatak bi bio vrlo lagan d a imamo plavu i crvenu kocku - na svakoj od kocaka moZe biti broj izmedu 1 i 6, pa bi broj ishoda bio jednak 6 . 6 .

U ovom zadatku, Eak i d a moZemo razlikovati kocke, ne smijemo razlikovati ishode u ovisnosti o tome na kojoj je kocki ispao koji broj. Ne moiemo postupiti na isti naEin kao u prethodnim zadacima, jer imamo ishode koje moZemo dobiti na dva naEina (to su

54

4. PREBROJAVANJE

dva razliEita broja) i ishode koje moiemo dobiti samo na jedan naEin (na obje kocke isti broj). Promatrajmo ipak kao i ranije skup svih uredenih parova. Oni uredeni parovi koji imaju jednake koordinate generiraju svaki po jedan ishod u smislu zadatka. Njih ima 6 komada. Uredeni parovi s razliEitim koordinatama daju po dva zajedno jedan ishod. Dakle ishoda s razliEitim brojevima na obje kocke ima 6 . 512. Ukupni broj ishoda je 6+6.5/2=6.7/2.

tP Ovaj rezultat moiemo interpretirati i tako da umjesto bacanja kocke imamo moguCe ishode 1,2,3,4,5,6 i "ponovi broj s druge kocke" i ako zahtijevamo d a su dva ishoda koje dobivamo razliEiti.
Kako biste organizirali rukovanje devetorice ljudi, tako da se svaki od njih r5 uje s toCno trojicom od preostalih?
Rjedenje. Kad bi svaki od te devetorice ljudi nakon rukovanja napisao s kim se sve rukovao, popis bi sadriavao 9 . 3 imena. BuduCi d a je za rukovanje potrebno dvoje ljudi, po dva listiia bi trebala predstavljati jedno rukovanje. Rukovanja bi tada bilo 9.312. To je nemoguCe - broj rukovanja bi trebao ipak biti cijeli broj.
Morali bismo zahtijevati d a se svaka od neparnog broja osoba (9) rukovala s parnim brojem drugih. Posebno, 8 je jedan od tih dobrih brojeva, gto smo mogli i oEekivati.

n ljudi ieli sjesti za okrugli stol. Na koliko naEina to mogu napraviti ako
' je

(a) bitno na koji stolac sjeda koja osoba, (b) bitan samo relativni poloBaj ljudi (tko kome sjedi zdesna, tko kome slijeva), (c) n paran, u drustvu ima jednako Bena kao i muskaraca, svaka osoba ieli sjediti izmedu osoba suprotnog spola i bitno je tko sjedi na kojem stolcu, (d) kao (c), ali je bitan samo relativni poloBaj.
Rjedenje.
(a) Stolce oznaEimo brojevima 1 , 2 , . . . ,n , a rasporede ljudi promatramo kao uredene n -torke. Rezultat: n! .

4. PREBROJAVANJE

55

(b) Po n od razmjeitaja iz prethodnog dijela zadatka daje jedan od razmjegtaja koje traiimo ovdje - nakon ;to ljudi jednom sjednu za stol, mogu se svi pomicati za po jedno mjesto ulijevo, tako d a svaki od njih nakon jednog od pomaka ima najbolji pogled na kuhinjska vrata. MoguCih pomaka ima n , pa je rezultat jednak n!/n = (n - I)!. (c) Razmjestimo prvo iene: odredimo da li C e sjediti na stolcima s parnim ili s neparnim oznakama, t e ih pustimo da sjednu. Izbor stolaca moiemo napraviti na 2 natina, a iene rasporediti na za njih predvidene stolce na (n/2)! naEina. Nakoe toga pustimo muLkarce d a sjednu. Oni mogu izabrati svoja mjesta na (n/2)! natina. Zene moie razmjegtati domakin, a muLkarce domaCica - nezavisno. Dakle svaki raspored oko stola je uredena trojka koja se sastoji od dogovora d a li C e iene sjedati na stolce s parnim ili s neparnim oznakama, zatim rasporeda iena, t e rasporeda mu'skaraca. Tih ureaenih trojki ima 2 . (n/2)! . (n/2)! = 2(n12)!~.
1

(d) Vidi rjegenja (b) i (c) dijela! Rezultat: 2 ( ~ ~ / 2 ) ! ~ / n .

tP Zadatak smo rijegili pozivajuCi se na analogni, jednostavniji sluEaj. Time smo dobili rjeienje koje malo zaEuduje - rjeienje mora biti cjelobrojno, ali na prvi pogled ne izgleda tako. No, uz koristenje Einjenice da je n paran, moie se brzo utvrditi da n dijeli 2(n/2)!2.

.d
,

Ako imamo m ) 3 razlieitih perli, na koliko naEina moiemo od njih napiti ogrlicu (bez kopEe)?

Rjes'enje. Ogrlicu, kad je jednom stavimo oko vrata, moiemo vrtjeti do mile volje. Oko vrata je moZemo staviti na dva natina koji se jedan od drugog dobivaju rotacijom rotacijom za 180' oko horizontalne osi naprijed-nazad. Ukupno jednu ogrlicu moiemo postaviti oko vrata na 2m natina, a da jedna perla bude okrenuta prema naprijed (ovaj uvjet je poireban zbog usporedbe!). Dakle broj slaganja ogrlice je m!/(2m) = (m- 1)!/2.

9 Za m = 2 ("ogrlica" sastavljena od dvije koine trake zavezane jedna za drugu) ovaj broj nije cjelobrojan. Pogledajte gdje smo koristili uvjet m > 2.
Cetrdeset ljudi treba prijeCi graniEni prijelaz, jedan po jedan. Na koliko je to moguCe napraviti: (a) ako je svejedno kojim C e redoslijedom prelaziti granicu, (b) ako je u grupi desetero djece medu kojima nema braCe, a svako dijete mora biti uz svoju majku (neposredno ispred ili neposredno iza), (c) ako svako od desetero djece mora biti ispred svoje majke (ne nuino neposredno ispred) .

(a) Takav zadatak smo veC nekoliko puta rjeiavali. Rezultat: 40!. (b) U ovom slutaju bilo bi najbolje ne brojati majku i dijete kao dvije osobe (neka se drie za ruke). Tada rasporeda ima (40 - lo)! = 30'. BuduCi d a svako dijete moie biti ili neposredno ispred ili neposredno iza svoje majke, traieni broj rasporeda iznosi 30! .21.

56

4. PREBROJAVANJE

(c) Svakom rasporedu u kojem je dijete a ispred svoje majke A moiemo pridruiiti jedan raspored u kojem je majka ispred djeteta - zamijenimo im mjesta. Stoga je broj rasporeda u kojima je mali a ispred svoje majke upola manji od broja svih rasporeda.

To vrijedi za svako dijete, pa je rjesenje zadatka 40!/21

9+ Neka je p broj rasporeda papiriCa s napisanim prezimenima svih osoba u redu. Za svaki od rasporeda papiriCa moiemo na 2'' naEina rasporediti ljude (za svako od prezimena kaiemo ili LLmajka ispred djeteta" ili 'Ldijeteispred majke") i na t a j naEin dobivamo sve rasporede ljudi. Dokaiite t u tvrdnju tako da konstruirate injekcije iz skupa svih rasporeda u skup svih ovakvih rasporedivanja i obratno. Rezultat je prema ovakvom naEinu prebrojavanja jednak p . 2'' = 40!, ili ponovno p = 40!/21 .
% (preskoEite pri prvom Eitanju) Ovaj rezultat moie se zapisati i kao
40 2,...,2,1, ..., 1 gdje je 10 dvojki i 20 jedinica. Interpretirajte to rjegenje!

( @ Na koliko se naEina moie odigrati prvo kolo lahovskog turnira s 271 sudionika? Redoslijed ploEa nije bitan. A s 2n

+ 1 sudionika?

Rjegenje i. Poredamo sudionike, na primjer prema redoslijedu prijave. Tada prvi sudionik moie izabrati svog protivnika na 2n - 1 naEina. BuduCi da je svejedno tko igra na kojoj ploEi, oni mogu sjesti za prvu ploEu. Prvi od preostalih sudionika moie izabrati svog protivnika na 2n - 3 naEina i s njim sjesti za drugu ploEu, itd. Kad ostanu joi samo dva sudionika, prvi od njih moie izabrati protivnika na samo jedan naEin, pa rezultat glasi: (2n - 1) . (2n - 3) . . . . . 3 . 1.

Ako je prijavljeno 2n 1 sudionika, gahista koji C e u prvom kolu pauzirati moZemo izabrati na 2n 1 naEin, a ostale raspodijeliti kao i ranije. Rezultat je stoga 2 n + 1 puta veCi.

9-t UoEite da je broj naEina igranja prvog kola s 2n i objasnite zaBto je to tako.

+ 1 i s 2n + 2 sudionika jednak + 1 igraEa izaberemo

>' brvo protivnika za prvog igraEa na 2n nalina, zatim na 2n - 2 naEina protivnika za


.

- 9 Gdje je pogreika u sljedeCem zakljuEivanju:

ako igra 2n

. drugog itd?

RjeSenje 2. Ako nekako poredamo sve sudionike u niz, onda moiemo reCi d a prva dva Bahista trebaju igrati zajedno, druga dva zajedno itd. Tada C e medu svim nizovima sudionika biti ekvivalentni (u smislu igranja prvog kola turnira) oni koji se razlikuju jedan od drugog

u rasporedu 2k -tog i 2k l -vog igraEa (za neki k E N n ) ili u rasporedu parova igraEa. Razmjeztaja unutar parova ima 2n , razmjegtaja parova i ~ n! a , pa je broj nizova Bahista iz kojih se generiraju razliEiti rasporedi igranja jednak:

4 . PREBROJAVANJE

57

Teie je izraEunati broj naEina igranja drugog kola turnira. Uo8te sami gdje je problem. Zadaci takvog tipa nalaze se u poglavlju o formuli ukljuEivanja-iskljutivanja.

9 tP tP

Dokaiite d a su ova dva rezultata jednaka. Da je turnir teniski, ne bi bilo svejedno koji par igra na srediLnjem terenu.. .

Na koliko naFina rnoiemo presloiiti slova rijeFi baobab?


Rjes'enje. Na raspolaganju imamo dva slova a, tri slova b i jedno slovo o. Da su sva slova medusobno razliEita, moguCih preslagivanja bi bilo 6 ! . Dakle, slova a, a, b, b, b, o moZemo presloiiti na 6! naEina.
b b b b b b b b b b b b a a a a a a a a a a a a o o o o o o o o o o o o b b b b b b b b b b b b a a a a a a a a a a a a b b b b b b b b b b b b

No u svakom od rasporeda iz postavljenog zadatka mogu se na 2! naEina presloiiti slova A (tj. a i a), a na 3! naEina slova B (tj. b, b i b). Skup svih rasporeda Lest razliEitih slova moiemo podijeliti na klase, tako da se rasporedi unutar pojedine klase razlikuju samo u rasporedu istih slova razliEitog tipa. Svaka klasa sadriavat C e prema prethodnom po 2!3! rasporeda i predstavljati po jedan raspored slova baobab. Zato je broj raspored a iz zadatka 2!3! puta manji od broja rasporeda s razliEitim tipovima slova. Rezultat: 6!/(2!3!) .

$% koliko permutacija slova rijeFi JUPITER samoglasnici dolaze po abec e d d m redu?


Rjes'enje. Buduii d a sarnoglasnici imaju toEno odreiten redoslijed, moiemo sve samoglasnike zamijeniti npr. slovom S. Tada C e bilo tko tko pozna abecedu moCi na jednoznaran naEin popuniti bilo koju od dobivenih permutacija.
Sada permutiramo multiskup slova { j,S,c$,t,S,r ) . Kao i u prethodnom zadatku dobivamo rezultat: 7!/3!. i

(1 Upravo ste uSli u knjignica u kojoj je n razliEitih knjiga. Na koliko n a ~ h a moiete posuditi tri knjigeiz te knjiiiice?
RjeSenje. Da ielite posuditi jednu knjigu, mogli biste je izabrati na n naEina. I sutradan biste mogli posuditi sljedeCu na n - 1 natina, a dan kasnije treCu na n - 2 naEina.

( ' 2

58

4. PREBROJAVANJE

Tako bi broj naEina posudivanja bio jednak n ( n - l ) ( n - 2). No, nevolja je u tome Bto posudujete sve tri knjige u isti dan, pa iigovi na kartici zaduienja neCe biti razliEiti. Posudivanje triju knjiga A, B i C odjednom moie se na 3! naEina simulirati posudivanjem u tri dana ( ABC, ACB, BAG, BCA, CAB, C B A ) . Dakle, posudivanja u tri dana ima 3! puta viBe nego posudivanja u jednom danu. Broj naEina posutiivanja triju . knjiga odjednom je zato jednak n(n - l ) ( n - 2)/3! =

.--tP
OI

(t)
1)
7

Posudivanjem triju knjiga smo u stvari izabrali jedan troElani skup knjiga iz knjiinice. OpCenito, broj naEina izbora k-Elanog podskupa n-Elanog skupa bio bi n ( n - 1 ) . . . . . (n - I c f k!

ito oznaEavamo s i Eitamo n povrh k . Brojevi ( ) ; zovu se binomni Eoeficijentz jer se pojavljuju u razvoju potencije binoma: (a b)n = C;=O (;)akbn-k .

(z)

9 Interpretirajte broj k-Elanih podskupova n-Elanog skupa, zapisan u obliku: n!/(lc!(n - k)!) . Objasnite zaito ima jednako k-Elanih i (n - k) -Elanih podskupova! Ta Einjenica se Eesto zapisuje u obliku pravila

12. IzraEunajte broj najkrakih puteva od ishodiSta do svih toeaka koje se nalaze u prvom kvadrantu, a ispod ili na pravcu x y = 10.

'I

Neka su zadani nenegativni cijeli brojevi p i q . IzraEunajte broj najkrakih puteva od ishodigta do toEke (p, q) .
Rjeienje. NajkraCi putevi od ishodiita do neke toEke u prvom kvadrantu sastoje se od koraka desno i gore (prema istoku, odnosno sjeveru).

PoEnimo raEunati. Ukupno trebamo izraEunati 1 1 . 1212 brojeva, Bto nije malo. Krenimo zato od otiglednih situacija i nadimo neki sistem za raEunanje. Od ishodista do ishodiita postoji samo jedan (prazni) put. Od ishodiBta do bilo koje toEke na s ,odnosno y -osi postoji takoder samo po jedan najkraCi put - ti putevi idu Sam0 ravno. Do toEke (1,l) moiemo doCi na jedan od dva naEina: ili posjeCujuCi toEku (1,O) ili posjeCujuCi toEku ( 0 , l ) . Ukupno imamo dva puta do toEke (1,l) . Zapiiimo rezultate koje imamo tako da uvijek iznad toEke u koordinatnom sustavu stoji broj puteva do te toEke. Zasad imamo

4. PREBROJAVANJE

Do toEke ( 2 , l ) moiemo doCi ili iz toEke ( 1 , l ) ili iz toEke (2,O) , pa je ukupni broj puteva jednak zbroju broja puteva do tih dvaju toEaka: 2+1 = 3 . SliEno vrijedi i za toEku (1,2), ali se sada radi o toEkama (0,2) i ( 1 , l ) : 1 2 = 3 . Ovaj postupak nastavljamo i za ostale toEke, te uz ukupno 9.10/2 zbrajanja (ili manje, ako smo bili dovoljno lukavi) dobivamo konaEni rezultat:

Rijegimo i drugi dio zadatka. Za proizvoljnu toEku (p, q) ne moZemo provesti postupak kao u prvom dijelu zadatka, nego traiimo formulu koja C e davati ieljeni broj puteva. Da bismo izabrali jedan put, trebamo odluEiti npr. kojih C e p od ukupno p + q koraka biti koraci udesno. Prema napomeni iz prethodnog zadatka, tih p elemenata od p q nazina, pa je to i trageni broj puteva. moiemo izabrati na (P:4)

+f Mogli smo istaknuti i one korake koji vode prema gore. Rezultat bi tada bio . Uvjerite se d a je to jednako prvom rezultatu zadatka. zapisan u obliku
JoL jedna interpretacija istog rezultata glasila bi ovako: onaj tko treba iCi od ishodiLt a do toEke (p, q) dobio je p papiriCa s uputom "odi jedan korak na sjever" i q papiriCa s uputom "odi jedan korak na istok". Svaki moguCi put odgovara j\ednom razmjeLtaju tih uputa, a njih ima (p q)!/(p!q!) .

+P OEigledna je bijekcija izmedu p -Elanih podskupova p q -8anog skupa i puteva iz ovog zadatka. To bi to moglo znaEiti d a ili puteve ili podskupove moiemo zaboraviti i raditi dalje samo s jednom vrstom objekata. No to se ne Eini, jer se se za neke probleme zgodnijom pokazuje interpretacija pomoCu skupova, a za druge pomoCu puteva.
+f Prvi dio zadatka daje jednostvani naEin raEunanja binomnih koeficijenata, zvan Pascalov trokut. On se obiEno crta malo drugaEije nego Lto smo ga mi nacrtali.

60

4. PREBROJAVANJE

Prvi i zadnji element u svakom redu su jednaki 1 ( = ) ( : = I ) , dok je svaki od preostalih elemenata jednak sumi dvaju elemenata koji se nalaze iznad njega (lijevo i desno).

(g)

4-t IzraEunajte broj najkraCih puteva od cjelobrojne toEke (a, b) do cjelobrojne toZke ( c ,d) .

tP U jednom od prethodnih zadataka bio je izraEunat broj svih puteva od ishodiLta do pravca x y = n . IzraEunajte t a j broj jog jednom koristeCi rezultat ovog zadatka. Rezultat koji Cete dobiti samo se naizgled razlikuje od u tom zadatku predloBenog rezultata.

1 q Na tomboli treba izmedu n brojeva pogoditi k brojeva koji C e biti izvueeni. Koliki treba biti k da bi bila najmanja Bansa da netko pogodi cijelu kombinaciju?
Rjes'enje. Sansa C e biti to manja :to ima viLe k-Elanih podskupova skupa N n

r\
"

) ( ;

Pogledajmo kako se za fiksni n binomni koeficijenti mijenjaju s k - za koje k je vea, a za koje manji od ( k ; l ) .

Usporedujemo li brojeve koji se sastoje preteino od sliEnih faktora, moBemo primijeniti sljedeCu oEiglednu Einjenicu: pozitivni broj a je veCi od pozitivnog broja b ako je alb > 1, manji ako je t a j omjer manji od jedan, a brojevi a i b su jednaki ako je omjer alb jednak 1. Zato raEunamo:

Taj broj je veCi od 1 ako je Ic < (n - 1)/2, manji od 1 ako je k > (n - 1)/2, a jednak 1 ako je k = (n - 1)/2.

( ; rastu s k od ) ( : do , a za Vidimo da za paran n = 2n' binomni koeficijenti ) neparan n = 2n' + 1 do = (n,:l) . Zato je za paran n najveCi binomni koeficijent , a za neparan bonomni koeficijenti i (n,"+) .

(2)

(z,)

(z,)

(2)

Na koliko naEina moiemo podijeliti 30 uEenika u tri jednakobrojne skupine

(a) od kojih C e prva saditi cvijeCe, druga kositi travu, a treCa rezati grane, (b) koje C e sve kositi travu.
Rjes'enje. U svakoj skupini te biti 10 uEenika.
(a) Rjes'enje 1. Izaberimo prvo skupinu uEenika koja t e saditi cvijeCe. To moHemo : ( ; naEina. Nakon toga, od preostalih uEenika izaberimo one koji Ce kositi napraviti na ) travu ( (30i10) naEina), te one koji C e rezati grane ( (30-::-10) natina). Rezultat :

4. PREBROJAVANJE

61

Rjes'enje 2. Na stolu moZemo pripremiti snop od 30 papiriCa - 10 dozvola za posudivanje lopatice iz Zikolskog skladiBta, 10 za kose, 10 za Zikare. UEenici de dolaziti unaprijed odredenim redom (npr. po abecedi) i s vrha snopa uzimati po jedan papirid. Papiride moZemo presloIiti na 30!/(10!10!10!) naEina. Uvjerite se da je ovaj rezultat jednak rezultatu dobivenom u prvom rjeBenju.
+f'

I optenito je:

za n = nl

+ n 2 f . . . + nk , gdje su svi ni 2 0 . Taj broj se oznaEava s

(b) Ako uEenici ostanu podijeljeni u skupine kao u prethodnom dijelu zadatka, ond a moZemo i svim skupinama podijeliti kose i kosilice. No, ako svi uEenici imaju kose, moIemo im na 3! naEina podijeliti lopatice, kose i Bkare. Dakle triju grupa za koBnju ima 3! puta manje nego triju grupa za razliEite poslove:

9+ U ovom zadatku smo birali neke podskupove. Sto smo toEno prebrojali u zadatku (a)? U zadatku (b)? .

/ $

Na koliko naEina moierno izabrati 5 od 52 igrab karte, ako medu njima m a biti barem jedan tref.
Rjes'enje 1. MoZemo izabrati 1,2,3,4 ili 5 trefova. Ako izabiremo i trefova, broj naEina izbora je (If) ( 5 2 5 1 i 3 .) Rezultat zadatka se dobiva zbrajanjem po i :

5
z=1

6 "

( I : )

9 ) .' . . , Rjes'enje 2. Sjetimo li se odmah da znamo izraEunati broj naEina izbora pet karata bez ijednog trefa, onda rezultat rezultat dobivamo oduzimanjem tog broja od ukupnog broja izbora:

("iT i3). --.

t,~,ztr ,

? .

c ,. f a.

)ti

4-,

DokaIite da su ova dva rezultata jednaka. Generalizirajte jednakost medu nji(113) (5i)

ma!

9+ Nadite pogreZiku u rezultatu

( 9

Full u pokeru je kombinacija pet karata, od kojih su tri jedne, a preostale dvije druge vrijednosti, bez obzira na boje. Npr. jedan moguCi full je

62

4. PREBROJAVANJE

90,94,90,

J h , JV . Ako se zna da se poker igra s 32 karte (po osam svake boje), koliko ima mogudih fullova?

Rjedenje. Moiemo prvo izabrati vrijednost koja C e se u fullu pojaviti triput, zatim izabrati boje tih karata, a nakon toga izabrati vrijednost koja C e se pojaviti dvaput i boje karata te vrijednosti. Rezultat:

( ) (:)
4-t Sto je mogla misliti osoba koja je kao rjeSenja napisala:

4-t U pravom pokeru postoji moguknost zamjene karata nakon dijeljenja. Premda su moguCnosti zamjene razliEite, ograniEit Cemo se samo na sluEaj u kojem se dozvoljava da se do fulla dode i tako da se prvo odloii jedna karta, a zatim se dobije jog jedna karta. Jasno, na ovaj naEin Cemo pokuHati doCi do fulla samo onda kad veC u rukama nemamo full, ali imamo ili po dvije karte jednakih vrijednosti ili tri karte jednake vrijednosti. Na koliko naEina moiemo uz opisanu strategiju doCi do fulla ako dozvoljavamo i zamjenu?
Na tulumu se okupilo m djevojaka i m deiki. No n djevojaka ne ieli p esati. Na koliko naiina se mogu preostali gosti "sloiiti" u plesne parove?
Rjedenje 1. DeEki se mogu prvo dogovoriti tko C e od njih plesati )na , : ,( Oni koji i e plesati mogu izabrati partnerice na (m - n)! naEina. Rezultat:

(I/

naEina.

Rjes'enje 2. Na m naEina moie se iiabrati partner za prvu plesaEicu (prvu, npr. po visini), medu preostalih m - 1 moie se izabrati partner za drugu plesaEicu, medu m - 2 za tretu, itd, medu n 1 za posljednju, m - n-tu. Rezultat:

me (m - 1) . . . . . ( n

+ 1) = (m),-,.

' 18 Neka su m i n zadani prirodni brojevi i neka je m 2 n . Koliko ima 'najkradih puteva od ishodigta do toike (nz,n) koji izgledaju kao stepenice, tj. ne sadrIe dva uzastopna vertikalna koraka?
Rjedenje 1 . Svaki C e vertikalni korak biti iza "svog" horizontalnog koraka, osim moi d a prvog koraka u cijelom putu. Dakle, poslaiemo li sve horizontalne korake jedan do drugog na x-os, onda postoji ukupno m 1 pozicija na kojima smijemo umetnuti jedan vertikalni korak i time podignuti dio puta desno od tog mjesta za jedan. Te pozicije su: na prvo mjesto, iza prvog horizontalnog koraka, iza drugog, itd, sve do zadnjeg moguLeg mjesta - iza n-tog koraka. Da bismo umetnuli n vertikalnih koraka, izabiremo n , f l ) . pozicija od moguiih m 1. Broj naEina d a se to uradi je jednak (

" : ,

4. PREBROJAVANJE

63

Rjegenje 2. Vertikalne korake koji C e biti iza nekog horizontalnog koraka veiemo s horizontalnim korakom u par. Osim iza horizontalnog koraka, vertikalni korak moie biti i na poEetku puta. Tako se svi putevi raspadaju na dvije klase kojima se lako odredi veliEina. Putevi koji poEinju horizontalnim korakom sadrfe n parova koraka (desno, gore) i jog m - n horizontalnih koraka. Putevi koji poEinju vertikalnim korakom sadrfe osim njega jog n - 1 parova i m - n 1 horizontalnih koraka. To su i jednine moguCnosti za formiranje niza, pa je rezultat jednak

(n+

;(

-n))

( ( n - 1)

+ ( m- n + 1)
n-1

($+(.:I)

p
(g)

U hotel u jedno malo planinsko mjesto stiglo je n sudionika nekog savjetoanja. Sve sobe u hotelu su jednosobne i imaju krasan pogled na okolne vrhove, ali ih samo b od ukupno n ima balkon. v sudionika savjetovanja su VIP i moraju dobiti sobu s balkonom. Medu sudionicima savjetovanja je s studenata kojima se trebaju dati sobe bez balkona. Ostale sudionike moZe se smjestiti bilo kako . Na koliko naEina se mogu sudionici tog savjetovanja smjestiti u sobe?
Rjegenje 1. UoEimo prvo d a zadatak ima rjegenje samo ako je v b i s Pokugajmo rijegiti zadatak tako da oponagamo smjegtavanje osoba u sobe.

<

< n - b.

Izaberimo prvo u koje C e sobe s balkonom useliti VIPovi - to je moguCe uEiniti na naEina. Nakon toga smjestimo VIPove u te sobe (na v! naEina). Broj razliEitih smjeS. v! . SliEno moiemo napraviti i sa studentima, n a z ~ .s! ~ naEina. ) tavanja VIPova je Ostale goste smjestikno u preostalih n - v - s soba na ( n - v - s)! naEina. Rezultat:, ,.;,-

it)

(:) (n;
v ! .

, <

>

b, . s ! . ( n - v - s ) ! = (b), ( n - b ) ,

( n - d - s)!.

G '

Mofe se direktno interpretirati i desna strana posljednje jednakosti.

RjeSenje 2. Druga strategija za rjezavanje ovog problema je prvo razdvojiti ljude koji C e imati sobe s balkonom od ostalih, pa onda svakoj grupi odvojeno dijeliti kljuEeve naEina moiemo izabrati koji C e ljudi osim VIPova imati sobe s od soba. Na (ni:iS) balkonom, a onda kljuEeve soba sa balkonima moiemo podijeliti na b! , a kljuEeve soba bez balkona na ( n- b)! naEina. Rezultat:

Za desert na nekoj gozbi bilo je priredeno 140 komada kolaEa i 100 zdjelica oleda. Na gozbi je bilo 200 ljudi. Pretpostavimo da 70 ljudi Beli bezuvjetno J jesti kolaE, da 50 ljudi Beli bezuvjetno jesti sladoled, dok je ostalima svejedno. Na

Uvjerite se d a je i ovaj rezultat jednak rezultatu iz prvog rjegenja.

64

4. PREBROJAVANJE

koliko naEina se mogu podijeliti kolaEi i sladoledi, ako svaka osoba smije dobiti najvige jednu porciju deserta? Porcije kolaEa, kao i porcije sladoleda medusobno su jednake.
Rjedenje. Onim osobama koje su odluEile Sto C e jesti moZemo desert podijeliti samo na jedan naEin. Ostalih 80 ljudi moCi demo ponuditi sa 70 komada kolaEa i 50 porcija sladoleda. Bududi da svaki Eovjek treba dobiti porciju deserta, znamo da Cemo podijeliti izmedu 30 i 70 komada kolaEa. Ako namjeravamo podijeliti toEno k komada kolaEa, broj . Ukupni broj podjela iznosi: podjela je

1
I

(to)

F I -

-,

k=30

Ovaj se izraz ne moZe 'srediti'.


+f'

Da je bilo 150 porcija kolaEa i 130 porcija sladoleda, rezultat bi bio

C
k=O

80

(y)

! 4+ Ako ste veC barem jednom protitali ovo poglavlje do kraja, pokugajte odgovoriti ~ , na ovo pitanje: na koliko naEina se mogu podijeliti svi kolaEi i svi sladoledi, a k nFqa

"LoograniEenjana broj porcija koje jedna osoba smije (moZe) pojesti?

lo"

BL t A

<-

Uputa: ponovno promatrajte odvojeno raspodjele u kojima razliEiti brojevi ljudi iz skupine neodluEnih dobivaju kolaE. Pazite, neodluEni mogu pojesti i kolaE i sladoled!

Z n uEenika doelo je na praksu u jedno poduzete. Polovina tog broja su momci, a polovina djevojke. Jedan odjel tog poduzeta zatraZio je da dobije barem ml momaka i barem dl djevojaka, a drugi odjel barem mz momaka i barem d2 djevojaka. Pretpostavimo da su zahtjevi tih pogona uskladeni, tj. da je ml mz n i dl d2 n . Na koliko se naEina mogu rasporediti u pogone ako u oba pogona treba raditi jednak broj uEenika?

<

+ <

Rjedenje. Dovoljno je izabrati koji L e uEenici raditi u prvom odjelu. Broj momaka mora biti takav da zadovolji potrebe tog odjela, a da istovremeno ostane dovoljno momaka za drugi odjel. Isto vrijedi i za djevojke.

4-t Usporedite posljednja tri zadatka!

m w.

DostavljaEi trebaju dostaviti 70 raznih poruka. Da ne bi shvatili sadrBaj poruka koje nose i da se ne bi previge umorili trEeCi od jednog do drugog primatelja, poruke su podijeljene na sljedeti naEin. Svaka poruka rastavljena je na 5 dijelova, a svaki dostavljaE nosi toeno 7 poruka. Koliko je dostavljaea potrebno?

RjeSenje 1. Da je svaka poruka u jednom komadu, bilo bi potrebno 10 = 70/7 dostavljaFa. BuduCi da svaku poruku treba nositi toeno 5 dostavljaEa, potrebni broj dostavljaEa je 5 puta veCi i jednak 50.

4.

PREBROJAVANJE

65

Na temelju izraEunatog broja dostavljaEa joL ne moiemo zakljuEiti da je potrebni raspored dostavljanja poruka mogud. Neki rasporedi se vrlo jednostavno konstruiraju. Konstruirajte barem dva bitno razliEita rasporeda dostavljanja.

9 Ovo rjeSenje ima smisla samo za ove posebne brojeve. PokuLajte ga provesti za dostavljaEe koji nose po 25poruka. Pogledajte drugo rjegenje.
R j e i e n j e 2. Brojimo zaduienja dostavljaEa - svaki dostavljaE zaduien je za sve poruke koje nosi. Ako ima d dostavljaEa, onda to znaEi da je broj zaduienja jednak d . 7 . S druge strane, svaka poruka zaduiena je kod 5 dostavljaEa, pa je broj zaduienja jednak 70 . 5 . Dakle imarno d.7=70.5,
i t o ponovno daje d = 50.

tP NaEin prebrojavanja koriLten u drugom rjezenju zove se princip dvostrmkog prebrojavanja i Eesto se koristi. Umjesto direktnog prebrojavanja traienih objekata brojimo - __neke Eiji broj ovisi o nepoznatom-broju traienih Ti drugi objekti -- objekata. - -- dru~e-objekte moraju biti izabrani tako d a se mogu preb;ojati i na neki drugi naE&>ako d a se moie dobiti jednadiba u nepoznatoj veliEini.

W Nacrtajte tablicu u kojoj su u recima dostavljaEi, a u stupcima poruke (moiete to uEiniti i za neki manji primjer). NapiSite kriZiC na presjek onih redaka i stupaca koji odgovaraju zaduienju za poruku. Sto moiete reCi o toj tablici?
W Nacrtajte jog jedan prikaz rasporeda. Nacrtajte prvo skup poruka i skup dostavljaEa. Poveiite zatim crtom svaku poruku sa svim dostavljaEima koji je nose. Interpretirajte zadane i traiene veliEine na ovom prikazu. Sto predstavljaju crte? RijeSite ponovno zadatak.

Q
,

3. Povjerenstvo se sastajalo 40 puta i svaki je put bilo prisutno 10 Elanova. o se niti jedan par Elanova nije susreo vise nego jednom, dokaBite da je broj Clanova povjerenstva najmanje 60.
RjeSenje. $to znaEi d a se niti jedan par Elanova nije susreo viSe od jedanput? To znaEi d a za svaki par Elanova povjerenstva postoji najviLe jedan sastanak na kojem su oboje prisustvovali.
Brojimo zato ureitene trojke (jedan Elan, drugi Elan, zajedniZklci sastanak), gdje je na primjer ime prvog 8 a n a po abecednom redu ispred imena drugog. Krenemo li od Elanova, vidimo d a takvih trojki ima manje ili jednako , gdje je c ukupni broj Elanova povjerenstva. Krenemo li od sastanaka, te brojimo parove prisutnih po sastanku, dobivamo broj trojki: 40. (I2') . Time smo doHli do nejednakosti

(g)

iz Eega, uz uvaiavanje Einjenice da c mora biti prirodni broj, slijedi rjeSenje c 2 60.

?-+ Ako veC niste, nacrtajte tabliEni i grafovski prikaz ovog problema. Objasnite na oba prikaza: Lto je bilo zadano, Lto se traZilo. Ponovite rjeLenje komentirajuti akcije i zakljuEke na svakom od prikaza. Koji od njih vam se za ovaj problem Eini prikladnijim?

66

4. PREBROJAVANJE

9+ Nadite pogregku u sljedetoj izjavi: uvjet zadatka ekvivalentan je uvjetu da ukupni broj prisustvovanja sastancima (parova (Elan, sastanak)) mora biti manji ili jed. 2 2 40. i pogreSno nak dvostrukom broju parova Elanova. Iz toga bi slijedilo rjeLenje.

(z)

(iO)

24. Na ispitu je bilo 28 zadataka. Svaki student je toEno rijeSio po 7 zadataka i za svaki par zadataka postoje toEno dva studenta koji su ga rijegili. Koliko je bilo studenata na ispitu?
Rjegenje. Ovaj zadatak malo nalikuje na prethodne. UsporedujuCi ga s prvim, moZemo reCi da su studenti rjeLavali zadatke sliEno kao Lto su dostavljaEi nosili dijelove poruka. NaZalost zadane veliEine ne odgovaraju sasvim veliEinama zadanim u tom zadatku. UsporedujuCi ovaj zadatak s prethodnim, vidimo d a je par zadataka rijeSio par studenata, te da je zbog toga situacija u ovom zadatku razliEita od situacije u prethodnom zadatku.
Nacrtajte skicu! Vidimo da uvjet o parovima zadaje na neki naEin broj veza izmedu skupa zadataka i skupa studenata. Te veze moZemo kao i ranije prebrojati sigurno jog na jedan natin. Brojimo ono ;to nam daje najkompliciraniji uvjet - sve parove toEnih rjegenja, tj. trojke (student, jedan toEno rijedeni zadatak, drugi toE;no rijes'eni zadatak), gdje je prvi zadatak na ispitu imao manji redni broj od drugog. Neka je ispitu pristupilo s studenata. Brojimo li te uredene trojke tako d a krenemo od zadataka, dobivamo rezultat (228) . 2 . Krenemo li od studenata, dobivamo s . . IzjednaEavanjem ovih relacija dobivamo jednadZbu

(i)

Eije je rjegenje s = 3 6 .
Ct, Konstruirajte jedan primjer s dva ili tri rijeLena zadatka po studentu. Ct, Usporedite posljednja tri zadatka. Postavite s priEom svakog od njih uvjete i pitanja zadana u preostalima.
+f' TabliEni i grafovski prikaz sluie nam za olakgavanje rjeiavanja ovakvih zadataka. Postoji jog jedna interpretacija veliEina zadanih u ovom zadatku. Nazovimo zadatke totkarna, a studente (ili njihove ispite, svejedno je) pravcima. ToEka leii na pravcu ako je student rijeSio odgovarajudi zadatak. Tada moZemo reCi d a imamo ukupno 28 toEaka, da svaki pravac sadrii 7 toEaka, te da kroz svake dvije toEke prolaze toEno dva pravca. Ovakav pristup vodi u teoriju blok-dizajna.

25. Neka je S skup veliEine n i neka je Ic neki prirodni broj. Koliko ima uredenih k-torki A = (A1, Az, . . . , Ak) E 'P(s)~takvih da je:

(a) U;=,A~ = S i svi A i , A j su u parovima disjunktni, (b) n,,,Ai = 0 i A ; U A j = S zasve i , j E N k , i # j , (c) U;=,A~ = S , svi Ai , Aj su u parovima disjunktni, te je [Ail = n i , za svaki i E Nk , gdje su n1, n2,. . . ,n k unaprijed zadani nenegativni cijeli brojevi i

4.

PREBROJAVANJE

67

Rjeienje.
(a) Za svaki element skupa S moramo reCi u koji od Ai -ova ulazi. Dakle, za svaki od n elemenata imamo k moguCnosti izbora. RjeLenje: kn .

9+ Neki skupovi Ai Ce moida ostati prazni. PokuLajte za neke male k (npr. 2,3) izraEunati broj k -torki (A1, . . . ,Ak) s dodatnim uvjetom da su svi Si neprazni.
(b) PokuLajte i ovaj zadatak rijeLiti direktnim prebrojavanjem. Mi Cemo ovdje krenuti drugim putem. Usporedimo li uvjete ovog zadatka s uvjetima prethodnog, vidimo d a tam0 gdje u prethodnom zadatku stoji presjek, ovdje stoji unija i obratno, te tam0 gdje u prethodnom stoji cijeli skup S , ovdje stoji prazan skup. To nas moZe navesti na to da uvjete ovog zadatka dobijemo iz uvjeta prethodnog komplementiranjem (uz primjenu de Morganovih zakona).

Kako nam to moBe pomoCi u brojanju rjegenja? Promatrajmo preslikavanje P ( S ) P ( S ) definirano s X C I+ X . To preslikavanje je bijekcija s P ( S ) na P ( S ) jer si je ono samo obostrani inverz. I ne samo to, nego zbog upravo otkrivene veze izmedu uvjeta prethodnog i ovog zadatka ono je i bijekcija izmedu njihovih skupova rjeLenja (svojstvo injektivnosti je naslijedeno od preslikavanja P ( S ) + P ( S ) za koje smo pomoCu inverza dokazali d a je bijekcija; svojstvo surjektivnosti dokaiemo tako d a uzmemo neku k -torku rjegenja ovog zadatka, komplementiramo svaki od njenih skupova i dokaiemo da je to rjegenje prethodnog zadatka - ono C e onda komplementiranjem biti preslikano u rjeeenje ovog zadat ka) .

Sada, prema principu jednakosti znamo d a rjeSenja ovog zadatka ima jednako koliko i rjesenja prethodnog! (c) U ovom dijelu zadatka ne moiemo, analogno prema (a), za svaki element reCi u koji de od Ai -ova uCi, jer A, -ovi imaju unaprijed odredene veliEine. Zato Cemo prvo e tvoriti skup Al , zatim elemente za skup A2 itd. Za izdvojiti elemente skupa S koji C A1 elemente moBemo izabrati na naEina, za Az na (n,znl) itd. Rezultat:

(c)

9+ Sto se dobiva komplementiranjem ovog dijela zadatka?


% OpiSite rijeEima: Sto smo prebrojali u (a), Sto u (c) dijelu zadatka?

26. Neka je X skup veliEine n i neka je k prirodan broj manji ili jednak n . Koliko ima uredenih k-torki skupova (XI,X2,. . . ,Xk) takvih da je

XI

x2c . . . CX,

CX?

Rjeienje. Ovaj niz skupova veC je jedinstveno odreeten "prirastom" skupova, tj. preslikavanje
(Xl,X2,..9,Xk) je bijekcija

(Xl,X2\XltX3\XZ,...,Xk\Xk-l,X\Xk)

{ ( x i , x2,. . . ,X k ) : X1

S Xz

.. .

Xk

X)

{(y1,y2 ,...,Y ~ + ~ ) : u ~ = $ ~ ~Y Yi ~ nY = jx=, O z a i # j ) .

68

4. PREBROJAVANJE

X
Yk+l

Xk

y2

x1

Yl

Bijektivnost tog preslikavanja moEe se najjednostavnije provjeriti konstruiranjem inverznog preslikavanja: (Yl,Y2,...,Yk+l)
H

(Y1,Yl UY2,Yl UY2 UY3,... , u f = l ~ i ) .

Provjerite da je ovo preslikavanje obostrani inverz prethodno definiranom preslikavanju. Time dobivamo jednakobrojnost svih padajutih k -torki X -eva i svih k 1 -torki disjunktnih Y -a. Prema prethodnom zadatku rezultat je jednak (k 1)" .

27. Koliko postoji uredenih k -to& (xl , x2, . . . ,x k ) s elementima iz N, svojstvom 1 x1 < xz < . . . < x k n?

<

<

, sa

Rjes'enje. PopiSemo li jedan kraj drugoga sve brojeve od 1 do n , vidimo d a izbor ovakve k -torke odgovara izboru k razlizitih elemenata skupa Nn . Primjer. n = 12, k = 4 .

Rezultat:

(E) .

+P Ovaj rezultat se pokatkad interpretira na drugaEiji na8n. Naime, svaki podskup skupa prirodnih brojeva moZe se urediti (prirodno, tj. po veliEini) na jednoznatan naEin. Dakle svakom k-Elanom podskupu od Nn moiemo na jedinstveni naEin pridruiiti uredenu k-torku brojeva iz istog skupa kojoj su koordinate uredene rastute. Jasno je d a i svakoj takvoj uredenoj k-torci moEemo na jednoznaEan naEin pridruiiti podskup, te d a su t a dva preslikavanja jedno drugom inverz, pa i bijekcije.

4-t Usporedite ovaj zadatak s prethodnim. Prvo za neke male n nacrtajte skice. Za ovaj zadatak nacrtajte n toEaka u ravnini i oznaEite ih brojevima od 1 do n . Nacrtajte zatim strelice od svake toEke i do njenog neposrednog sljedbenika i 1. Za prethodni zadatak nacrtajte totke koje odgovaraju podskupovima skupa N, (ne zaboravite na prazni skup!). Strelice crtajte zatim za svaki skupa A izmedu toEke koja odgovara njemu i toEke koja odgovara sljedetem vetem skupu A U {x) za x !$ A .

Objasnite sada na crteZu: Lto je bio zadatak ovog, a Sto prethodnog zadatka.

4. PREBROJAVANJE

69

28.

Koliko postoji rjeienja jednadBbe yl

+ yz + . . . + yk = n u Nk ?

Rjebenje. Ako ponovno popizemo sve brojeve od 1 do n jedan kraj drugoga, onda moiemo vidjeti d a svako rjegenje ove jednadibe odgovara izboru k - 1 elemenata izmedu prvih n - 1 elemenata. Naime, svaki izabrani element sluii tome da kaie "sad je dosta uzimanja jedinica za ovaj sumand, prijedi na drugoga". Na kraju za posljednji sumand uzmemo sve preostale jedinice. Primjer. n = 1 2 , k = 4 .

OdgovarajuCi rastav broja 12 je 3

+ 4 f 1+ 4 = 12.
( ; I : )

Biramo k "posljednjih jedinica" medu n - 1 elemenata (posljednju jedinicu ostavljamo kao zalihu za posljednji sumand), pa je rezultat jednak .

9+ Usporedite tri prethodna zadatka. Ako prva dva proglasimo u nekom smislu analognima, formulirajte i zadatak sa skupovima analogan ovom zadatku.
29. IzraCunajte na koliko se naCina moBe podijeliti b jednakih bombona na d djece. Pojedino dijete smije ostati bez ijednog bombona, a ne smeta niti ako neki dobiju i po nekoliko bombona.
Rjebenje. Jedan uobiEajeni naEin raspodjele u svakodnevnom Zivotu je napraviti pakete s bombonima, poredati ih, poredati djecu, te prvom djetetu dati prvi paket, drugom drugi, itd.

U ovom sluEaju moiemo smatrati da su djeca ved poredana. Trebamo jog samo napraviti pakete i poredati ih. BuduLi da su bomboni svi jednaki, moiemo postupiti tako da ih poslaZemo u niz i uzimarno u svaki paket po nekoliko bombona. Na koliko naEina moiemo iz niza bombona napraviti d paketa? Moiemo postupiti na sljededi naEin: iza posljednjeg bombona koji ulazi u prvu skupinu stavimo vreCicu u koju C e se kasnije staviti ti bomboni, iza posljednjeg bombona koji ulazi u drugu skupinu stavimo vreCicu u koju C e se kasnije staviti bomboni druge skupine itd. sve do d - 1-ve skupine. Sve bombone koji tim postupkom nisu stavljeni niti u jednu skupinu, stavit Cemo u posljednju (d-tu) skupinu i uruEiti ih bez vretice.
Primjer. Situaciju u kojoj prvo dijete dobiva 2, drugo niti jedan, treCe 4, Eetvrto 1, a pet0 2 bombona moiemo prikazati Gematski:

Stoga se broj raspodjela b bombona na d djece svodi na raspodjelu d - 1 jednakih vreCica izmedu b bombona. Dalje se to moie svesti na odredivanje na kojima C e od

70

4. PREBROJAVANJE

b + (d - 1) pozicija biti bomboni (znamo da te na ostalima biti vredice). Ovakav problem znamo rijeiiti i broj takvih raspodjela iznosi:

+P

Vidimo da smo mogli i birati pozicije na kojima su vretice, jer je

Usporedite ovaj zadatak s rjeiavanjem jednadibe yl N ~ .

+y2 + . . .+yk = n u skupu

30. Odredite broj svih r-Elanih podskupova skupa Nn koji ne sadrZe niti jedan par uzastopnih brojeva.
Rjes'enje. Znamo izraEunati broj podskupova nekog konaEnog skupa i uz ograniEenje na njihovu veliEinu. Kako svesti zadani problem na t a j ili neki drugi problem koji znamo rijeiiti? Na primjer koliki treba biti skup da bismo mu mogli izabirati npr. r -Elane podskupove bez ikakvog uvjeta i da ih bude jednako kao i zadanih skupova?

U traienim podskupovima razlika izmedu svaka dva susjedna broja veta je od jedan. U "obiEnom" podskupu dozvoljava se i razmak 1. U podskupu koji ielimo dobiti, recimo {x1,x2,. . . ,xT) , uredimo elemente tako da bude XI < s 2 < . . . < z r . Pokusajmo njemu pridruiiti skup bez dodatnih uvjeta na razlike susjednih elemenata preslikavanjem
To preslikavanje pridruiuje svakom r -Elanom podskupu od Nn r -Elani podskup skupa Nn-,+1 . Ono je bijekcija, jer mu je preslikavanje
{~1,~2,...,~T)~{~1,~2+1,~3+2,...,y~+r-1)

obostrani inverz. Stoga traienih podskupova ima:

4-t Usporedite ovaj zadrqtak s prethodna dva, kao i njihova rjeienja. Izvedite postupkom analognim rjeienju ovog zadatka iz broja svih rjeienja jednadibe y ~ y2 + ... yk = n u Nn broj svih rjeienja iste jednadibe u Nn U (0).

+ +

+P SliEno rjeienju ovog zadatka moiemo dobiti i broj podskupova od Nn Eiji se elementi razlikuju barem za I , gdje je 1 E N . Rezultat:

r ,

Iz tog rezultata moiemo lako provjeriti za koliki je 1 takav izbor mogud: n- (1- l)(r- 1) 2 tj.

4.

PREBROJAVANJE

71

4-t Ranije u ovom poglavlju bio je zadan zadatak u kojem je trebalo prebrojati sve puteve od ishodista do toEke (m, n) koji nemaju susjednih vertikalnih koraka.
Ako interpretiramo puteve duljine m + n kao n-Elane podskupove skupa Nm+, koji predstavljaju pozicije vertikalnih koraka, onda znamo da ih ima

Pogledajte jog jednom rjegenje upravo spomenutog zadatka i konstruirajte pomodu njega drugo rjeLenje ovog zadatka!

31. Neka je S skup svih vrhova nekog konveksnog n-terokuta i neka je t E N . Koliko ima t-Elanih podskupova od S koji ne sadrZe niti jedan par susjednih vrhova?
Rjedenje 1 . Zadatak nalikuje na prethodni, ali je relacija LLbiti susjedan" proLirena novim parovima: (n, 1) i (1, n) . Tako je broj podskupova skupa vrhova n-terokuta bez susjednih vrhova manji od broja podskupova od N n bez susjednih elemenata toEno za broj onih dobrih podskupova koji sadrie i vrh 1 i vrh n . Ako je podskup dobar, a sadrBi i 1 i n , onda ostale potrebne elemente ( t - 2 komada) treba birati me& vrhovima 3 , 4 , . . . ,n - 2 . Rezultat:

Rjes'enje 2. Nezgodnog 'Lkruinog"uvjeta moiemo se rijegiti i na drugi naEin. UoEimo jedan vrh i nazovimo ga X . Tada de biti lako prebrojati odvojeno one dobre podskupove koji sadrie X i one dobre koji ne sadrie X .
Naime, oni podskupovi koji sadrie X moraju sadriavati jog t - 1 elemenata, ali ne smiju sadriavati susjede od X . Ostalo je joL n - 3 elemenata na izbor, a ograniEenje na susjednost elemenata je kao u "ravnom" zadatku. Dakle, takvih podskupova ima ( ( n - 3 ) ~ L i - 1 ) + 1 )Oni . podskupovi koji ne sadrie X moraju sadriavati jog t elemenata, izabranih medu vrhovima poligona razliEitim od X . Njih ima: ( ( n - l ) t - t S 1 ) traHenih podskupova ima:

Ukupno,

72

4. PREBROJAVANJE

32. Koliko postoji uredenih r-torki (XI,xz,. . . ,x,) s elementima iz N, svojstvom 1 xl x2 . . . x, n?

< < <

< <

, sa

Rjegenje. Kao i u jednom od ranije rijegenih zadataka, i ovdje moiemo zaieljeti svesti ovaj zadatak na prebrojavanje L'obiEnih" podskupova. Ovdje bi to preslikavanje trebalo razmaknuti elemente. PokuHajmo s: To preslikavanje pri&u~ujesvakoj uredenoj r -torci s neopadajuLim koordinatana iz N, r -Elani podskup od N,+,-i . Ono je bijekcija, jer mu je preslikavanje: ( Y I , Y Z , . . . ~ Y T )( Y I , Y Z - ~ , Y ~ - ~ , . . . , ~ T - ~ + ~ ) obostrani inverz. -Dakle, traienih r -torki ima:

?+ Pokugajte sami formulirati i rijeHiti zadatak u kojem bi se iz rjeHenja ovog zadatka zakljuEivalo neito, o rjegenjima jedndkbe yl yz . . . yk = n iz skupa N ! !

+ + +

Formula ukljuzivanja-iskljuzivanja
U prethodnom poglavlju su skupovi, Eije veliEine je trebalo izraEunati, bili zadavani kao unije disjunktnih skupova ili kao Kartezijevi produkti skupova ili su se mogli jednostavno prevesti u neku od tih kategorija. U ovom poglavlju skupovi C e biti zadavani iskljuEivo kao unije ili kao presjeci nekih skupova. Ti skupovi opCenito neCe biti disjunktni, niti C e biti u bilo kakvom pravilnom medusobnom odnosu. Osnovna ideja koja vodi k rjesenju zadataka tog tipa je rastaviti zadane skupove na podskupove koji se medusobno disjunktni. Tada se moBe primijeniti princip sume. Postoji formula koja olaksava rjesavanje takvih problema, no pokusajmo prvo rijeBiti nekoliko jednostavnih zadataka primjenom ranije nauEenih principa.

1. IstraBivanje provedeno na uzorku od 110 bolesnika s rakom na pluCima pokazalo je da je medu njima 70 pusaEa (sadaSnjih ili bivgih), 60 stanovnika gradskih podruEja, te da je 35 bolesnika zaposleno na opasnim poslovima. Nadalje, 40 puSaEa Bivi u gradu, 15 radi na opasnim poslovima, a 5 ih je u obje kategorije. 10 pacijenata s opasnim zanimanjima nije nikad puSilo niti Bivjelo u gradu. IzraEunajte:

(a) Koliko je pacijenata iz grada koji rade na opasnim poslovima?

4. PREBROJAVANJE

(b) Koliko je pacijenata iz grada koji nisu nikad puSili niti su na poslu bili izloBeni opasnim tvarima? (c) Koliko pacijenata puSi ili Eivi u gradu? (d) Koliko pacijenata niti ne pugi, niti ne Zivi u gradu niti ne radi na opasnim poslovima?
Rjes'enje 1. Ako s P oznaEimo skup svih puEiaEa, s G skup svih stanovnika grada, s R skup svih ljudi koji rade opasne poslove, a s U skup svih ispitanih bolesnika (uobiEa-

jena oznaka - univerzalni skup, skup koji obuhvata sve "interesantne" elemente za neki problem) moBemo nacrtati te skupove shematski:

Poznato nam je

(a) (b) (4 (d)

IG n pC n

l G n Rl

~~l

IGUPl n RCI. IGCn PC

IzraEunajmo prvo veliEine svakog od "osnovnih", meausobno disjunktnih skupova:

(uoEite te skupove na slici). Ako izraEunamo sve te veliEine, primjenom principa sume moCi Cemo dobiti i veliEinu bilo kojeg drugog skupa sa slike. Pronaaimo dva skupa Eije veliEine znamo, a od kojih je jedan podskup drugoga. Tada iz I R n P n G I = 5 i I P n G I =40, dobivamo I ( P n G ) \ R I = 3 5 . SliEnoiz ( P n R I = 1 5 slijedi I(P n R) \ GI = 1 0 . KombinirajuCi dalje uvjete s dobivenim rezultatima, moBemo dobiti sve veliEine potrebnih skupova, :to je pregledno prikazano slikom:

74

4. PREBROJAVANJE

Sada moZemo lako rijegiti zadatak: (a) (b) (c) (d)


IG n RI = 5 10 = 15, IG n PC n RCI= l o ,

~G~P~=20+10+35+5+10+10=90, ( G c n P c n R C= Ilo.

+f Ova metoda, koliko god je zabavna za manje probleme, nije prikladna za rjeSavanje problema s veCim brojem skupova. Naime, lako se moZe vidjeti da disjunktnih podruWa na slici s n podskupova univerzalnog skupa ima toEno 2". Ako se uzme u obzir da nas nakon raEunanja tih veliEina Eeka jog i njihovo zbrajanje, vidimo da je posla previge. No, ovom metodom se isplati raditi u sluEaju da moramo izratunati veliEine mnogo (ili Eak svih) skupova sa slike (skupova je na slici 22" . . . ).
RjeSenje 2. Pokugajmo direktno iz zadanih elemenata izraEunati tragene veliEine skupova. Rijegimo tako npr. zadatak (a).

Ostale zadatke rijegite sami.

Iarazunajte broj svih peteroznamenkastih brojeva koji:

(a) sadr3e barem jednu devetku (b) sadr3e barem jednu devetku ili barem jednu osmicu. (c) sadr3e barem jednu devetku i barem jednu osmicu
RjeSenje.

(d'Ovakav zadatak smo veC rijegili, ali ponovimo ipak rjesenje.


1
RaEunamo broj svih peteroznamenkastih brojeva kao i broj peteroznamenkastih brojeva bez ijedne devetke. Broj brojeva s barem jednom devetkom je njihova razlika. Rezultat : 9 . lo4 - 8 . g 4 .

4. PREBROJAVANJE

75

/ybj Oznatimo s U skup svih peteroznamenkastih brojeva, s D skup svih brojeva koji sadrZe devetku, a s 0 skup svih brojeva koji sadrie barem jednu osmicu. PostupajuCi
kao i u (a) dijelu zadatka koristimo formulu u kojoj su nam poznati svi elementi. Rezultat: (c) Traiimo D n 0.Znamo:

2elimo izratunati ( D n 0 1 . BuduCi d a je dobivamo

l u l=l u \ (D u o ) I+ ID(+ 1 0 1- I Dn0 1 ,


1 U I +I U \ ( D U O)I
-9

I D n0 1

= = =

. lo4 + 7 . 84 + ( 9 . lo4 - 8 . g4) + ( 9 . lo4 - 8 . g4)

+ ID1 + 1 0 1

9.10~-2.8.9~+7.8~.

9 Formulirajte i rijeLite zadatke analogne zadacima (b) i (c) u kojima se radi o brojevima koji sadrBe sedmice, osmice i devetke!
3. U jednom gradu, koji ima samo 40000 stanovnika, organiziraju se u dobrotvorne svrhe razne aktivnosti. GradonaEelnik se na kraju godine pohvalio da su odrzana dva koncerta i lutrija, te da je svaki drugi gradanin sudjelovao u dobrotvornim aktivnostima. Poznato je da je na koncertu ozbiljne glazbe bilo 2000 posjetilaca, na rockkoncertu 8000, te da je po jedan listiC lutrije kupilo 12000 stanovnika. Nadalje, poznato je da je 500 ljudi bilo na oba koncerta, da je 50 ljudi koji su bili na koncertu ozbiljne glazbe sudjelovalo u lutriji, te da je 3000 posjetilaca rock-koncerta kupilo lutriju. Nitko nije kupio vi5e od jednog listiCa lutrije. 3 to mislite o gradonacelniku?
Rjes'enje. BuduCi da nemamo razloga misliti iLta lose o gradonazelniku, jedino Bto moZemo uEitini je provjeriti da li je broj prodanih ulaznica, odn. listiCa lutrije u skladu s gradonaEelnikovom izjavom o broju gradana koji su sudjelovali u dobrotvornim aktivnostima.
OznaEimo s 0 skup svih ljudi koji su bili na koncertu ozbiljne glazbe, s R skup ljudi koji su bili na rock-koncertu, a s L skup ljudi koji su kupili listiC lutrije. Tada znamo:

1 0 1 = 2000, I R I = 8000, ILJ = 12000, 1 0 nR l = 500, 1 0nL I = 50, I R nL I = 3000, 1 0u R u L I = 20000.


Nacrtate li skicu tih skupova, lako Cete vidjeti da je:

~ouRuL~=~o~+~R~+~L~-~onR~-~onL~-~RnL

76

4. PREBROJAVANJE

UvrHtavanjem poznatih veliEina u t u jednakost, dobiva se:

1 0 n R n LI = 1550,
Hto bi trebalo znaEiti da je 0 n R n L veii od nekih svojih nadskupova (npr. od 0 n R ) . Lako se moZe vidjeti d a tome moZe biti uzrok Einjenica d a je gradonaEelnik izjavio da je u dobrotvornim akcijama sudjelovao vedi broj sudionika nego Hto ih je zaista bilo.

tP Cak i d a je gradonazelnik u sudionike uraEunao i organizatore tih aktivnosti, pretjerao je - da bi njegova. izjava mogla biti u skladu s ostalim podacima, njih bi trebalo biti najmanje 1500!
Formula ukljuzivanja-iskljuEivanja daje izraz za veliEinu unije n skupova, ako su poznate veliEine presjeka svakih i od tih skupova, i = 1,. . . ,n . Alternativnom formulom ukljuEivanja-iskljuEivanja iz istih brojeva se raEuna veliEina presjeka komplementarnih skupova. OpCenito, ako su A1, A2, . . . , A, konaEni skupovi, onda se veliEina skupa A1 U Az U . . . U A, moBe izraEunati formulom:

SliEno, ako su svi Ai podskupovi konaEnog skupa S , onda je IA;

n A; n . . . nA;l

Cesto su zbog simetrije problema svi Ai medusobno jednake veliEine, svi AinAj medusobno, svi Ai fl Aj fl Ak itd. Sumanada u I-toj sumi ima , pa prva formula prelazi u:

(7)

.. + (-1)"-lI~1 n A2 n . . . nA,I.
SliEno se moBe prilagoditi i druga formula. Formula ukljuEivanja-iskljucivanja se primjenjuje onda, kad velicine skupova A1 U . . . U A,, odnosno A T fl . . . n A : ne moBemo izraEunati jednostavnije, a vidimo naEin da se izracunaju veliEine svih presjeka skupova Ai . Usporedite prethodne zadatke s prvim dvjema formulama, a drugi zadatak i njegovu napomenu joS i s treCom.

4. PREBROJAVANJE

4. Pustinjom hoda karavana od devet deva. Na koliko naEina se nakon odmora u oazi one mogu presloziti tako d a niti jedna deva ne hoda iza one deve iza koje je hodala prije oaze?
RjeSenje. BuduCi d a devama ne znamo imena, numerirajmo ih onako kao :to su bile sloZene prije oaze. Nakon oaze druga deva ne smije hodati iza prve, treCa iza druge, itd, niti deveta iza osme. Izgleda da je jednostavnije izraEunati broj slaganja deva takvih da neki par (ili neki parovi) deva hodaju jedna iza druge - zaveZemo devu za prethodnicu i tretiramo ih kao jedan element karavane. Zato Cemo pokuLati, umjesto da raEunamo brojeve rasporeda u kojima niti druga deva ne hoda iza prve, niti treCa iza druge, itd, niti deveta iza osme, raEunati brojeve rasporeda u kojima su neki podnizovi karavane ostali vezani jog od prethodne etape - koristit Cemo formulu koja povezuje presjek skupova sa svim presjecima komplemenat a.
Uvedimo oznake - neka je S skup svih rasporeda deva, A1 skup svih rasporeda deva u kojima druga deva hoda iza prve, Az skup svih rasporeda deva u kojima treCa deva hoda iza druge, itd. A8 skup svih rasporeda deva u kojima deveta deva hoda iza osme. KonaEni cilj je izraEunati [A: n A$ n . . . n Ail . gelimo li primijeniti formulu UI, morat demo izraEunati veliEinu skupa S , te veliEine svih presjeka Ai -ova - one za koje nam se Einilo da Cemo ih lAko izraEunati i zbog kojih smo se odluEili na formulu UI. I zaista, raEun je jednostavan, pa napiLimo samo rezultate:

I S 1
\Ail ]Ai n Aj( ]Ai n Aj

9!,
8!

=
=

za svako i , za svaki par razliEitih i, j (bez obzira d a li je )i - jl = 1 ili ne), za svaku trojku razliEitih i, j, k ,

7! 6! l!

n Ak]

= =

( A l n A2 n . . . n A 8 1
Uvrgtavanjem u formulu UI:

I A ; ~ A $ ~ . . . ~ A ~ I

4.

PREBROJAVANJE

9+ ProuEite kako lete rode. Gdje nastaju problemi u analognom zadatku s rodama? RijeLite ga za n = 3 i n = 5 .
e se naEina moCi presloiiti deve na? + Razmislite kako bi se izraEunalo na koliko C kon druge oaze, ako niti jedna od njih ne ieli hodati iza neke od deva iza kojih je na tom putu vet hodala? Zadatak je bitno teii od LLoriginalnog" zadatka.

5. 2n ljudi ruEalo je za okruglim stolom. Oni Zele 'za istim stolom i veEerati, ali tako da nitko ne gleda sebi nasuprot istu osobu koju je gledao za vrijeme ruEka. Bitno je tko sjedi na kojem stolcu.
RjeBenje. Ponovno je lako izraEunati brojeve razmjeztaja s fiksiranim parom odn. parovima koji se iele gledati.
OznaEimo stoga parove ljudi koji su se gledali za vrijeme rutka s 1,2,. . . ,n te je ponovno S skup svih rasporeda oko okruglog stola, Ai skup svih rasporeda u kojima ljudi i-tog para ponovno sjede jedan nasuprot drugom. Sada raEunamo:

I S 1

[Ail

lA;nAjI

(2n)! 2 n . ( 2 n - 2)! jer prva osoba iz para izabire mjesto na 2 n naEina, a druga nema ;to birati 2n.(2n-2).(2n-4)!

I A 1 n A 2 n . . . nA2,( = 2 n e ( 2 n - 2 ) . ... .2. Uvrgtavanjem u FUI i sredivanjem dobivenog izraza zatadak de biti rijeSen: IAy

n A$ n . . . n A$,I

W Kad se veC radi o sjedanju za stol, moiete rijeZiti i sljedeCi zadatak: na koliko naEina moie 2 n ljudi veEerati za istim stolom za kojim su ruEali, ako niti jedna osoba ne ieli imati pokraj sebe niti jednu osobu od onih koje su kraj nje sjedile za ruckom? PokuZajte prvo rije6iti ovaj zadatak "pjeSicen za neke male n .
Usporedite rjegenje ovog zadatka s rjeienjem zadatka o devama-.

W Gdje nastaju problemi, ako zadatak postavimo ovako: na koliko naEina mogu ljudi iz ovog zadatka veEerati, ako niti jedna osoba ne ieli za veEerom sjediti u istom druZtvu kao za ruEkom (tj. barem jedna od osoba koje sjede pokraj nje mora biti razliEita)? ,RijeSite ovaj zadatak za n = 3 .

4.

PREBROJAVANJE

6. Koliko ima brojeva od 1 do 1000 koji su djeljivi s 3 a nisu djeljivi niti s jednim od brojeva 2, 5 i 7?
Rjeienje. Brojeva manjih ili jednakih 1000 djeljivih s 3 ima [1000/3] (najveCe cijelo od z). OpCenito, lako je izraEunati broj brojeva djeljivih proizvoljnim brojem. Lako je i izraEunati broj brojeva koji nisu djeljivi proizvoljnim brojem, ali u ovom zadatku problem je u tome Sto treba izraEunati broj brojeva koji nisu djeljivi s viSe brojeva istovremeno. PokuSajte sami vidjeti gdje nastaju poteSkote u direktnim pokuSajima prebrojavanja. Mi Cemo primjenom formule UI izraEunati broj brojeva manjih od 1000 (manjih ili jednakih, svejedno je.. . ) koji su djeljivi s 3, ali nisu djeljivi s 2,5 i 7 brojeCi brojeve djeljive tim brojevima.

OznaEimo s S skup svih prirodnih brojeva manjih od 1000 djeljivih s 3, s A2 c S skup brojeva djeljivih s 2 (odnosno s 2 . 3 ) , s A5 C S skup brojeva djeljivih s 5 (s 5 . 3 ) , a s A7 C S skup brojeva djeljivih sa 7 (sa 7 . 3 ) . Da bismo rijeLili zadatak, trebamo izraEunati IAg n A; n AFl. VeC smo u uvodu odluEili da Cemo koristiti formulu UI, pa izraEunajmo sada potrebne veliEine.

I"'
14421 lA51
1-42

= = = =

[ E l
[a1 [a1 [a] [rn] [[ %TI m l

1 4 1 =
IAznA71 IA5nA71 IA2 n A5 n A7 1
= = =

Uvrgtavanjem u formulu UI dobiva se rezultat:

m je uoEila da medu kupci7 . Uprava robne kude u j e d n o ~ r i s t i ~ k omjestu ma ima stranih turista i odluEila poslati osoblje na teEajeve stranih jezika. Svaki prodavaE treba upisati teEaj jednog jezika, a cilj je da za svaki od 9 za to mjesto bitnih jezika postoji prodavaE u robnoj kudi koji govori tim jezikom. Na koliko naEina se moZe poslati p prodavaEa te robne kude na tecajeve?
RjeSenje. Kad bi prodavaEi mogli slobodno birati, bilo bi gp raznih izbora teEajeva. No tada bi se moglo dogoditi da neki od bitnih jezika ne bude zastupljen u toj robnoj kuCi. Nas zanimaju samo takvi izbori u kojima su zastupljeni svi jezici. UoEavamo da je lako izratunati broj izbora teEajeva pri kojima neki od jezika nisu zastupljeni, pa je to razlog za odluku d a se krene formulom ukljuEivanja-iskljuEivanja.

OznaEimo jezike brojevima od 1 do 9, te s Ii skup izbora teEajeva pri kojima nije zastupljen jezik i . Neka je I skup svih izbora teEajeva. Trebamo izraEunati veliEinu skupa If n I; n . . . nI6 . VeC smo odluEili d a Cemo krenuti formulom UI, pa izraEunajmo

4. PREBROJAVANJE

potrebne elemente:

1 1 1 lIil

=
=

IIinIj)

gp 8p, 7p,

...

za svako i zasve i , j , i # j

11~n1~n...n1~ =1 oP
Ukupni broj dobrill izbora je jednak

Ctt Provjerite d a je za p < 9 rezultat jednak nuli. Koliki je rezultat za p = 9 ? Obrazloiite zagto se to dogodilo (znanje deriviranja moie koristiti).

9+ Izbore stranih jezika mogli smo prikazati i kao surjekcije sa skupa svih prodavaEa u skup devet izabranih stranih jezika.
IzraEunajte koliko ima surjekcija s n-Elanog na m-Elani skup.

9 Da su se prodavaEi prvo organizirali u 9 grupa koje iele zajedno pohadati teEajeve, onda im se moglo na 9! naEina pridruiiti strane jezike.
IzraEunajte broj particija n-Elanog skupa u m dijelova. Dijelovi particije nisu nikad prazni skupovi.

9+ U nekoj drugoj robnoj kuCi uprava je bila lukavija, pa se odmah sjetila d a prodavaEi rade u dvije smjene i da svaki jezik trebaju znati barem po dva prodavaEa.
IzraEunajte broj naEina izbora stranih jezika i u toj robnoj kuti. Moiete raditi sa skupovima Ii svih izbora u kojima jezik i ne uEe barem dva prodavaEa. Tada je formula nalik na originalnu formulu UI, ali je teie raEunati veliEine skupova i presjeka. Jednostavnije je raditi sa skupovima N; svih izbora u kojima jezik i ne uEi niti jedan prodavaE i skupovima J; svih izbora u kojima jezik i uEi toEno jedan prodavaE. Tu su mnogi presjeci jednaki praznom skupu!

8. UEenici jednog razreda trebaju pregledati svoje domaCe zadaCe iz matematike. Sve zadaCe stave se na jedan kup, promijesaju i podjele uEenicima na pregledavanje. U razredu je 30 uEenika, a od toga dvoje iz matematike ima ocjenu nedovoljan. Na koliko naEina se mogu podijeliti zadaCe. . .

ako svi nedovoljni trebaju dobiti na pregledavanje upravo svoje zadaCe? ako nedovoljni ne smiju dobiti na pregledavanje svoje zadaCe? ako niti jedan uEenik ne smije dobiti na pregledavanje svoju zadaCu? Rjes'enje. Dijeljenja domakih zadata odgovaraju preslikavanjima sa skupa svih uEenika tog razreda na njega samog: svakom uEeniku pridruiimo onog uEenika koji C e pregledavati njegovu zadatu. Dakle brojimo permutacije skupa uEenika tog razreda koje zadovoljavaju dodatna zadana svojstva.
(a) Nedovoljne preslikamo svakog u njega samog, a ostalih 30-2 bilo kako. Rezultat: (30 - 2)!.

4. PREBROJAVANJE

(b) Upravo smo vidjeli d a lako moiemo izraEunati brojeve permutacija razreda koje neke unaprijed zadane uEenike preslikava u njih same. Ako je k takvih uEenika, broj permutacija je jednak (30 - k ) ! . Izrazimo onda broj permutacija u kojima niti jedan od nedovoljnih ne pregledava svoju zadaCu pomoCu brojeva permutacija skupova u kojima neki od nedovoljnih ispravljaju svoje zadaCe. OznaEimo uEenike s nedovoljnom ocjenom brojevima 1 i 2. Neka je R skup svih permutacija uEenika promatranog razreda, a za i {1,2) neka je Ni skup svih permutacija u kojima se i-ti nedovoljni preslikava u samog sebe. Traiimo veliEinu skupa Nf n N; . Prema formuli ukljuEivanja-iskljuzivanja imamo

INinN2I

= =

IRI-INII-IN~I+IN~~N~I
30! - 2(30 - l)!

+ (30 - 2)!.

(c) Postupamo analogno prethodnom rjeienju. OznaEimo s Ri skup svih permutacija razreda koji i -tog uEenika (po abecedi, na primjer) preslikava u sebe samog. Traiimo IRE n R$ n . . . n Rso I . Formula UI daje

.. . +

U R2 U

. . . U R301.

Za svaki k 2 0 svi presjeci k skupova Ri su medusobno jednake veliEine i jednaki . Uvritavanjem tih veliEina u pret(30 - k ) ! . Za prizvoljni k takvih presjeka ima hodnu formulu dobivamo konaEni rezultat

(to)

4-t OpiSite rijeEima presjek nekih k skupova Ri .


+f' Elemente domene koji se nekom permutacijom preslikaju sami u sebe zovemo fiksnim totkama te permutacije.
Upravo smo prebrojali permutacije skupa uCenika sa zadane dvije fiksne toEke (ostale su mogle, ali nisu morale biti fiksne), zatim permutacije kojima zadane dvije toEke nisu fiksne, te na kraju permutacije bez .fiksnih toEaka.

9. Na koliko se naEina mo5e 2mn predmeta, po 2n svake vrste, podijeliti u dvije jednakobrojne skupine.
Rjeienje. UoEimo prvo d a je dovoljno iz zadane skupine predmeta izdvojiti skupinu od m n predmeta.
Ako nisu ograniEeni izvori svake od vrsta predmeta, broj naEina izbora m n predmeta iz m skupina je jednak (m",-",-"'+l) . No, medu tim izborima postoje i oni koji imaju vise od 2n primjeraka nekog od predmeta. Za fiksnu vrstu predmeta postoji ( m n - ( 2 ~ ~ ~ ) ) l)- m izbora f m n predmeta iz m skupina s viie od 2n izabranih primjeraka te vrste. Vrstu se moie izabrati na ( y ) naEina. Tako od ukupnog broja naEina izbora moramo oduzeti ove "loEe" izbore. No, ako to napravimo, vidimo d a smo dvaput oduzeli one izbore koji imaju dvije skupine predmeta iz kojih je uzeto viie od 2n komada. Tako t e izbore sada moramo dodati, itd.

4. PREBROJAVANJE

Optenito, broj naEina izbora m n predmeta iz m skupina, ako se iz k skupina uzima viBe od 2n predmeta, iznosi ( m n - k ( ~ ~ : ) - m f l) . Broj "dobrih" izbora iznosi: ;m :( ; n
1)

):(

(mn

- (2~+-1; - m + 1

) +...

% Formalno, ova suma je beskonaEna. Medutim, lako je uoEiti d a je samo konaEni broj binomnih koeficijenata ( m n - k ( ~ ~ : ) - m + l ) razliEtih od nule. Za koje k su pribrojnici razliEiti od nule?
% ZapiSite precizno skupove o kojima bi se radilo u formuli UI.

9-t Preuredite ovaj rezultat tako da dobijete . . . ,z,) E N r za koje je z l + 2 2 + . . . + s, = n .

broj svih uredenih m-torki (xl, 2 2 ,

10. Koliko prirodnih brojeva n manjih ili jednakih lo6 su potencije nekog prirodnog broja, n = mk za prirodne brojeve m i k , k > 1?
Rjes'enje. Broj brojeva manjih ili jednakih lo6 koji su potencije toEno odredenog prirodnog broja m lako je izraEunati - ima ih [log, lo6]. Ovdje je problem Bto neki brojevi mogu biti potencije razlieitih brojeva. Ti brojevi su tada redovito Elanovi istog niza potencija nekog broja mk , za k E N . Kad bismo jednostavno mogli odrediti koji se tu disjunktni nizovi pojavljuju, zadatak bi bio rijeSen - samo bismo zbrojili njihove veliEine.
Bududi da to ne moiemo (jednostavno) napraviti, promatrajmo sve skupove potenA , cija prirodnih brojeva, A, := {mk1 k E N, m k E N106). ielimo izraEunati jer su sve ostalo prazni skupovi). VeliEina skupa Al je (unija sigurno ne ide preko jednaka 1, velieine ostalih skupova su lA,I = [log, lo6]. No, veC pri raEunanju presjeka parova skupova, vidimo prve probleme - naime Ai n Aj je neprazan samo ako su i i j potencije istog broja. Tako vidimo da se polako poEinjemo vrtjeti u krugu.

m,

UZzl

Druga moguCnost je promatrati skupove Bk := {mk, m E N, mk E N106}. VeliEine Pri raEunanju presjeka, ponovno tih skupova moiemo lako izraEunati: lBkl = nastaju problemi: vidimo na primjer da su sve Eetvrte, Beste itd. potencije veC sadriane u skupu svih kvadrata. OpCenito, vidimo da je svaka potencija moie zapisati kao potencija s prostim eksponentom, pa nam trebaju samo oni Bk-ovi koji kao indeks k jmaju neki prosti broj. Maksimalni k odredimo kao [log2 lo6] = 1 9 , jer C e najvedi eksponent biti dostignut na najmanjoj bazi, bazi 2. MoguCi indeksi su tada elementi skupa {2,3,5,7,11,13,17,19). Presjeci skupova su Bi n B j = Bij , pa im takoder znamo odrediti veliEinu. Svaki par skupova irna u presjeku element 1, a samo oni kojima je 2ij < lo6 ( i j 19) imaju jog neki element. Parovi indeksa za koje je presjek vedi od jednoElanog skupa SU: (2,3), (2,5), (2,7) i (3,5) . Ne postoji niti jedna trojka skupova koja bi imala presjek vedi od jednoElanog ikupa.

[m]

<

Da bismo se rijeBili presjeka veliEine 1, prijedimo na skupove B i := Bk \ (1). IzraEunat Cemo veliEinu unije tih skupova i dobivenom broju dodati 1.

4. PREBROJAVANJE

Uvriitavanjem veliEina koje smo ranije izraEunali u formulu UI dobivamo da potencija veCih od jedan a manjih ili jednakih lo6 ima:

I ~ b l +PSI + I ~ b l +lBfl+ l ~ i i I + I ~ i s l +l ~ i 7 + l I~isl


- 1 ~ 4 . 3 1- 1 ~ h . 5 1- 1 ~ 4 . 7 1- IB3.51
=999+99+14+2+1+1+1-9-2-1-1

= 1110.
Ukupni broj trazenih potencija iznosi 1111.

11.

Jedan od dokaza formule ukljuEivanja-iskljuEivanja,

zasnovan je na brojanju pojava pojedinih elemenata u skupovima koji se spominju u formuli. Naime, na lijevoj strani se svaki element koji se nalazi u bilo kojem od Ai -ova broji jednom. Provjerimo da se svaki element i na desnoj strani broji samo jednom. Ako se neki element pojavljuje u toEno k Ai-ova, onda se k puta pojavljuje u <, IAi n Ajl itd, te toEno jednom u sumi velilAil, ( : ) puta u 6 n a presjeka od po k skupova. d$upno, taj element se na desnoj strani brojao

xlCiCn

puta. To vrijedi bez obzira na izbor k , pa formulu mogemo smatrati provjerenom. Dokagite sliEne relacije: (a) Za svaki n E N i za sve a l , . . . a n E R : max{al, az, . . . , a,) =

C(-l)j-'
j=1
JCNn,

min{ai : i E J).

I JI=j

(b) Zasvaki n E N i z a s v e a l l . . .an E N :

Rjezenje.
(a) Da li se zaista svaki broj osim maksimalnog na desnoj strani poniiitava?

84

4. PREBROJAVANJE

Maksimalni element pojavljuje se samo jednom kao minimalni element nekog skupa. Taj skup se sastoji samo od njega samog, tj. veliEina mu je jedan, tako da se t a j minimum pojavljuje s pozitivnim predznakom. Promatrajmo element koji nije maksimalan, nego od njega postoji k veCih ( 0 < k < n ) . Tako se promatrani element pojavljuje kao minimum svih skupova koji sadrie njega i neke od uoEenih k veCih elemenata: jednom kao minimum jednoElanog skupa, k puta kao minimum dvoElanog skupa, ( : ) puta kao minimum troElanog skupa itd, te na kraju jednom kao minimum k 1-Elanog skupa. Ukupni njegov doprinos sumi na desnoj strani stoga iznosi:

Sto smo i ieljeli pokazati. (b) I ovaj dio zadatka moiete dokazati sliEnim razmatranjem kao ranije. Jednostavnije je na svaki od eksponenata prostih faktora koji se pojavljuju u rastavu broja V(al, a 2 , . . . , a n ) primijeniti prethodni dio zadatka!

Rekurzivne relacije
Primjeri rekurzivnih relacija
Ponekad pri konstruiranju objekata koji nekako ovise o parametru n E N postupamo tako, da konstruiramo prvo familiju objekata za n = 1, zatim za n = 2 i postupno dalje za 3 , 4 , . . . . Pritom je Eesto moguCe uoCiti pravilo kako iz familije objekata za neki n konstruirati familiju objekata za sljedetu vrijednost parametra, n 1. Posebno Eesto dolazimo do takvih konstrukcija onda, kada smo prisiljeni traZiti sistem umjesto navoditi objekte bez ikakvog reda, a to je pri pisanju programa za generiranje objekata. UoCeni postupak moZemo iskoristiti za prebrojavanje. MatematiEka indukcija ovdje ima manje znaEenje, jer pretpostavlja poznavanje onog svojstva objekata koje se dokazuje. Ovdje se radi o izvodenju pravila, odnosno broja objekata. Neka je definiran nix brojeva (a,). Tada relcurzivnom relacijom zovemo svaku relaciju koja elemente niza izraZava pomoCu njihovih prethodnika (elemenata s manjim vrijednostima indeksa). Rekurzivna relacija moZe elemente niza izraZavati uvijek pomoCu fiksnog broja p&thodnika (za neki fiksni k E N proizvoljni element niza a, izraZava pomoCu a,-1,. . . ,a,-k ) ili pomoCu svih njegovih prethodnika. U prvom sluEaju govorimo o rekurzivnim relacijama konaEne, u drugom o rekurzivnim relacijama beskonaEne prosZosti. Da bi se rekurzivna relacija mogla koristiti za raEunanje elemenata niza, potrebno je imati poEetne uvjete - vrijednosti elemenata niza za neke vrijednosti indeksa. To su najEeSCe prvi elementi niza. Za rekurzivne relacije konaEne proSlosti uzimamo obiCno prvih k vrijednosti niza, npr. a l , az, . . . , ak , a za rekurzivne relacije beskonacne proiilosti prvi element niza. PoCetni uvjeti se ili zadaju ili su dostupni jednostavnim raEunom. U sljedebim zadacima traZit Cemo rekurzivne relacije za broj raznih tipova objekata. U jednostavnijim sluEajevima izraEunat Cemo i njihov broj. InaEe ih,

86

5. REKURZIVNE RELACIJE

do sljedekeg poglavlja, moBete iskoristiti za racunanje, odnosno pisanje programa koji raEuna Beljene brojeve. Kad vidite nix brojeva, moZda uoEite pravilnost koja se u njemu pojavljuje i naslutite formulu za opCi Elan niza. Nju se lako provjeri uvrStavanjem u rekurzivnu relaciju (to je onda dokaz u kojem skriveno leBi matematiEka indukcija).

1. Neka je za prirodni broj n s p, oznaCen broj permutacija n-Elanog skupa. Nadite rekurzivnu relaciju koja izraBava p, pomoku p,-1 . IzraCunajte pomoCu nje p n .
Rjeienje. Primijetimo prvo da su brojevi pn dobro zadani, jer broj permutacija skupa ne ovisi o tipu njegovih objekata, nego samo o njihovom broju. DokaZite sami da, ako je poznata bijekcija izmedu neka dva skupa (tj. oni su jednakobrojni), onda postoji i bijekcija izmedu skupova njihovih permutacija.

Uzmimo kao n-Elani skup N n . Sve njegove permutacije moZemo dobiti tako, d a prvo promatramo sve permutacije skupa N,-i, a zdesna svakoj od njih pripigemo element n . Time smo dobili sve permutacije koje zavrLavaju elementom n . Permutacije koje zavrLavaju elementom n - 1 dobivamo tako, da promatramo sve permutacije skupa Nn \ {n - 1) i zdesna im pripiLemo element n - 1. Na isti naEin dolazimo i do ostalih permutacija - onih koje zavrgavaju elementima n - 2, n - 3,. . . , 2 , 1 . Iz ove konstrukcije vidimo da permutacija skupa Nn ima n puta viLe nego permutacija skupa Nn-l . Dakle imamo pn = npn-1. Dobili smo rekurzivnu relaciju za niz (p,). Znamo li d a je pl = 1 (poEetni uvjet), moiemo raEunati ili napisati program i pustiti ga da raEuna: pa = 2 . pl = 2 , p3 = 3 . p2 = 6 itd. No nama je zadatak iz ove rekurzivne relacije nadi formulu za opdi Elan niza pn . Za ovu posebnu, vrlo jednostavnu rekurzivnu relaciju moZemo provesti trik, koji se u svojim razliEitim oblicima Eesto koristi. NapiLimo naLu rekurzivnu relaciju za sve k od 1 do nekog proizvoljnog n :

Pomnoiimo li sve lijeve strane medusobno i sve desne strane medusobno, dobivamo jednakost p n . p n - l . . . . . p 2 = n ( n - 1 ) . . . . . 2 . p n - l . ..:pg.pl, i nakon kradenja pn = n!, Hto smo i oEekivali.

5. REKURZIVNE RELACIJE

87

.pf&vacau ravnini?

2.4 Koliko neomedenih podruzja u ravnini odreduje n medusobno neparalelnih

Rjegenje. Jedan pravac dijeli ravninu na dva podruqa - oba neomedena. Dva pravca dijele ravninu na Eetiri podruEja - sva Eetiri neomedena. Dodamo li im i treCi pravac, on dijeli dva neomedena podruEja u po dva dijela, pa se broj neomedenih podruEja povedava za dva. Nasuprot tome, broj omedenih podruiija se moZda, u ovisnosti o poloiaju treteg pravca u odnosu na prva dva, poveCava za jedan. Nacrtajte skicu. Pogledajte Lto se dogada dodavanjem Eetvrtog pravca. OpCenito novi pravac uvijek dijeli po dva "stara" neomedena podruiija stvarajuki dva nova neomedena podruEja odredena njegovim prvim i zadnjim presjekom s vet postojeCim pravcima. Ako s pn oznaEimo broj neomedenih podruiija koja su odredena s n pravaca, onda dobivamo rekurzivnu relaciju

pn = pn-1 2. PoEetni uvjet je pl = 2. Ovo je jednostavna rekurzivna relacija, pa je moiemo rije'siti. RaEunanjem dobivamo pn = 4 , ps = 6 , p4 = 8 . MoZemo naslutiti da je pn = 2 n . I zaista, ovaj izraz za pn zadovoljava i poEetni uvjet i rekurzivnu relaciju ( 2n = 2(n - 1) 2 ) za svaki n E N , pa je njeno rjeLenje. 9-t Ova rekurzivna relacija moie se rijegiti i trikom opisanim u prethodnom zadatku - umjesto mnoienja relacija, ovdje ih treba zbrajati. Provedite t a j postupak! +f Nacrtajmo kruInicu tako da sva sjecigta nacrtanih pravaca leie unutar nje. Tad a su neomedena podruEja ona podruEja koja kruZnica presijeca. To znaEi d a kruZnica presijeca upravo one polupravce koji su granice neomedenih podruiija. UoEimo na crteiu jedan neomedeni kraj jednog pravca (npr. onaj koji gleda na sjever) i oznaEimo t a j pravac brojem 1, sljedeCi (npr. u pozitivnoj orijentaciji) brojem 2, itd. Tada moiemo, kruieCi od sjevernog pravca u pozitivnog smjeru podruEja oznaEiti kao "podruiije iza I", "iza 2",. . . ,"iza n " , pa opet (radi se sada o drugim krajevima pravca) "iza I", "iza 2",. . . ,"iza n ". Iz toga se odmah vidi da ima 2n neomedenih podruEja. Da je redoslijed presjeka zaista ovakav vidi se najjednostavnije ako smjestimo u istu ravninu koordinatni sustav sa ishodistern u sredigtu kruinice (uvjerite se u to sami!). 9+ Rijegite analogni zadatak s ravninama u prostoru umjesto pravaca u ravnini.

3. Neka je za prirodni broj n s f n oznaEen broj poploFavanja n x 1 pruge ploEama dimenzija 1 x 1 i 1x 2 . Nadite rekurzivnu relaciju za niz ( f , ) . Nadite i dovoljno poEetnih uvjeta da bi se dobivena rekurzivna relacija mogla iskoristiti za raEunanje.
Rjes'enje. Ako poEnemo poploEavati prugu, moiemo to uEiniti ili jednom malom ili jednom velikom ploEom.

Ako poEnemo malom ploEom, onda nam ostaje za poploEiti jog pruga duljine n - 1. Broj njenih poploEavanja je jednak fn-l . Ako poEnemo ploEom duljine 2, onda trebamo

5. REKURZIVNE RELACIJE

poploEiti joB prugu duljine n - 2 . To moiemo uEiniti na fn-2 broj poploEavanja pruge duljine n jednak

naEina. Zato je ukupni

Da bismo ovom rekurzivnom relacijom mogli raEunati elemente niza (f n ) , potrebno je zadati dva poEetna uvjeta. Lako se vidi d a mora biti f l = 1 (jedino poploEavanje sastoji se od jedne male ploEe) i f2 = 2 (dvije male ili jedna velika). Sada moiemo dalje raEunati: f3 = 2 1 = 3 , f4 = 3 2 = 5 itd.

+f Moiemo izraEunati i fa : 2 = 1+ f a , pa je fo = 1. To se moie i interpretirati: postoji samo jedno poploEavanje pruge duljine 0, a to je ono koje ne sadrii niti jednu ploEicu. Ponekad je zgodno "spustiti" se do 0-tog Elana niza i njega iskoristiti kao poEetni uvjet (npr. u sljedetem poglavlju pri raEunanju elemenata niza).

4-t Brojevi fn zovu se Fibonaccijevi brojevi. Izvorno su se pojavili u vezi s razmnoiavanjem zeEeva. Na poEetku imamo par zeEeva: zeca i zeEicu. Kad navr'se dva mjeseca, dobiju prvi put par mladih - ponovno zeca i zeticu. Nakon toga svaki mjesec dobivaju ponovno po par mladih, a i mladi se nakon Bto navrLe dva mjeseca poEinju razmnoiavati na isti naEin. Pretpostavlja se da zeEevi u promatranom razdoblju ne ugibaju. Pratite broj parova zeEeva kroz prvih desetak mjeseci. Moiete nacrtati i njihovo rodoslovno stablo naznaEujuti uz svaki par zeEeva mjesec u kojem su se rodili. UoEite vezu s poploEavanjem iz zadatka.

4-t Cesto se koriste i "pomaknuti" Fibonaccijevi brojevi Fn := f n - i . Uvjerite se da oni zadovoljavaju istu rekurzivnu relaciju kao i brojevi f n - razlika je samo u poEetnim uvjetima.
4-t Na primjeru problema poploEavanja dokaiite da je f n = Ck>o ( n ; k ) :. Uvjerite se zatim i direktnim raEunom da ove sume zadovoljavaju rekurzivnu relacqu za Fibonaccijeve brojeve.
UoEite na kraju na Pascalovom trokutu koji su se binomni koeficijenti nalazili u sumi. UEinite to za neki n , zatim za n - 1 i n - 2 , te se uvjerite jog jednom da za te sume vrijedi ista rekurzija kao i za Fibonaccijeve brojeve.

4. Promotrimo sljedeke poploEavanje: prvo stavimo na st01 jednu ploEicu domina. U drugom koraku stavimo oko prve ploCice joS odgovarajudi broj ploEica domina, tako da se nove ploEice naslanjaju na staru i da je sasvim okrufuju. U tredem koraku okruBimo lik dobiven u drugom koraku. Na isti naEin nastavljamo i u sljedekim koracima. OznaEimo s dn broj ploEica domina koje se nalaze u liku nakon n-tog koraka. Nadite rekurzivnu relaciju za niz (d,) i izraEunajte iz nje dn .
Rjes'enje. Upoznajmo se prvo sa zadanim naEinom slaganja domina - slijedeti upute napravimo prvih nekoliko koraka.

5. REKURZIVNE RELACIJE

Vidimo d a je d l = 1. Prvi lik kojeg treba obrubiti je 1 x 2 pravokutnik. U drugom koraku dodali smo 5 ploEica domina. Njihova (gornja) povrSina je trebala biti jednaka opsegu prvog lika uveCanom za 4 jer trebamo poploEiti i Eetiri kvadratika u kutevima koji nemaju zajedniEku duZinu s prvim likom. Dakle broj novih ploEica u drugom koraku je pl = (6 4)/2 = 5 ( p kao prsten). Time smo dobili 3 x 4 pravokutnik opsega 14. Zato C e broj novih domina potrebnih za provodenje treCeg koraka biti pa = (14 4)/2 = 9 .

NasluCujemo da C e pravokutnik kojeg de trebati obrubiti u n-tom koraku biti (2n - 3) x (2n - 2). Dokaiite t u tvrdnju matematickom indukcijom. Opseg tog lika je jednak 8 n - 1 0 , pa je broj novih domina potrebnih za provodenje n-tog koraka jednak (8n - 10 4)/2 = 4n - 3 . Za ukupni broj ploEica u liku vrijedi

Zbrajanjem ovih relacija za vrijednosti indeksa od 2 do n , dobivamo nakon kraCenja elemenata niza koji se pojavljuju na lijevoj i na desnoj strani
n

d n = d l +)(4kk=2

3),

iz Eega dalje sredivanjem slijedi rezultat

+f Rekurzivna relacija C e vrijediti i za n = 1 ako definiramo do := 0 .

9 Pogledajte rezultat i objasnite zagto smo ovakav rezultat i oEekivali (presloZite domina u istom liku).

9 Usporedite rekurzivne relacije i rjeLenja ovog zadatka sa zadatkom o neomedenim podruEjima u ravnini. Obratite paZnju na stupnjeve polinoma u varijabli n koji se pojavljuju u relacijama i rjegenjima.
5. DokaZite da je broj uredenih trojki s medusobno razliEitim elementima iz n-Elanog skupa jednak ukupnom broju kamenih blokova u tri piramide visine n - 2 , Tlocrt baze piramide visine k je k x (k 1) pravokutnik, a nacrt i bokocrt su kao na slici. Piramide su iznutra popunjene kamenim blokovima.

90

5. REKURZIVNE RELACIJE

Rjegenje. Neka je bk broj kamenih blokova u zadanim trima piramidama visine k . OdluEimo li od njih napraviti piramide visine k 1, moramo za svaku od njih izgraditi temelje (k 1) x (k 2) i postojete piramide preseliti na nove temelje. Zato za broj kamenih blokova vrijedi rekurzija

k 3(k l)(k bk+l = b Za broj uredenih trojki elemenata iz npr. Nn vrijedi

+ + +

+ 2).

tn+l = tn 3n(n - I), jer su uredene trojke skupa Nn+l ili ureCtene trojke skupa Nn ili sadrEavaju element n + 1 na jednoj od tri pozicije i dva medusobno razlieita elementa skupa Nn na ostalima. Dobivene rekurzivne relacije imaju slieni oblik, ali nisu sasvim jednake. Ipak moZemo iz njih naslutiti d a je tn = bn-2. Pogledajmo prvih nekoliko vrijednosti svakog od nizova. ti = O t2 = 0 tj =6 t4=24 bl = 6 b2 = 24 b3 = 60

...

Posebno imamo t3 = bl . Iz rekurzivnih relacija vidimo da, ako za neki prirodni broj n vrijedi tn = bn-2, onda je i t n t l = bn-2+i ( t n + i = tn 3n(n - 1) = bn-2 3(n - 2 2)(n - 2 1) = bndztl ). Time je dokaz zavrlien jer prema principu matematieke indukcije znamo da je onda svaki tn jednak broju blokova bn-2.

Formulirajte i rijeHite sliEni zadatak za parove.

Ctt Napizite broj karnenih blokova u piramidi kao sumu broja blokova na pojedinim visinama. Objasnite kako bi trebalo brojati uredene trojke d a bi se rezultat dobio u tom obliku.

9
6.

Generalizirajte sada zadatak na proizvoljne t-torke.

Za broj permutacija skupa vrijedi rekurzija: n! = ( n - l ) [ ( n- I)! ( n - 2)!]. Za broj permutacija bez fiksnih toEka vrijedi ista t a rekurzija: D, = (n - l)[Dn-1 Dn-21. No poznato je da sve permutacije nisu permutacije bez fiksnih toEaka, pa permutacija bez fiksnih toEaka ima manje nego svih permutacija. DokaBite svaku od ovih rekurzivnih relacija i objasnite za6to se ovim rekurzivnim relacijama dobivaju razliEiti brojevi.

Rjes'enje. DokaZimo prvo prvu rekurziju. Faktor n - 1 mogao bi potjecati od smjeztavanja prvog elementa. Krenimo tim putem. Promatrajmo permutacije skupa Nn . Permutacija moZe element 1 preslikati ili u neki drugi element skupa Nn ili u samog sebe. Ako se jedinica ne preslikava sama u sebe, onda ima n - 1 mogutnosti izbora za njenu sliku, a ostale elemente se moEe preslikati na (n - I)! naeina. Ako se jedinica preslikava sama u sebe, onda se dvojka moIe preslikati u neki od n-1 preostalih elemenata, a ostalim elementima se slika bira na (n-2)! naEina iz skupa {2,3,. . . ,n) \ f (2) . Tako dobivamo rekurziju za faktorijele (brojeve permutacija):
n! = ( n - l ) ( n - I)!

+ (n-

l ) ( n - 2)!.

5. REKURZIVNE RELACIJE

SliEnim razmatranjem dobiva se i druga rekurzija. Element 1 se mora preslikati u neki od preostalih elemenata. To daje n - 1 moguknosti izbora. Ako se slika elementa 1, f ( l ) , istom tom permutacijom preslikava u element 1, onda je broj odgovarajukih permutacija bez fiksnih toEaka jednak Dn-2. InaEe moramo preslikati skup {2,3,. . . , f ( 1 ) ,. . . ,n ) bijektivno na skup {2,3,. . . , I , . . . ,n ) tako d a ne bude niti f ( i ) = i za i # f ( 1 ), i 2 2 niti f ( 1 ) = 1 . Uz identifibciju f ( i )* i radi se ponovno o permutacijama bez fiksnih toEaka i ima ih Dn-l . Tako smo dobili i drugu rekurzivnu relaciju

D, = (n- l)(Dn-2

+ Dn-1).

Ostalo nam je joi obrazloiiti zaito su rjeienja ovih dviju rekurzivnuh relacija razliEita. Ako su rekurzije za Elanove dvaju nizova jednake, onda se jedina iansa da se oni medusobno razlikuju nalazi u poEetnim uvjetima.

D1=0 D2=l D3=2 D4 = 10 Vidimo d a se poEetni Elanovi niza razlikuju, pa to onda znaEi i da se Elanovi niza medusobno razlikuju ( n! # Dn za svako n E N ).

1!=1 2!=2 3!=6 4! = 24

+f' Ostaje joi otvoreno pitanje: jesu li moida, kao u prethodnom zadatku, Elanovi ovih nizova jednaki do na neki pomak u indeksima? Pretpostavimo d a je to istina i da za neka dva uzastopna Elana niza ( D n ) vrijedi Dn = ( n - k ) !, Dn+l = ( n- k I ) ! . Tada se iz rekurzivnih relacija vidi d a je D n f 2 # ( n- k 2)!.

9 Dokaiite korigtenjem ovih rekurzivnih relacija inaEe oEiglednu Einjenicu: n! > Dn , za svaki n E N .

Tehnika r-j e Eavanja


Linearne homogene rekurzivne relacije s konstantnim koeficijentima su relacije oblika gdje su r E N i ci E R fiksni brojevi, a (ai) neki niz brojeva (neCe nam smetati niti kompleksni brojevi). Za ovu relaciju kaBe se jog da je reda r . Linearna je zato jer se Elanovi nepoznatog niza (ai) javljaju u jednadBbi u obliku linearne kombinacije (mnoBeni s konstantnim koeficijentima (ci) ). Homogena je zato gto se s desne strane nalazi broj nula. Rijegiti rekurzivnu relaciju znaEi naCi skup svih nizova koji je zadovoljavaju. Za njeno rjegavanje potrebno je prvo rijegiti njenu karalcteristic'nu jednacE3u xT-l . . . cn = 0. c,+,xr cn+,-1 Ako su xl,x2,. . . x, rjegenja te jednadEbe, te ako su svi medusobno razliEiti, onda se sva rjegenja polazne rekurzivne relacije mogu zapisati kao: an = Alx;" A ~ x ; . . . A,x:, Ai E R.

+ +

+ +

5. REKURZIVNE RELACIJE

Kad je skup svih rjeSenja ovako zapisan (pomoCu parametara Ai), an se zove op&__rjienje-te rekurzivne relacije. eAko je neki x, = x i , onda u rjeSenju umjesto Aixr A x'." ----I , Z treba stajati A , z l + ~ ~ n ~ ~ - . n - Z l ud da & je j koriiei&SBve_Ce u K F a t----n osti*~~a&_u_es~~~e ----potencije od n . "Linearna rekurzivna relacija reda r ima jedno rjeSenje ako su za r uzastopnih Elanova niza koji je njeno rjeSenje zadane vrijednosti koje svaki pojedini od njih treba poprimiti. Medu svim rjeSenjima homogene rekurzivne relacije tada se direktnim uvrgtavanjem odreduje ono koje zadovoljava poEetne uvjete. Relacija tipa

1,

zove se n-liaearna re-a,reLacija reda r . Relacije tog tipa rjeSavaju se tako da se prvo rijeSi pripadna homogena relacija Neka je njeno opCe rjeSenje akh) . Nakon toga se nade jedno (bilo koje) rjeSenje polazne rekurzivne relacije - to je tzv. p_rtikularno ries'enje rekurzivne relacije. OznaEimo li ga s a t ) , onda je qpCe rjeSenje -.n e h o m o g e e ~ s , -- --relacije d a n ~ [ a ; ) ' at). ;
-A

--- - -

Ne postoji pravilo za odredivanje partikularnog rjeSenja. Kao pomoC pri pogadanju rjeSenja moie posluiiti sljedeCa tablica:

+ ------.

gdje su Bi realne konstante. Iz opCeg rjeSenja dobiva se rjeSenje nehomogene rekurzivne relacije uvrStavanjem poEetnih uvjeta, kao i u sluEaju homogenih relacija.

4-t Za lakSe pamCenje ovih postupaka zgodno je poznavati njihove dokaze. PotraCite ih i nastojte razumjeti! 4-t Ako ste se vet susreli s diferencijalnim jednadibama, usporedite ove postupke s rjegavanjem linearnih diferencijalnih jednadibi. Ponekad se rekurzivne relacije zovu i diferenczjskim jednadibama.
Sustavi rekurzivnih relacija mogu se eliminacijom varijabli ili nekim supstitucijama pokuSati dovesti do jedne jednadzbe, iz koje onda izraEunamo prvi nepoznati nix. Nakon toga raEunamo ostale nizove. Linearne rekurzivne relacije mogu se rjeSavati i pomoCu funkcija izvodnica. Ideja se sastoji u tome da iz rekurzivne relacije dobijemo funkciju izvodnicu za tra3eni niz (rjegenje), koji onda, po potrebi "raspakiramo". U sljedekem poglavlju bit C e na primjerima opisan i taj postupak.

5. REKURZIVNE RELACIJE

rir) Nadite opCa rjeSenja rekurzivnih relacija:


an+2 - 5an+l 6 a n = 0 , 4an+2 an = 0 , an+2 4an+1 5an = 0 , ,@ an+3 2an+2 - an+l - 2% = 0, (e) an+r - 8an+3 18an+2 - 27an = 0 , (f) an+3 - 2an+l - 4 a n = 0.

+ + +

Rjes'enje.
(a) KarakteristiEna jednadiba zadane rekurzivne relacije je x2 - 5x 6 = 0 . Njena rjelienja su XI = 3 , 2 2 = 2 . Stoga je opCe rjegenje zadane rekurzivne relacije a n = A . 2n B 3n , gdje su A i B realni brojevi.

(b) KarakteristiEna jednadiba 4x2 - 42 x = 112. Rezultat: an = A(1/2)n B n ( l / 2 ) n .

+1= 0

ima jedno dvostruko rjegenje,

- (c) Rjegenja karakteristiEne jednadibe x2+4x+5 = 0 su kompleksna: xl,2 = - 2 f i . Rezultat: an = A(-2 i)n B(-2 - i ) n .

+ +

i cp := arc tan(-112) , onda je -2+i = r(cos p + i sin cp) , +P Ako oznaEimo r := a -2 - i = r(cos cp - i sin cp) (polarni prikaz kompleksnog broja). Tada primjenom Moivreovog stavka dobivamo d a je rjelienje oblika an = rn(A B ) cos(ncp) + P ( A - B) sin(ncp) = C r n cos(ncp) D r n sin(np) , za proizvoljne konstante C i D . Ponekad se ovaj oblik rjeHenja spominje kao pravilo za nalaienje rjegenja ako karakteristiEna jednadiba ima kompleksne korijene.

+ B(-l)n + C(-2)n = A + B ( - I ) ~ + C(-2)" (e) KarakteristiEna jednadiba glasi: x4 - 8x3 + 18x2- 27 = 0 (pazite, u jednadibi se nije pojavio an+l ). -1 je jednostruki, a 3 trostruki korijen te jednadibe, pa je rezultat: a n = A(-l)n + B . 3n + Cn3n + ~ n ~ 3 ~ . (f) Rezultat: an = A . 2n + B(-1 + i)n + C(-1 - i ) n .
(d) Rezultat: an = A . In

'3+ Uvrstite dobivena rjegenja u pripadne jednadibe i pogledajte kako se krate pojedini sumandi. UoEite znaEenje karakteristizne jednadibe. $to se dogada kod vigestrukih korijena - razmislite o vezi viiestrukih korijena jednadibe, deriviranja funkcije koja se nalazi na'lijevoj strani jednaibe i rjegenja rekurzivne relacije u tom sluEaju!

Rijesite rekurzivne relacije:


3an = 0, uz poEetne uvjete a1 = 1, a2 = 7 , f,, = fn-l f n - 2 , uz poEetne uvjete fo = 0 , f l = 1, 1 (c)/ an+3 - an = 0 , uz poEetne uvjete a0 = a1 = 0 , a2 = 1.
an+2 - 4an+l

Rjes'enje. Treba prvo naCi opCe rjelienje, a onda izmedu moguCih rjeSenja izabrati ono koje zadovoljava zadane poEetne uvjete.

5. REKURZIVNE RELACIJE

(a) OpCe rjeienje je an = A B . 3,. Uvrgtavanjem poEetnog uvjeta a1 = 1 u formulu za opCe rjegenje za n = 1 , te poEetnog uvjeta a2 = 7 u istu formulu za n = 2 , dobivamo sustav jednadibi:

RjeHenja tog sustava su A = - 2 , B = 1 , pa rjeienje zadane rekurzivne relacije glasi: an=-2+3,. (b) Rjeienja karakteristicne jednadibe x2 - z - 1 = 0 su (1 &)/2 i ( 1 - & ) / 2 , pa je opCe rjegenje jednako f n = A ( ( 1 &)/2)" B ( ( 1- & ) / 2 ) n . Uvritavanjem poEetnih uvjeta dobivamo sustav jednadibi 0 = A+B, B ( 1 - &)/2. 1 = A(1+ &)/2

Rjegenja tog sustava su A = I / & ,

B = -I/&,

pa je

h = [ ( ( I + ~ 5 1 1 2 ) "- ( ( 1 -

/&.

(c) RjeSenja karakteristicne jednadibe z3- 1 = 0 su treCi korijeni iz 1, pa je opUvrgtavanjem poEetnih de rjegenje an = A B ((-1 i f i ) 1 2 ) ~ C ((-1 - i&)/2),. uvjeta dobiva se sustav jednadibi:

= =

A+B+C

o
Slijedi A =

+ B ((-1 + i & 1 / 2 ) ~+ c ((-1 - i f i 1 / 2 ) ~


te a, = 5 + ( 4 ( - 1 + i & ) / 2 ) ~ + ' +

i,B =

&(-l+i&), C = &(-1-i&),

(;(-I

malo bolje, odmah se vidi da je an = 1 ako je n djeljivo s 3, a an = 0 inaEe. Provjerite da dobiveni izraz za an zaista poprima tako jednostavne vrijednosti (npr. predoEavanjem treCih korijena iz jedinice u kompleksnoj ravnini nactite sumu njihovih n-tih potencija).

- i&)/2)n+1. 4+ Pogleda li se zadatak

3.

Nadite opCa rjeSenja sljedetih rekurzivnih relacija: 2an+2 - an+l - a , = 2,, an+2 - 4an+1 4an = Z n ,
3an+2 2an+l - a , = 3 2 . 3 " 1/3", a,+l - 5an = 4 n 2 2 n 5 , an+3 - 7an+2 1 5 ~ , + ~ 9an = n .

+
+

+ +

Rjes'enje. Za svaki od zadataka prvo treba rijegiti pripadnu homogenu jednadibu. Dobiveno rjegenje oznaEit Cemo s a i h ) . Nakon toga Cemo pomoCu tablice s poEetka poglavlja pogoditi jedno rjegenje zadane nehomogene jednadibe i oznaEit ga s a?). OpCe rjeienje polazne jednadibe bit C e tada zbog njene linearnosti jednako aih) a;') .

(a) KarakteristiEna jednadiba pripadne homogene jednadibe je 2z2 - x - 1 = 0 , a njena rjegenja su -112 i 1. Dakle opCe rjegenje pripadne homogene rekurzije je

5 . REKURZIVNE RELACIJE

95

akh) = A(-112)" B . Partikularno rjegenje traiit Cemo, prema tablici, medu nizovima oblika C . 2". UvrZtavanjem u rekurziju dobiva se da C mora biti jednak 5. Jedno partikularno rjeHenje je a?) = 5.2", a opCe rjeSenje je a n = akh)+aip) = A ( - 1 / 2 ) " + B ~ 5 . 2 ~ . (b) KarakteristiEna jednadiba je s2- 4 s 4 = 0 , a njeno rjegenje je s = 2 (dvostruko), pa je opCe rjeHenje pripadne homogene rekurzivne relacije akh) = A . 2n Bn2". Partikularno rjegenje trebali bismo prema tablici traiiti u obliku a$) = C . 2" . No, to je besmisleno, jer je t a funkcija vet sadriana u opCem rjegenju. PokuLajmo zato kao kod viSestrukih korijena karakteristiEne jednadibe pomnoiiti rjeSenje sa "sljedeCom slobodnom" potencijom od n i traiiti rjegenje u obliku: a $ ) ' = c A ' ~ ~ .I zaista, uvrgtavanjem u rekurzivnu relaciju dobiva se d a je niz ovog oblika zadovoljava za C = 118. OpCe rjeSenje 2n-3. glasi a n = aih) a$) = A . 2" Bn2"

%-+ Uvrgtavajte potencijalna partikularna rjegenja 2", n2", n 2 2 n , n 3 P , uoEite Hto se dogada!
(c) Za pripadnu homogenu jednadibu karakteristiEna jednadiba glasi 3s2 2 s - 1 = 0,pa je njeno opCe rjegenje jednako aih) = A(1/3)"+B(-1)". Partikularno rjegenje Cemo traiiti u dva koraka: prvo za desnu stranu rekurzije jednaku 3", a onda za 1/3n . Zbrajanjem tih rjegenja dobit Cemo jedno partikularno rjegenje polazne jednadibe (uvjerite se da za linearne rekurzivne relacije mdiemo uvijek traiiti odvojeno partikularna rjegenja za svaki pribrojnik na desnoj strani rekurzije, a onda ih zbrojiti i dobiti partikularno rjegenje polazne rekurzije). Uvrgtavanjem izraza C.3" u rekurziju koja na desnoj strani ima 32.3" , dobivamo prvo partikularno rjegenje a k l ) = 3" . Postupimo li analogno i za drugi pribrojnik, vidimo d a ne moiemo izraEunati potrebnu konstantu. To je zato, jer je 113" veC rjeHenje homogene rekurzije. Kao i u sluEaju vigestrukih korijena, potraBimo rjegenje u obliku Cn(1/3)" . Tako se moie izrazunati konstanta C , pa imamo drugo partikularno rjegenje = (314) . n(1/3)" = (114) . n1/3"-' . Opte rjegenje nehomogene rekurzije koju je

.~ - ' trebalo rijegiti glasi an = akh)+a$')+akp2) = ~ ( 1 / 3 ) ~ + ~ ( - 1 ) " + 3 ~ + ( 1 / 4 ) . n ( 1 / 3 )


%-+ (za one koji znaju funkciju sin napisati pomoCu eksponencijalne funkcije) Nadite "pravilo" za pogadanje rjegenja nehomogene rekurzije ako je na desnoj strani sin a n , gdje je a neka konstanta. Izmislite u t u svrhu prvo nekoliko takvih rekurzija i pokugajte ih rijegiti.
(d) Opde rjegenje homogene rekurzije je aih) = A . 5". Kad bismo slijepo koristili postupak iz prethodnog dijela zadatka i tablicu s poEetka poglavlja, partikularno rjeLenje traiili bi5mo tako, da ga prvo traBimo za desnu stranu jednaku 4 n 2 , zatim za 2n te na kraju za $ No, prvo partikularno rjegenje bio bi polinom 2. stupnja, drugo bi bio polinom 1. stupnja, a treCe neka konstanta. Njihovim zbrajanjem dobili bismo ponovno polinom 2. stupnja, pa vidimo da moiemo odmah raditi sa cijelom B n C . Uvritavadesnom stranom i pretpostaviti d a je rjeHenje oblika a?) = ~n~ njem u jednadibu izraEunaju se konstante A = B = -1, C = -514, pa je opCe rjegenje jednako a n = aih) a?) = A . 5n - n2 - n - 514.

(e) KarakteristiEna jednadiba s3- 7s2 15s - 9 = 0 ima 1 kao jednostruko i 3 kao dvostruko rjegenje. Dakle opCe rjeienje pripadne homogene rekurzije je akh) = A B . 3" Cn3".

Desna strana polazne rekurzivne relacije je polinom stupnja 1. Dakle, partikularno rjegenje trebali bismo traiiti medu polinomima stupnja 1. No, buduCi da je 1 korijen karakteristizne jednadibe, na t a j naEin ne bismo mogli dobiti rjegenje (uvrstite sami opti

96

5 . REKURZIVNE RELACIJE

polinom stupnja 1 u rekurziju, pogledajte ;to se dogada i objasnite kakve to ima veze sa Einjenicom da je 1 korijen karakteristizne jednadzbe). Ako pokugamo s polinomom stupnja 2, dobivamo rjesenje, i ono glasi: a?) = n(n 1)/8.

n(n

+ l)/8.

OpCe rjegenje polazne rekurzivne relacije je a n = a?)

+ aLh) = A + B . 3n + Cn3" +

Ftijegite rekurzivne relaGje:

+ 2an = 10, uz poEetne uvjete a0 = 6 , a1 = -2, an+l - an = n 2 , uz poEetni uvjet a1 = 2 , te an+2 - 4 a n + l + 4an = 3n - 2 n , uz poEetne uvjete a0 = 1, a1 = 3 .
2an+2 - 5an+l

Rjes'enje. Treba prvo naCi opCe rjesenje svake od jednadibi, a zatim izabrati ona rjegenja koja zadovoljavaju poEetne uvjete.
(a) OpCe rjegenje homogene rekurzije 2an+2 - 5an+l +2an = 0 je aLh) = A(1/2)n -iB2n. Na desnoj strani nehomogene jednadibe stoji konstantni Elan, koji moiemo interpretirati kao polinom stupnja 0. Stoga pretpostavimo da je partikularno rjeSenje polinom stupnja 0, tj. da je oblika a?) = C . Uvrgtavanjem tog partikularnog rjegenja u polaznu jednadibu dobivamo da ono moie biti rjegenje polazne jednadibe i da tada mora biti C = -10. Dakle opCe rjegenje polazne jednadibe glasi: Uvrgtavanjem poEetnih uvjeta vidi se d a je A = 1 6 , B = 0 . Rezultat: an = (1/2)"-~ 10.

4-t Interpretirajte desnu stranu kao potenciju i pokuzajte tako pogoditi partikularno rjeSenje.

4-t BuduCi da su u rjegenju preostale samo jedna potencija i Elan koji dolazi od nehomogenog dijela, sigurno je da postoji nehomogena rekurzivna relacija reda 1koju ovo rjegenje zadovoljava. Nadite jednu!
(b) Rjegenje pripadne homogene jednadibe je an = A . 1. Partikularno rje'lenje za desnu stranu jednaku n2 trebali bismo traiiti medu polinomima drugog stupnja. No, buduCi da je rjegenje homogene jednadibe veC polinom nultog stupnja (vidi prethodni zadatak, dio (e)), partikularno rjegenje traiit Cemo medu polinomima stupnja 3. UvrStavanjem opCeg polinoma treCeg stupnja a?) = ~n~ c n 2 D n E u polaznu rekurziju i izjednaeavanjem koeficijenata uz svaku pojedinu potenciju od n dobiva se sustav jednadLbi u B, C, D i E . Rjegavanjem tog sustava dobiva se jedno partikularno rjegenje: an(p) = n3/3 - n2/2 n / 6 .

Opde rjegenje polazne jednadibe je stoga an = n3/3 -n2/2 +n/6 +A . PoEetni uvjet a1 = 2 moiemo rekurzijom "pretvoriti" u jednostavniji poEetni uvjet ag = a1 - 0' = 2 , pa se onda bez raEunanja vidi da je A = 2 .

tP Onaj tko je bolje promotrio zadanu rekurzivnu relaciju mogao je bez raEunanja partikularnim rjesenjem proglasiti oEigledno rjegenje sip) = i2 = n(n+1)(2n+1)/6 . SluEajno smo isto partikularno rjegenje dobili i pogadanjem.

zzl

(c) KarakteristiEna jednadiba ima 2 kao dvostruko rjegenje, pa je opde rjegenje pripadne homogene jednadibe aih) = A . 2n B n 2 n . Partikularno rjegenje traiimo posebno za jednadibu koja bi na desnoj strani imala 3n i za jednadibu koja bi na desnoj

5. REKURZIVNE RELACIJE

97

strani imala -2,, t e dobivena dva partikularna rjelienja zbrojiti. BuduCi da je 2 veC korijen karakteristicne jednadHbe (i to dvostruki), onda partikularno rje'senje traiimo u ~ n ~ 2 ,Slijedi . A = 1 , B = -118. OpCe rjelienje je jednako obliku: a?) = A3" a, = A . 2, Bn2n n22n-3 3,.

+ +

UvrEtavanjem poEetnih uvjeta dobiva se rezultat: a n = n ( n - l)2n-3

+ 3,.

5.

a i = i , z a i=0,1,

RijeSite rekurzivnu relaciju an+20 ...,19.

+ 2an+lo + a,

= 4 uz poEetne uvjete

RjeSenje 1. Ovo je rekurzivna relacija reda 20. Ona se moHe rjeliavati kao i ostale rekurzivne relacije, a zbog njenog posebnog oblika (pogledajte indekse!), karakteristiena jednadHba bi se, unatoE velikom stupnju, mogla rijeliiti jednostavnom supstitucijom. PokuEajte je sami rijeliiti tim natinom. Problemi nastaju pri uvrLtavanju poEetnih uvjeta, jer se time dobiva sustav 20 jednadibi s 20 nepoznanica. On se takotter moie zgodno pojednostavniti, ali ostaje Einjenica d a treba pun0 pisati. Pokugajte ipak! RjeSenje 2. Promatrajmo rekurziju blc+2+2bk+l+bk = 4 . Za i E { O , l , . . . , 9 ) njeno rjelienje za poEetni uvjet bo = i , bl = i 10 daje rjegenje polazne rekurzije alOk+i = bk . OpCe rjegenje nove rekurzije je bk = A(-ilk ~ k ( - l ) ~1. Uz poEetni uvjet bk = i , dobivamo bk = (i - l)(-ilk - 2(i - 4 ) l ~ ( - l ) ~ 1. Stoga je za n = lOk i , gdje je i E {0,1,. . . ,9) rjelienje an = (i - l)(-ilk - 2(i - 4)k(-ilk 1 .

1 -

6.

(a) (b)

RijeSite rekurzivnu relaciju an+2 - a,+l a0 = 0 , a3 = 0 , a0 = 1 , a3 = 2

+ a,

= 0 uz poCetne uvjete:

RjeSenje. Opde rjeEenje ove rekurzivne relacije je

Zbog potenciranja u nastavku zadatka zgodno je napomenuti d a su korijeni karakteristiEne jednadHbe treCi korijeni iz -1 - to moie uHtedjeti potenciranje. (a) PoEetni uvjeti daju sustav: A+B -A-B

= =

0 0.

Iz toga slijedi d a svako rjelienje zadane rekurzivne relacije u kojem je A = -B zadovoljava ao=a3=O. U tom sluEaju, koristeCi Einjenicu da se radi o treCim korijenima iz -1, vidimo da je: an =

0 Afi

-A&

ako 31n, ako 31n - 1, a ako 3/72- 2.

(b) PoEetni uvjeti daju sustav:


A+B -A-B

= =

1 2.

5. REKURZIVNE RELACIJE

Taj sustav nema niti jedno rje'lenje, pa ne postoji niti jedno rje'lenje zadane rekurzivne relacije koje zadovoljava poEetne uvjete a0 = 1 , a3 = 2 .

'=I+ Za utjehu nadite poEetne uvjete (ne nuino za a0 i a1 ) koji odreduju po jedno rje'senje zadane rekurzivne relacije. Postoji ih beskonaEno mnogo, jednako kao i ovih LLdivljih" !
7.

Nadite po jednu rekurzivnu relaciju kojoj su rjeSenja:

w n i z (a,) kojem su a0 = 0, a1 = 1 , a2 = 2 , as = 1 , O'niz a , = 3n 5n + 7

Rjebenje.
(a) Traiit Cemo homogenu rekurzivnu relaciju. Prvi problem je odrediti kojeg bi ona trebala biti reda. Rekurzija reda k ima u sebi k 1 koeficijent, no, d a iz razmatranja izbacimo medusobno ekvivalentne rekurzije, podijelimo svaku koeficijentom uz an+k (tako da sve karakteristiEne jednadibe imaju vodeCi koeficijent 1). Time smo izjednaEili niz rekurzija koje se jedna od druge razlikuju samo u konstantnom faktoru i daju iste nizove kao rjeHenja. Uz to, vidimo d a su homogene rekurzivne relacije reda k odredene s po k koeficijenata (usporedite ovo razmatranje s polinomskim jednadibama!).

Bududi da imamo zadana Eetiri Elana niza, pokuHat Cemo naCi rekurziju reda 2 kojoj je neki niz sa zadanim poEetkom rje'lenje (uvjerite se sami da je razumno poku'lati sa stupnjem rekurzije jednakom polovini broja zadanih Elanova niza). Ako rekurziju zapi'semo kao a n t 2 Aan+l Ban = 0,

onda uvr'stavanjem zadanih Elanova niza dobivamo sustav jednadibi:

te njeno rje'lenje A = -2, B = 3 (oEekivali smo B # 0). Jedna rekurzivna relacija koji 3an = 0 . zadovoljava zadani poEetak niza rje'lenja je an+2 - 2a,+l

'=I+ Da smo pokuHali dobiti rekurziju vi'leg reda, rje'lenje ne bi bilo jedinstveno. To je i logicno, jer je npr. bilo koja linearna kombinacija ove rekurzije i bilo koje rekurzivne relacije koja moie zadovoljiti poEetne uvjete ao = a1 = a2 = a3 = 0 takoder rekurzivna relacija koja zadovoljava uvjet zadatka. PokuHajte s redom 3 ili 4!
BeskonaEno mnogo rjeSenja pri poku'laju traienja rekurzije reda 2 mogli smo dobiti s nekim drugim poEetnim uvjetima (npr. 1 , 1 , 1 , 1 ) . Objasnite za'lto je to tako i nadite jo'l jedan skup takvih poEetnih uvjeta.

'=I+ Za neke druge poEetne uvjete moglo se dogoditi i da dobiveni sustav nema rjeSenje (npr. 1,1,1,2). Za'lto je tome tako? Tada bismo morali pokuSati s rekurzijom vi'leg stupnja. Uvijek postoji homogena rekurzivna relacija kojoj je dani konaEni niz poEetni podniz jednog rje'senja.
(b) Rjes'enje 1. Ako zadano rjegenje interpretiramo kao da je linearna kombinacija funkcija 3", 5n i I n , onda karakteristizna jednadiba traiene rekurzivne relacije moie biti (x - 1)(x - 3)(x - 5) = 0 , tj. x3 - 9x2 - 7x 15 = 0 . Rekurzivna relacija glasi:

5. REKURZIVNE RELACIJE

Rjes'enje 2. Zadano rjeHenje moiemo interpretirati i kao rjeHenje nehomogene linearne rekurzivne relacije s karakteristitnom jednadibom (x - 3)(x - 5) = 0 i polinomom stupnja 0 na desnoj strani. UvrHtavanjem zadanog rjeHenja u rekurziju s odgovarajuCim karakteristiEnim polinomom i ieljenom desnom stranom
dobivamo C = 56.

8. RijeSite sljedeCe rekurzivne relacije svodeCi ih pogodnom supstitucijom na linearne:

/I$( a:+,-2a:=l,
Cb-)
(n

a n + l - n u n = n! , a0 = 2 . (d) a n + b n - 1 = 4 n - 2 , bn+an-1 = 2 , a o = O , b o = l

+ l)an+l + n u n = 2,,

a0=2,
a0

= 59,

Rjes'enje.
(a) U ovoj rekurzivnoj relaciji pojavljuju se samo kvadrati Elanova niza (a,), pa moiemo staviti b, := a:. Relacija prelazi u b,+~ - 2bn = 1 uz poEetni uvjet bo = 22 = 4 . OpCe rjegenje te rekurzivne relacije je bn = A2, -1 , a uvrgtavanjem poEetnog uvjeta dobiva se rjeHenje nove rekurzije: b , = 5.2n - 1. RjeHenje zadatka je a n = 6= d m . (b) Svaki Elan niza pojavljuje se pomnoien sa svojim indeksom, pa moiemo staviti b, := nan . Dobivamo rek rziju bn+l bn = 2, i poEetni uvjet bo = 0 . RjeHenje te rekurzije je bn = ((-1)" + k n ) / 3 , a rjegenje polazne rekurzije a n = ((-1)" 2")/(3n) za n > O i a o = 5 9 .

(c) Do rekurzije s konstantnim koefisijentima dolazimo supstitucijom bn = an/(n I)! za n 2 1. Nova rekurzija je tada bn+1 - bn = 1 , a poEetni uvjet bl = a1 = 1. RjeHenja su bn = n , odnosno a n = n! (za sve n 2 0 ). (d) Eliminirat Cemo Elanove niza (b,) . Iz druge jednadibe imamo bn = 2 - an-1, gto nakon pomicanja indeksa i uvrgtavanja u drugu jednadibu daje a n - a,-2 = 4n - 4 (mogete li ovu rekurzivnu relaciju rijeHiti napamet?). OpCe rjegenje pripadne homogene a jedno partikularno rjegenje aLp) = n2 . PoEetni jednadibe je aLh) = A B(-l),, uvjet a0 = 0 imamo, a a1 = 1 izraEunamo iz sustava jednadibi i poEetnog uvjeta za bo . Dobivamo rjezenje a, = n 2 . Iz druge jednadibe slijedi onda bn = 2 - (n - I ) ~ .

9+

RijeHite zadatak eliminirajudi Elanove niza (a,)

Rjes'enje. Promatramo li veze izmedu Elanova traienog niza i izraza koji se pojavljuje na desnoj strani, vidimo d a je:

S :

an

da,-l . . . . Eliminiranjem an iz ovog sustava dobivamo rekurziju gdje je Sn = Jan S : = 2Sn-1 , koja se moie rijeHiti logaritmiranjem (najbolje je logaritmirati po bazi 2 uvjerite se zaHto) i supstitucijom un := log2 S,i . Imamo:

+ Sn-1,

5 . REKURZIVNE RELACIJE

RjeSenja su redom: un = 1, pa S n = 2"" = 2 i konaEno an = 2 za n

2 1, a0 = 4

Ako se ovaj zadatak rijeSi za proizvoljni poEetni uvjet a0 = c , maze se vidjeti da se u svakom sluEaju Elanovi niza ( a n ) za rastuCi n monotono priblizavaju broju 2 (limn+, an = 2 ) . ZaSto je tome tako?

tP ttt

Mogli smo eliminirati i an -ove.

Funkcije izvodnice
U prethodnom poglavlju vidjeli smo Sto moZemo uEiniti ako kombinatorno razmatranje umjesto izraza za opB Elan niza da rekurzivnu relaciju. Ovdje demo se baviti jednim drugim naEinom zapisivanja nizova. Naime za neke primjene zgodno je promatrati nizove "zapakirane" u formalne redove - sume oblika f (x) = fnxn koje promatramo kao spremiSta za elemente niza (f,) . Pritom nas ne zanima eventualna konvergencija tih redova, kao niti vrijednosti u pojedinim toEkama. Ipak, ako imamo formalni red,za koji red s jednakim koeficijentima konvergira na nekom podruEju (kao npr. 2"xn = 1/(1- 22) za 12x1 < l ) ,onda to moZemo iskoristiti. Zapisujemo za formalni red 2nxn = 1/(1 - 22) i desnu stranu zovemo analiticxm prikazom ili prikazom u zatvorenom obliku. Ako su a(x) = En>,o anxn i b(x) = bnxn funkcije izvodnice za nizove (a,), odnosno (b,) , onda definiramo operacije s formalnim redovima:
7

'

zbmjanje mnozenje skalamm mnozenje d eriwimnje integrimnje kompozicija

+ b)(x) (aa)(z) (ab)(x) af(x> J a(x)dx a(b(x))


(a

:= := := := := :=

+ bn)xn , a a n z n, ' \ En>O (Ey=O(aibn-i))x n , , nanzn-I , an/(n l)xn+l En>O anb(x)", ako je b o = O .
&,(an

Nadalje, dva formalna reda smatramo jednakim, ako im se podudaraju svi koeficijenti (tj. ako oba formalna reda odgovaraju istom nizu). Sve opisane operacije odgovaraju istoimenim operacijama s pripadnim analitiEkim prikazima, Sto formalne redove Eini praktiEnima. Tako npr. za formalne redove 1 + x + x 2 + . . . i x + x 2 + . . . imam0 ( 1 + x + x 2 + . . . ) . (x + x 2 + ...) = 1/((1 - x) . x/(1 - x) = x/(l - x ) ~ (Sto znaCi, razvijemo li desnu stranu u red, dobit demo dva jednaka reda).

102

6. FUNKCIJE IZVODNICE

NaBalost C e zbog ovog dio zadataka ovog poglavlja biti teZe rjegiv za sve one koji jog ne znaju derivirati i integrirati "obiEne" funkcije. Da se radi npr. o deriviranju formalnih redova lako se prepozna iz definicije derivacije formalnog reda, ali za sredivanje izraza potrebno je znati i kako se deriviraju i odgovarajuCe funkcije (njihovi prikazi u zatvorenom obliku). Kad dodete do te toEke u rjegavanju zadataka, posluBite se slobodno nekim tablicama derivacija funkcija - sa stanovigta kombinatorike to je jedno prihvatljivo privremeno rjegenje. Za niz (a,) definira se obiEna funlccija izvodnica kao formalni red: f (x) := a eksponencijalna funlccija izvodnica je:

C a,.",
n>O

Oznaka za funkciju izvodnicu moie se izabrati proizvoljno, a ako se radi samo o jednom tipu funkcije izvodnice za neki niz najzgodnije je uzeti slovo koje oznaEava elemente tog niza (npr. a($) za niz (a,)). ObiEna i eksponencijalna funkcija izvodnica su osnovni tipovi funkcija izvodnica. Za neke sloBenije probleme pokazuje se praktiEnijim elemente niza zapakirati na druge naEine. Koju funkciju izvodnicu Cemo izabrati, ovisi o problemu kojeg rjegavamo. Da bi rad s funkcijama izvodnicama bio koristan, Belimo da slaganja objekata koja prouEavamo odgovaraju nekim operacijama s funkcijama izvodnicama. Ovisno o vrsti objekata, to C e biti moguCe samo s odgovarajuCim vrstama funkcija izvodnica. Kako funkcije izvodnice prate kombiniranje objekata vidjet Cemo na primjeru mnofenja formalnih redova. Pokazuje se da je u mnogim kombinatornim problemima jednostavnije izraEunati funkciju izvodnicu nego Elanove niza ili neki Clan niza direktno. No, iz funkcije izvodnice se moHe (barem teorijski) izraEunati bilo koji Elan niza: ponekad direktno, a ponekad formulom koja ukljuEuje formalno deriviranje (usporedite s Taylorovom formulom):

Cak i ako ne moBemo izraEunati koeficijente (npr. raEunanje 100-te derivacije bi bilo prekomplicirano), funkcija izvodnica se moBe iskoristiti za raEunanje funkcija izvodnica nekih sIoBenijih objekata, kojima je moBda moguCe izraEunati koeficijente. Ovdje se neCemo baviti problemom izbora tipa funkcije izvodnice za pojedini problem, nego C e u zadacima uvijek biti navedeno kojim ga tipom funkcije izvodnice treba rijegiti. U svim zadacima traBi se da se rezultat zapiSe u anaIitiEkom obliku. Na poEetku dano je nekoliko tehniEkih zadataka koji pokazuju kako se manipulira funkcijama izvodnicama. U njima se traBi da se izraEunaju Elanovi niza, te da se oni uvrste u definiciju funkcije izvodnice.

6. FUNKCIJE IZVODNICE

1. Napigite obiEnu i eksponencijalnu funkciju izvodnicu za broj podskupova konaEnog skupa.


Rjes'enje. n-Elani skup ima ukupno 2n podskupova (prisjetite se, za svaki od njegovih n elemenata treba redi d a li C e biti ukljuEen u skup ili ne - dvije mogutnosti izbora). BuduCi d a je ovdje jedini parametar velitina skupa ( n ) , imamo niz veliEina podskupa indeksiran veliEinom polaznog skupa i stavljamo an := 2 n . ObiEna funkcija izvodnica glasi 1 f (x) = 2nxn = (2x)n = 1- 22' n>O n>O

Eksponencijalna funkcija izvodnica je:

4-, Odredite za vjeZbu za koje objekte bi f(x) bila eksponencijalna funkcija izvodnica. Uputa: izraEunajte prvo odgovarajute koeficijente bn , takve da je f (x) = bn/n!xn , a zatim kaZite Zto broje ti bn -ovi.

2. Neko n-Elano drugtvo odluEilo je otiCi na veEeru u restoran. Poznato je da se dobro jede u restoranu Riba, gdje im je na raspolaganju r ribljih specijaliteta, zatim u vegetarijanskom restoranu, gdje nude v specijaliteta od povrda, te u restoranu Ovca, gdje se moIe kugati o specijaliteta s ovEjim sirom. NapiSite obiEnu funkciju izvodnicu za broj razliEitih narutivanja jela za tih n o&ba. Pretpostavimo daXe cijelo drugtvo jesti u istom restoranu, te da C e svaka osoba naruEiti toEno jedno jelo.
Rjes'enje. To drugtvo moZe u prvom restoranu izabrati jelo na T~ naEina, u drugom na vn , u treCem na o n . Ukupni broj naEina izbora iznosi rn vn o n , pa je funkcija izvodnica:

+ +

f(x)

C ( T+ ~vn + on)xn
n)O

W Napigite i eksponencijalnu funkciju izvodnicu.


4 ' Ova funkcija izvodnica je suma pojedinih funkcija izvodnica narudZbi. Analogno vrijedi i za ekponencijalne funkcije izvodnice.

6. FUNKCIJE IZVODNICE

3. Napigite obiEnu funkciju izvodnicu za broj nizova koji poEinju (pod)nizom slova, a zavrsavaju (pod)nizom znamenaka. Bilo koji od podnizova moBe biti i prazan. Nizovi trebaju biti grupirani prema ukupnoj duljini.
Rjes'enje. Nezavisno treba izabrati podniz slova i podniz znamenaka. Funkcija iz30nxn, a za broj izbora niza znamenaka vodnica za broj izbora niza slova bila bi

E n 2 0 10nxn. "Veliki" niz duljine n mofe se slofiti od praznog niza slova i niza znamenaka duljine n , ili od niza slova duljine 1 i niza znamenaka duljine n - 1 itd, opdenito od niza slova duljine i i niza znamenaka duljine n - i , za bilo koji i E N n U ( 0 ) . Broj naEina izbora takvog niza je stoga 30'10~ 30'10n-' . . . 30~10' = EL03 0 ~ 1 0 ~ Zto - ~ je , jednako koeficijentu uz xn u produktu funkcije izvodnice za niz slova i funkcije izvodnice za niz znamenaka. Dakle, traiena funkcija izvodnica odgovara produktu funkcija izvodnica za podnizove, tako da rezultat glasi

+ +

+P Da bismo provjerili rezultat - jesu li stvarno koeficijenti u dobivenoj funkciji izvodnici feljeni brojevi - napiSimo je prvo u obliku parcijalnih razlomaka.

Sada se lako vidi da je koeficijent uz xn u f ( x ) : ( x n ) f ( x ) = 312 .30n - 112 . lon = ( 3 . 30n

- 1on)/2,

a to je upravo jednako broju od kojeg smo krenuli. 4-t Za eksponencijalne funkcije izvodnice ovih nizova ne bi vrijedilo da je funkcija izvodnica na opisani naEin slofenog niza jednaka produktu pojedinih funkcija izvodnica. Provjerite to. Preuredite zatim tekst zadatka tako da produkt eksponencijalne funkcije izvodnice za niz slova i eksponencijalne funkcije izvodnice za niz brojeva bude jednak eksponencijalnoj funkciji izvodnici za nekakve nizove slofene od slova i brojeva.

4. NapiSite funkciju izvodnicu za broj naEina izbora uredenog para razliEitih elemenata iz n -Elanog skupa.
Rjes'enje. Uredeni par razliEitih elemenata moBe se iz n-Elanog skupa izabrati na n(n - 1) naEina. Skup iz kojeg biramo takve parove mora imati barem dva elementa, pa je odgovarajuta funkcija izvodnica:

f (x)

)n(n - l ) x n = x 2
n22

n(n
n22

- 1)xnW2

+ I ' Sve sume su mogle biti i za n

2 0.

s)3 Zto prema binomnom teoremu iznosi 2(-l)n-2

CP Koeficijent uz xn u f(s) je jednak dvostrukom koeficijentu uz xn-2 u 1 / ( 1 -

(n22) = 2(;)

= n ( n - 1) .

6. FUNKCIJE IZVODNICE

105

'3+ SliEno se moIe izraEunati funkcija izvodnica za bilo koju Ic-torku razliEitih elemenata iz n-Elanog skupa. IzraEunajte je prvo za k = 3 , zatim za proizvoljni k . Rezultat: fk(x) = k!xk/(l - x ) ~ " .
Lako se izaEunaju i eksponencijalne funkcije izvodnice.

4+ IzraEunajte za fiksni k obiEnu i eksponencijalnu funkciju izvodnicu za broj Ic Elanih podskupova n-Elanog skupa. Usporedite rezultat s funkcijama izvodnicama za k -torke.
?+ Ako vam ovo nije prvo Eitanje ovog poglavlja, pokuSajte kombinatorno doCi do funkcije izvodnice za broj k -torki, npr. do njenog oblika: fk(x) = (x x2 . . .lk(1 x x2 . . .) .k! . NapiSite rekurzivnu relaciju za niz funkcija (fk) , pa pokuSajte interpretirati i nju!

+ +

+ +

5. NapiGite funkciju izvodnicu za broj naEina izbora uredenog para elemenata n -Elanog skupa.
Rjegenje. Uredeni par elemenata moIe se iz n-Elanog skupa izabrati na n2 naEina. Stoga je odgovarajuka funkcija izvodnica:

Da bismo izraEunali ovu sumu, napigimo n2 kao linearnu kombinaciju izraza n(n - 1) i n . Tada Cemo svaku od suma moLi interpretirati kao derivaciju formalnog reda l + x + x 2 + . . . :

f (x)

= =

n)O

C (n(n - 1) + n) xn C n ( n - I)xn + C nxn


n20

n20

% Za vjeBbu izraEunajte koeficijent uz xn u f (x) .

?+ RijeSite analogni zadatak za uredene trojke. % SliEno se moZe izraEunati funkcija izvodnica za bilo koju k-torku elemenata
n-Elanog skupa. To je posljedica toga Lto su padajuCi faktorijeli jedna baza vektorskog prostora svih polinoma, pa se bilo koji polinom (pa tako i polinom n k u varijabli n ) mo2e napisati pomoCu njih. PokuHate li raspisivati potencije n k po bazi sastavljenoj od padajuCih faktorijela u varijabli n , vidjet dete da to ne ide sasvim lagano. No, moIe se vidjeti d a su koeficijenti uz (n)l u razvoju n k po padajuCim faktorijelima jednaki Stirlingovim brojevima S(k, 1 ) , tj. brojevima particija k -Elanog skupa u 1 dijelova. Usporedite koeficijente i brojeve particija za male k . DokaIite jednakost tih koeficijenata brojeCi sva preslikavanja iz k -Elanog u n-Elani skup. Grupirajte ih prema veliEini slike.

6 . FUNKCIJE IZVODNICE

Neka je n E N zadan. NapiSite funkciju izvodnicu za broj podskupova n-Elanog skupa, sortiranih prema veliEini.
6.
Rjes'enje 1. Ranije smo izraEunali d a k -Elanih podkupova n-Elanog skupa ima Funkcija izvodnica glasi:

(E) .

Rjes'enje 2. "Prava" metoda za nalaienje funkcije izvodnice ne bi u sebi trebala imati uvrgtavanje gotovih rezultata, veC bi se do nje trebalo doCi nekim kombinatornim putem. Nakon toga se funkcija izvodnica mofe upotrijebiti za raEunanje raznih veliEina.

Za svaki element zadanog n-Elanog skupa treba reCi ulazi li u podskup ili ne. Nakon toga treba prebrojati one podskupove u koje je uzlo 0, 1, 2 itd, elemenata, svaku skupinu posebno. Rezultat brojanja za svaku od skupina treba napisati kao koeficijent uz odgovarajuku potenciju od x . Lako se vidi da C e u produktu (1

+ x) . (1 + x) . . . . . (1 + x)

( n faktora)

stajati uz potencije od x upravo ti feljeni koeficijenti. Naime, ovaj dugaEki produkt mofe se izraEunati tako da se mnoZi "svaki sumand sa svakim". Sumande prvog faktora moiemo protumaEiti kao odluku - ulazi li prvi element u podskup ili ne. Ako u raEunanju e odgovarati situaciji produkta ovih polinoma iz prvog faktora izaberemo sumand 1, to C u kojoj prvi element ne ulazi u podskup, a ako izaberemo x , to C e odgovarati situaciji u kojoj prvi element ulazi u podskup. SliEno i ostale faktore pridruIimo ostalim elementima. Uzimanjem x -a s potencijom 1iz nekog faktora poveCavamo veliEinu izabranog podskupa za 1, a uzimanjem x-a s potencijom 0 ne mijenjamo izabrani podskup. Na kraju C e se u produktu (nesreaenom) pojaviti potencija xk onda i samo onda ako smo u k faktora izabrali sumand x , a u ostalima 1, Eto znaEi d a C e svaka pojava xk u produktu odgovarati jednom k -Elanom skupu. Nakon zbrajanja (grupiranja) svih Elanoe upravo biti jednak broju svih k-Elanih podskupova va, koeficijent koji C e stajati uz xk C zadanog n -Elanog skupa.

tP Sada, kad imamo ovu funkciju izvodnicu, moZemo ponovno izraEunati broj svih podskupova n-Elanog skupa. On je jednak sumi koeficijenata u dobivenoj funkciji izvodnici, tj. f(1) = (1 l)n = 2".

tP Primjenom binomnog teorema mofemo izraEunati koeficijent uz bilo koju potenciju od x .


Za koje objekte bi ovo bila eksponencijalna funkcija izvodnica?

7. NapiSite funkciju izvodnicu za broj izbora n komada kamena iz izvora od neograniEeno mnogo granitnih kocaka, neograniEeno mnogo kamenih ploEa i neograniEeno mnogo oblutaka. Bitno je samo koliko C e biti komada kamena koje vrste.
Rjes'enje. Mofemo uzeti koliko Ielimo kamenih kocaka, koliko ielimo kamenih ploEa i koliko Ielimo oblutaka. Kad god de suma broja kocaka, broja ploEa i broja oblutaka biti jednaka n , to C e biti izbor n komada kamena. I obratno, svaki izbor n komada kamena

6. FUNKCIJE IZVODNICE

bit de karakteriziran brojem kocaka, brojem ploEa i brojem oblutaka, Eija je suma jednaka
n.

Dakle razliEiti naEini izbora odgovaraju pojedinim produktima u izrazu: ( 1 + x + x 2 + x 3 + ...).( 1 + x + x 2 + x 3 + . . . ) . (1 + x + x 2 + x 3 + ...), jer umjesto kocaka uzimamo odgovarajuti broj x-eva iz prve zagrade, umjesto ploEa iz druge, umjesto oblutaka iz trete. Tako potencije od x u prvoj zagradi predstavljaju izbore broja kocaka, potencije u drugoj zagradi izbore broja kamenih ploEa, a potencije u tredoj izbore broja oblutaka, a produkt xaxixk odgovara izboru i kocaka, j ploEa i k oblutaka za sve i, j i k . Nakon sredivanja (zbrajanja Elanova koji predstavljaju istu potenciju od x ) uz xn de stajati broj sumanada oblika xixixk , tj. broj naEina izbora n komada kamena. U zatvorenom obliku dobivenu funkciju izvodnicu moiemo zapisati kao: 1 (1 - x)3 '

+f OEito je d a smo zadatak mogli formulirati i kao traZenje funkcije izvodnice za broj nenegativnih cijelih rjeLenja jednadibe i j k = n , sloiienih prema broju n .

+ +

+f Koeficijent uz xn u ovoj funkciji izvodnici je jednak (-l)n(;3)


(n$2)

(n:2)

, Lto kao broj izbora znamo izraEunati i direktno. 9+ Za Lto bi dobiveni formalni red bio ekponencijalna funkeija izvodnica? 9+ Ovdje se, kao i u zadatku o nizovima slova i brojeva, pojavilo mnoienje formal-

nih redova. Usporedite t a dva zadatka. Objasnite ;to znaEi mnoZenje obiEnih funkcija izvodnica. Usporedite jog ovaj zadatak s prethodnim.

9+ Kako Einjenica da smo pretpostavili beskonaEne izvore objekata (kamenja, u ovom sluEaju) utjeEe na konaEni rezultat? Zahvaljujudi tome Lto su redovi s kojima smo raEunali bili beskonaEni, raEun je bio jednostavan (ponekad to i bitno vise olakgava raEun nego u ovom sluEaju!), no d a li je to utjecalo na rezultat?
Prisjetimo se zagto raEunamo funkcije izvodnice - zato da dobijemo konkretne koeficijente. A koeficijent uz npr. xn je isti ako mnoZimo beskonaEne redove i ako svaki od redova u produktu ograniEimo do potencije xn (uvjerite se u to). Dakle, za pojedini n nije potrebno imati beskonaEno, nego samo "dovoljno mnogo" kocaka, ploEa i oblutaka koje moiemo uzeti (po n od svake vrste, a to bismo pretpostavili i u direktnom raEunanju).

8. Na Skolskim priredbama sudjeluju pjevaEi, mali i veliki plesaEi, te ponekad glumci. Treba naCi obiEnu funkciju izvodnicu za broj razliEitih sastava za Skolske priredbe sloiene prema broju izvodaEa ako je bitan bitan samo broj uEenika u pojedinoj skupini.

Svako dijete moBe preuzeti najvige jednu ulogu. PjevaEa treba biti barem 30, broj velikih plesaEa treba biti djeljiv s 2, a broj malih plesaEa mora biti djeljiv s 3. Pretpostavimo da 6kole imaju po 1000 uEenika koji mogu pjevati i glumiti, dok po 500 djece mo5e biti ukljuEeno u grupu velikih, odnosno malih plesaEa.

6. FUNKCIJE IZVODNICE

Rjeienje. I ovdje, kao i u prethodnom zadatku uzimamo po nekoliko "objekata" svake od raspoloZivih vrsta. Izbore trebamo ponovno sortirati prema broju objekata koje smo izabrali. Razlika u odnosu na prethodni zadatak je u tome ;to sada ne moZemo sasvim slobodno birati objekte pojedinih vrsta. Ako nam izbor potencije varijable x iz prvog faktora (zagrade) oznaEava broj pjevaEa, onda se u njoj smiju pojavljivati samo potencije od x vede ili jednake 30 i manje ili jednake 1000. Drugi faktor odredimo za broj malih plesaEa - u njemu C e biti samo potencije od x Eiji eksponent je djeljiv s 3, a manji ili jednak 500. SliEno odredimo tredi faktor za brojanje velikih plesaEa i Eetvrtu za glumce. Funkcija izvodnica glasi:
f (x)

+ + . . . + xlooO 1. .(a3 + z6 + z9 + . . + 2498) . .(x2 + x4 + xS + . . . + X5O0) .(1 + x + x 2 + ... + X l o o 0 1


(X3O
X3l

+f' Izbore plesaEa mogli smo obrazloZiti i ovako: biramo broj parova ili trojki, a svaki par donosi dva nova sudionika ( x 2 ), a svaka trojka tri ( x 3 ).
+f RjeSavali smo jednadZbu i 21j, 3lk, te i,1 6 1000 i j , k 500.

<

+ j + k + 1 = n u skupu N:

uz ogranizenja: i 3 30,

9. Napis'ite obiEnu funkciju izvodnicu za broj naEina da se postigne suma n pri bacanju deset razliEitih igrakih kocaka.

Izvedite iz te funkcije izvodnice funkciju izvodnicu za broj naEina da se postigne parna suma, te izraEunajte broj razliEitih dobivanja parne sume.
Rjeienje. Ovdje izabiremo (ili l1sreCa" izabire umjesto nas) broj toEkica s 1 , 2 , . . . , l o te kocke. Kao i do sada, umjesto toEkica s i-te kocke uzimamo odgovarajuh potenciju varijable x iz i-tog faktora. Na svakoj od kocaka nalaze se napisani brojevi 1 do 6, pa je funkcija izvodnica jednaka:

f (x) = (a: x2

+ + x3 + x4 + x5 + X6)l0.

U funkciji izvodnici za sve parne sume trebaju uz parne potencije varijable x biti koeficijenti jednaki odgovarajutim koeficijentima u f (x) , dok koeficijenti uz neparne potencije
od x trebaju biti 0. Stoga je funkcija izvodnica za parne sume jednaka:

Ovdje se, kao i kod svih ost'alih operacija s formalnim redovima, radi o formalnom uvrstavanju: umjesto x stavimo u red -x i postupamo po pravilima raEunanja.

6. FUNKCIJE IZVODNICE

Broj svih parnih suma je jednak sumi koeficijenata u formalnom redu p ( z ) , a to je jednako p ( 1 ) = ( f ( 1 ) f ( - 1 ) ) / 2 = (61 0 1 ) / 2 = 3 .6' .

IzraEunajte obiEnu funkciju izvodnicu za broj neparnih suma, te broj svih neparnih suma.

9 9

Zagto smo 1 uvritavali u poEetni, a ne u zavrgni izraz za p ( z ) ?

9-t Izvedite rezultat za broj svih parnih suma direktno, dokazujuki d a je broj parnih suma jednak polovini ukupnog broja ishoda.
10.

IzraEunajte broj rjeSenja jednadzbe

Ck21 kxk = n

iz skupa N o .

Rjegenje. Da bi se dobila suma jednaka n , moramo uzeti z l puta jedinicu, 2 2 puta dvojku, 23 puta trojku itd. Ili drugim rijeEima: a1 puta jedinicu, 22 puta paket sastavljen od dvije jedinice, 23 puta paket sastavljen od tri jedinice itd.

Neka, kao i do sada, formalna varijabla x predstavlja jednu jedinicu u sumi koja treba doseCi n . Tada izbor z l mokemo prikazati kao izbor sumanda iz 1 s + z 2 z 3 . . . . Sumi 22 pridonosi dvostruku svoju vrijednost, pa njegove izbore prikazujemo izborom ~ ( z 2 ) 3 . . . . SliEno za izbore z 3 promatramo sumu potencija sumanda iz 1 z2 ( z ' ) + z 3 itd. Kao i ranije, d a bi se veliEina objekata koje sastavljamo poveiavala s eksponentom formalne varijable, koristimo mnokenje formalnih varijabli da bismo opisali sastavljanje objekata. Funkcija izvodnica tada glasi:

+ +

+ +

(1+z+z2+...).(1+z2

+ ( z 2 ) 2 + . . . ) . ( I + ~ ~ + ( ~ ~ ). .~ ... . . ) .

Nakon mnoienja opCi Elan u produktu je:

.( z 2 ) ~ 2 .(z3)~3 .... = zx1+2~2+3~3+...


Potenciju an dobit Cemo onda i samo onda ako je Sredimo li izraz za funkciju izvodnicu, dobivamo:

k z k = n , kao ito smo i keljeli.

9-t Usporedite ovaj zadatak sa zadatkom s uEenicima. I ovaj zadatak broji rjegenja y2 . . . yn = n s nekim ograniEenjima. Izrecite t a ograniEenja. jednadibi yl

+ + +

11. IzraEunajte obiEnu funkciju izvodnicu za razliEite sume brojeva na igraCim kockama ako bacamo jednu kocku proizvoljno mnogo puta, a nije bitan redoslijed dobivanja brojeva.
Rjegenje. Sumu n mozemo dobiti kao sumu nekoliko jedinica, nekoliko dvojki, itd, te nekoliko gestica. OznaEimo li broj bacanja u kojima je "pao" broj i s x i , onda vidimo da trebamo rjegavati jednadibe:
xl

+ 2x2 + 3x3 + 4 z 4 + 5 z 5 + 6x6 = n.

Funkcija izvodnica je, kao i u prethodnom zadatku jednaka:

6. FUNKCIJE IZVODNICE

12. U jednom gradu ulice u smjeru istok-zapad su glavne ulice i u njima se nalaze trgovine, restorani i kafiCi. Ulice u smjeru sjever-jug sluSe iskljuEivo kao ulazi u stambene zgrade. Taj grad ima tlocrt u obliku trokuta i leZi uz rijeku.

Na koliko naEina mozemo stiCi od ulaza u grad koji se nalazi u ishodigtu do rijeke, ako pritom moSemo svratiti i u neke lokale. Izmectu svaka dva raskrgta je u svakoj ulici u smjeru istok-zapad toEno 1 raznih lokala. Puteve klasificirati prema mjestu dolaska na rijeku (npr. prema ordinati toEke izlaska na rijeku).
Rjes'enje. BuduCi d a smo se vet navikli na promatranje puteva u koordinatnoj mreii, promatrajmo plan grada umjesto samog grada.

Da nema lokala, puteve bismo birali tako, d a za svaki horizontalni korak uzimamo po jedan x , dok horizontalni koraci ne bi utjecali na stvaranje sumanada u funkciji izvodnici. Prvi korak bi bio horizontalni ili vertikalni - birali bismo jedan od Elanova izraza 1 x . SliEno i za drugi, treCi i ostale korake, pa bismo dobili produkt sastavljen od izraza 1 x . Koraka je ukupno n , pa bi funkcija izvodnica bila (1 x)lO.

+ +

NaS je problem u tome Sto u horizontalnim koracima moiemo navratati u lokale. U svakom horizontalnom koraku moiemo birati izmedu I lokala koliko ih ima izmedu dva raskrSLa. Dakle izmeau dva raskrSCa na istoj visini moIemo na 2' naEina provesti vrijeme. Za svaki horizontalni korak broj puteva (koji sada ukljuEuju i posjete lokalima) se poveCava 2' puta. Zato je ova funkcija izvodnica jednaka (2' s)lO.

9+ Pogledajte jog jednom zadatak o nizovima slova i brojeva. Objasnite porijeklo konstanti u tim funkcijama izvodnicama.
13. Dano je 2mn predmeta, po 2n svake vrste. IzraEunajte broj raspodjela tih predmeta u dvije jednakobrojne skupine.
Rjegenje. Dovoljno je izdvojiti samo jednu skupinu. NapiSimo funkciju izvodnicu za broj izbora jedne skupine i potraIimo u njoj koeficijent uz xmn .

6. FUNKCIJE IZVODNICE

Iz svake od m skupina nezavisno biramo 1 , 2 , . . . , 2 n - 1 ili 2n elemenata:

f(x)

(1+x+x2+...+x2n)m

+P Da smo imali izbore iz beskonaEnog izvora, dobili bismo rezultat 1 / ( 1 - x ) ~ . Tako je faktor ( 1 - x2n+1)ms jedne strane odgovoran za ograniEenja, a s druge strane u sumu donosi faktore (-ilk .
To smo i oEekivali, jer zadatak bez ograniEenja znamo rijeiiti direktno, a u sluEaju da izvori predmeta nisu beskonaEni moramo koristiti formulu ukljuEivanja-iskljuzivanja.

14. Ako s a ( x ) oznaEimo funkciju izvodnicu za niz (a,), a s b(x) funkciju izvodnicu za niz (b,) , dokagite da vrijedi:

(a) ako za neki p E N vrijedi a, = b,+, za svaki n E N o , onda je a ( x ) = ~ - p ( b ( ~ -) bo - blxl - . . . - b p - l ~ ~ - l ) , (b) ako je a, = za svaki n E N o , onda je a ( x ) = b l ( x ) , (c) ako je a, = b, b,+l za svaki n E N o , onda je a ( x ) = b(x) x b ( x ) , (d) akoje an=bo+bl+ b, zasvaki n E N o , o n d a j e a ( x ) = b ( x ) / ( l - x ) , bn+2 . . . za svaki n E N o , onda je a ( x ) = (e) ako je a, = b,+l (b(1)- b ( x ) ) / ( l- 7 (f) ako je a, = bz, za svaki n E N o , onda je a ( x ) = ( b ( f i ) b ( - f i ) ) / 2 .

...+ + +

Rjes'enje. Sve ove formule lako se provjere direktnim uvritavanjem. Pronadite u prethodnim primjerima gdje smo ove formule veC koristili.

Nemojte formule iz ovog zadatka uEiti napamet kao "recepte" - moiete ih uvijek +f' po potrebi izvesti, tim vise Lto se najEeHCe ne javljaju u ovom obliku, nego s nekim pomakom u indeksima. Vrijedno ih je samo zapamtiti kao osnovne principe, npr. d a se moie realizirati pomak u indeksima, razdvojiti parni od neparnih Elanova, zatim d a zbrajanje koeficijenata ("Eistih" ili pomnoienih s nekim faktorom), pri Eemu se duljina zbroja mijenja s indeksom, obiEno potjeEe od mnoienja formalnih redova, d a mnoienje koeficijenata nekim faktorom koji ovisi o indeksu upuCuje na deriviranje ili integriranje i sl.

% Za koje od ovih relacija moiete napisati analogne relacije za ekponencijalne funkcije izvodnice?

15. Nadite obiEne funkcije izvodnice za nizove rjeSenja sljedecih rekurzivnih relacija:

112

6. FUNKCIJE IZVODNICE

Iz tih rekurzivnih relacija izraeunajte rjegenja.


Rjes'enje. Trik koji se primjenjuje pri rjegavanju rekurzivnih relacija je jednostavan. Relacija se pomnoti s xn ili nekom viSom pogodnom potencijom, a zatim se tako dobiveni izrazi za n = 0,1,2,. . . zbroje. Zbrajanjem se dobiju traiene funkcije izvodnice. Nakon toga po potrebi moiemo izraEunati rjeHenja - koeficijente u funkciji izvodnici.
(a) Izraz Cemo pomoiiti s sn+',te rezultat pripremiti za zbrajanje (podsjetite se, nakon zbrajanja Zelimo dobiti funkciju izvodnicu): Zbrajanjem po n E N slijedi (a(z) - ad

-0

- alz) + 5z . (a(z) - ao) + 6z2a(z) = 0,

iz Eega se lako uvrZtavanjem poEetnih uvjeta i sredivanjem dobiva rezultat

NapiSimo rezultat u obliku iz kojeg se mogu odmah oEitati rjeZenja:

Sada se vidi d a je prvi sumand odgovoran za potencije (-2)n koje se javljaju u rjeZenju, a drugi za potencije (-3)n . RjeHenje rekurzivne relacije se dobiva razvijanjem ovih razlomaka u red i oduzimanjem i glasi: a n = 4 . (-2)n

- 3 . (-3)n.

9+ PokuZajte rekurzivnu relaciju pomnoiiti s xn+' ili s z n i onda zbrojiti! Rezultat treba biti jednak dobivenom. (b) Mnotenjem relacije s zn+' i zbrajanjem relacija za n E N dobiva se izraz:

Izraz na desnoj strani se moie lako sumirati - u njemu treba samo prepoznati derivaciju reda CT=o ( 2 ~. ) Imamo ~ redom ? 2 ^ P = 22 n(2x)"-' = z (I/('- 2")))' = 2x/(1 - 22)' . Uvrstavanjem slijedi rezultat:

Cn>o

znao

Izraz se moie joS malo srediti rastavljanjem na parcijalne razlomke. Tako se dobiva pregledniji oblik rezultata:

iz Eega se mote lako proEitati rjegenje:

9+ UoEite u funkciji izvodnici dio koji potjeEe od homogene jednadzbe. Kao dopunski primjeri mogu vam posluiiti zadaci prethodnih poglavlja. Pokugajte naCi pravilo koje bi govorilo kako C e izgledati rjeSenje proizvoljne homogene jednadtbe. Moiete li ne'ito reCi o izgledu funkcija izvodnica za nehomogene jednadtbe tipova obradenih u prethodnom poglavlju?

6 . FUNKCIJE IZVODNICE

16. NapiSite eksponencijalnu funkciju izvodnicu za broj permutacija bez fiksnih toEaka.
Rjes'enje 1. Broj permutacija n-Elanog skupa bez fiksnih toEaka smo veC izraEunali u poglavlju o formuli ukljueivanja-iskljdivanja i on iznosi n! . ~ i , ~ ( - l ) ~ / . k UvrBtava! njem u opCi izraz za eksponencijalnu funkciju izvodnicu dobivamo:

Rjes'enje 2. Puno je ljepBe direktno izraEunati funkciju izvodnicu - bez uvritavanja L L g ~ t o ~rjegenja. ihli Sve permutacije 'n-Elanog skupa moiemo razvrstati prema broju fiksnih toEaka koje sadr8e. Permutacija s k fiksnih toEaka ima (E)Dn-k, gdje je Dn-k broj permutacija bez fiksnih toEaka (n - k) -Elanog skupa. Slijedi:

( Do = 1). Dijeljenjem s n dobivamo:


1

= =

1 1 -Dn Dn-1 O!n! l!(n - I)! (xn>exe(x).

1 D,-k + . . . + -Do + . . . + k!(n 1 - k)! n!O!

Formalni red koji ima sve koeficijente jednake 1 je 1/(1 - x) , pa imamo:

9-t U poglavlju o rekurzivnim relacijama susreli smo se s rekurzivnom relacijom za broj permutacija bez fiksnih toEaka. Bila je to rekurzija reda 2 s nekonstantnim koeficijentima. IzraEunajte iz nje, mnoieki je odgovarajukim faktorom i sumiranjem izraz koji povezuje eksponencijalnu funkciju izvodnicu e i njenu derivaciju e' . Oni koji znaju rjeBavati obiEne diferencijalne jednadibe moki C e napraviti i zavrBni korak - iz tog izraza izraEunati e(x) .
17.
Nadite funkciju izvodnicu za broj kvadrata na n x n Sahovskoj ploCi.

Rjes'enje 1. Na n x n Bahovskoj ploEi ima n2 kvadrata 1x 1, (n- 112 kvadrata 2 x 2 , (n - 212 kvadrata 3 x 3 itd. Ukupno, broj kvadrata iznosi 1 ' 22 . . . (n - 1)2 n2 .

+ + +

Sada mogete dalje izraEunati funkciju izvodnicu tako da dobiveni rezultat uvrstite u opki oblik funkcije izvodnice koju potom interpretirate kao neki produkt ili tako da se prvo prisjetite da je 1 ' 22 . . . (n - 1)2 n2 = n(n 1)(2n 1)/6, pa onda t a j

+ + +

6. FUNKCIJE IZVODNICE

rezultat uvrstite u opCi oblik funkcije izvodnice. U oba sluEaja ima jog dosta posla oko sredivanja dobivene funkcije izvodnice. Pokuiajte barem skicirati osnovne korake.

Rjes'enje 2. Smjestimo sve n x n iahovske ploEe tako da im je lijevi donji vrh u ishodiitu koordinatnog sustava. Nazovimo n x n gahovsku ploEu n-tom ploEom.
OznaEimo broj kvadrata na n-toj ploEi s kn . Svaka cjelobrojna toEka (n - 1) -ve ploEe jedinstveno odreduje jedan kvadrat koji je sadrgan u n-toj ploEi, ali nije u n- 1-voj (kvadrat koji ima lijevi donji vrh u toj toEki, i ide do ruba n-tog kvadrata). Broj cjelobrojnih toEaka u n - 1-vom kvadratu je n2 je n2 , pa vrijedi kn = n2 kn-l . Mnogenjem relacije s xn i sumiranjem za n E N dobiva se izraz za funkciju izvodnicu.

Rjeienje 3. Prethodna dva rjeienja opisana su samo ukratko, jer koriste metode koje su vet upotrijebljene u prethodnim zadacima. Ovo je jog jedno rjeienje koje nalazi funkciju izvodnicu bez prethodnog poznavanja rezultata.
Rubom n-te ploEe nazovimo skup toEaka (x, y) za koje vrijedi max{x, y) = n . IzraEunat Cemo prvo broj svih kvadrata (bez obzira na veliEinu) u kojima je desni gornji vrh na na rubu n-te ploEe. Ako funkciju izvodnicu za te brojeve oznaEimo s p(x) , a pripadne brojeve kvadrata s pn , onda C e za brojeve kn , n E N vrijediti: kn=pl+pz+ te C e funkcije izvodnice biti povezane formulom:

...+pn,

(koristili smo poEetne uvjete po = ko = 0 ) . IzraEunajmo sada funkciju izvodnicu za broj kvadrata kojima je gornji desni vrh na rubu n-tog kyadrata. Svaki od kvadrata je odreden svojim donjim lijevim i gornjim desnim vrhom. Zelimo da svim kvadratima koji "zavrZavaju" na rubu n-tog kvadrata bude pridrufena potencija xn .

Izbor donjeg lijevog vrha indeksirat Cemo brojem kvadrata na kojem se on nalazi (za kvadrat na slici to bi bio 2), a umjesto gornjeg desnog vrha birat Cemo duljinu stranice. Broj naEina izbora toEke s ruba n-tog kvadrata je 2n 1, pa je funkcija izvodnica za izbor donjeg lijevog vrha jednaka 1 32 5s2 . . . . Funkcija izvodnica za izbor duljine stranice je x z 2 x3 . . . . BuduCi da su ovi izbori medusobno nezavisni, izboru kvadrata odgovara formalni red koji je jednak produktu formalnih redova koji opisuju dva

+ + +

+ +

6. FUNKCIJE IZVODNICE

115

"osnovna" izbora:

p(z)

= =

( 1 3 z 5 z 2 7 z 3 . . .) . ( x z2 z3 . . .) (neparne koeficijente prikaiemo kao k = 2i + 1 ) [2.(1+2z+3z2+4z3+

+ +

+ + +

...)-( 1 + z + z 2 + z 3 +

...

Rezultat, prema prethodnom, glasi

Kombinat orno dokazivanje identiteta


U ovom poglavlju bavit Cemo se dokazivanjem identiteta koji mogu koristiti pri pojednostavljivanju raznih izraza. VeCinu njih moHe se dokazati matematiEkom indukcijom ili direktnim raEunom. No ti dokazi su sa stanovista kombinatorike nuHno zlo, jer se iz njih ne moHe lako oEitati bit dokazivane formule. Sigurno ste pun0 puts: rjesavali zadatak zajedno s kolegom i dobili naoko razliEite rezultate, oboje uvjereni da ste do njega doSli toEnim postupkom. Ako ste zaista oboje zadatak rijeSili bez pogregke, oba rezultata moraju biti toEna, te ste u stvari dobili identitet koji povezuje vaga dva rjeSenja. Kombinatorni dokazi priEaju o formulama. U svakom dokazu ispriEat Cemo prvo priEu o jednom naEinu prebrojavanja nekih objekata - takvom naEinu koji kao rezultat daje izraz koji nalazimo na lijevoj strani jednadHbe. Zatim Cemo naCi priCu koja opisuje naEin prebrojavanja istih objekata koja kao rezultat daje izraz koji nalazimo na desnoj strani jednakosti. BuduCi da smo dvaput prebrojavali iste objekte, rezultati moraju biti jednaki, te je time zadani identitet dokazan. Ponekad Cemo interpretirajuti lijevu stranu brojati objekte jedne vrste, a interpretirajuki desnu stranu objekte druge vrste. U tom sluEaju dokaz zavrSava uspostavljanjem bijekcije izmedu navedenih dvaju skupova objekata. PriEu moHemo priEati o razliEitim objektima. U veCini zadataka ispriEat Cemo viSe priEa: o prebrojavanjima nekih podskupova zadanog skupa, o prebrojavanju nekih puteva u cjelobrojnoj koordinatnoj mreZi, te Cemo pokusati dokazati identitet raEunski, pomoCu formalnih redova, odnosno funkcija izvodnica. Direktni dokazi bit C e provedeni samo tam0 gdje nema drugog izbora. U ostalim situacijama provedite direktne dokaze, ako Helite, sami. IspriEajte i svoje prize o objektima koji su vam bliski. Dokazi pomoCu puteva i dokazi pomoCu skupova su uglavnom jednaki, uz prirodnu bijekciju izmedu podskupova zadanog skupa i razmjegtaja horizontalnih koraka unutar puta. Neke od ovih formula Eesto su korisne u sredivanju nekih izraza. No ako zaista nauEite dokazivati kombinatorno, potreba za njihovim uEenjem, odnosno

7. KOMBINATORNO DOKAZIVANJE IDENTITETA

zapisivanjem bit C e bitno manja, jer Cete postati toliko vjeSti da Cete u svakom trenutku moCi ponovno ispriEati priCu kojom sredujete izraz. Ako budete crtali skice dokaza, odnosno objekata koje brojite, bit C e vam joS lakSe podsjetiti ih se. Shvatite ovo poglavlje kao ponavljanje prethodno svladanih vjegtina! Ponovimo prije rjeSavanja zadataka definiciju binomnih koeficijenata koja C e nam ovdje koristiti. Binomni koeficijenti (L) mogu se definirati kao broj k -Elanih podskupova n Elanog skupa, ali i formulom:

Ova formula omoguCava definiranje binomnih koeficijenata i za n < 0 , a Eak i za proizvoljne realne n . Nasuprot tome, za ovu formulu k mora biti element skupa N o . Prema ovoj definiciji binomni koeficijent jednak je 0 za k E { n 1,n 2, . . .) . Binomni koeficijent (L) definiramo da je jednak nuli za k @ N o . Cinjenica da je binomni koeficijent jednak nuli izvan prirodnog podruzja definicije korisna je pri zapisivanju raznih suma. Naime, Eesto je teSko zapisati toEne granice sumacije, pogotovo ako se zbrajaju izrazi koji u sebi sadrtie vise binomnih koeficijenata. Tada se napiSe da se sumira npr. po svim k E Z , a da Elanovi u kojima se nalazi binomni koeficijent koji nije definiran formulom ne smetaju, jer su jednaki nuli.

1.

Dokatiite da za svaki prirodni broj n i svaki k E N, U (0) vrijedi:

(1)=

Rjezenje. Skupovi. broji k -?lane podskupove n -Elanog skupa. broji (n - k) -Elane podskupove npr. istog skupa. Jednostavno je nati jednu bijekciju izmedu skupa svih k -Elanih podskupova n-Elanog skupa i skupa svih njegovih (n - k) -Elanih podskupova: svakom podskupu pridru6imo njegov komplement. Zaista, komplement k Elanog podskupa u n -Elanom skupu jest ( n - k) -Elan. Izborom k -8anog podskupa nekog n-Elanog skupa istovremeno je izabran i njegov n - k-Elani komplement. Komplementiranje je bijekcija, jer je sama sebi (obostrani) inverz.

(i)

(,Fk)

Putevi. ) ( ; je npr. broj svih puteva u cjelobrojnoj koordinatnoj mreii koji poEinju u ishodigtu a zavrgavaju u toEki (k, n - k) . No, tih puteva ima jednako kao i puteva do toEke (n - k, k) gto je lako vidjeti zrcaljenjem slike oko pravca y = x. Puteva do toEke i tvrdnja je dokazana. (k, n - k) ima prema istoj formuli

(,Fk)

Prisjetite se kako smo izraEunali broj puteva od ishodigta do neke toEke: u nizu ' = I + koji se sastoji od n koraka odredili smo na kojim t e mjestima stajati koraci udesno i time jedinstveno odredili na kojim C e mjestima stajati koraci prema gore. Rezultat je bio ) ( ; . Jednako tako mogli smo prvo odrediti na kojim C e mjestima stajati koraci prema gore i dobili bismo rezultat .

(,Ek)

118 Formalni redovi.

7. KOMBINATORNO DOKAZIVANJE IDENTITETA

Izrazi (1

+ z)n i

(z

+ l)n

su jednaki, pa su im nuino jednaki i koeficijenti uz xk .

2.

DokaEite da je za svaki n E N i za svaki k E N, :

Rjeienje. Skupovi. Lijeva strana broji k-Elane podskupove skupa Nn . Desna strana broji prvo (k - 1) -Elane podskupove skupa Nndl , a zatim k -Elane podskupove Nn-1 . Kakve to ima veze jedno s drugim? Svaki k-Elani podskup od Nn moZe ili sadrzavati element n ili ne. Onih koji sadriavaju element n ima koliko i (k - 1) -Elanih podskupova od Nn-1 , a ostalih koliko i k-Elanih podskupova od NnF1 . Putevi. Lijeva strana broji sve puteve od (0,O) do (k, n - k) . Desna strana raEuna sumu broja puteva od ishodigta do toEke (k - l , n - k) i broja puteva do toEke (k, n - k 1 ) . To su jedine dvije toEke iz kojih se moie doCi do (k, n - k) , pa moiemo reCi d a je izraz na desnoj strani napisan tako d a klasificira sve puteve do (k, n - k) u ovisnosti o zadnjem koraku - d a li je on u smjeru prema desno ili prema gore.

Formalni redovi. ( 1 + ~= ) (1 ~ +z)"-l(l +z).

) ( ; je jednak koeficijentu uz zk u izrazu na lijevoj strani, a ( n ~ l ) uz istu potenciju od x u izrazu na desnoj strani.

+ (;I:) koeficijentu

+f Ova relacija daje pravilo kojim se raEunaju binomni koeficijenti pomoCu Pascalovog trokuta.

3. DokaEite da za svaki prirodni broj n i svaki k E Nn vrijedi:

(i)= i (;I:) .

Rjeienje. Transformirajmo izraz kojeg treba dokazati d a bismo pojednostavnili njegovu interpretaciju. ObiEno lako prepoznajemo od kakvog slaganja objekata potjeEe mnoienje, pa pokugajmo dokazati: k(;) = n ( ; ~ t ) . Slcupovi. k(;) nam kaie d a smo prvo izabrali k-Elani podskup n-Elanog skupa, a zatim izabrali jedan njegov element. Dakle, imamo par (A, z ) , gdje je A k -Elani podskup od Nn , a x neki njegov element. Desna strana n(;~:) nam kaie d a smo prvo izabrali neki element polaznog, n Elanog, skupa a potom medu preostalim elementima zadanog skupa izabrali jog (k - 1) Elani podskup. Sada imamo par (y, B ) , gdje je B neki (k - I ) -Elani podskup od N, , a y E Nn nije element iz B . Lako je naCi preslikavanje izmedu ovih dvaju skupova uredenih parova: ('49x1 (2, A \ {XI).

7. KOMBINATORNO DOKAZIVANJE IDENTITETA

To preslikavanje je bijekcija, jer mu je (y,BIc, (BU{Y),Y) obostrani inverz (dokaiite!). Dakle uredenih parova koje smo brojali na lijevoj strani ima jednako kao i uredenih parova koje smo brojali na desnoj strani, pa vrijedi identitet kojeg je trebalo dokazati.

tP

Lijevu stranu smo interpretirali kao ( t ) k .

Putevi. k ( t ) broji sve puteve od ishodizta do toEke (k, n - k) s istaknutim jednim horizontalnim segmentom (segment po kojem neEemo trEati, nego Eemo hodati). Desna strana, n ( $ ~ : ) , broji sve puteve s istim tim poEetkom i krajem (k - l , n - k), ali s istaknutom jednom (cjelobrojnom) toEkom na tom putu.
Bijekcija izmedu tih dvaju tipova objekata je oEigledna - vadenje istaknutog horizontalnog segmenta i stavljanje istaknute toEke na mjesto na kojem je poEinjao. Lako se konstruira i inverz ovog preslikavanja.

Formalni redovi. Izraz na lijevoj strani, k ( t ) , pokazuje da bi bilo dobro promatrati derivaciju polinoma C L O x k = (1 x ) ~ .Lijeva strana je koeficijent uz sk-' u (Cr=o zk)'. S druge strane t a derivacija je jednaka n ( l x ) ~ - ' , pa je koeficijent uz 2 ~ - 1 jednak desnoj strani izraza kojeg treba dokazati.

4.

DokaZite da je za svaki n E Z i svaki k E No :

RjeSenje. Dokaz se moie lako provesti direktnim raEunom (Eak za svaki realni n!). Formalni redovi. PotraZimo brojeve iz ovog zadatka u nekom poznatom izrazu! Za n 2 0 ako faktor (-l)k prebacimo na lijevu stranu, dobivamo tvrdnju da je (1 - x ) - ~ funkcija izvodnica za broj raspodjela k jednakih predmeta na n ljudi, za ;to otprije znamo d a je istina.
Da tvrdnja vrijedi za svaki n E Z dobivamo na sljededi naEin. Lijeva i desna strana jednakosti koju treba dokazati su polinomi stupnja k u varijabli n . Oni poprimaju iste vrijednosti u k 1 toEaka (;tovise, u beskonaEno mnogo - u svim pozitivnim n ) , pa su zbog toga jednaki (imaju jednake koeficijente). Zato imaju jednake vrijednosti u svim toEkama n E R .

? + U dokazu je bio spomenut jedan vrlo vaZni teorem iz algebre: ako dva polinoma stupnja k imaju iste vrijednosti u k 1 toEaka, onda su oni jednaki. ProEitajte ga za k = 1. Potraiite t a j teorem, prouEite dokaz i njegove posljedice. Zapamtite t a j teorem, koristan je!

tP citamo li ovu jednakost zdesna nalijevo, dobivamo formulu kojom se pri raEunanju suma moiemo rijegiti Eesto neugodnog indeksa sumacije koji se pojavljuje i "gore" i "dolje" u binomnom koeficijentu i to s istim predznakom. Primjeri primjene pojavljuju se u nekim od sljededih zadataka.

120

7. KOMBINATORNO DOKAZIVANJE IDENTITETA

5.

Dokagite da za svaki n E No i sve r, k E Nn U (0) vrijedi:

Rjegenje. Skupovi. Lijeva strana broji uredene parove skupova kojima je na prvom mjestu k-Elani podskup od Nn , a na drugom neki (T - k) -Elani podskup od Nn disjunktan s prije izabranim podskupom (toEnije, izabran iz preostalih elemenata skupa). Desna strana broji uredene parove skupova kojima je na prvom mjestu skup od Nn , a na drugom k -Elani podskup upravo izabranog skupa.
T -Elani

pod-

Bijekcija izmedu skupa svih objekata prebrojanih na lijevoj strani i skupa objekata na desnoj strani je: (A1,442) (A1 U A2, A2), s inverzom:

Putevi. Ovaj put Cemo brojati puteve u trodimenzionalnoj cjelobrojnoj koordinatnoj mreZi. n C e biti ukupni broj koraka, te npr. k broj koraka u smjeru x-osi, a T broj koraka u smjeru y -osi. Izvedite daljnju analizu sami precizno!

9-t Zadatak se moie rijegiti i u dvodimenzionalnoj koordinatnoj mreIi (malo je "ukosite" za ovu priliku): od n dionica trebamo pretrEati svih k horizontalnih i neke s usponop, a ukupno r komada.
9
6.
Jedan sliEni zadatak smo veC rijegili - nadite ga!

Dokagite da je za svaki n E No :

Rjes'enje. Skupovi. Lijeva strana broji sve podskupove skupa Nn , sortirane po veliEini: prazni skup, skup koji sadrii 1 element, dva elementa, itd. Desna strana broji ponovno sve podskupove istog skupa, ali tako d a za svaki od n elemenata kaZe d a li je u podskupu ili ne (po dvije moguCnosti izbora). Putevi. Lijeva strana broji sve puteve koji se sastoje od n koraka, sortirane po broju horizontalnih koraka. Desna strana opisuje lutanje dug0 n koraka - u svakom koraku odluEujemo da li Cemo gore ili desno. To je isto!

9-t

Kamo Cemo stidi ako krenemo iz ishodigta?

Fomalni redovi.

2 (7)
i=O

xi = (1

+x ) ~ .

Uvrstimo li x = 1 (izraEunamo sumu koeficijenata), dobivamo traienu jednakost.

7. KOMBINATORNO DOKAZIVANJE IDENTITETA

7.

Dokaiite da je za svaki n E N :

RjeHenje. Skupovi. Kako izabrati podskup parne veliEine skupa Nn ? Ponovno moZemo za element 1 r e 6 da li C e uCi u skup ili ne, zatim isto za 2, pa za 3 itd. KonaEno, kad dodemo do zadnjeg elementa, nemamo viSe izbora - naime ako smo do tada vet uzeli paran broj elemenata, zadnji element ne smijemo uzeti, a ako smo uzeli neparan broj elemenata, zadnji element moramo ukljuEiti u izabrani podskup. Dakle, slobodno moiemo samo za elemente 1,2,. . . ,n - 1 reCi da li ulaze u podskup ili ne. To nam daje 2n-1 moguCnosti izbora. Jasno je da smo ovim naEinom izbora obuhvatili sve podskupove parne veliEine.

tP Analogno se dobiva i = 2n-1. Osim toga, relaciju za "neparne" binomnne koeficijente moiemo dobiti i kombiniranjem upravo dokazanog identiteta s identitetom iz prethodnog zadatka.
Rac'unski dokaz. Zbrojimo li jednakosti:

i podijelimo rezultat s 2, dobivamo upravo traiienu jednakost.


4 ' BuduCi da se Ci20 za k > 2 ne moie izraEunati kombinatornim razmatranjem nalik na prvo rjegenje, takva suma se raEuna analogno ovom posljednjem dokazu. RaEuna se suma izraza (1 c j ) " , za j = 0,1,. . . ,k - 1, gdje je E neki primitivni Ic-ti korijen iz jedinice (tj. ck = 1, E' # 1 za 0 < i < k ) .

(G)

8.

Dokaiite da je za svaki n E N :

RjeHenje. Skupovi. Lijeva strana broji sve podskupove s istaknutim jednim elementom. Desna strana broji isto to, ali se prvo bira istaknuti element, a zatim se medu preostalim elementima bira ostatak podskupa kojem C e on pripadati.

, odnosno n koji mnoii 2n navode Formalni redovi. Koeficijent i koji mnoii . je jednaka n(1 x ) ~ - ' . nas d a promatramo formalnu derivaciju izraza (1 z ) ~ Ona Uvrstimo li x = 1 direktno u dobiveni izraz (izraEunamo sumu koeficijenata), a zatim u

(7) +

izraz

(zto (;)zk)'

x;=o k(;)xk-I

, dobivamo traienu jednakost.

JoHjedan dokaz. Podijelimo li obje strane jednakosti koju trebamo dokazati s 2 n , dobivamo tvrdnju da je srednja vrijednost veliEine podskupa n -Elanog skupa jednaka n/2. To je istina, jer svaki i-Elani podskup moiemo "spariti" s (n - i) -Elanim podskupom

7. KOMBINATORNO DOKAZIVANJE IDENTITETA

- njegovim komplementom.

Tako dobivamo parove podskupova koji zajedno doprinose sumi veliEina podskupova toEno n (tj. svaki po n/2).

Putevi. Na lijevoj strani su pobrojani svi putevi koji se sastoje od n koraka, ali svaki od njih s oznaEenim (ukradenim) jednim horizontalnim segmentom. Na desnoj strani su pobrojani svi putevi koji se sastoje od n - 1 koraka, s oznaEenom jednom cjelobrojnom toEkom na putu. To je upravo toEka na koju bi trebalo vratiti ukradeni segment.
Pronadite medu prethodnim zadacima jedan sa sliEnom pritom, pa rije'site ovaj zadatak joi jednom, ovaj put uz kori'stenje tvrdnje prethodnog zadatka.

++

9.

DokaBite da je za sve n, r E No :

Rjeienje. Skupovi. Desna strana broji ( n 1) -Elane podskupove (n T 1) -Elanog skupa. Kako te objekte broji lijeva strana? Tu se broje n -Elani podkupovi skupova raznih veliEina. Gdje je ( n + 1) -vi element? MoZemo smatrati da su na lijevoj strani podskupovi sortirani prema svom najveCem elementu. NajveCi element moZe biti n 1, n 2 ,. . . , n r 1. U sluEaju da je najveCi element jednak n 1, preostalih n elemenata biramo iz skupa Nn (na naEina), ako je jednak n 2 , preostale elemente biramo iz skupa naEina). OpCenito, ako je najveCi element jednak n 1 i , preostali Nn+l (na (n:l) n : i ) naEina. Time elementi se moraju nalaziti u skupu Nn+i ,pa ih moZemo izabrati na ( je jednakost dokazana. Putevi. Radi se o putevima od ishodi'sta do toEke (n 1,T) . Na lijevoj strani t i su putevi sortirani u ovisnosti o visini toEke na kojoj je put napustio pravac x = n .

+ +

+ +

(E)

+ +

miranjem se dobiva

Formalni redovi. Lijeva strana je koeficijent uz x n u izrazu C;=,(l x ) ~ + Su~. (1 x)"+'+' - (1 x)"] /x . Dakle, koeficijent uz xn u polaznom

izrazu je jednak koeficijentu uz xn+' u (1

[ +

+x ) ~ + ~ + -'(1+ 2)" , 'st0 iznosi (n:~il) .

P+ Upravo dokazanu relaciju moZemo zapisati na jo's nekoliko naEina:

Koliko je to u stvari razliEitih relacija? Koliko ih je ako izbacimo trivijalne ( (;I)= Pokuiajte i te relacije direktno dokazati kombinatorno.

(,zb))?

7. KOMBINATORNO DOKAZIVANJE IDENTITETA

9 Iz navedene relacije vidi se, medu ostalim, da je C:=l 2' = n(n 1)/2. ProEitajte t u relaciju kombinatorno. Osim toga, iz upravo dokazane relacije moZemo lako vidjeti da je Cy.oik polinom k 1-vog stupnja u varijabli n (dokaz teEe npr. matematiEkom indukcijom po k). Dalje se vidi da je i C:=o Pk(i) takocter polinom k 1-vog stupnja Eim je Pk polinom stupnja k . PokuSajte dokazati ove tvrdnje (prvo prvu, zatim drugu iz prve).

Zapamtite dokazane tvrdnje, Eesto se pokazuju korisnima, jer pri raznim zbrajanjima omogudavaju da barem djelomiEno predvidimo rezultat (ili uoEimo "glupe" pogreLke).

10.

DokaZite:

Rjes'enje. Skupovi. Desna strana broji sve npr, neprazne podskupove od Nn . Na lijevoj strani moiemo smatrati d a su ponovno pobrojani svi neprazni podskupovi od Nn , ali sortirani prema najvedem elementu. Naime, postoji toEno jedan podskup od N n s najvedim elementom jednakim 1, dva podskupa s najvedim elementom jednakim 2, itd, te opkenito toEno 2' podskupova s najvedim elementom jednakim i 1. NajveCi element 2i. moie biti izmedu 1 i n , pa nepraznih podskupova ima Putevi. Brojimo sve puteve od ishodiSta do pravca x y = n koji imaju barem jedan horizontalni korak. Na lijevoj strani moiemo ih prepoznati sortirane prema broju vertikalnih koraka na kraju Letnje. Formalni redovi. Na lijevoj strani nalazi se suma promjenjive duljine - znak da lijevu stranu trebamo interpretirati kao produkt nekih formalnih redova. Vidimo zatim da se radi o produktu (1 22 ( 2 ~ ) ' ( 2 x 1 ~ . . .) . (x x2 x3 . . .) . Desnu stranu traiimo u razlici formalnih redova 1 22 (22)' ( 2 ~ ) .~ . . i 1 x x2 x3 . . . . Tako dobivamo relaciju koja zaista vrijedi (provjerite!):

~yro].
+

+ +

+ + + + + + + + + + + + +

tP I u ovom dokazu se radi, kao i npr. kod puteva o kombiniranju nekih objekata (mnoienje formalnih redova!). Prvi faktor je odgovoran za dio getnje u kojem slobodno biramo korake, a drugi dio za vertikalni istek.
+f Relacija je u stvari trivijalna, jer je posljedica opdenitije formule za sumu prvih n Elanova geometrijskog niza. Za jednostavni raEunski dokaz trebalo bi od zadane relacije oduzeti nju pomnoienu s 2. Time se dobiva identitet 1- 2n = 2n - 1- 2n+1 2 i tvrdnja je dokazana.

11. DokaZite da za sve m, n, r E No vrijedi:

RjeHenje. Skupovi. Iz skupa od m n elemenata biramo nekih r elemenata. Da bi se pri interpretaciji desne strane skup mogao prirodno raspasti na dva skupa, pretpostavim0 da se on sastoji od m djevojEica i n djeEaka. Tada se skupina od r djece moie

7. KOMBINATORNO DOKAZIVANJE IDENTITETA

sastojati od jedne djevojEice i T - 1 djeEaka (jasno, ako postoji barem jedna djevojEica i barem T - 1 djeEaka), ili od dvije djevojEice i r - 2 djeEaka itd. OpCenito, mofemo izabrati prvo k djevojEica, a zatim T - k djeEaka i to za bilo koji k izmedu 0 i T . Za svaki , bez obzira na to irnamo li dovoljno djevojEica, takav k je broj izbora jednak odnosno djeEaka da t a j izbor provedemo. Time je jednakost dokazana.

(7)(,nk)

'3+ Napigite precizno: za koje k


nule ?

E N T U (0) je odgovarajuCi sumand razliEit od

Putevi. Na putu duljine m+n koraka, od kojih je r horizontalnih, nakon m koraka se odmorimo (odmor Cemo najaviti s "kad stignemo na pravac x y = m ") i pogledamo koliku smo horizontalnu udaljenost progli.

Ctt Nacrtajte skicu. UoEite kako moIe pravac x y = m sijeCi Setaligte (pravokutnik unutar kojega su sadrfani svi putevi) i poveiite te sluEajeve sa situacijama kad ponestaje djevojEica, odnosno djeEaka, te sa situacijom kad moiemo slobodno birati.
o
T

-k

Fomnalni redovi. Pojavljivanje produkta s izrazom koji ovisi o k i izrazom koji ovisi moie nas podsjetiti na mnoienje formalnih redova:

Ako jednakost poEnemo interpretirati zdesna, imamo:

+f' Ova relacija se zove Vandermondeova konvolucija. OpCenito, konvolucije su sume (u diskretnoj matematici, inaEe i integrali) oblika C f (z)g(r - x) , gdje su f i g neke funkcije, a T konstanta. Na temelju steEenih iskustava znamo da je zgodno konvolucije interpretirati kao koeficijente u produktu formalnih redova.

12. Neka su m, n E No zadani. IzraEunajte:

Rjes'enje. Prelaskom na komplementarni binomni koeficijent u bilo kojem od faktora m : n ) , dobivamo izraz nalik na izraz iz prethodnog zadatka. Rezultat: ( Skupovi. Ovaj put biramo npr. djevojEice koje Ielimo uzeti u skup i djeEake koje ne Ielimo. Formalni redovi. Promatramo ( 1

+ x)"(x +

'3+ Po kojim se k ovdje sumira?

7. KOMBINATORNO DOKAZIVANJE IDENTITETA

CEO (z
13.

tP

Za m = n ova relacija daje sumu kvadrata binomnih koeficijenata:

)= ~ (3 .

DokaBite da je za svaki n E N :

Rjes'enje. Kao i u prethodnom zadatku, umjesto

(E)~ promatrat

Cemo

(z)(nrk).

Skupovi. n djevojEica i n djeEaka stoje u redu Eekajuti autobus koji C e ih odvesti na izlet. U redu stoje prvo djevojEice, a zatim djeEaci. Polovina njih smije otiCi malo progetati, a od ostalih treba nekog zaduiiti d a pazi na ruEak koji se nalazi zapakiran u jednoj kutiji. Ako u redu ostaje k djevojEica i n - k djeEaka, onda nam lijeva strana jednakosti kaie d a ruEak mora ostati ili ispred prve djevojEice ili ispred druge djevojEice ili . . . ili ispred k -te djevojEice ili iza k-te djevojEice. Jasno, u sluEaju da su sve djevojEice otiile u Setnju, ruEak ostaje na brizi djeEacima. Pokuiajmo sada dobiti isti broj u malo kompaktnijem obliku koji nalikuje na desnu stranu. Jednu djevojEicu moiemo zaduiiti na n naEina da pazi na ruEak: to moie biti ili prva ili druga ili . . .ili n - t a djevojEica koja pazi na ruEak koji se nalazi ispred sebe. Nakon toga biramo ostalu djecu koja ostaju u redu. To je ukupno n(2nz:) naEina. Osim toga, kutija s ruEkom je mogla ostati stajati izmedu skupine djeEaka i skupine djevojEica. U tom sluEaju bi na nju pazila zadnja djevojEica, ako je ijedna ostala u redu, odnosno djeEaci, ako su sve djevojEice otiile u getnju. Takvih na8na izbora ima (2nn) . Ukupno, broj izbora iznosi:

Formalni redovi. Faktor k+ 1 moie nas podsjetiti na deriviranje, a na mnoienje formalnih redova.

(z)2= (r)(rink)

14.

Dokagite da je za sve m, r E No :

Rjes'enje. Skupovi. Zadatak nam je na lijevoj strani prepoznati broj svih r -Elanih podskupova m-Elanog skupa. Vidimo da svaki pojedini sumand lijeve strane broji neke

126

7. KOMBINATORNO DOKAZIVANJE IDENTITETA

izbore kojima biramo to manje elemenata Sto je manji skup iz kojeg biramo. To nam moZe sugerirati da su oni elementi koji nam nisu na raspolaganju veC izabrani.

I stvarno, lijeva strana dijeli r -Elane podkupove m-Elanog skupa prema najmanjem elementu koji nije u tom podskupu. Ako je t a j element jednak i , onda imamo izabranih i - 1 elemenata, a preostalih r - i f 1 elemenata biramo iz (m - i) -Elanog skupa { i + l , i + 2 ,...,m } .
Putevi. Sve puteve od ishodiLta do toEke (r, m - r ) puta kojeg provodi na pravcu y = 0 . klasificiramo prema duljini dijela

Formalni redovi. Za interpretaciju ove sume (duljina promjenjiva s r ) moZe nam zasmetati Einjenica d a se ne sumira od nule. Osim toga, indeks sumacije se pojavljuje i "gore" u binomnim koeficijentima. Radi olakzavanja raEuna prvo Cemo se rijesiti tih problema.

mo5ete napisati medukorak s j = r - i


=

(a:T)(l - x)-m+T.

2
1-a:

15. Neka su k,n E N o . IzraEunajte:

Rjes'enje. Skupovi. Biramo neki skup ( r -Elani, za r = k, . . . ,n ) koji C e biti nadskup k -Elanog skupa kojeg Cemo izabrati nakon toga. To isto smo mogli napraviti i tako da smo prvo izabrali k -Elani podskup, a zatim za svaki od preostalih elemenata rekli da li ulazi u njegov nadskup ili ne. Rezultat: (;)2n-k . Rjes'enje transformacijom izraaa.

7. KOMBINATORNO DOKAZIVANJE IDENTITETA

127

16.

Dokafite da je za m, n, i, j E No :

Rjes'enje. Skupovi. _Zamislimoda u redu stoji m + n + 1 dijete - prvo n + l djeEaka, a zatim m djevojEica. Zelimo od njih izabrati i j 1 dijete. Napravit Cemo to tako, da prvo izaberemo jedno dijete tako da mu stavimo u ruku zastavicu, a zatim biramo i djece lijevo i j djece desno od njega. Ako poloZaj djeteta kojem smo stavili u ruku zastavicu odredimo relativno u odnosu na zadnjeg djeEaka - k = 0 znaEi da je zastavica u ruci zadnjeg djeEaka, k = 1 znaEi da je zastavica u ruci prve djevojzice, k = -1 u ruci predzadnjeg djeEaka itd, dobivamo traZenu formulu.

+ +

Formalni redovi. TraZena suma je konvolucijskog tipa i izgleda kao koeficijent uz xm+n u produktu dvaju formalnih redova oblika: El (:)xl (jedan za a = i , a drugi za a = j ) . Ovaj formalni red se lako moZe sumirati - uoEite samo da su svi produkti u koeficijentima sastavljeni od jednakog broja faktora. To nam daje ideju da bi se moglo raditi o derivaciji formalnog reda xa .

Ci

Prema lijevoj strani slijedi d a je suma koju trebamo izrazunati jednaka:

% Drugi nazin za raEunanje sume formalnog reda

El ( ,1 ) x 1

ukljuEuje "izbaci-

vanje" indeksa sumacije s gornje pozicije u binomnom koeflcijentu:

(-lafa'))

( - l ) l - a. Provjerite rezultat i ovom metodom.

( : )

= (l-a) =

17. Dokafite da za sve n, k E N (k!)"n! dijeli (kn)! .


Rjes'enje. U izrazu ( k ~ ~ ) ! / ( ( k ! = ) ~ (n k! , / )n k ) / n ! prepoznajemo broj naEina grupi,"., ranja k n predmeta u n grupa jednake velitine, pri Eemu nije bitan redoslijed elemenata unutar grupa, kao niti redoslijed grupa. Jasno je da je t a j broj prirodan, pa zakljuEujemo da ( k ! ) n n !dijeli ( k n ) !.

+f'

Direktno rjezenje takoder je jednostavno.

7. KOMBINATORNO DOKAZIVANJE IDENTITETA

18. DokaBite kombinatorno da je z a sve n, m E N :

Rjes'enje. Na desnoj strani su izbrojane sve funkcije iz N n u N m . Na lijevoj strani one su klasificirane prema broju elemenata koji se preslikavaju u element m . Naime, ako se k elemenata treba preslikati u m , onda njih moiemo izabrati na (E) naEina, a ostale elemente treba preslikati u preostalih m - 1 elemenata. To moiemo napraviti na (m - l ) k naEina.

tP Ova polinomna jednakost vrijedi za svaki m E N , pa onda i za svaki m E R . Ako umjesto m - 1 u jednakost uvrstimo a l b , onda mnoienjem s bn dobivamo binomni teorem, ponovno!
19. Ako je S(n,k) Stirlingov broj druge vrste (broj particija n-Elanog skupa u k dijelova), d o k 5 i t e da z a prirodne brojeve n i m vrijedi:
(a) S(n 1,m 1) = ( m l)S(n,m 1) S(n,m), (b) C, (;)S(k, m) = S(n + 1,m I).'

+ +

+ 1 blokova, a na desnoj o particijama nekog n-Elanog skupa. Istaknimo stoga element n + 1 i pogledajmo kako se on moie ukljuEiti u particije skupa N, .
m

(a) Na lijevoj strani se radi o particijama nekog ( n + 1) -Elanog skupa (npr. N,+i ) i

Ako se particija skupa N n vet sastoji od m + l blokova, onda element n + l moramo prikljuEiti jednom od njih. Broj izbora particija je u tom sluEaju jednak S(n, m I ) , a broj naEina izbora jednog bloka m 1 . Ukupni broj izbora odgovara prvom pribrojniku. Ako particija skupa N n ima m bfokova, onda, da bismo na kraju dobili particiju skupa Nn+l u m + 1 blokova, element n + 1 mora biti sam u svom bloku. Broj izbora odgovara drugom sumandu. Jasno je da particija skupa N n ne smije imati manje od m niti vi'se od m + 1 blokova, pa je time tvrdnja dokazana.

9+ IzraEunajte S(n, 1) i S(n, n) za proizvoljni n E N . RaEunajte zatim za male n Stirlingove brojeve. Za zapis koristite ideju Pascalovog trokuta.
(b) Ovaj put podijelimo sve particije skupa Nn+i u m 1 dijelova u ovisnosti o tome koji sve elementi nisu u bloku s n 1. Njih mofe biti ili 0 ili 1 ili itd. ili n. Ako ih ima k , onda ih moiemo izabrati na (E) naEina i podijeliti u m blokova na S(k, m) naEina. Jasno je da C e neki od S(k, m) biti jednaki 0,ali to ne smeta da rezultat zapi'semo u obliku kakav je na lijevoj strani jednakosti.

20.

DokaBite da je z a n E N : 1.1!+2.2!+

...+n . n ! = ( n + l ) ! - 1 .

Rjes'enje. SudeCi prema desnoj strani, na lijevoj bismo trebali prepoznati sve permutacije n 1-Elanog skupa osim npr. identitete (uzimamo identitetu kao jednu istaknutu permutaciju). Za'sto se tam0 pojavljuju brojevi permutacija skupova razliEitih veliEina?

8. ZBRKA ZADATAKA

141

91. 3n braEnih parova treba podijeliti u trojke (zbog prijevoza malim Eamcima). Na koliko je naEina to mogude napraviti ako niti u jednoj trojci ne smije biti par suprugnika?
92. Poznato je da u nekoj populaciji toeno a0 posto ljudi nije u stanju nikoga prevariti za prvi travnja, a1 posto ljudi prevari jednog Eovjeka, a2 dvoje ljudi, itd, a, prevari n ljudi (a1 a2 . . . a, = 100).

+ + +

IzraEunajte maksimalni broj Bala ( m ) za koji se sigurno moBe redi da de barem jedan Eovjek nasjesti na m Sala. U populaciji se nalazi barem dvoje ljudi.
93. Na koliko naEina se lik sa slike moEe popuniti brojevima 0 i 1 a da se niti jednom izometrijom praznog lika u samog sebe (rotacije, osne i centralna simetrija) popunjeni lik ne preslika sam u sebe?

94. Nadite obiEnu funkciju izvodnicu za broj rastava prirodnog broja na sumu razliEitih rastudih neparnih prirodnih brojeva. Pritom duljina rastava mora biti barem 2 (tj. moraju postojati barem dva sumanda). Iz te funkcije izvodnice izraEunajte broj rastava broja 6 na sumu razliEitih rastutih neparnih prirodnih brojeva.
95. Neka je t, broj nesukladnih trokuta sa stranicama cjelobrojnih duljina, manjim ili jednakim 2n - 1. Nadite rekurziju za t, i izraEunajte t, , za proizvoljni n E N .

96. Na koliko naEina moZemo otkinuti pravokutnu ploEu Eokolade proizvoljne veliEine od m x n ploEe Eokolade? Treba pogtivati strukturu Eokolade, tj. otkidati du?, naznaEenih brazdi ili koristiti rub!

97. Pravokutnu ploEu veliEine n x 2 treba pokriti ploEicama: crvene su veliEine 1 x 2 , a plave veliEine 1 x 1. Na koliko se naEina to moBe napraviti, ako imamo neograniEen izvor ploEica svake boje - napiSite funkciju izvodnicu za te brojeve!

Na koliko naEina moZe d djece medu sobom podijeliti j jabuka, k krugaka i n naranEi, tako da svako dijete dobije barem jedan komad voda (bilo koje , vrste)?
1

142

8. ZBRKA ZADATAKA

99.

DokaBite kombinatorno:

100. Neka je a, broj izbora r kuglica iz beskonaEnog izvora crvenih, bijelih i plavih kuglica, u kojima je broj crvenih kuglica paran. PomoCu obiEne funkcije izvodnice izraEunajte a, , za r E N . 101. Na koliko se naEina moBe izabrati n olovaka iz skupine od N 2 2n 010vaka u kojoj je n jednakih obiEnih olovaka tvrdoCe "H", n jednakih obiEnih olovaka tvrdoCe "B", a ostalo su medusobno razliEite kemijske olovke ? 102. Neka je n E N parm. Neka je f : N, -t N, zadano preslikavanje bez fiksnih toEaka koje je samo sebi inverz. Nadite broj uredenih parova skupova (A, B) takvih da je A U B = N, , A n B = 0 i da vrijedi f(A) = B i f ( B ) = A . 103. Neka je a, maksimalni broj disjunktnih podruEja u koja dijagonale konveksnog r -terokuta dijele njegovu unutrsnjost. Pretpostavimo da je r -terokut takav da se nikoje tri dijagonale ne sijeku u istoj toEki. DokaZite da vrijedi:

104. Na koliko naEina mogu Ana, Boris, Cvjetko, Damir, Eva i Frmka podijeliti sljedeCe poslove: izmiSljanje jelovnika, nabavka hrane, kuhanje, serviranje jela, pranje posuda, spremanje kuhinje ? Ana ne moie smisliti jelovnik, Boris ne zna gdje je duCan, Cvjetko loge kuha, Damir ne zna gdje se nalazi pribor za jelo, Eva ne voli prati posude, a F'ranka ne voli spremati kuhinju. Svi trebaju biti zadovoljni, poslovi moraju biti dobro obavljeni i svatko od navedenih ljudi mora sam obavibi toEno jedan posao.
105. Pomodu funkcija izvodnica izraEunajte broj raspodjela r jednakih kugala u 4 razliEite posude tako da broj kugala u prvim dvjema posudama bude djeljiv s 3, a u drugim dvjema ne bude?

106. Neka su n = 2 j i k zadani prirodni brojevi. Koliko postoji najkraCih puteva od toEke (0,O) do toEke (n, n ) dui bridova cjelobrojne koordinatne mreZe koji prolaze barem jednom cjelobrojnom toEkom segmenta s krajevima (j-k,j+k) i (j+k,j-k)? 107. IzvjeStaj s ljetovanja: od n dana, koliko smo bili na ljetovanju, barem svaki drugi dan je padala kiSa. StoviSe, svaki treCi dan je padao poslijepodnevni pljusak, svaki Eetvrti dan je rominjala jutarnja ki'sica, a svaki peti dan kiSa nam je pokvarila veEernji izlazak.

8. ZBRKA ZADATAKA

Koliko je najviSe moglo biti suncanih dana? Koliki je taj broj za n = 407
108. Objasnite kombinatorno zaSto je broj dvoElanih podskupova (n Clanog skupa jednak sumi prvih n prirodnih brojeva.

+ 1)-

109. DokaBite da za m 2 3 suma m Fibonaccijevih brojeva ne moBe biti jednaka nekom Fibonaccijevom broju. 110. Zaostaci se gomilaju ovako: svaki dan zaostatak u poslu (koji se moBe numeriEki izraziti) je za konstantu ovisnu o danu (za n-ti dan ona iznosi c,) veCi od sume zaostataka svih prethodnih dana, od poEetka posla. Prije poEetka posla zaostatak je zo = 0 .

Nadite obiFnu funkciju izvodnicu za niz zaostataka (2,) , ako je zadan proizvoljni niz (c,) . Iz toga izraEunajte z, , za n N , ako je zadani niz konstanti C , = 1, za EN, co = O .
111. Na koliko naEina mo5emo s ove slike proEitati reCenicu "Koliko ima kombinacija?" pomiCuCi se uvijek do susjednog slova (gore,dolje, lijevo ili desno)?
?

K K O K O L K O K

K O L I L O K

K O L I K I L O K

K O L I K O K I L O K

K O L I K O I O K I L O K

K O L I K O I M I O K I L O K

K O L I K O I M A M I O K I L O K

K O L I K O I M A K A M I O K I L O K

A J I C A N I B M O K A M I O K I L O K

? A J I C A N I B M B I N A C I J A ?

? A J I C A N I B I N A C I J A ?

? A J I C A N I N A C I J A ?

? A J I C A N A C I J A ?

? A J I C A C I J A ?

? A J I C I J A ?

? A J I J A ?

? A ? J A ? A ? ?

144

8. ZBRKA ZADATAKA

1. Ako je n = 1 , n-terokut je u stvari duiina, pa moiemo jedan njen kraj oznaEiti s 0 a drugi s 1.

U sluEaju kvadrata, brojevi mogu biti redom: 00,01,11,10. Prvu znamenku driali smo fiksnom, a drugu mijenjali. Kad smo iscrpili obje moguCnosti za drugu znamenku, promijenili smo prvu znamenku, te drugu mijenjali, ali ovaj put drugim smjerom. Konstruirajte primjer za sluEaj n = 3 sami.
Postupak proveden za kvadrat moiemo provesti i za opCi sluEaj. Pretpostavimo d a imamo konstruiran niz brojeva pridruien 2n -terokutu i da je on a l , a2,. . . ,apt . 2n+1 terokutu pridruiimo niz Oal, Oaz,. . . ,Oazn, l a p , . . . ,l a z , l a l . Svaka dva susjedna Elana ovog niza se razlikuju samo u jednoj znamenci, a isto tako i prvi i zadnji Elan.

9 Prikaiite sluEaj n = 3 grafiEki na jog jedan naEin. Smjestite jediniEnu kocku u prvi oktant trodimenzionalnog koordinatnog sustava tako d a je jedan njen vrh u ishodistu. OznaEite koordinate vrhova i spojnice izmedu susjednih elemenata niza. OpiHite i na ovom primjeru Sto se dogada pri prijelazu na veCi n .

tP Brojevi ovog niza mogu se pridruiiti brojevima skupa {O,1,2,. . . , 2n - 1). Tako dobivamo Grayev kod, koji je zbog svojstva d a se kodovi susjednih brojeva razlikuju samo u jednoj znamenki bio korigten u brojaEima.
2. Da bi se od duiina duljina a, 6 i c mogao sastaviti trokut, nuino je i dovoljno d a vrijede nejednakosti trokuta: a < b+ c , 6 < a + c i c < a 6 . BuduCi da su nama brojevi veC poslagani po veliEini, moBemo zahtijevati a b c , pa od triju uvjeta ostaje samo jedan: c < a b .

< <

Prvo rjeienje - pokus'aj. Prvo se vidi da je stvar trivijalna ako u nizu postoje tri jednaka broja. Zatim se sliEno zakljuEi i ako postoje dva jednaka broja koja nisu minimalna u nizu. Dakle, svi brojevi su medusobno razliEiti, osim eventualno prva dva u nizu. Tako vidim0 da je sigurno i-1 xi 1994-19+i. Tako se traienje trojke brojeva xi x j < xk takve da je xk < xi +sj moie svesti na traienje trojke brojeva 1 i < j < k 19 takve da je 1975 k < i j - 2. No oEito je da takva trojka ne mora postojati.

< <

<

< <

Mana ovog pristupa bila je u tome Hto se ocjena veliEine xi -ova radila bez obzira na veliEinu ostalih elemenata niza, a jasno je da moguCa veliEina svakog od xi -eva ovisi o izabranim elementima niza manjim od xi . Drugo rjeienje. PokuSajmo nadi niz s minimalnim elementima takvim d a se od duiina odgovarajuie duljine ne moie sloiiti niti jedan trokut. Moida nam uspije dokazati da, ako su prva dva elementa niza minimalna (jednaka I), te ako za svaki sljedeCi broj uzimamo najmanji moguCi, s 19. elementom moramo prekoraEiti 1994. Dakle, x l = 1 i x2 = 1. 2 3 ne smije biti 1, jer bismo tada imali jednakostraniEni trokut. Najmanji moguCi 2 3 je 2. xr ne smije biti manji od sume bilo kojih dvaju vet izabranih elemenata, jer bismo inaEe mogli sloiiti trokut s jednom stranicom duljine 2 4 . Najmanji x4 je jednak 2 3 2 2 = 3 .

SliEno zakljuEujemo i opLenito, te je xn = xn-1 xn-2. Ponovno su u igri Fibonaccijevi brojevi - zbog poEetnih uvjeta imamo xn = F n . Dobivamo da je najmanji moguCi z18 jednak 2584, Sto je veCe od 219 = 1994.

8. ZBRKA ZADATAKA

e biti rijegen ako do3. Vrijednosti suma od po dva broja iz Nloo ima 199. Zadatak C kaiemo da postoji klasa u kojoj ima toliko brojeva da je broj suma veCi od 199. Tada Ce, prema Dirichletovom principu, postojati barem dvije razliEite sume jednake vrijednosti. Ako je t a klasa veliEine k , mora biti tj. k 2 15. No, buduCi da 100 elemenata moramo podijeliti u 7 klasa, sigurno postoji klasa veliEine 15 (prema Dirichletovom principu, ponovno).
4. Za prvim stolom moie sjediti 1,2, . . . ,n - 1 ljudi. Kao i obiEno, izraEunajmo prvo brojeve rasporeda za sluEaj d a razlikujemo stolove, pa onda rezultat podijelimo s 2.

Ako ielimo k ljudi posjesti za prvi stol, treba ih prvo izabrati ( naEina), zatim posjesti oko stola uvaiavajuCi samo relativni raspored ( (k - I)! naEina) te na kraju ostale smjestiti oko drugog stola ( (n - k - I)! naEina). Rezultat zadatka se dobiva zbrajanjem po svim k i dijeljenjem s 2 (da ne razlikujemo prvi od drugog stola):

(i)

e imati samo osoba koja je bila usreCena direktno od zaEetnika 5. (a) Prvi dan pismo C lanca. Ona C e poslati k pisama i drugi dan C e k ljudi u rukama imati pismo koje C e odmah prepisati i poslati kopije. TreCi dan C e od svakog od tih k pisama stiCi svojim primateljima k kopija, dakle treCi dan C e biti k2 primatelja. SliEno zakljuEujemo i optenito, pa C e d-ti dan od slanja pisma kd-l ljudi primiti pismo s porukom lanca srede. (b) Prva osoba s liste dobit C e 10 novEiCa od osobe koja je primila pismo od zaEetnika igre. Druga osoba s liste dobit C e novac od ljudi koji su pismo dobili drugi dan. Dakle primit C e lOk novEiCa. OpCenito, i - t a osoba s poEetne liste primit de novac od onih koji pismo dobivaju i -ti dan. Njih ima ki-l , pa C e i -ta osoba primiti 10ka-l novEiCa. OEito je da C e osoba koja je zapoEela lanac srede sebe skromno staviti na zadnje mjesto u listi. OpCenito, svaka osoba koja primi i pogalje pismo moBe se smatrati zaEetnikom "svog" lanca (jer se u pismu kojeg Zalje nalazi na posljednjem mjestu). Zato Ce, ako se lanac odrii dovoljno dug0 (vidi i (c) dio zadatka) primiti 10ki-' novEiCa. (c) Rezultat dobivamo zbrajenjem rezultata (a) dijela zadatka.

4-t Citatelji upoznati s osnovnim pojmovima teorije grafova i uredenih skupova uoEit C e da ova igra vise nalikuje stablzl nego lancu. Uz neke dodatne pretpostavke ona moBe postati stablo.

146

8. ZBRKA ZADATAKA

6. Broj naEina izbora druge i treCe karte ovisit de o tome koju smo vrijednost karte izabrali kao prvu kartu (pik). Prirodno je izbore podijeliti u tri grupe: prva izabrana kart a je pik 7, prva izabrana karta je pik 3 i prva izabrana karta ima neku drugu vrijednost. VeliEine tih skupina lako je prebrojati, a rezultat zadatka jednak je njihovom zbroju:

1.3~(12.4-2)+1.4.(12.4-1)+11~4.(12.4-2). 7, Trebamo dokazati d a medu dobivenim zbrojevima postoje dva zbroja koji imaju jednake ostatke pri dijeljenju s n . Kad to ne bi bila istina, buduCi da zbrojeva ima 2 n , medu njima bi bili zastupljeni svi ostaci modulo 2n: 0,1,2,. . . , 2 n - 1 . Ostataka ima jednako kao i suma, pa nam direktna primjena Dirichletovog principa ne daje rjeSenje.
Sto joS moZemo reCi o tim zbrojevima? BuduCi da su brojevi u prvom redu ispermutirani, ne moiemo niLta reCi o svakom posebno. No, poznat nam je njihov zbroj i on iznosi:
2n

2 . C i = 2 . 2n(2n + 2
i=l

= 2n(2n + 1).

Kad bi medu zbrojevima bili zastupljeni svi ostaci modulo 2 n , prethodni izraz bi trebao imati isti ostatak pri dijeljenju s 2n kao i zbroj svih ostataka:

Provjerimo to! 2n(2n 1)2n -2n.+ 1) - (2n 2 2n+3 2 .

Razlika promatranih zbrojeva sigurno nije vigekratnik od 2 n , te zakljuEujemo d a oni sigurno nemaju isti ostatak pri dijeljenju s 2n. Time smo doSli do kontradikcije, pa zakljuEujemo da medu promatranim ostacima nisu zastupljeni svi moguCi ostaci pri dijeljenju s 2 n . A kad nije tako, onda postoji ostatak koji se ponavlja, pa je razlika odgovarajuCih suma djeljiva s 2n.

+f' MoguCe sume su 2,3,. . . ,4n - ukupno ih je 4n- 1 komada. Moglo se pokuSati promatrati mogu li one imati medusobno razliEite ostatke pri dijeljenju s 2 n .
8. ielimo od n ljudi izabrati m . ToEnije, ielimo ih izabrati tako d a Perica (koji se ) inaEe nalazi medu njima) ne bude izabran. Broj moguCnosti izbora: ( n ~ l .

S druge strane, ne moramo Pericu odmah iskljuEiti: prebrojimo sve izbore, zatim . oduzmemo one u kojima se nalazi Perica. Svih izbora m-Elanih podskupova ima Izbora u kojima se nalazi Perica ima toEno , ali Cemo ih prebrojati na malo drugaEiji naEin. Identificirat Cemo ih sa svim m - 1-Elanim podskupovima zadanog skupa ljudi, osim jasno onima koji veC sadrZe Pericu. m - 1-Elane podskupove koji sadrie Pericu prebrojat Cemo na sliEni naEin, itd. Slijedi:

(G)

( : I )

Sto je samo malo drugaEije zapisana tvrdnja zadatka.

=):( +f' Ovdje smo u stvari primijenili formulu ( n ~ l ) ponovno na drugi Elan itd.

-( : I )

, te istu formulu

9+

Istraiite moie li se ovdje primijeniti formula UI s veliEinama presjeka skupova.

8. ZBRKA ZADATAKA

147

tP Svi oni koji tele vjeibati matematiEku indukciju mogu to u ovom zadatku uEiniti na dva naEina - naime tvrdnja se mote dokazati indukcijom po m (trivijalno) ili po n (neSto tete).
Ovo je jedan od klasiEnih primjera za primjenu formule ukljuEivanja-iskljuEivanja 30m oduzmemo broj onih koji su djeljivi s 6, onih koji su djeljivi s 10 i onih koji su djeljivi s 15. Time smo neke brojeve oduzeli dvaput, i to one koji su djeljivi s po dva od navedenih brojeva. Broj takvih brojeva treba dodati, ali nakon toga se u ukupnoj sumi ne nalaze prebrojani oni brojevi koji su djeljivi s 6 i s 10 i s 15. Njihov broj treba joS dodati sumi. Prikatite spomenute skupove skicom!

. . . Od broja svih brojeva manjih ili jednakih od

9.

Rezultat :

10. Prebrojimo prvo uredene parove sa zadanim svojstvom - to je obiEno lakSe!


Svaki od elemenata skupa A treba biti Elan nekog od traienih podskupova: bilo samo Xl , samo X2 ili oba. Dakle, za svaki element skupa A dovoljno je odrediti u koji C e element particije skupa A uCi: u X I \ X2 , X2 \ X1 ili u Xl n X2 . Zato je broj izbora uredenih parova (XI, X2) C ?(A) takvih d a je X1 U X2 = A jednak 3". Broj uredenih parova koji odgovaraju dvoElanim skupovima je za jedan manji, jer za izbor X1 = X2 = A ne dobivamo dvoElani skup. Bududi d a smo skupove {XI, X2) radi prebrojavanja pretvorili u uredene parove, konaEni rezultat glasi: 3" - 1

11. Broj najvige n-znamenkastih brojeva koji sadrie barem jednu znamenku 3 ili 5 je jednak lon - 8" za n 2 1. Za n = 0 t a j broj je jednak 0. ObiEna funkcija izvodnica za broj prirodnih brojeva s maksimalno n znamenaka i znamenkama 3 i 5 je:

Broj brojeva s toEno n znamenaka u kojima se pojavlju zadane dvije znamenke iznosi 9 . lon-' - 7 . 8"-l, za n 2 1 , a 0 za n = 0 . ObiEna funkcija izvodnica za broj n-znamenkastih brojeva u kojima se nalazi barem jedna od zadanih dviju znamenaka je jednaka: f2(x)=-10 9 ( 1 -Ilox
1 -(

--

l)

Bududi da brojevi koji imaju manje ili najvige n znamenaka imaju ili 1 ili 2 ili n znamenaka, provjerite vidi li se to i na funkcijama izvodnicama, tj. u kakvoj su vezi fl (x) i f2 (x)/(l - x) . Kako biste proEitali izraz x fl (x) - fl (x) - Sto broje njegovi koeficijenti?

. . .ili

9+

148

8. ZBRKA ZADATAKA

12. Cudno je u ovom zadatku ;to tvrdnju moramo dokazati ne za svaki prirodni broj, ili npr. svaki neparni ili parni, nego za svaki prosti broj. To nas navodi na to, d a ne pokugavamo izraz koji treba biti djeljiv s p protumaEiti kao broj nekih objekata, koje zatim treba moCi prikazati kao Kartezijev produkt skupova od kojih je jedan veliEine p , nego da idemo raspisivati izraz. raspisati pomoCu Znamo da pl(f) za svaki i E N p e l . Stoga moiemo pokugati binomnih koeficijenata oblika (f) . Pritom se nadamo da Cemo, ako dobijemo sumu, u svakom sumandu imati dva faktora traienog oblika. Zaista, Vandermondeova konvolucija daje:

(2)

a to je izraz upravo onog oblika kojeg smo ieljeli.


- 2 raspisivanjem 9+ Kako vam izgleda ideja o pokugaju sreitivanja izraza binomnog koeficijenta do oblika koji je pogodan za odredivanje faktora tog izraza?

(2)

4 ' -2 nas moie podsjetati na broj izbora p predrneta iz skupine od 2p = p+p predmeta, takvih da je izabran po barem jedan predmet iz svake skupine. Tako dobivamo formulu jednaku zadnjoj formuli prvog rjesenja.
13. Svejedno je popeli se jednom na n-ti kat ili n puta na prvi. IzraEunajmo onda na koliko se naEina moiemo popeti na prvi kat, pa Cemo nakon toga rezultat potencirati brojem katova. Sva uspinjanja moiemo klasificirati u ovisnosti o tome jesmo li prvo zakoraEili na prvu ili na drugu stepenicu. Ako je izmedu katova niz od k stepenica, pomoCu te podjele dobivamo rekurzivnu relaciju za broj uspinjanja na prvi kat, bk :

(';)

bk = b k - i -k bk-2.
Direktnim prebrojavanjem slijedi:

Dakle,

(Fibonaccijev broj), pa ukupni rezultat glasi: Fr+l Za zadanu zgradu broj uspinjanja iznosi: F : ~= 8g3 = 704969. 14. Prvi je problem kako izgubiti dodatni uvjet x l

Pokugajmo ovako: prvo biramo xk =: tk xk-2 - xk-1 =: tk-2 itd. Tada je: n = tl

, zatim

2 x 2 . . . 2 xk > 0 . razliku xk-1 - xk

=: t k - l

, pa

+ 2tz + . . . + ktk,

i t i -ovi su medusobno nezavisni. Svi su oni nenegativni, osim t k koji je nuino prirodan. ObiEna funkcija izvodnica za rastave ovog oblika je poznata i glasi ( t l sumand doprinosi

8. ZBRKA ZADATAKA

sumi 1, t g doprinosi 2 itd.):

9 Pravi znatifeljnik bi se sada sjetio da smo izaEunali funkciju izvodnicu za broj svih particija zadanog broja, bez obzira na broj dijelova (zadatak se nalazi u zbirci, ali je bio malo drugaEije zadan). Ona glasi:
Ako upravo izraEunatu funkciju izvodnicu oznaEimo s f k (x) , onda mora vrijediti: 9(x) = 1+ Pokullajte to dokazati sami!. Evo i skice jednog elegantnog dokaza - izostavljeni su samo raEunski detalji. Dva reda potencija su jednaka ako su im jednaki svi koeficijenti. Uzmimo zato proizvoljni n E N i dokafimo d a su koeficijenti uz xn u oba reda potencija jednaki. U stvari, dokazat Cemo d a su svi koeficijenti do n jednaki. U t u svrhu uoEimo prvo d a je svejedno gledamo li cijelu sumu f -ova ili samo sumu do k = n . Isto tako, svejedno je gledamo li cijeli g ili samo produkt prvih n 8anova. Krenite sada na dokazivanje. MatematiEka indukcija daje jedno vrlo jednostavno rjellenje!

C fk(~).
k>l

15. Za svaku od skupina moramo izabrati felimo li iz nje uzeti 0 ili 1 ili 2 predmeta. Jedini izbor koji nije dobar je onaj pri kojem ne uzimamo niti jedan predmet ni iz koje skupine. Rezultat: 3n - 1.
+ I ' Da smo prvo odredili koliko predmeta felimo uz&i (npr. k ) , broj izbora bi bio . Premda se ovaj izraz ne mofe srediti, lako se mofe izraEunati jednak suma tih izraza za k = 1 , 2 , . . . , 2 n (uputa: zamijenite redoslijed sumacije). Rezultat je, jasno, ponovno 3" - 1.

' & ; : (1)(;1ii)

16. Prvo rjeienje - primjenorn formule ukljutivanja-islcljuffvanja. Od ukupnog broja permutacija oduzet Cemo prvo one koje imaju jedno od slova na pogrellnom mjestu. Time Cemo broj permutacija koje imaju dva slova na pogrellnom mjestu oduzeti dvaput, pa t a j broj treba dodati prethodnoj razlici. Tako nastavljamo dalje, tj. koristimo formulu ukljuEivanja-iskljuEivanja raEunajuCi presjek skupova koji nemaju pojedino slovo na pogrellnom mjestu pomoCu veliEina skupova koji imaju neka slova na pogrellnim mjestima.

IzraEunajmo prvo brojeve potrebne za primjenu formule UI: broj svih permutacija:

&

broj permutacija s jednim slovom na zadanom mjestu: napraviti na 6 naEina. broj permutacija s dva slova na zadanim mjestima: -dva slova iste vrste: -dva slova razlititih vrsta: komada; 3 izbora slova

& ; izbor slova moie se

& komada; 3 . 2 . 2 izbora slova

8. ZBRKA ZADATAKA

broj permutacija s tri slova na zadanim mjestima: -dva slova iste vrste: -sva

& komada; 3 . 4 izbora slova slova razliEitih vrsta: & komada; 2 . 2 . 2 izbora slova
$
komada; 3 izbora slova

broj permutacija sa Eetiri slova na zadanim mjestima: -po dva slova iste vrste: -sve vrsta slova zastupljene:

& komada; 3 . 2 . 2 izbora slova

broj permutacija s pet izbora slova iznosi 1 i to je identiteta za svaki od 6 izbora slova zadamo li mjesta za 6 slova, postoji oEito samo jedna permutacija koja tako smjeZtava slova, a i izbor tih slova moie napraviti na jednoznaEan naEin. Uvriitavanjem u formulu ukljuEivanjaiiskljuEivanja dobivamo rezultat: 90 - 180

+ 18 + 144 - 36 - 48 + 3 + 24 - 6 + 1= 10.

Dmgo rjei'enje - direktno. Radi jednostavnosti pretpostavimo d a imamo k simbola svake vrste. U konkretnom sluEaju k iznosi 2.
Smjestimo prvo elemente prve skupine. Oni C e zauzeti mjesta elemenata druge ili treCe skupine. Neka ih x komada zauzme mjesta elemenata druge skupine. Tada ih jog k - x treba zauzeti mjesta elemenata treCe skupine. Kad smo njih smjestili, znamo koji C e elementi doCi na preostala mjesta druge i trete skupine: k - x elemenata druge skupine morat C e zauzeti mjesta u tretoj, a x elemenata treCe u drugoj. Sad joii samo trebamo smjestiti npr. x elemenata druge skupine na mjesta u prvoj skupini. Sve u svemu, rezultat glasi:

UvrZtavanjem se ponovno dobiva rezultat 10.

4 ' SljedeCa moguCa generalizacija jest: k elemenata, svaki kratnosti 2.


17. Moiemo biti sigurni da, ako 51 toEku podijelimo u 25 ili manje skupina, postoji skupina koja sadrii barem tri toEke (Dirichletov princip). No, problem je kako napraviti podjelu kojom Cemo zadovoljiti uvjete zadatka. Krugovi kao predloieni skupovi su neprikladni, jer ih se ne moie organizirati tako da Eine particiju skupa toEaka kvadrata.
Pokuiiajmo podijeliti kvadrat na 25 dijelova na najprirodniji natin: podijelimo svaku stranicu kvadrata na 5 jednakih dijelova. Spajanjem odgovarajuCih toEaka s nasuprotnih stranica dobivamo 25 kvadrata, svaki stranice duljine 115. Sad znamo da postoji kvadratit u kojem se nalaze barem tri toEke. Bududi da se svaki od dobivenih kvadratiCa moie pokriti krugom radijusa 117, time je tvdnja dokazana.

+ I ' 6 a k vrijedi i vise nego Lto se traii u zadatku: ne pokrivamo toEke bilo kakvim, nego nekim od 25 toEno odredena kruga.
e 18. Lako moiemo naCi broj traZenih rasporeda. Za svaku kuglicu izabiremo u koju C e u kantu za smeCe). Broj od kutija uCi - 1.,2.,. . . , p - t u ili niti u jednu od njih (tj. uCi C naEina izbora je (p l ) n , a eksponencijalna funkcija izvodnica glasi:

8. ZBRKA ZADATAKA

151

% Zgodno je pogledati kako se ova funkcija izvodnica ponaia u odnosu na dodavanje jedne kutije.
Znamo d a c(x) = a(x) . b(x) za eksponencijalne funkcije izvodnice znaEi cn = (;)akbn-k. Za c(z) = e ( ~ + ~ )funkciju ", izvodnicu za p 1 kutiju, te a(x) = e(pS1)" dobivamo b(x) = ex , te rekurzivnu relaciju medu koeficijentima koja trivijalno vrijedi

( p + 2)" =

Ona kaie da kuglice smjeitavamo u p 1 kutiju tako da prvo kaiemo koje kuglice neCe iCi u p 1-vu kutiju, a zatim ih rasporedimo.

k=O

2 ($b+ ilk.
+

9+ Ako preformuliramo zadatak tako da se radi o jednakim kuglicama, obiEne funkcije izvodnice C e se sliEno ponaiati u odnosu na dodavanje jedne kutije. ProuEite to!
19. Od broja svih puteva od ishodiita do pravca x y = t oduzet Cemo broj onih puteva koji pritom prolaze toEkom (p, q) . Prvi od traienih brojeva iznosi 2 t , jer od ishodigta do zadanog pravca trebamo napraviti t koraka, ali nije bitno koliko ih je horizontalnih, a koliko vertikalnih. Drugi broj je jednak:
I
\

jer smo prvo morali doCi do toEke (p, q) , a zatim slobodno do pravca z

+ y = t . Rezultat:

+ I ' Sve moguCe ciljeve mogli smo prvo podijeliti u dva skupa: one do kojih se moie stiCi prolazeCi kroz (p, q) i ostale (ovaj drugi skup se prirodno raspada na dvije klase).
20. Prvo rjezenje - razunsko. Sumanada je paran broj, pa to moiemo pokuSati iskoristiti zbrajajuti dva po dva sumanda, redom:

(FlF2

+ F2F3) + (F3F4 + F4F5) + . ' . + (F2n-lF2n + F 2 n F ~ n + l )

+ ((F3F5 - F3F3) + (F5F5 - F5F5)) + . . .


+ ((F2n-lF2n+l
- F2n-1F~n-1)
-F;+F;~+~
~ & + ~ - 1 .

((FlF3-FlFl)+(F3F3-FlF3))

+ (Fzn+lF2n+l - Fzn-lFzn+l))

= =

Drugo rjegenje - kombinatorno. Prisjetimo se da Fibonaccijeve brojeve Fn moiemo interpretirati kao brojeve poploEavanja pruge 1 x n ploEicama 1 x 1 i 2 x 1 . Tada produkte Fibonaccijevih brojeva FkFl moi emo interpretirati kao brojeve poploEavanja Ic x 1 pravokutnika 1 x 1 , 1 x 2 i 2 x 2 ploEicama, tako d a iljebovi izmedu ploEica Eine u polaznom pravokutniku duiine duljina Ic, odnosno I . Ta poploEavanja moiemo interpretirati i kao poploEavanja zadana particijama dvaju medusobno okomitih stranica pravokutnika u duZine duljina 1 i 2.

152

8. ZBRKA ZADATAKA

Poku2ajmo pomoCu primjera dokuEiti kako treba brojati! Za n = 1 imamo Fl F2 + F2F3. To ne djeluje kao neko sistematsko prebrojavanje, ali drugaEije zapisano, kao Fl F2 F2F1 F2F2 , vet je bolje:

Ovime su dana sva poploEavanja 3 x 3 kvadrata do na jedno, koje u gornjem lijevom kutu ima jedan 1 x 1 kvadrat, a sve ostale ploEice imaju barem po jedan brid duljine 2 (zbog tog poploEavanja rado bismo u sumi vidjeli jo2 i Fl Fl - vidi nastavak izvoda!). Krenimo dalje. DopiSemo li sumi za n = 1 joS dva Elana koja nam trebaju d a dobijemo sumu za n = 2 , te raspiSemo li ih kao F3F4 F4F3 F4F4, moiemo zakljuEivati kao i ranije. Sumandi koji su sp veC nalazili u sumi za n = 1 odgovaraju poploEavanjima 5 x 5 kvadrata koja na posljednjim trakama desno i dolje imaju ploEice duljine barem jednog brida 2: na desnoj traci to je horizontalni, na donjog vertikalni brid. Dva sumanda F3F4 F4F3 broje poploEavanja s po jednom trakom Sirine 1 i jednom Sirine 2, dok sumand F4F4 broji poploEavanja s obje trake 2irine 1.

Izrecite slutnju i dokaiite je matematiEkom indukcijom!

9 Kako bi izgledala odgovarajuta tvrdnja koja bi opisivala poploEavanja kvadrata stranica parne duljine? Izvedite je kombinatorno, a dokaiite raEunski!
!.?+eke rjeienje - g e o r n e t ~ j s k o . Prikaiimo F;F;+l kao povr2ine pravokutnika stranica F; i Fi+l , a zbrajanje kao lijepljenje pravokutnika d u i bridova. Prva dva pribrojnika opisuju povrSinu lika:

Dodavanje jog dvaju pribrojnika vidi se na sljedetoj slici:

8. ZBRKA ZADATAKA

Dokaz provedite matematiEkom indukcijom slijedeCi ovu ideju.

21. Prikaiimo particije iz zadatka tako da iz svakog faktora izluEimo njegov zadani djeljitelj:
n = n l + 2 n 2 + 3 n 3 + . . . + p np, gdje je n i , za svaki i E Np proizvoljni nenegativni cijeli broj. Dakle, broj traienih rastava je jednak broju rastzva koji se sastoji od proizvoljnog broja jedinica, pa od nekoliko dvojki, malo trojki itd, sve do eventualno nekoliko brojeva p . ObiEna funkcija izvodnica za ovakve rastave je oEigledno:

22.

UoEite prvo da m dijeli n.

(a) Svaki prosti faktor broja V(a, b) ulazi u V(a, b) s veCom od potencija s kojima se pojavljuje u a , odnosno u b. Faktori broja M(a, b) su takoder faktori brojeva a , odnosno b, ali u t a j broj ulaze s manjom potencijom.

U zadatku imamo zadane brojeve m i v , a traiimo brojeve a i b Eiji bi zadani brojevi bili najveCa zajedniEka mjera i najmanji zajedniEki viiekratnik, redom. "Viikovi" e uCi ili cijeli u a , ili cijeli u b . Zanima nas koji 6e od potencija u v u odnosu na m C tih viikova udi u a , a koji u b. OznaEimo li:

tada je rezultat jednak 2' (b) Brojeve a i b moZemo birati nezavisno jedan od drugoga, a za svakog od njih je bitno da eksponent uz svaki p i bude veCi ili jednak si , a manji ili jednak ri . Rezultat:

%+ Neka su na primjer m = 1 i v = 12. Nacrtajmo m r e h svih djeljitelja broja 12 na sljedeCi naEin: u ravnini napiiemo sve pozitivne djeljitelje broja 12. Zatim nacrtamo strelicu od broja i do broja j ako i dijeli j i ako je pripadajuti kvocijent prosti broj.

154

8.

ZBRKA ZADATAKA

Na strelice moiete napisati i odgovarajuie djeljitelje. Nacrtajte mreie i za sljedeie parove brojeva (m, v) : (1,162), (1,4375), (9,108), (1,216) . Analizirajte sliEnosti medu dobivenim crteiima. Objasnite pomoCu mreia: o Eemu se radi u zadatku? 23. Iz sume moiemo ukloniti produkt promjenjivih binomnih koeficijenata koristeCi formulu:

(:) (Y) (Y)(:) ):(


= =
Nakon toga koristimo binomni teorem:

(TI:).

U sluEaju m = n rezultat je jednak 1, a inaEe 0.

tP Kombinatorni dokaz ovdje je moida malo izvjeHtaEen, no zgodno ga je navesti za vjeibu. Brojimo sve bijekcije Nn ,Nm , tako da definiramo Ai kao skup svih injekcija koje ne pogadaju element i , te umjesto veliEine skupa A: n A: fl . . . n A k raEunamo veliEine svih Al n A 2 n. . .n A k i koristimo formulu UI. S druge strane broj bijekcija znamo izratunati i direktno, pun0 jednostavnije nego upravo opisanim naEinom. IzjednaEimo li dobivena dva izraza, dobivamo rezultat kojeg joH treba podijeliti s n! da bismo dobili rezultat kao i u prvom rjeHenju.
24.

Prvi dokaz - ozigledni. RaspiHite desnu stranu po binomnom teoremu.

Dmgi dokaz - kombinatorni. Znamo da podskupova neparne veliEine proizvoljnog skupa ima jednako mnogo kao i podskupova parne velitine. Prelaskom na karakteristiene funkcije vidimo da uredenih n-torki sastavljenih od brojeva 0 i 1 s parnih brojem nula ima jednako kao i onih s neparnim brojem nula. To i odgovara formuli koju treba dokazati za k = 1 . Sto se dogada za ostale k ? Odakle viHak n-torki s parnim u odnosu na n-torke s neparnim brojem nula? Naime, znajuii da ukupno n-torki ima pn n n = (k l ) n (suma n-torki s parnim i n-torki s neparnim brojem nula), iz predloiene formule dobivamo pn - nn = (k - l ) n . S druge strane, ako uspijemo dokazati ovu relaciju, onda je

8. ZBRKA ZADATAKA

zadatak rijeSen, jer iz nje i oEigledne relacije za pn nn moiemo izraEunati pn . Uzmimo proizvoljni k i pogledajmo Sto se dogaaa dok povekavamo n . Za n = 1 sve n-torke su (0), (I), (2), . . . , (k) . Jedna n-torka ima neparni, k imaju parni broj nula. Razlika je, kao Sto smo i oEekivali k - 1. NapiSite primjere za n = 2 i moida jog n = 3 . Pri prijelazu na za jedan veLi n , od svake n-torke s neparnim brojem nula dobivamo po jednu ( n 1) -torku s parnim i po k s neparnim brojem nula. Od svake n -torke s parnim brojem nula dobivamo po jednu (n 1) -torku s neparnim i po k s parnim brojem nula. Ili, kompaktnije zapisano:

Traiena razlika iznosi: Pn+l RjeSavanjem ove rekurzivne relacije po razlici pn Pn - nn = (k - l ) n .

- nn+l = (k - l)(pn - nn). - nn dobivamo ono Lto smo i trebali:

25. "Zaobilaienje" vigekratnika podsjeka na formulu ukljuEivanja-iskljuEivanja. I zaista, zbog apsolutne konvergencije reda C 112~ moHemo raEunati po analogiji:

tP
26.

Pola sume

Ci21 112'

Prvo vidimo da je jednostavno izraEunati broj k x k kvadrata (on iznosi (p+l-k)2 ) Smatramo li toEku 0 x 0 kvadratom, funkcija izvodnica glasi:

156
OEito je, kako god raEunali ovu sumu, bit C e nam potrebna suma rezultat. IzraEunajmo to!

8. ZBRKA ZADATAKA

k2tk kao pomoCni

Sjetimo li se da deriviranjem izraza t k dobivamo faktor k ispred potencije od t , to nas moie potaknuti da krenemo od neke sume potencija od t . Uzmemo npr. sumu prvih p 1 potencija. Na kraju Cemo vidjeti kako se broj sumanada u zavrgnom izrazu pona'sa u ovisnosti o broju potencija u poEetnom izrazu, pa poEetni izraz lako moiemo korigirati.

Imamo:

iz Eega deriviranjem dobivamo:

Mnoienjem cijelog izraza s t (kako bismo osigurali koeficijente oblika k2 umjesto k(k-1) ) i jog jednim deriviranjem dobivamo:

a jog jednim mnoienjem s t konaEni rezultat:

Nakon i t o smo dobili traieni metturezultat, funkciju izvodnicu moiemo urediti na dva nacina.

Prvi natin - direktni. RaspiZemo izraz (p 1- k)2 i pod znakom sumacije uz potencije od t ostavljamo samo Elanove koji sadrie indeks sumiranja (ili njegovu potenciju)

Ostatak raEuna provedite sami, pogotovo u sluEaju ako vam se ne svida drugo predloieno rjeienje ili ako nemate vlastito!

Dmgi n a S n - trik, s manje ratuna. Zamijenimo indeks sumacije!


P

x ( p
k=O

+ 1- k12tk =
P+ 1

(zamjena i = p + 1 - k )

xi2tp+1-i
i=l

8. ZBRKA ZADATAKA

157

27. Analizirajmo sluEaj p z -osi. Vidi sliku !

q . SluEaj q

3 p dobiva se zrcaljenjem ovog rjeLenja oko

Da bi se provukao kroz zadanu prugu, put mora posjetiti toEke i duiine oznaEene dvostrukom crtom. Tako se dolazi na jedinstveni naEin do prve zaokruiene toEke. Nakon toga ostaje jog samo izabrati na koji C e naEin (gornjim ili donjim putem) put doCi od jedne do druge zaokruiene toEke. Postoji jog min{p, q) zaokruienih toEaka koje treba posjetiti. Rezultat: 2min{~,q1.

% Ovo je bilo jednostavno! RijeLite analogni zadatak za prugu Sirine 4: raEunajte broj puteva do prvih nekoliko toEaka (npr. napredujuCi na naEin kao u Pascalovom trokutu), uoEite pravilo, generalizirajte i dokaiite. Sto se zbiva s prugom Birine 6? Za koju girinu pruge postaje za zadanu toEku (p,q) situacija ponovno vrlo jednostavna?
28. Zbog centralne simetrije i uvjeta da u svakom retku treba biti po jedna kuglica, na ploEu neparne duljine stranice moramo postaviti kuglicu na srednje polje. BuduCi da u svakom retku i svakom stupcu smije biti najviSe jedna kuglica, to znaEi da u srednjem retku i u srednjem stupcu viLe neCe biti kuglica (v. sliku), a ostale kuglice ostaju centralno simetriEno rasporedene i nakon Lto izvadimo srednji stupac i srednji redak. Dakle vrijedi:
G2n+l

= G2n.

Vidimo da je dovoljno pronaCi rekurziju samo za Lahovske ploEe parne ili samo za gahovske ploEe neparne duljine stranice. Nadalje raEunamo samo za parne duljine stranice. .4ko postavimo kuglicu u prvi stupac (2n) x (2n) Lahovske plote, onda trebamo nakon toga popuniti 'Lpreostalu" (2n - 2) x (2n - 2) ploEu s istim svojstvom (v. sliku).

158
Rekurzija stoga glasi: a uvrgtavanjem poEetnog uvjeta G2 = 2 lako dobivamo:

8. ZBRKA ZADATAKA

9 Ovaj rezultat, zapisan u obliku Gzn = 2n. (2n - 2). . . . . 2 moie se lako direktno kombinatorno interpretirati. IspriEajte i priEu o rjeSenju zapisanom u obliku n!2n .
29. Prvi element niza moiemo izabrati na n naEina, a svaki sljedeti na 2 naEina ( + 1 ili -1 modulo n). Stoga broj traienih nizova iznosi cn = n . 2n-1 .

z
n>l

n ( 2 ~ ) ~ -= ' (uoEite derivaciju geometrijskog reda)

Ctt Moiete li kombinatorno interpretirati ovaj rezultat, zapisan u obliku z ( 1 + 2z +


22s2

+ . . J2

e razgovarati. Prvi od njih (npr. po abe30. PoEnimo direktno birati pretplatnike koji C cedi) moie u telefonskom imeniku zadanog grada biti prvi, drugi, tredi itd. ili (n-2r+l) -ti po redu. Dakle, njega biramo na ( n - 2r 1) natina. Broj naEina izbora njegovog sugovornika sada ovisi o tome kojeg smo pretplatnika izabrali kao prvog. Izgleda d a se s ove toEke ne moie lako nastaviti raEunanje.

Prvo rjes'enje. Izaberimo prvo pretplatnike koji C e razgovarati, a nakon toga Cemo ih "spariti". Izbor pretplatnika moiemo izvrziti na ( n ) naEina. Sugovornika za prvog izab27". ranog (npr. prvog po abecedi) traiimo na 2r - 1 naclna. Nakon toga biramo sugovornika za prvog neizabranog (na 2r - 3 naEina) itd. U r -tom koraku sugovornika biramo na jedan naEin. Ukupni broj naEina razgovaranja iznosi:

tP

Na kraju smo ipak birali istom logikom kao i u poEetku, ali nije bilo neizabranih!

Dmgo i treCe rjes'enje. IspriEajte price koje dovode do sljedeCih rjeSenja:

Cetetvrto rjes'enje. Ovo rjeSenje je prema obliku rezultata koji se na kraju dobije nalik prethodnom. Izaberimo prvo koji C e telefonski pretplatnici razgovarati. Zatim, izabiremo li parove za razgovor moiemo izabrati prvi par, a onda ostale spariti bilo kako. Time smo prvi par favorizirali u odnosu na ostale, pa C e broj naEina sparivanja, koji se dobije odvojenih formiranjem jednog para od ostalih, trebati podijeliti s brojem mogudih (razliEitih)

8. ZBRKA ZADATAKA

159

mjesta za par koji je u izabranom rasporedu prvi. Stoga, ako s pi oznaEimo broj naEina sparivanja 2i telefonskih pretplatnika, onda vrijedi:
pi = ( : ) p i - l

.i'

za 2i E Nn,

te p l = 1. Rjegenje zadatka je

(g). pr . IzraEunajte iz rekurzivne relacije sami p, .

+f Izraz sparivanje ovdje nije upotrijebljen sluEajno. Naime, u sliEnom znaEenju


se sparivanje pojavljuje u teoriji grafova. 31. Broj manji od 280 ili je prost ili je djeljiv s barem jednim od prostih brojeva iz , jer najmanji prosti faktor broja ne moie biti veCi od njegovog

Formulom ukljuEivanja-iskljuEivanja ("klasiEni primjer") moHe se izraEunati broj brojeva manjih od 280 koji nisu djeljivi niti s jednim od brojeva iz navedenog skupa. Da bismo dobili rjegenje zadatka, broju dobivenom formulom treba dodati 6 (broj elemenata skupa, oni nisu brojani) i oduzeti 1 (jer je broj jedan uzet u obzir). Broj brojeva manjih ili jednakim 280 koji nisu djeljivi niti s jednim od brojeva iz skupa {2,3,5,7,11,13) ima:

[GI -I *[
[2 .:8011]

[&]

- -

[&] - [&] - [&I

[A]

160
Rezultat je tada 54

8. ZBRKA ZADATAKA

+ 6 - 1 = 59.

+f Ovaj pristup moie se opravdati samo Einjenicom da je "formula za broj brojeva manjih ili jednakih nekom prirodnom broju koji nisu djeljivi s brojevima iz nekog skupa" poznata. UobiEajeni postupak za nalaienje svih prostih brojeva manjih ili jednakih nekom zadanom broju ( n ) zove se Eratostenovo sito, a provodi se na sljededi naEin: napi'su se svi brojevi manji ili jednaki n . Zatim se u toj listi prekriie svi vigekratnici broja 2 osim samog broja 2. Prvi neprekrizeni broj je 3, pa se sada prekriie svi vigekratnici broja 3 osim broja 3 (neki su veC prekriieni, gto ne smeta - ne moramo ih prekriiiti dvaput). Prvi sljedeCi neprekriieni broj je 5, koji je ujedno i prvi prosti broj nakon 3. Prekriiimo sada i sve njegove vi'sekratnike, te nastavimo postupak dok ne dodemo do posljednjeg neprekriienog broja.
Primjetimo da se postupak uvijek moie prekinuti Eim se prede broj neprekriieni brojevi su prosti! Primjer ( n = 29 ):

fi,svi ostali

ProuEite ovaj postupak i pogledajte kakve on ima veze s izrazom dobivenim iz formule ukljuzivanja-iskljuEivanja.

32. Navikli smo raditi s binomnim koeficijentima, tj, prikazivati razne izraze izborima podskupova. I ova se suma moie lako pretvoriti u sumu binomnih koeficijenata. UoEimo da po dva broja pod faktorijelima u sumandima na lijevoj strani rastu, po dva padaju. Dakle, moiemo spariti te brojeve tako da im sume iznose n - 1, odnosno n (od drugog sumanda nadalje: 1 (n - 2) , 2 (n - 3) , itd; 2 (n - 2) , 3 (n - 3) itd). Da dobijemo binomne koeficijente, trebamo jo's pomnoZiti zadani izraz s (n - l)!n! . Dobivamo:

Sad veC znamo da se radi o izborima n-Elanog podskupa (2n - I) -Elanog skupa. Lijeva strana govori da bismo te izbore trebali promatrati kao izbore dijela predmeta iz n-Elanog skupa i dijela iz (n - 1) -Elanog. No, zagto onda ne biramo svih n elemenata? U stvari, mi biramo n elemenata, ali se to ne vidi jasno iz ovog oblika izraza. Malom promjenom - prelaskom na komplementarne binomne koeficijente u prvom faktorima - dobivamo (mogli smo imati sreCu i odmah pogoditi L'pravi" izraz?):

Sada vidimo d a su svi izbori n elemenata podijeljeni u ovisnosti o tome koliko elemenata uzimamo iz n - 1-Elanog skupa.

8. ZBRKA ZADATAKA

33. Broj je djeljiv sa 4 ako je dvoznamenkasti broj koji je sastavljen od zadnje dvije znamenke zadanog broja djeljiv sa 4.
Dakle, ako su barem dvoznamenkasti, traieni brojevi mogu imati bilo koje znamenke na prvih n - 2 mjesta, a na zadnja dva moraju imati neku od kombinacija: 12, 24, 32, 44. 52,

Zadanih n-znamenkastih brojeva postoji dakle 5n-2 . 5 , za n jednoznamenkasti broj s t r J e n i m svojstvom, i to je broj 4. Funkcija izvodnica glasi:

> 1.

Postoji samo jedan

34.

Zadatak zahtijeva rutinsko rjegavanje, pa navodimo samo osnovne toEke rjegenja. KarakteristiEna jednadiba: z2+2a:+1=0

ima jedno dvostruko rjegenje: pa rjeLenje pripadne homogene jednadZbe glasi: a; = A(-1)"

+ Bn(-1)".

Desna strana zadane jednadibe je polinom prvog stupnja, pa tragimo partikularno rjegenje u obliku: a% = X n + Y . UvrStavanjem u polaznu jednadZbu dobivamo X = 1, Y = -1. OpCe rjeLenje zadane jednadZbe glasi: h an = a, +a: = A(-1)" Bn(-1)" n - 1.

IzjednaEavanjem vrijednosti za n = 1 i n = 2 dobivamo konkretne vrijednosti za A i B , te konaEno rjegenje: a n = 3(-1)*+' 2n(-1)" n - 1.

35. Prvi naEin - svodenje nejednadzbe na nejednadzbu s nenegativnim rjes'enjima. RjeBavanje ove nejednadibe u skupu nenegativnih brojeva je privlaEno, bez obzira na to kako je promatramo. Na primjer, moiemo je promatrati kao jednadibu s y z = n koju rjegavamo u skupu nenegativnih brojeva (pokugajte s drugim interepretacijama, npr. s geometrijskom interpretacijom u koordinatnoj ravnini). U svakom sluEaju, lako je doCi do do rjeLenja "pojednostavljene" nejednadibe.

+ +

No, buduCi d a u zadatku stoje apsolutne vrijednosti, neka od tih rjegenja davat C e Eetiri, neka dva, a neka samo jedno rjelienje iz Z. RjeLenja oblika: (0,O) postoji jedno

162
(0, y) , x

8. ZBRKA ZADATAKA

a a

a (x,O), y

# 0 postoji n pozitivnih, ukupno 2n cijelih # 0 postoji n pozitivnih, ukupno 2n cijelih (x, y) , x, y # 0 : traiiimo li samo pozitivna rjelienja ovo je ekvivalentno rjezavanju jednadiibe x + y + v = n , za x, y 2 1, v >, 0 - njih postoji ((n-2)2+3-1) =) ; ( ;
prelaskom na polaznu nejednadibu bilo koji od x, y moiemo uzeti kao pozitivni ili kao negativni, pa je ukupni broj rjelienja 4(;) .

Zbrajanjem ovih Eetiriju stavki dobivamo da nejednadiba ima 1 2n

+ + 2n2 rjelienja.

Drugi naSn - svodenje nejednadzbe na niz jednadzbi. Jednadibu moiemo zamiy 1 = k za 0 k n , gdje su takoder x, y E Z . Svaku jeniti nizom jednadiibi 1x1 1

< <

od ovih jednadiibi moiemo ponovno svesti na jednadibu bez apsolutnih vrijednosti i onda "preraEunati" rezultat ili je promatrati kao skup toEaka u koordinatnoj ravnini. U svakom sluEaju dobiva se da jednadiba x y = k ima 4 4(k - 1) = 4k rjelienja za k > 0 , a jedno rjelienje za k = 0 . Sumiranjem dobivamo rezultat polaznog problema:

1x1

+ Iyl < n

D e 6 na6n

iisasvim geometrijski". Brojimo cjelobrojne toEke unutar kvadrata

i dobivamo rezultat (n

+ 1)' + n2 - vidi sliku!

9-t

Ovime nisu iscrpljena sva rjelienja - nadite sami jog neka!

36. Umjesto relacija ekvivalencije brojimo particije. To je u redu, jer svaka relacija ekvivalencije inducira jednu particiju istog skupa, a svaka particija jednu relaciju ekvivalencije, te je opisana veza relacija ekvivalencije i particija bijektivna. Razlog ovakvoj odluci je u lakoCi predoEavanja particija za razliku od relacija.
Lijeva strana jednakosti koju treba dokazati kaie da treba promatrati particije n + l Elanog skupa. Desna strana izgleda kao d a biramo prvo neki i-Elani skup, a zatim ga particioniramo.

8. ZBRKA ZADATAKA

163

Interpretacija desne strane: Prvo biramo blok u kojem C e stajati (n 1) -vi element. On moie biti j-Elani skup za bilo koji j E Nn+?. Ako osim elementa n 1 on sadrii toEno n - i elemenata, onda ga moiemo izabrati na ( n : i ) = (1) naEina. Nakon toga moramo particionirati jog i ostale elemente, Lto je moguCe napraviti na ri naEina.

UoEimo da trebamo staviti ro = 1, Hto je i logicno. OznaEimo eksponencijalnu funkciju izvodnicu za niz (ri) s r(x) := Ci20 rixi/i! . Dokazana relacija, dijeljenjem s n! (da bi se faktorijeli "pravilno rasporedili"), mnoienjem s xn i sumiranjem za n E N daje: rl(x) = ~ ( x .)ex. Ova diferencijalna jednadiba lako se rijeSi separacijom varijabli, iz Eega slijedi: ~ ( x= ) eez-c, a uvazavanjem r(0) = TO = 1 dobiva se konaEni rezultat: ~ ( x= ) eez-l.

37. BuduCi d a u svakom retku treba stajati po jedna jedinica, mokemo smatrati prirodnim d a traiene matrice interpretiramo kao funkcije NT + Nn+p (svakom retku pridruii se broj stupca u kojem je njegova jedinica). Drugi uvjet, preveden na jezik funkcija, zahtijeva d a izabrane funkcije pogadaju elemente 1,2,. . . ,n.
RjeLenje (usporedite s izrazom za broj surjekcija
N T -+ Nn

):

38. Moiemo prvo poslati signal 1 ili 2 ili . . . ili m. U ovisnosti o poslanom signalu, ostalo nam je jog n - t l , n - ta , . . .ili n - t, sekundi za poslati ostatak poruke. Tako dobivamo rekurzivnu relaciju: bn = bn-tl

+ bn-t2 + . . . + bn-t,.

U zadanom specijalnom sluEaju imamo rekurzivnu relaciju: uz poEetne uvjete: bl = 2, b2 = 5. Nakon Lto se dobije opde rjegenje rekurzivne relacije, vidi se da ovi poEetni uvjeti nisu idealni, jer zahtijevaju potenciranje "nezgodnih" brojeva. Uvritavanjem poEetnih uvjeta u rekurzivnu relaciju za n = 2 dobiva se bo = 1 (1ogiEno i bez raEuna, zar ne?). Sada se rjeHenje dobiva izjednazavanjem opCeg rjeLenja za n = 0 i n = 1 s bo i bl , te na kraju izlazi:

Jednostavni zadaci koji se odmah Sam po sebi nameCu su: koliko ima poruka +f' Eije slanje traje najviLe n sekundi ako (a) pauza (interval bez poruka) mora biti na kra,ju tih n sekundi, (b) izmedu pojedinih signala smije biti pauza cjelobrojne duljine.

164

8. ZBRKA ZADATAKA

39. Prvi nac'in - prebrojavanje. Desna strana nam daje do znanja da je cijeli izraz zgodno promatrati kao broj naEina izbora k-Elanih multiskupova nad n-Elanim skupom. 0 Eemu se radi na lijevoj strani? Prvo za neki s izaberemo s-Elani multiskup nad n Elanim skupom, pa svaki element tog multiskupa uzmemo dvaput. To radimo zato jer u sljedetem koraku trebamo birati k - 2s elemenata (a ne k - s ) n -Elanog skupa. Izborom tih k - 2s elemenata se odreduje koji C e elementi u zavrSnom k-multiskupu biti neparne kratnosti. Tako je jednoznaEno opisan k-Elani multiskup nad n Elanim skupom. I obratno, svaki k -multiskup nad n-Elanim skupom moie se opisati na predloieni naEin. Postupak ponavljamo za sve s za koje on ima smisla (podsjetite se, n<k.
' = I ) IspriEajte jednu "Zivu" priEu o ovoj jednakosti.

) ( ;

= 0 Eim je

Drugi naSn - funlcije izvodnice. Broj na desnoj strani je koefici'ent uz xk u izrazu (1 - XI-". Traiimo sada izraz kojem je lijeva strana koeficijent uz xi. Na lijevoj strani imamo izraz koji je skoro konvolucijskog tipa - kvari ga samo to Bto u prvom faktoru stoji k - 2s umjesto k - s. No usprkos tome moiemo lijevu stranu pokusati traiiti kao koeficijent u nekom produktu, ali C e jedan faktor morati sadriavati x2 umjesto x . Brzo se vidi da t a j produkt treba biti (1 x)"(l- x2)-".

Dakle i lijeva i desna strana su koeficijenti uz istu potenciju varijable x u istom izrazu, pa su stoga jednake.
40. Zadnji element niza moie biti 0 ili neki element razliEit od nule. Ako je zadnji element jednak nuli, onda prvi dio niza ima neparni broj nula - takvih prvih dijelova onda postoji (k l)"-' - an-1. Ako je zadnji element jedan od k elemenata razliEitih od nule, onda prvi dio niza sadrii paran broj nula, a takvih prvih dijelova postoji an-1.

Dakle: an = [(k RjeBavanjem dobivamo:

+ I)"-'

- a,-l]

+ kanml.

4-t Zanimljivo je d a za k 2 2 postoji viBe nizova s parnim, nego nizova s neparnim brojem nula. IstraZite t u pojavu.
41. Prvo rjeienje - izraEunati prvo traieni broj permutacija, zatim funkciju izvodnicu. Ukupni broj k-permutacija skupa Nn iznosi C ; = o ( n ) k , pa direktnim raEunom slijedi:

Drugo rjeienje - funkcija izvodnica izraEunata iz rekurzije za broj svih permutacija. Kombinatorno ili iz izraza za Pn lako se dobiva rekurzija:

8. ZBRKA ZADATAKA

165

Skica kombinatornog izvoda: k -permutacija n-Elanog skupa je ili prazna ( 0 -permutacija) ili je moiemo izabrati tako da na n naEina izaberemo prvi element u njoj, pa onda za svaki moguCi prvi element ostatak promatramo kao k - 1-permutaciju n - 1-Elanog skupa (i k i n su u ovom sluEaju veCi ili jednak 1). MnoZimo ovu relaciju s xn/n! i sumiramo po n 2 2. Nakon uvrgtavanja poEetnih (l)o = 2 dobivamo uvjeta PI =

p(x)

- 2%= xp(x) + e x - 1 - x,

te isti rezultat kao u prethodnom rjeLenju.


42. Treba izraEunati broj parova disjunktnih podskupova od S . To je isto kao brojati funkcije iz S u troElani skup izjava {"ulazi u A ", "ulazi u B ", "niti u A niti u B ") , uz oEiglednu bijekciju. Takvih funkcija postoji 3".

43.

MoBe se prepoznati da je:

44. Ako ne ielimo niti jednu gljivu svrstati u pravu skupinu, onda za svaku gljivu imamo toEno dva izbora. Gljiva imamo o b j komada, pa rezultat glasi:

y+b+j.

4 , Kad bismo znali koliko je gljiva koje vrste, rezultat bi bio npr.:

(n) (, $ (, -36

On se moie zapisati i u nekoliko drugih oblika - samo naizgled nije simetriEan u varijablama b, o i j . Provjerite to!

+ +

+ i)

U ovom sluEaju rezultat treba biti manji od rjegenja zadatka.


45. Razdvojimo prirodno izbore s razliEitim brojem paketa od po dvije olovke. Tada moZemo prvo izdvojiti olovke koje C e biti zapakirane u paru od olovaka koje C e biti same u paketu.

Ako je broj paketa s po dvije olovke jednak d , onda trebamo izabrati 2d olovaka koje C e uCi u velike pakete. Pri pakiranju tih olovaka, buduCi da nam je svejedno koji je sadriaj kojeg para (zec C e ih u vreCi ionako sve izmijegati), biramo prvo par za prvu olovku, zatim za prvu jog nesparenu, pa opet za prvu nesparenu itd. Stoga je broj slaganja paketa, ako treba biti toEno d paketa s po dvije olovke, jednak:

166

8.

ZBRKA ZADATAKA

a ukupni broj rasporeda:

9-t

Razmislite o direktnoj kombinatornoj interpretaciji posljednjeg izraza.

46. Sve ovisi o tome kako poEnemo poploEavanje - jednom vertikalnom ili dvjema horizontalnim ploEicama:

Zato je: uz poEetne uvjete: P1=l Vidimo d a je Pn = Fn+1 P=2. Sada znamo ili izraEunamo:

, n + 1 -vi Fibonaccijev broj.

47.

Zadatak rje'iavamo 'iablonski - mnoZimo rekurziju s xn : anxn = x[x(k

+ l)ln-l + x ( k - l ) a n - l x n - l ,

zbrojimo po n 2 1, pa uz oznaku a ( x ) = En),-, anxn imamo:

i konaEno, uz primjenu poEetnog uvjeta: 112 112 1 - x(k 1) 1 - x(k - 1) ' Funkciju izvodnicu smo napisali u obliku sume parcijalnih razlomaka d a bismo mogli lako oEitati rje'ienje rekurzije. an je tako suma koeficijenta uz xn u prvom i koeficijenta uz xn u drugom razlomku, a oba se lako izraEunaju. Rezultat: (k l)n (k - l)n an = 2 a(x) =

+ +

% TraZe se kombinatorne interpretacije ove rekurzije i rje'ienja.


48. GledajuCi desnu stranu, vidimo da moZemo razmi'iljati o k -?lanim podskupovima n -Elanog skupa.

Prvo rjegenje. PromatrajuCi lijevu stranu vidimo da se biraju ( k - 2 ) -Elani podskupovi - kao da su neka dva elementa skupa vet izabrana. Uz to vidimo da je skup iz kojeg

8 . ZBRKA ZADATAKA

biramo tih k - 2 elemenata sve manji i manji - kao d a ga t a dva elementa ograniEavaju. Zato je prirodno kao polazni n-Elani skup uzeti Nn (ili neki drugi linearno uredeni skup), a kao dva istaknuta elementa pokugati uzeti najmanji i najveCi element. I zaista, sumandi su sloieni u ovisnosti o razlici izmedu prvog i zadnjeg elementa podskupa, tako da i -ti Elan sume odgovara podskupu s razlikom n - i . Takav podskup biramo na sljedeCi naEin: najmanji element biramo izmedu i najmanjih elemenata skupa N n , najveCi je jedinstveno odreden, a ostale biramo izmedu preostalih elemenata na (n&l) naEina.
Drugo rjes'enje. Moie se redi i da su k-Elani podskupovi na lijevoj strani sloBeni prema drugom po veliEini elementu u podskupu. On moZe biti jednak 2 (prvi sumand), 3 (drugi sumand), . . . , n - (Ic - 2) (zadnji sumand).

49. Premda to neCe osigurati red u razredu, niti jedan par uEenika ne smije ostati zajedno - niti prvi, niti drugi itd. Izgleda d a je ovo dobar teren za primjenu formule UI, jer se broj rasporeda s fiksiranim pojedinim parom moie lako izraEunati.
RaEunat demo prvo broj rasporeda u klupe za situaciju u kojoj se toEno zna mjesto svakog uEenika (klupa i mjesto unutar klupe), a onda Cemo rezultat prilagoditi zadatku dijeljenjem s ukupnim brojem dogovora o mjestima unutar klupa ( 2" ). Ako je odredeno kojih C e k parova sjediti zajedno, onda postupamo na sljedeti naEin. Prvo nademo klupe za one koji ostaju zajedno (na (n)k naEina), a zatim ih pustimo da se smjeste unutar klupa (na 2k naEina). Nakon toga pustimo ostalih 2n - 2k uEenika d a sjednu kako Bele na preostala mjesta (oni to mogu uEiniti na (2n - 2k)! naEina). Broj naEina sjedanja 2n uEenika u n klupa, tako d a nekih k parova ostane sjediti zajedno tada iznosi: (n)k2k . (2n - 2k)!. Primjenom formule ukljuEivanja-iskljueivanja dolazi se do broja rasporeda u kojima se zna tko sjedi lijevo, a tko desno u svakoj pojedinoj klupi: C (-ilk (n)k21c(2n-2k)!. k20 Rezultat zadatka dobiva se dijeljenjem s 2" :

(i)

9+ ObrazloBite odluku da se radi s rasporedima u kojima se zna tko sjedi lijevo, a tko desno u klupi.
50. Svaku buhinu Zetnju duljine 1 moZemo prikazati uredenom 1 -torkom brojeva iz N, i obratno, svaka uredena 1 -torka iz predstavlja jednu Eletnju. Traiene getnje imaju na zadnjem mjestu broj 1 te toEno t trojki.

NL

Biramo prvo gdje C e stajati te trojke, a zatim biramo brojeve koje Cemo staviti na ostala mjesta. Ako buha moBe skakati na mjestu, onda je rezultat:

Ako buha ne moZe skakati na mjestu, pojavljuju se neka ograniEenja. Pri izboru trenutaka u kojima C e ona biti u toEki 3 moramo paziti da ne budu uzastopni - izabrat Cemo t -Elani podskup ( I - 1) -Elanog skupa koji nema susjednih elemenata (prvih t - 1 elemenata koje treba izabrati vuku za sobom svoje sjene, od kojih svaka pokriva sljedeCi

168

8. ZBRKA ZADATAKA

element, tako da ga ne moiemo izabrati; broj naEina izbora je ). Pri izboru ostalih toEaka moramo paziti d a buha ne skoEi u toEku 3 (jer su svi boravci u toEki 3 vet odredeni) i da ne skoEi na mjestu. To znaEi da za svaki od koraka u kojima ne dolazi u toEku 3 ima n - 2 moguCnosti izbora. Rezultat iznosi:

('-'-lt-'))

51. OEito je da u ovom zadatku Englezi sluie kao pregrade izmedu Francuza i Arapa. Stoga C e se broj grupa Francuza i Arapa malo razlikovati u ovisnosti o tome je li broj Engleza paran ili neparan.
Za paran e = 2e' moguCi su rasporedi: A . . . A E F . . . F E A . . .A E F . . . F E A . . . A E F . . . FE

... E A . . .A, . . . E F . . . F, . . . E F . . . F, . . . EA . . .A,

s e' 1 grupom Arapa i e' grupa Francuza ili obratno. Za neparan e = 2e' - 1 moguCi su rasporedi: A . . .A E F . . .F E A . . . A E F . . . F E A . . .A E F . . . FE oba s po el grupa Arapa i el grupa Francuza. IzraEunajmo prvo broj ovakvih shema. Kasnije moiemo pustiti pripadnike pojedinog naroda da se smjeste unutar pozicija predvidenih za njih. BuduCi d a Englezi sluie samo kao pregrade, da dobijemo broj shema slaganja ljudi po narodima, raEunamo broj rasporeda Arapa i Francuza u zadani broj grupa. Npr. za 1 i Francuza u e' grupa, te Arapa u paran e brojimo naEine grupiranja Arapa u e' e' i Francuza u el 1 grupa. Analogno se broje i rasporedi za neparni e . Stoga je broj shema slaganja ljudi po narodima jednak:

a+(el+l)-1

f + e 1 - - 1 + (a+:-1)

za neparan e. Rezultat zadatka je a!e!f! puta vedi jer moramo jog poslagati ljude unutar rezerviranih' mjesta za pojedini narod.

%+ Obrazloiite zagto je sljedeCe zakljuEivanje pogregno: Postavimo Engleze tako da nikoja dva nisu medusobno susjedna, pa Cemo u prostore izmedu njih stavljati naizmjeniEno grupe Francuza i grupe Arapa.
52. Prirodno je d a izbore poslaZemo po broju izabranih predmeta iz skupine jednakih. Ako t a j broj oznaEimo s j , rezultat je:

tP ToEno polovina svih podskupova 2 n 1-Elanog skupa ima manje ili jednako n elemenata (dokaz uspostavljanjem bijekcije A I+ A'). Svaki od takvih podskupova
moiemo nadopuniti na jedinstveni naEin do n-Elanog izbora elemenata polaznog multiskupa uzimanjem "jokera" - proizvoljne koliEine jednakih elemenata. To daje direktnu kombinatornu interpretaciju dobivenog rezultata.

8. ZBRKA ZADATAKA

169

53. Prvo trebamo rasporediti trojke po stolovima, a zatim se ljudi unutar svake trojke trebaju dogovoriti tko be gdje sjediti:

Eksponencijalna funkcija izvodnica (uz dogovor

a0

= 1):

Dobiveni formalni red bio bi obiEna funkcija izvodnica za neke izbore Eiji je broj jednak 6n = (3!)2. Dakle svejedno je koja trojka sjedi za kojim stolom (strana svijeta na koju svaka pojedina osoba gleda je jog uvijek bitna).
54. OpiHimo prvo rijeEima uvjete zadatka: u zadanim nizovima jedan se element iz Nn pojavljuje samo jednom, i to na zadnjoj poziciji. Na ostalim pozicijama nalaze se zastupljeni svi ostali elementi iz Nn (neki moida i viHe puta).

Iz ovoga odmah vidimo d a mora biti n d . Izaberimo element iz Nn koji C e se pojaviti samo na zadnjem mjestu niza. To moiemo napraviti na n naEina. Kako god t o napravili, ostaje nam jog raspodijeliti preostalih n - 1 elemenata na d - 1 mjesta, tako da se svaki element upotrijebi barem jednom. To odgovara surjekciji s Nd-l na Nndl , pa je ukupni rezultat jednak (za dobivanje broja surjekcija v. poglavlje o formuli UI):

<

e koji igraE dobiti karata koje boje, t j , koji C e igraE 55. Prvo trebamo izabrati koliko C dobiti koju boju "viGkan. To moiemo napraviti na 4! naEina. Nakon toga, za svaku boju dijelimo karte posebno d a bismo saEuvali upravo izabrani raspored boja medu igraEima. Za svaku boju izaberemo prvo skupinu od Eetiri karte za onog igraEa koji treba dobiti Eetiri karte upravo te boje. Zatim izdvojimo jog za svakog od preostalih igraEa po jednu skupinu od tri karte. Karte jedne boje mogu se podijeliti na

naEina, pa ukupni rezultat glasi:

56.
m

Prvo rjeienje. Na visinu I moiemo prvi put zakoraEiti na apscisi 0,1,2,. . . ili - k . Nakon toga k koraka idemo po pravcu y = 1 , a zatim napuiitamo t a j pravac

korakom prema gore.

170

8. ZBRKA ZADATAKA

Trebamo dakle izabrati apscisu i dolaska na pravac y = 1 , a zatim put od toEke (0,O) do (i, 1 - 1) te put od (i k, I 1) do (m,n) . Rezultat:

jer vrijedi jednakost:

za sve a, b, c E No (radi se o izborima a b 1-Elanog podskupa a skupa sortiranim po (a 1) -vom izabranom elementu).

+ +

+ b + c + 1-Elanog

Drugi naEin. Ako pogledamo sliku uz prvo rjegenje, vidimo d a je za definiranje traIene Zetnje dovoljno razmjestiti m- k horizontalnih koraka duljine 1i jedan "korak" duljine k prikazan na slici. Pritom moIemo svih m - k 1 koraka smatrati ravnopravnima, jer ne moramo birati poziciju dugog koraka - on mora biti na visini I . SliEno moHemo razmjeZtati n - 2 vertikalnih koraka i LLkorak" sa slike. Ukupno koraka ima: ( n - 2) (m- k) 1, pa rezultat glasi:

+f' Motemo reCi i da biramo puteve koji imaju m - k horizontalnih i n - 1 vertikalnih koraka smatrajudi d a nas pri planiranom vertikalnom koraku s visine 1 - 1 na visinu 1 nest0 prisili da napravimo dugaEki korak sa slike.

Na sliEni naEin moHemo direktno izraEunati i broj svih Zetnji koje provode barem K jedinica na visini I . IzraEunajte t a j broj i sumiranjem prethodno dobivenog rezultata po k 2 K i uvjerite se da su oni jednaki.

9+

Zadajte analogni zadatak sa skupovima.

57. Prvo rjegenje - direktno. Lijeve strane zadanih rekurzija podsjekaju na koeficijente u produktu. Bududi da se uk -ovi pojavljuju i samo s indeksima odreitene parnosti, dobro

8. ZBRKA ZADATAKA

je pogledati :to 'bi se dobilo mno8mjem OBI u(zf s l/(& - 2') :

l t

i\

Sada se moie izraEunati traI

Drugo rjes'enje - prvo odredzti u k , zatzm OFI. Zbrajanjem zadanih re1

mo:
2n

iz Eega oduzimanjem slijedi:

Oduzimagjeq ist* relacija sa sumama ,

uI dobiva se rel?cija(:

ovih relacija, l z zadani poEetni uvjet, idazi da su ur -oii ~ibon&cijeui brojevi: uk = F k , za k E IU. Dalje rai;npoq dobivamp: , , ,, ' ,

omb bin iron^&

U2n

'u2n-1 + u2n-2.

' +Pi Lukavije bi bilo defintrati uo = 1 , nakon Eega se zadane dvije rkkurzije mogu zbrojiti (na dva naEina!) f %k#isatijednom forrmrlom: , ' .
I

u o f 2 ~ 1 + ~ 2 - t - . . . + U n -= lua+1.

Ponovno oduzimanjem dviju uzartopnih r,elacij? izlbzi rek~rzija za ~ i b b o n a c i j e ~ e ~ ~ r o f e ~ Na kraju treba jo5 aduzimarijem jedinici ispraviti rjezenje koje se dobije, jef je niz kojeg trebamo d&0i definirpn samo &a n N . , . , , i ' , 58. Umjesto da zbrqjamo gotbve brojeve; zbrajat demo njihovei saetavne dijelow, tj, moguCe jedinice, desetice, stotice, tisuCic ko puta u koliko se brojeva moie pojaviti.
, I

1 ,

MoguCe znamenke jedinica su 2, 4 i 6, a za svaku od njih moZemo na 6 naEina izabrati znamenku desetica, na 6 naEina znarnenku st6tica i na 6 naEina zaamenku tisutica. 'MoguCi doprinosi znamenke desetica su 10, 20! 30, 40, 50 i 60, a za svaki od njih moiemo na 3 naEina izabrati znamenku jedidica (broj treba biti paran!), te na po 6 naEina znamenku

8 . ZBRKA ZADATAKA

stotica i znamenku tisutica. SliEno vrijedi i za ostale znamenke. Rezultat: (2+4+6).6*6,6


(10+20+30+40+50+60).6.6.3

(100 200 300 400 500 600) . 6 . 6 . 3 (1000 2000 3000 4000 5000 6000) . 6 . 6 . 3

+ + + + + + + + + +

+ + +
=
2520072.

59. ielimo li postaviti n figurica na n x n gahovsku ploEu, moiemo postupiti ovako: prvo izaberemo mjesto za prvu figuricu (razliEite su, pa ih moiemo urediti, npr, po veliEini). Za ostalih n - 1 figurica ostao je dio ploEe koji je u smislu smjegtavanja figurica ekvivalentan (n - 1) x (n - 1) ploEi. Stoga rekurzija glasi:

uz poEetni uvjet: RjeHavanjem slijedi:

4-t Moiete li n2 iz rekurzije interpretirati jog na koji naEin?

tP Ovaj rezultat moie se interpretirati i na treCi naEin: Prvo smo na Hahovskoj ploE i izabrali mjesta za figurice, a svaki izbor mjesta odgovara jednoj permutaciji: u svakom retku oznaEimo po jedan stupac, tako da niti jedan stupac ne oznaEimo dvaput. Takvih izbora ima n! . Nakon toga rasporedimo n figurica na n mjesta - to je moguCe uEiniti na n! naEina. Ukupni rezultat je ponovno n! . n! .
60.

Zbog tuzemnih linija potrebno je barem n boja.

Medunarodne linije moiemo shvatiti kao bijekciju izmedu skupa aerodroma prve zemlje i skupa aerodroma druge zemlje. BuduCi da su aerodromi unutar prve zemlje raznobojni, moiemo (nakon bojanja aerodroma prve zemlje) medunarodne linije shvatiti i kao funkciju s nekog n-Elanog podskupa skupa boja u skup svih boja. Pritom t a funkcija mora biti injekcija i ne smije preslikavati niti jednu boju u istu t u boju. Minimalni broj boja je n , jer ovo preslikavanje moiemo realizirati tako d a se prva boja preslikava u drugu, druga u treCu, itd. n - t a u prvu. Ako imamo na raspolaganju b 2 n boja, moramo prvo obojiti aerodrome jedne zemlje - to je moguCe uEiniti na (b), naEina. Zatim trebamo obojiti aerodrome druge zemlje u medusobno razliEite boje tako da niti jedan aerodrom nije obojen istom bojom kao aerodrom prve zemlje s kojim on ima direktnu liniju. BuduCi da je jednostavnije brojati bojanja u kojima je zadano da su aerodromi obojeni istom bojom kao i njihovi "parovi" iz prve zemlje, primijenit Cemo formulu ukljuEivanja-iskljuEivanja. Broj svih bojanja aerodroma druge zemlje iznosi (b), , a broj bojanja ako nekih zadanih i aerodroma mora biti zadane boje (b - i),-i. Tih zadanih i aerodroma moiemo izabrati na naEina, pa je broj dobrih bojanja aerodroma druge zemlje jednak:

(7)

Ukupni broj bojanja aerodroma u obje zemlje tada iznosi:

8.

ZBRKA ZADATAKA

173

+f Broj dobrih bojanja aerodroma druge zemlje u sluEaju b = n jednak je broju permutacija bez fiksnih toEaka.
61. Pogledajmo prvo najjednostavniji sluEaj. Ako su oznake vrhova dodijeljene tako da su sva slova A (a onda i sva slova B ) jedno do drugog, onda je traieni produkt oEito jednak: 2-1 . (1/2)b--1 = 2a-b.

Bilo koji drugi raspored moZe se dobiti iz ovog uzastopnim zamjenama susjednih elemenata - guramo npr. slova A na njihova mjesta. Zamjena susjednih elemenata ne mijenja produkt (kad se zamjenjuju A i B faktor na strani gdje je bio A se smanjuje, a na strani gdje je bio B poveCava dva puta): Prije zamjene:

?
?

2 A A

1
B

112 B
1

? ? ?

...
Poslije zamjene:

? ?

1
B A

...

...

Niti proizvoljnim brojem zamjena susjednih oznaka neCe se promijeniti produkt. BuduCi da se svaki raspored A -ova i B -ova moie dobiti zamjenama susjednih elemenata (dokaz konstrukcijom plana zamjene), time je tvrdnja dokazana.

62. Za funkcije f koje su same sebi inverz vrijedi: ako f preslikava x u y , onda preslikava i y u x ( f : x I+ y f : y I+ x ) , pa one prirodno induciraju particiju s blokovima veliEine 1 ili 2 na svojim domenama. I obratno, svaka takva particija inducira preslikavanje koje je samo sebi inverz. Obje ove veze su injektivne - uvjerite se sami u Einjenicu d a razliEita preslikavanja induciraju razliEite particije i da razliEite particije induciraju razliEita preslikavanja.

BuduCi d a mi ielimo preslikavanja koja nemaju fiksnih toEaka, moramo gledati samo one particije koje imaju blokove veliEine 2. Zadatak nam se dakle sveo na brojanje podjela elemenata n-Elanog skupa na neuredene parove. Njihov broj je: (n-l)(n-3)(n-5)

. . . . . 1,

n neparan, nparan.

63. Iz definicije brojeva ak jasno je da je ak broj naEina izbora k predmeta, ako imamo na raspolaganju n vrsta predmeta, od svake vrste po dva komada.
Od svake vrste predmeta moZemo uzeti 0 , l ili 2 predmeta, pa je:

Lto svodi rjeLavanje zadatka na dokazivanje relacije:

Prvo rjegenje - pronalazenje bijekcije izmedu skupa svih izbora s parnim odnosno neparnim brojem predmeta. Svaki izbor moIemo prikazati uredenom n-torkom (xl,. . . ,~ n ) , gdje 4xi znaEi da od i-te vrste predmeta uzimamo xi komada, i = 1,.. . ,n . Definirajmo funkciju koja preslikava izbore s parnim brojem predmeta u izbore s neparnim brojem

174

8. ZBRKA ZADATAKA

predmeta tako, da promijeni parnost zadnjeg izabranog predmeta:

. . ,xk, 1 , 0 , . . . ,O) H (21,. . . ,xk, 2, 0,. . . , 0) ( x ~. ,


ako je (X ako je

~
k

f xi + = 1 parno, ~
xi

Xk,2,0, ...,o ) ~ ( ~ ~ , . . . , X k , 1 , 0 , . . . , 0 )

+ 2 parno.

Lako se vidi da je ova funkcija dobro definirana na skupu svih nepraznih izbora parnog broja predmeta i da je ona traiena bijekcija na skup svih izbora neparnog broja predmeta (nije joj teLko pronaCi obostrani inverz). Drugo rjes'enje - direktnim prebrojavanjem. Lijeva strana traiene jednakosti je jednaka broju izbora parnog broja elemenata. Izaberimo prvo iz kojih C e skupina u konaEni izbor uCi po jedan element (takvih treba biti paran broj), a zatim za ostale kaiimo d a li C e u izbor uCi 0 ili 2 puta. Imamo za lijevu stranu:

i sliEno za desnu:

gdje se u obje sume sumira po j , broju predmeta koji su u'31i u izbor bez svog duplikata. Dalje imamo: L-D=(2-l)n-1=1-1=0.
64.

Iz desne strane moiemo zakljuEiti d a se radi o n + l -permutacijama b-Elanog skupa.

Opis lijeve strane: neka je, odredenosti radi zadani b-Elani skup potpuno ureden, npr. Nb . Sortiramo njegove n 1-permutacije skupa prema vrijednosti najveCeg Elana koji je uLao u t u permutaciju. Neka je on i + l . Njega moiemo smjestiti na jedno od n+ 1 mjesta, a ostalih i elemenata koji eventualno ulaze u permutaciju slaiemo na preostalih n mjesta.

tP Umjesto najveCeg mogli smo istaknuti i npr. posljednji element permutacije. Takoder smo mogli promatrati i injekcije NnS1 -+ Nb .
65. Komplicirani uvjet zadatka samo kaie da na uietu trebaju stajati naizmjeniEno crvene i plave zastavice.

Mjesta na kojima C e stajati zastavice npr. crvene boje moiemo izabrati na dva naEina ako je broj zastavica crvene boje jednak broju zastavica plave boje, a na jedan naEin ako se ti brojevi razlikuju za jedan. U svim ostalim sluEajevima nije moguCe napraviti traieni razmjeztaj. Nakon odredivanja mjesta za zastavice pojedinih boja treba razmjestiti zastavice svake od boja na za t u boju predvidena mjesta. Rezultat: 2c!p!, c = p, , ! . ; { c-p, =I, inaEe.
66. Dovoljno je promatrati ostatke komponenata uredenih trojki pri dijeljenju s 3. Da bismo dobili uredenu trojku brojeva Eija je suma djeljiva s 3, suma njihovih ostataka modulo 3 mora biti djeljiva s 3.

8. ZBRKA ZADATAKA

Kako god izabrali prve dvije komponente uredene trojke, postoji toEno jedna dopustiva moguCnost za treCu komponentu. Dakle ukupno ima 32 moguCnosti za ostatke po komponentama i to su: 2) (0,2,1) (O,O, 0) , 0) (LO, 2) (2,0,1) (2,1, 0) (2,2,2). Medu 3n uzastopnih brojeva postoji: n brojeva djeljivih s 3, n brojeva koji pri dijeljenju s 3 imaju ostatak 1, n brojeva koji pri dijeljenju s 3 imaju ostatak 2. Ako se koordinate smi'u ponavljati, onda za svaki izbor trojki ostataka, trojku brojeva ' naEina. Stoga je rezultat jednak 9n3. moIemo izabrati na n 3 Ako sve koordinate moraju biti medusobno razliEite, onda nemamo problema ako izaberemo kao predloIak trojku ostataka s razliEitim koordinatama. Takvih izbora ima 6n3. InaEe sve tri koordinate moraju imati isti ostatak modulo 3, ali moraju biti medusobno razliEite, pa je broj takvih izbora jednak 3n(n - l ) ( n - 2). Ukupni broj izbora je jednak 6n3 3n(n - l ) ( n - 2) . 9+ Razmislite o poopCenjima ovog zadatka. Neka su vrlo jednostavna. Gdje vidite probleme?

67. SydeCi prema desnoj strani, trebamo brojati sve m-Elane podskupove 2n-Elanog skupa. Citamo li lijevu stranu, izgleda d a prvo biramo k elemenata n-Elanog skupa kao da su elementi polaznog skupa bili podijeljeni u dvije jednako velike grupe. I u drugom faktoru vidimo d a je ostalo jog samo m - 2k elemenata za izabrati, umjesto m - k kao d a je prvih k elemenata koje smo izabrali bilo dvostruko.
PokuSajmo zato 2n-Elani skup predoEiti kao {1,2,. . . ,n, l', 2', . . . ,n') . Iz njega biram0 m-Elani podskup na sljededi naEin: prvo izaberimo koji C e elementi doCi u skup zajedno sa svojim parom (tj, i s i'). To moIemo napraviti na ) ( ; naEina. Sad biramo : I & ) naEina, a zatirn elemente koji u izbor ulaze bez svog para - prvo brojeve na ( odredujemo hodemo li uzeti crtane elemente ili elemente bez crtica na 2m-2k naEina. % IspriEajte neku priEu o ovom rjeEIenju - Gto bi sve mogli predoEavati parovi elemenata?
68. OEito je lakGe pratiti razvoj bakterija ako promatramo odvojeno brojeve bakterija starih 0, 112, odnosno 1 sat u svakom pojedinom trenutku stvaranja novih bakterija, tj. svakih pola sata.

OznaEimo redom s nn, s n , un broj novorodenih, starih i umiruCih bakterija nakon sz,. Imamo sljedete relacije: n/2 sati. TraIimo n2,

Prva relacija se pokazuje nepotrebnom jer' veC uvrgtavanjem druge relacije u treCu dobivamo: 12,-2, nn = 12,-1

;to zajedno s poEetnim uvjetom za broj novih bakterija daje:

8. ZBRKA ZADATAKA

Dalje imamo: Broj Zivih bakterija minutu nakon n = 2n/2 sati iznosi:

tP Brojevi umirukih bakterija, koji su se pojavili u modeliranju Bivota bakterija, pokazali su se u raEunu nepotrebnim.
9+ Problem s ovim zadatkom nastaje u trenutku kad se sjetimo kako je Fibonacci definirao "svoje" brojeve. Naime na poEetku imamo jedan par zeEeva (zeca i zeEicu). ZeEevi se razmnaZaju na sljedeki naEin: nakon Bto navrBe dva mjeseca, svaki mjesec par zeEeva dobije par mladih - ponovno zeca i zeEicu. Ako uz to zeEevi ne ugibaju, onda nakon n mjeseci imamo Fn+l parova zeEeva.
To znaEi d a se, uz odgovarajuki pomak u vremenu (indeksima, promatrajuki jedinice vremena kao jedinice, a ne kao mjesece, odnosno polovine sata), zeEevi i bakterije razmnaZaju istom brzinom. To nije sasvim oEigledno, s obzirom na to da bakterije ugibaju, a zeEevi ne. ProEitajte paZljivo jog jednom obje sheme razmnozavanja, nacrtajte rodoslovna stabla za oba primjera, te objasnite zagto u oba problema kao rjeLenje dobivamo Fibonaccijeve brojeve.
69.

PokuBamo li konstruirati tablice za i = 9 , 8 , 7 , lako se vidi d a je:

U sluEaju i = 6 elementi matrice mogu biti ili n ili 0, s tim d a mora biti toEno po jedan n
u svakom retku i svakom stupcu. Stoga postoji onoliko rasporeda koliko i permutacijskih matrica reda 3, tj.: P6 = 3!. Za i = 5 vidimo d a je raspored nula po recima (stupcima) jednak 2,2,1 ili nekoj permutaciji tog niza brojeva. No u onim recima koji imaju po dvije nule stoji n na trekem mjestu, Hto se zbog zahtjeva na sume po stupcima ne moZe uskladiti s retkom u kojem je s a i o jedna nula. Dakle,

P5 = 0.
Za i = 4 raspored nula po recima (stupcima) je jednak 2,1,1 ili nekoj permutaciji tog niza. Redak i stupac koji imaju po dvije nule moraju imati zajedniEki element razliEit od nule. Taj element mma biti jednak n , a mjesto mu moZemo odrediti na 9 naEina. Nakon toga treba popuniti jog 2 x 2 tablicu elementima razlititim od nule, tako d a se saEuva uvjet da sume po recima i stupcima moraju biti jednake n . Za to je dovoljno u jedan od preostalih kvadratika staviti broj izmedu 1 i n - 1. Ostali elementi time su jednoznaEno odredeni, pa je ukupni broj naEina izbora jednak:

P4 = 9(n - 1).
Tri nule mogu biti samo tako razmje'stene, da se u svakom retku i svakom stupcu nalazi po jedna od njih. Dakle, broj rasporeda nula jednak je broju 3 x 3 permutacijskih matrica, 3! = 6 . Da bi se popunile ostale pozicije, dovoljno je na bilo koju od njih

8. ZBRKA ZADATAKA

staviti neki broj izmedu 1 i n - 1. Time su jedinstveno odredeni brojevi na svim ostalim pozicijama. Slijedi: P3 = 3!(n- 1). U sluEaju da u tablicu trebamo upisati toEno dvije nule, one ne smiju biti u istom retku niti u istom stupcu. Broj naEina njihovog smjegtavanja iznosi 9 . 412 = 18. Bez smanjenja opdenitosti moBemo pretpostaviti d a se te nule nalaze kao na slici lijevo:

NapiHimo sada neki broj x u neki redak u kojem se nalazi jedna nula i stupac u kojem se ne nalazi nula. Time su jedinstveno odredeni i brojevi koje treba upisati u jog tri polja - vidi sliku u sredini. Sada vidimo d a treba upisati jog jedan broj - upigimo broj y u neko od slobodnih kvadratida prema istom kriteriju kao i broj x . Popunimo li odmah sve kvadratide Eija je vrijednost sada jedinstveno odredena, vidimo d a trebamo popuniti jog samo jedan kvadratid. Uvjet koji trebaju zadovoljiti x i y d a bi se i t a j posljednji kvadratit mogao popuniti je x+y <n, uz x, y E {1,2,. . . ,n - 1). Izbor takvih parova brojeva ekvivalentan je izboru dvoElanih podskupova {x, x y) C Nn-i . Dakle, za fiksni poloBaj nula u tablici, broj tablica iznosi , a ukupni broj tablica s po dvije nule je jednak:

(nzl)

+f Mjesta na kojima de biti npr. x , a na kojima n - x smo izabrali tako, da uvjet za popunjavanje devetog polja ispadne Lto ljepgi.
U sluEaju d a tablica treba sadrzavati toEno jednu nulu, postupamo kao i u prethodnom sluEaju. 9 je mogubnosti za izbor mjesta na kojem de stajati nula, a popunjavanjem dobivamo tablicu:

pa na brojeve x, y , z E Nn (ili mofemo odmah redi NnV1) postoji jedan jedini uvjet: a: y z < n . Broj naEina izbora takvih ureaenih trojki (x, y, z) jednak je broju naEina izbora podskupova {x, x y, x y z) C Nn-i (elementi su napisani u uzlaznom redoslijedu). Slijedi:

+ +

+ +

4-t Zbrojimo li sve dobivene brojeve tablica, vidimo d a je broj svih tablica koje sadrBe barem jednu nulu jednak 3n(3 n2)/2. Dovoljno privlaEno za pokugaj direktnog dokazivanja?

+f Ovaj zadatak potjeEe od sljededeg tezeg zadatka: na koliko naEina troje djece moBe podijeliti n kruZaka, n jabuka i n bresaka, tako d a svako od njih dobije po n komada vota?

8 . ZBRKA ZADATAKA

70. U ovom zadatku je nuino prvo nacrtati skicu podruEja po kojem moiiemo Eetati. Ovdje je prikazana skica za 1 = 3 .

Da dodemo do zavrine toEke, moramo se provuCi kroz jedan od vertikalnih kanala.

U svaki od njih moramo uCi posjeCujuCi jednu od toEaka njegovog donjeg segmenta (segmenti na visini 2 izmedu toEkaka 10k i lOlc 2 za k E {O,l,. . . , I) ) i izadi kroz jednu

od toEaka njegovog gornjeg segmenta (na visini 10 izmeau toEaka s istim apscisama kao i donji segment). U kanal moiemo uCi kroz prvu, drugu ili treCu toEku njegovog donjeg segmenta (parametrizirajmo je s d E {O,1,2)), a izaCi kroz jednu od toEaka njegovog gornjeg segmenta koja nema apscisu manju od ulazne toEke (neka je izlazna toEka parametrizirana s g { O , l , 2 ) , g d ) . Tada se svaki od dobrih puteva raspada na tri dijela: do kanala, kroz kanal i nakon kanala. Kana1 moie biti bilo koji od 0-tog do 1 -tog, pa je broj puteva jednak:

>

71. Uvjerite se prvo d a standardno rjeienje s dodavanjem i-tog vrha (i - I) -terokutu (kojem npr. veC znamo broj triangulacija - podjela na trokute) ne daje nikakav rezultat.
Moiemo pokuiati podijeliti sve triangulacije tako da ih sortiramo prema dijagonali iz proizvoljnog fiksnog vrha (npr. vrha 1) koju koriste. No, to ne moiemo provesti jer postoje triangulacije koje sadrie viie dijagonala iz promatranog vrha, pa se ne moie primijeniti princip sume (uvjerite se da niti primjena formule ukljuEivanja-iskljuEivanja ne izgleda perspektivno). Malom modifikacijom ovog razmatranja dolazimo do sljedeCe ideje: Izaberimo jednu stranicu i -terokuta. Ona C e uvijek pripadati toEno jednom trokutu. Taj trokut Ce, u ovisnosti o svom treCem vrhu dijeliti i-terokut na jedan k-terokut i jedan (i - k 1)-terokut, k E (2,3,. .. ,n - 1) . Triangulacije novonastalih poligona moiiemo promatrati nezavisno, pa je:

uz dogovor: Sz = 1. MnoieCi dobivenu relaciju s xi+' i sumiranjem za i relaciju za obiEnu funkciju izvodnicu za niz (Si), koju oznaBmo sa S(x) :

> 3 dobivamo

8. ZBRKA ZADATAKA

179

te: RjeBenje s pozitivnim predznakom odbacimo jer bi u tom sluEaju koeficijent uz z1 u S(z) bio razliEit od nule. Ostaje rezultat: S(z) = 2 (1- 4K-G) . je

fi= a l l 2

% Nije tegko niti izraEunati koeficijente Si u ovom redu potencija. Prisjetite se da i razvijte izraz prerna binomnom teoremu. Rezultat: Si= ( 2 / ~ . ) /( i1 ) .

Brojevi oblika ovog rjesenja zovu se Catalanovi brojevi i pojavljuju se kao rjegenja mnogih kombinatornih problema. Najpoznatiji medu njima je problem zagrada: postavlja se pitanje koliko naEina moiemo postaviti zagrade u izraz a1 a2 . . . a n , tako da nakon toga zbrajamo uvijek dva po dva broja (bilo sumanda ili medurezultata).

+ + +

Uspostavite bijekciju izmedu skupa rjegenja problema triangulacije i skupa rjeBenja problema zagrada.
72.

Uvjete zadatka prvo prevodimo u operativni oblik: (a) svaka plava toEka spojena je s barem jednom zelenom toEkom (inaEe bi ostala izolirana, jer s plavom ne smije biti spojena), (b) postoji plava toEka spojena s obje zelene toEke (da bi postojao put izmedu zelenih toEaka), (c) plava toEka spojena s obje zelene toEke je jedinstvena (zbog jedinstvenosti puteva),

i to su svi zahtjevi na spojnice. Dakle, trebamo izabrati plavu toEku koja C e biti spojena s obje zelene toEke (to moiemo uEiniti na p naEina), a nakon toga za svaku od preostalih p - 1 plavih toEaka izabrati po jednu zelenu s kojom C e biti spojena (biramo izmedu dvije moguCe toeke). Stoga je broj naeina izbora skupa spojnica jednak p . 2P-1 .
4-,

Razmislite o povetanju broja zelenih toEaka.

73. Od svake od n vrsta olovaka biramo ili jednu ili dvije ili itd. ili m olovaka, pa je funkcija izvodnica:

Broj naEina izbora k olovaka jednak je koeficijentu uz tk u dobivenoj funkciji izvodnici:

% ProEitajte ovaj rezultat pomoCu formule ukljuEivanja-iskljuEivanja.

180

8. ZBRKA ZADATAKA

74. Poku'sajmo s rje'senjem koje bismo primijenili da su gosti dolazili u hotel jedan po jedan: prvoj 90-torici dali bismo po jedan kljuE, a svakom od preostalih po 90. Time bismo podijelili 990 kljuEeva.

To je i mimimalni broj kljuEeva koji moramo podijeliti da bismo zadovoljili uvjete zadatka. Da ih je podijeljeno manje, postojala bi soba s najvi'se 10 podijeljenih kljuEeva. No tada bi se moglo dogoditi da su svi vlasnici kljuEeva te sobe istovremeno u Letnji, a u hotelu se nalazi 90 gostiju od kojih nitko ne moie uCi u tu sobu. Tada se gosti koji se nalaze u hotelu moraju raspodijeliti u najviSe 89 soba, pa nije zadovoljen uvjet zadatka.
75. Brojat demo brojeve s barem Eetiri faktora iz navedenog skupa brojeva i dobiveni broj oduzeti od 1000 (razmislite za'sto je ovaj put jednostavniji). Mogudnosti za Eetiri faktora iz zadanog skupa nema puno, pa ih moiemo popisati i prebrojati brojeve manje od 1000 sa svakom pojedinom Eetvorkom faktora:

Djeljivih s

3 . 3.3 . 3 3.3.3.5 3.3.3.7 3.3.5.5 3.3.5.7 3.3.7.7 3.5.5.5 3.5.5.7 3.5.7.7 ostalih

ima

I&[
* I[
[-] 1000

= 12 brojeva
= 7 brojeva

I-[ 1000 [rn] = 2 brojeva [=I = 2 brojeva 1-[ = 1 brojeva [=I = 1 brojeva
1000 1000
1000

[m]
1000

= 5 brojeva = 4 brojeva = 3 brojeva

Pritom smo brojali: brojeve djeljive s 3 3 . 3. 3.5 3 . 3 . 3. 5 . 5 3 . 3. 3.5 . 7 dvaput dvaput triput.

Primijetimo da se divizori, koji su ponovljeni u prvom brojanju Eetiriju faktora, pri drugom brojanju (5 faktora) vise ne mogu ponavljati. Stoga brojeva s barem Eetiri faktora iz skupa {3,5,7) ima:

12+7+5+4+3+2+2+1+1-1-1-2=37-4=33,
a brojeva s najvi'se tri faktora iz zadanog skupa: 1000 - 33 = 967.
76. Neka je a n broj raspodjela. U ovisnosti o veliEini skupine u kojoj se nalazi prvo stablo u nizu sortiramo rasporede i dobivamo:

an = a,-1 a(x)

+ an-2 + an-3.
2 3,
dobivamo:

Mnoienjem s xn i sumiranjem jednakosti za n

- a2x2 - a l x - a0 = x (a(x) - a l x - ao) + x 2 (a(x) - ao) + x 3 a(x).

UvrLtavanjem poEetnih uvjeta a0 = 1, a1 = 1, a2 = 2 slijedi:

+ I ' Rastavljanjem ovog izraza na parcijalne razlomke razvijanjem svakog posebno u red moie se dobiti i rjeLenje u eksplicitnom obliku.

8. ZBRKA ZADATAKA

181

77. Prvo rjes'enje - direktno prebrojavanje. Promotrimo traiene skupove odvojeno ovisno o njihovoj veliEini. JednoElani skupovi mogu sadriavati bilo koji element. Da bi svaki element dvoElanog skupa bio veCi od njegove veliEine, u njemu se ne smije nalaziti broj 1. Sliv cno, d a bi svaki element Ic -Elanog skupa bio veCi od njegove veliEine, u njemu se ne smije nalaziti niti jedan broj manji od k . Takvih skupova ima (n,k) . BuduCi da veliEina skupa moie biti izmedu 1 i n , rezultat glasi

9+

Po kojim se indeksima k zaista sumira?

Drugo rjes'enje - rekurzija. UoEimo li d a su skupovi koji su dobri kao podskupovi skupa Nn-l dobri i kao podskupovi skupa N n , ostaje nam jog samo pitanje koji su podskupovi skupa N n dobri, a d a nisu dobri podskupovi u Nn-i . To de biti oni podskupovi koji sadrie element n . Koliko ima takvih podskupova? Prirodno je pokuiati elementu n nalijepiti dobre podskupove skupa Nn-1 . No to ne ide, jer se dodavanjem n veliEina skupa poveCala, pa ne vrijedi nuZno k ) IAl . SljedeCa moguinost je pokugati s nekim dobrim podskupovima kojima Cemo, kad im dodamo element n , sve stare elemente poveCati za 1. Time C e se obje strane nejednakosti poveCati e ona i dalje biti zadovoljena. Zbog dodavanja jedinice pokuiajmo s dobrim za 1, pa C podskupovima skupa N n - 2 . Vet smo dokazali d a od svakog dobrog podskupa skupa Nn-2 moiemo napraviti jedan dobar podskup skupa Nn-2 koji sadrii n . Obratno, ako je A dobar podskup skupa N n koji sadrgi n , onda je k ) IAl za svaki k E A , te k - 1 ) IA \ {n)l za svaki k E A \ {n) . Zato, ako od svakog elementa skupa A \ {n) oduzmemo 1, dobivamo jedan dobar podskup skupa N,-2. Sada znamo kako dobivamo dobre podskupove skupa N n . Ako njihov broj oznaEimo s a n , onda imamo an = an-1 an-2.

PoEetni uvjeti su a1 = 1, a2 = 2 . Slijedi d a je an = fn (Fibonaccijev broj).

9+ Dokaiite na joi kojem modelu Fibonaccijevih brojeva d a se oni mogu izraziti pomoCu sume binomnih koeficijenata.
78. Pokugajmo poploEavati! Moiemo krenuti od bilo kojeg mjesta, ali je prirodno poEeti od nekog od donjih kuteva. Prvu ploEicu biramo na b naEina. PloEicu do nje biramo na b - 1 naEina (jer ne smije biti jednaka prvoj ploEici), a ploEicu iznad njih na b - 2 naEina (jer ne smije biti jednaka prvim dvjema). I svaku sljedeCu ploEicu moiemo izabrati na b - 2 naEina.

Na zid treba ukupno nalijepiti n

+ (n + 1) ploEica, pa rezultat glasi:

b(b - l)(b - 2)n+(nS1)-2 = b(b - l)(b - 2)2n-1

182

8. ZBRKA ZADATAKA

Najmanji broj vrsta ploEica potrebnih za ovakvo poploEavanje je b = 3 .

%+ Uvjerite se jog d a ovaj rezultat ne ovisi o mjestu s kojeg smo zapoEeli poploEavanje.
79. Prvo rjeHenje - direktnim prebrojo.vanjem. Traieni podskupovi mogu biti veliEine 0,4,8,. . . ,n (jer 41n), pa ih ima:

raEuna promatranjem Ovaj izraz bi sada trebalo zbrojiti. Sjetimo li se da se Cj2a izraza (1 (1 - I)", tada vidimo da bi bilo korisno pokuSati s raznim Eetvrtim korijenima iz 1. I zaista, buduCi d a 41n :

(G)

(1 i dalje:

+ 1)" + (1 - l ) n + (1 + i)n + (1 - i)n = 4Sn,

= =

i)"] 4 1 - . [2n-1 \/lncos IT^ = za n djeljiv s 8 2 2n-2 + 2n/2-1

1 . [2n + 2Re(l+

Drugo rjeienje - rekurzivnom relacijom. Ovako lijep rezultat privlaEi na traienje joi kojeg rjeSenja, na primjer nekog koje odraiava kombinatornu strukturu traiene familije skupova. Osim toga, formula koja izraiava S n pomoCu Eetvrtih korijena iz jedinice kaie d a bismo rjeienje mogli pokuiati traiiti kao rjeienje linearne rekurzivne relacije s konstantnim koeficijentima koja ima karakteristiEne korijene: 2, l + i i l-i . NajkraCa takva rekurzivna relacija ima karakteristitnu jednadibu (x - 2)(x - 1- i)(x - 1 i) = z 3 - 4x2 6 s - 4 = 0 , pa glasi: S n = 4Sn-1 - 6Sn-2 4Sne3.

Istu rekurzivnu relaciju mogli smo izvesti i kombinatornim razmatranjem. I<ao n Elani skup promatrajmo npr. Nn . UoEimo "rep skupa Nn " - podskup {n - 3, n 2, n - 1,n ) , te razdijelimo sve podskupove skupa Nn veliEine djeljive sa Eetiri na one koji sadrie sve elemente repa i one koji ne sadrie neki od elemenata repa. Poskupova koji sadrLe sve elemente repa ima S n 4 . Ostale Cemo klasificirati u ovisnosti o tome koji element repa ne sadrie. Za svaki od elemenata repa, postoji Sn-l podskupova koji ga ne sadrie. Zbrojimo li doprinose za sve moguCe elemente, dobivamo 4Sn-l, ali smo pri tome neke podskupove brojali viSe puta. Korigirajmo t a j rezultat dodajuii za svaki izbor para elemenata s repa broj podskupova koji ga ne sadrie (ima ih (;)sn-2 ), oduzimajuti za svaki izbor trojke elemenata broj podskupova koji je ne sadrZe ( (;)snm3 ) i dodajuii joi zadnji mogudi broj - broj podskupova koji ne sadrie niti jedan element repa. Upravo opisana relacija glasi: Sn = Sn-4

+ 4Sn-1

- 6Sn-2

+ 4Sn-3 - Sn-4,

Eito smo i Leljeli dobiti. Lako se dobiju poEetni uvjeti S1 = S2 = S3= 1, pa se onda moie i izraEunati rezultat.

8. ZBRKA ZADATAKA

183

4-, NapiLite ovaj postupak precizno, definirajudi skupove korigtene u formuli ukljuEivanja-iskljuEivanja.

9-+ Istu

rekurziju izvedite i ratunski iz prvog izraza za Sn .

9+ Kao izazov ostaju jog dvije moguCnosti za izvod rekurzije: za svaki n iz prvog izraza za Sn , te za n djeljive s 8 iz rezultata. Izvodi su ratunski, pokugajte dobivene rekurzije dokazati kombinatorno.
4-, UoEite d a smo u svim rjegenjima relativno kasno upotrijebili uvjet da je n djeljiv s 8.
80.

Plesni parovi sastoje se od po jednog muSkarca i po jedne Bene.

Pretpostavimo d a veC znamo kojih C e k muLkaraca plesati. Niti jedan od njih ne smije otiCi po svoju Benu. Lako se vidi da tada broj mogukih plesnih parova moZemo prirodno pokugati izraEunati pomoCu formule ukljuEivanja-iskljuEivanja (od ukupnog broja izbora prvo oduzimamo broj izbora u kojima jedan mugkarac plelte sa svojom ienom a zatim dobiveni broj korigiramo). No, pogledajmo nije li ipak jednostavno izraEunati potrebne brojeve - brojeve izbora partnerica, u kojima neki muLkarci plegu sa svojim suprugama. Broj izbora partnerica, ako nekih i , 0 iznosi:

< i < k muHkaraca pleHe sa svojim suprugama

jer je za prvog "nesparenog" ostalo jog n - i iena na izbor, nakon njegovog biranja za drugog je ostalo joB n - i - 1 Zena itd. sve do Ic - i-tog mugkarca koji treba izabrati partnericu. Ako je k = i , postoji jedna moguCnost izbora. Tih i muLkaraca koji pleLu sa svojim suprugama moiemo izabrati iz grupe onih koji su odluEili plesati na naEina. Dakle sada imamo sve potrebne podatke, te moiemo primijeniti formulu ukljuEivanja-iskljueivanja. Za zadani skup muLkaraca koji su odluEili plesati rezultat glasi:

(q)

InaEe treba izabrati jog i njih iz skupine od n muLkaraca, pa se rezultat povetava puta.

) ( ;

4' Za k = n rezultat je jednak broju bijekcija bez fiksnih toEaka skupa svih n braEnih parova.
81. Svako bojanje prirodno odgovara surjekciji s n-Elanog skupa predmeta na k-Elani skup boja. Broj surjekcija je poznat i iznosi:

8. ZBRKA ZADATAKA

pa eksponencijalna funkcija izvodnica glasi:

82. Ratunsko rjes'enje. Pokusajmo izraziti zadanu sumu kao koeficijent u nekom formalnom redu: CT=li2 = (xn) f (x) . BuduCi da se za svaki n koeficijent u f (a) uz xn mora sastojati od n sumanada, moiemo pokugati f(x) prikazati kao produkt triju formalnih redova: fl (x) , f2 (x) i x (zadnji zato jer nemamo n 1, nego n sumanada).

Lako se vidi d a bi produkt fi(x) . f2(x) mogao biti: (1

+ 22x + 32x2 + . . .)

(1

+ x + x2 + . . .)

(zagto nismo pokusali s dva reda Eiji koeficijenti linearno ovise o eksponentu varijable x . npr. x i x i ?). Izraz u drugoj zagradi je poznat, a izraz u prvoj zagradi je jednak:

Dakle,

Koeficijent uz x n u ovom forrnalnorn redu (tj. trazena suma) glasi:

Kombinatorno jes'enje. Promatramo li beskonatnu piramidu koja na vrhu ima jedan kameni blok, ispod njega 2 x 2 kamena bloka, pa 3 x 3 kamena bloka itd, onda je zadana suma jednaka broju naEina da iz te piramide izdvojimo konaEnu piramidu visine n blokova i u njoj oznaEimo jedan kameni blok. Taj izbor moiemo napraviti i tako, d a prvo na nekoj visini k (u stvari dubini, gledano od vrha piramide k = 1 ) izaberemo jedan blok na k2 naEina. Zatirn izaberemo jog visinu n - k 2 0 na kojoj C e u konaEnoj piramidi stajati t a j kameni blok. Tako dobivamo da je funkcija izvodnica za ovaj problem (potencije formalne varijable x predstavljaju visinu piramide kao zbroj dubine oznaEenog bloka i visine tog bloka u odnosu na novu bazu piramide):
(l2X

+ 22x2 + 32x3+ . . .) . (1 + x2 + x3 + . . .),

8. ZBRKA ZADATAKA

185

a dalje dokaz teEe kao i ranije.


83.

Iz uvjeta zadatka se vidi d a brojevi a r moraju zadovoljavati rekurzivnu relaciju:

PoEetni uvjeti su a1 = 0 i a2 = 1. Uvrgtavanjem u rekurziju mofemo dobiti i Elan s indeksom 0: a0 = 0 (s njim je obiEno lakge raEunati). Rjegenje pripadne homogene jednadfbe je oblika: aih) = A ( - I ) ~ B .

Partikularno rjegenje trafimo medu polinomima drugog stupnja:

jer je 1 korijen karakteristicne jednadfbe. Uvrgtavanjem u polaznu rekurzivnu relaciju dobivamo C = 112, D = -112. Uvrgtavanjem poEetnih uvjeta u a r = a$') aih) dobivamo A = 0 , B = 0 , pa je konaEno:

a~ = 2 .

r2 - r

9 Nadite jedan algoritam koji ima jednak broj usporedbi kao i algoritam opisan u zadatku. NapiLite program za sortiranje prema tom algoritmu.
84. OznaEimo s Si sumu triju brojeva oko i -tog odsjeEka (tj. sumu brojeva na (i - 1) vom, i -tom i (i 1)-vom odsjeEku). 0 pojedinim sumama ne znamo nigta. 0 brojevima jedino znamo d a se svaki od njih nalazi napisan toEno u jednom odsjeEku.

Svaki od brojeva napisanih na obodu kotaEa uraEunat je u po tri sume - jednom kao lijevi, jednom kao srednji, a jednom kao desni. Zbog toga je:
36

Si = 3 . (1
i=l

+ 2 + . . . + 36) = 1998.

Kad bi svaka od ovih suma bila manja ili jednaka 55, onda bi prethodni izraz mogao biti najvige jednak 55 .36 = 1980. Kontradikcija.
85.

Nacrtajte skicu i pokugajte naCi nekoliko trafenih puteva.

Lako se vidi da, ako felimo ostvariti barem jednu getnju, n mora biti paran (za dokaz ove tvrdnje promatrajte npr. parnost izraza x y i x - y za toEke duf Eetnje).

Sada trafimo moguCe startne toEke. Napigite jednadfbu skupa toEaka koje su na udaljenosti n koraka od barem jedne toEke (xo, yo) pravca koji sadrfi (n, 0)(0, n) . Uvrstite u t u jednadfbu uvjete za toEku ( z E (-n, 0)(0, n ) ): y = z n i x E (0, 1,. . . ,n ) . Tom ili kojom drugom metodom dolazimo do zakljuEka: od toEke (-n/2,n/2) mofe se u po n koraka stiCi do svake toEke segmenta (n, 0)(0, n) , a od bilo koje druge toEke segmenta (-n, 0)(0, n) samo do toEke (n/2,n/2) .

ZakljuEujemo sa getnja mora ili poEeti od toEke P s koordinatama (-n/2, n/2) ili zavrgiti u toEki K = (n/2,n/2). Obje ove klase puteva su jednako velike, a sastoje se od puteva duljine n koji mogu imati i E (0, 1,. . . ,n/2) vertikalnih segmenata. Na puteve nije bilo drugih ograniEenja osim duljine i izbora polazne i zavrgne toEke, pa puteva u jednoj klasi ima:

i=O

186
a ukupno puteva ima:

8. ZBRKA ZADATAKA

(-1 jer smo horizontalni put - od (-7112, n/2) do (n/2,n/2) u n koraka - brojali dvaput). Moiemo zaieljeti da put nakon Lto izade iz poEetne toEke viLe ne dotiEe duSinu (-n,O)(O, n) (ne izlazi iz unutraznjosti trokuta). Tada niti u jednoj poEetnoj etapi ne smije biti vise od polovine vertikalnih segmenata, pa rezultat glasi (usporedi s problemom zagrada, rjezenje zrcaljenjem):

86. Prikaiite spomenute skupove skicom! Ako je na sveuEili6tu p profesora, onda formulom ukljuEivanja-iskljuEivanja raEunamo broj profesora koji se bave trima sportovima:

IzjednaEimo li t a j rezultat s brojem profesora koji se bave svim trima sportovima:

dobivamo broj profesora na sveuEiliztu: p = 30. 87. Pretpostavimo da nam je dan neki redoslijed studenata. Da bi sjeli, prvo prvi student treba izabrati stolac, a zatim ostalih devet studenata treba izabrati udaljenosti, svaki od prethodnog studenta (broj stolca kojeg izabire prvi student je veCi ili jednak 1, a udaljenosti su veCe ili jednake 2). Dakle, zasad imamo: (x

+ x2 + . . .) . (x2 + x3 + . . J9.

To je obiEna funkcija izvodnica za broj rasporeda ved poredanih 10 studenata razvrstanih po zadnjem zauzetom stolcu. BuduCi da nas zanimaju rasporedi studenata na r stolaca razvrstani po ukupnom broju stolaca, treba jog ukljuEiti i broj slobodnih stolaca iza zadnjeg studenta (O,1, 2,. . . ). Dobivamo:

BuduCi d a smo umjetno poslagali studente, ukupni broj rasporeda je lo! puta veCi. Stoga traiena funkcija izvodnica glasi:

9+ Ako promatramo studente koji veC sjede, lako vidimo da a, mora biti lo! puta veCi od broja uredenih particija broja r - 10 u 11 dijelova, od kojih su prvi i zadnji dio nenegativni, a ostali su veCi ili jednaki 1 (zamiLljamo d a su stolci na kojima sjede studenti podijelili prazne stolce). Usporedite rjegenje koje biste na t a j naEin dobili direktno s odgovarajutim koeficijentom u funkciji izvodnici.

8. ZBRKA ZADATAKA

88. Rac'unanje broja izvlac'enja na ternelju sastava kutije na poc'etku izvlac'enja. Svaka kuglica uzrokuje neki broj izvlaEenja, a t a j broj ovisi samo o broju koji je napisan na toj kuglici (jer i pravilo zamjene ovisi samo o tom broju). OznaEimo t a j broj za kuglicu s napisanim brojem i s bi . Tada je b1 = 1,

jer nakon Sto izvuEemo kuglicu s brojem 1 ne stavljamo u kutiju nove kuglice koje bi poslije trebali izvlatiti. Za ostale i je

bi = l + b l b 2 . . +bi-2 bi_i, jer ako imamo kuglicu s napisanim brojem i , onda moramo izvuCi nju i jog obaviti sva izvlaEenja koja C e uzrokovati novoubaEene kuglice s brojevima 1 , 2 , . . . ,i - 1. Sada se indukcijom po i vidi d a svi bi moraju biti konaEni. + I ' Moiemo i izraEunati bi . Pomnoiimo rekurziju s xi i zbrojimo dobivene relacije bizi, imamo: za i = 1 , 2 , . . . . Uz oznaku za funkciju izvodnicu za niz (bi) , b(zj :=

+ +.

iz Eega slijedi:

i konaEno bi = 2i-1 . Rjes'enje indukcijom po najveiem broju napisanom na nekoj od kuglica koje se na poEetku nalaze u posudi. Baza indukcije: ako je najveCi broj napisan na kuglicama jednak 1, a imamo npr. N kuglica, onda Cemo isprazniti kutiju nakon toEno N koraka (a to je konaEni broj izvlaEenja). Pretpostavka indukcije: ako se u kutiji nalazi konaEan broj kuglica, a najveCi napisani broj je i , onda Cemo, neovisno o broju kuglica s napisanim pojedinim brojem, nakon konaEnog broja koraka isprazniti kutiju. Korak indukcije: u kutiji se nalazi konaEan broj kuglica, najveCi napisani broj je i 1. Ako ne prije, onda kad izvuEemo sve kuglice s manjim brojevima (a to se dogad a prema pretpostavci indukcije nakon konaEno mnogo koraka), doCi Cemo do kuglice s maksimalnim brojem. Nju zamijenimo kuglicama s manjim brojevima. Time se smanjio broj kuglica s maksimalnim brojem. PonavljajuCi postupak potroeit Cemo sve kuglice s maksimalnim brojem (bilo ih je konaEno), pa C e ostati samo kuglice s manjim brojevima. No njih se, prema pretpostavci indukcije, rjegavamo u konaEno mnogo koraka.

tP U ovom dokazu se radi o dvije ugnijeidene indukcije. Naime, unutar base i unutar koraka indukcije koristi se jog indukcija po broju kuglica na kojima je napisan maksimalni broj.
89. Rac'unanje koeficajenata iz svakog pribrojnika posebno. RaEunat Cemo odvojeno koeficijente aT za r n i r > n , jer zbog faktora zZ u prvom sluEaju doprinos koeficijentu ne daju svi sumandi. IzdvajajuB iz svakog od sumanada koeficijent uz zT , dobivamo:

<

za r < n : (zbog

~ f = ,( TI:),

= ("I)

8. ZBRKA ZADATAKA

(:-;")

(nadopunimo sumu da je moiemo zbrojiti)

tP

Za sve

moie se rjelsenje zapisati u obliku u kojem je zapisano za n

<r

< 2n

Prvo zbrojiti izrat, t a t i m r a h n a t i koeficijente. Moiemo uoEiti d a su sumandi zadanog izraza Elanovi geometrijskog niza i izraz prvo zbrojiti. U ovisnosti o tome Lto se sve izluEi ispred sume dobivaju se viLe ili manje praktiEni izrazi. Jedna moguCnost je:

Koeficijent uz xT u ovom izrazu je lako izraEunati, tako da u ovom rjelsenju otpada zbrajanje binomnih koeficijenata. Kombinatorno rjes'enje. Desna funkcija izvodnica broji podskupove skupa Nzn+' Elanog skupa kojima prvi element koji ni'e u skupu nije veCi od n 1 , sortirane prema njihovoj veliEini, Prvi sumand x O ( l x$^ pritom broji podskupove koji ne sadrie element 1, drugi one koji sadrie 1 ali ne sadrie element 2 , itd., opCenito sumand s indeksom i broji podskupove koji sadrie 1 , 2 , . . . ,i , ali ne sadrie i 1.

a, je zato broj r-Elanih podskupova skupa 2n+ 1 koji ne sadrie barem jedan od elemenata 1 , 2 , . . . ,n + 1. Njega znamo izraEunati - on je jednak razlici broja svih r -Elanih podskupova i broja svih ( r - n - 1) -Elanih podskupova skupa {n 2, n 3 , . . . , 2 n 1) :

90. Ako je a n broj ljudi u promatranom repu nakon n minuta Eekanja, a k konstanta proporcionalnosti koja odreduje prirast broja ljudi u repu, onda vrijedi:

an = an-1

U zadanom redu je a0 = 1, a1 = 1+ 2 = 3 , pa slijedi: k = 2


Rekurzivna relacija za promatranu ustanovu glasi: uz poEetni uvjet a0 = 1 . Rjelsavanjem dobivamo: a30 = 330

+ kan-l

= (k

+ l)an-1.

91. TraZimo broj podjela u neuredene trojke, takvih da niti jedan braEni par ne bude zajedno. OEito je laklse izraEunati broj podjela u kojima neki braEni parovi moraju ostati

8. ZBRKA ZADATAKA

189

zajedno, pa koristimo formulu ukljuEivanja-iskljuEivanja. Zasad pretpostavimo d a je bitno tko je u kojem Eamcu. Znamo: broj podjela u trojke je (3,;n.,3), broj podjela u trojke ako je jedan (odredeni) par zajedno u prvom Eamcu: (1,!~2,3) , bro' podjela ako su dva para zajedno, prvi u prvom, a drugi u drugom Eamcu: 6 1 . 4 (1,1,3,...,3) itd' ielimo li ostaviti zajedno nekih i parova, prvo trebamo izabrati koji C e to biti parovi ( (3?) naEina), zatim pronaCi im Eamce ( (2n)i naEina), a tek onda raspodijeliti ostale na preostala mjesta u Eamcima. UoEimo jog d a je maksimalni broj parova koje moiemo ostaviti zajedno jednak broju Eamaca ( 2n ). Dakle, broj "dobrih" rasporeda u Eamce je jednak:

BuduCi d a nije bitno tko je u kojem Eamcu, dobiveni rezultat treba podijeliti brojem rasporeda trojki u Eamce ( (2n)! ).

9 Dokaiite kombinatornim argumentima d a je izraz dobiven formulom ukljutivanja-iskljuEivanja djeljiv s 23n. Pritom vam mogu koristiti primjeri za n = 1 i n = 2 .
92. Ako je u populaciji p ljudi, ukupni broj izreEenih Sala je Egoiaip/lOO = p . Cgoiai/lOO. Kad bi svaki Eovjek primio jednaku koliEinu Sala, dobio bi ih ELoiai/lOO.

BuduCi d a t a kolizina nije nuino cijeli broj, postojat C e barem jedan Eovjek koji C e nasjesti iai/lOO1 ( [xl je najmanji cijeli broj veCi ili jednak a:) gala. na barem [CEO

tP Primijetimo jog d a Einjenica d a niti jedan Eovjek ne vara Sam sebe ne utjeEe na ovaj rezultat. Naime, za proizvoljni niz ai ne moie se reCi niSta preciznije od dobivenog rezultata (za razliku od, npr., situacije u kojoj samo jedan Eovjek vara).
9 , Usporedite ovaj intuitivno zapisani dokaz s Dirichletovim principom.
93. Broj kojim popunjavamo srednji kvadratiC ne utjeEe na simetriEnost lika. Stoga je dovoljno promatrati broj naEina popunjavanja lika bez tog kvadratita. Ukupni broj popunjavanja je dvaput veti od rezultata kojeg Cemo dobiti, jer u srednjem kvadratitu moie stajati 0 ili 1 . Dogovorimo se d a svaki krak gledamo kao niz od sredigta. Lako se vidi d a dva nasuprotna kraka kao nizovi ne smiju biti jednaki (zbog osne simetrije u odnosu na pravac koji sadrZi njima okomite krakove). Dva susjedna kraka ne smiju biti jednaka ako su ostala dva kraka jednaka (zbog osne simetrije u odnosu na simetralu kuta izmedu jednakih susjeda). Rotacije i centralna simetrija ne stvaraju nove uvjete.
Broj popunjavanja pojedinog kraka bez ikakvih uvjeta iznosi 22 = 4 . Popunjavanja krakova su sljedeta:

190

8. ZBRKA ZADATAKA svaka dva kraka razliEita - takvih popunjavanja ima 4 . 3 . 2 . 1 ili dva susjedna kraka jednaka, a druga dva razliEita -takvih popunjavanja ima 4.3.2.4 (zadnji " .4" zbog izbora mjesta za jednake krakove).

Oznatite razliEite nizove slovima A, B, C, . . . i nacrtajte skice moguCih popunjavanja! Broj popunjavanja cijelog lika je jednak: 2.(4.3.2.1+4.3.2.4) ~ 2 4 0 .

4' Rezultat se lako generalizira na bilo koju drugu duljinu krakova.


94. Biramo potencijalne pribrojnike: da li je 1 u sumi, da li je 3 u sumi, d a li je 5 u sumi itd. Funkcija izvodnica za ovakve izbore je (1 x ) ( l x3) . . . . . (1 . . ... No, ova funkcija izvodnica broji i "rastave" neparnih brojeva duljine 1 ( 2 n 1 = 2n 1). Da njih iskljuEimo, trebamo od nje oduzeti x x3 x5 . . . = x/(1 - x 2 ) .

+ + + + + + + + PoEetni dio funkcije izvodnice za sve rastave je x +x3 + x4 + x5 +x6 + x7 + 22' + . . . , a nakon oduzimanja dobivamo poEetak traiene funkcije izvodnice: x4 + x6 + 2x8 + . . . .
9-t
rojnika.
95.

Koeficijent uz x6 je jednak 1 (:to oEito tako i treba biti). Usporedite ovaj zadatak s rastavom prirodnih brojeva na sumu razliEitih prib-

Trokut sa stranicama duljina manjim ili jednakim 2n sve stranice duljina manjih ili jednakih 2n - 1, ili barem jednu stranicu duljine 2 n , a ostale manje, ili barem jednu stranicu duljine 2n 1, a ostale manje.

+ 1 moie imati:

Moie li ova podjela pomoCi pri brojanju? Broj trokuta u prvoj skupini je jednak tn . Trokute druge skupine moiemo brojati tako, da ih grupiramo prema duljini najkraie stranice: ako je ona jednaka 1, treCa stranica moie biti samo duljine 2 n , ako je ona 2, treCa stranica moie biti ili 2n ili 2n - 1 . Tako se poveCava broj izbora treCe stranice sve do najkraCe stranice duljine n . Za t a j izbor najkraCe stranice treCu stranicu moiemo izabrati na n naEina. No i za najkraCu stranicu duljine n 1 broj izbora treCe stranice je n (:to se dogodilo?). Dalje poveCavanjem duljine najkraCe stranice pada broj naEina izbora treCe stranice.

SliEno moiemo izbrojati i trokute treCe skupine, pa imamo:

Dobivena diferencijska jednadiba srestivanjem prelazi u: 2n2 tn+l = t, iz Eega se rjegavanjem dobiva ( t l = 1, otito): tn = (4n - l ) ( n

+ 3n + 1, + l)n/6.

9-t MoguCe izbore dviju stranica zgodno je promatrati kao cjelobrojne toEke u ravnini koje zadovoljavaju (npr. za sumu u drugom redu rekurzije):
oEigledno svojstvo: x, y > 0 , duljina stranice je manja ili jednaka 2n : x, y ,< 2 n , nejednakost trokuta: x y > 2 n , uvjet protiv ponavljanja sukladnih trokuta: x y.

<

8. ZBRKA ZADATAKA

Nacrtajte skicu - presjek svih spomenutih poluravnina za ovaj sluEaj i analogno za treCu skupinu trokuta (treCi red rekurzije). Prodite jog jednom izvod rekurzije pokazujuCi na slici Sto se dogada.

9 SliEno se uvjeti cijelog zadatka mogu prikazati u trodimenzionalnom prostoru pomoCu poluprostora umjesto poluravnina. Uvjeti su:
nejednakosti trokuta: x y z , y x z , z x y , ograniEenje na veliEinu trokuta: 0 < x, y, z 2 n , uvjet protiv ponavljanja trokuta: x y r . Ovi uvjeti nisu medusobni nezavisni - koje moiemo trivijalno izbaciti? Nadite nekakve kockice i napravite modele ovih nejednadibi za male n . Objasnite zagto razlika tn+l - t n kvadratno ovisi o n . Kakve veze to ima sa sliEnosti likova? Objasnite na ovom ili na prethodnom dvodimenzionalnom primjeru: koje su sliEnosti i razlike izmedu povrgina sliEnih likova i broja cjelobrojnih toEaka u sliEnim likovima? Brojanje cjelobrojnih toEaka moie biti vrlo tefak problem!

< +

< + < + < < <

'3-+ Sto moiete napraviti pomoCu funkcija izvodnica?


e biti rubovi nove, male Eokolade. U jednom smje96. Trebamo izabrati "brazde" koje C ru imamo m 1, a u drugom n 1 moguCnosti izbora 'edne brazde. Broj izbora dviju brazdi je onda u jednom smjeru (n:l) , a u drugom (m2 l ) . Ukupni broj naEina izbora . je ("f') (";I)

+f Ovo je diskretna varijanta price "slovo iie, a sira sve niie"


97. PoEnimo poploEavati traku, npr. slijeva nadesno. U ovisnosti o tome koje se ploEice nalaze u prvom stupcu, mogu nastupiti sljedede situacije:

jedna crvena uspravno dvije plave jedna iznad druge jedna crvena horizontalno, jedna plava u prvom stupcu ispod nje jedna crvena horizontalno, jedna plava u prvom stupcu iznad nje dvije crvene jedna iznad druge Ukupni broj poplotavanja jednak je dakle sumi brojeva poploEavanja u ovih pet sluEajeva. Trake s nepoznatim brojem poploEavanja kraCe su od polazne, pa moiemo dobiti rekurziju koja C e nas dovesti do cilja. OznaEimo s an broj poploEavanja zadane pruge duljine n , a s bn broj poploEavanja "krnje" pruge duljine n . Tada imarno: Krnju prugu moiemo poEeti poploEavati ili jednom plavom ili jednom crvenom ploEicom:
bn = an-1

+ bn-l.
2
2 , a drugu za

Pomnoiimo li svaku od ovih relacija s x n , te prosumiramo prvu za n n 2 1, dobivamo:

pri Eemu su a(z) i b(x) su funkcije izvodnice za nizove ( a n ) , odnosno (bn). PoEetni uvjeti koji se lako izraEunaju su a1 = 2 i bl = 1, iz Eega slijedi a0 = 1 i bo = 0 (Sto

192

8. ZBRKA ZADATAKA

je i logiEno!). Uvrstimo poEetne uvjete u jednadibe, te metodom eliminacije izraEunamo rje'ienje:

4-t Mogli smo prvo eliminirati Elanove niza (b,) iz sistema jednadibi, a onda traiiti funkciju izdvodnicu. Rekurzivna relacija koja sadrii samo elemente niza (an) je:

i=O

Moiete li ovu relaciju interpretirati direktno?

4-t Da li vam oblik rjesenja predstavlja iznenabenje? Ta to je oblik funkcije izvodnice za rje'ienja rekurzije reda 3, a mi smo imali (vidi prethodnu napomenu gdje je eksplicitno napisana rekurzija za niz (a,)) rekurziju beskonaEne proglosti. Gdje smo si zakomplicirali Zivot?
Jedna od mogudnosti za dobivanje rekurzije konaEne pro'ilosti je upravo iz posljednje relacije:

iz Eega oduzimanjem slijedi:

'ito smo prema obliku nazivnika funkcije izvodnice i oEekivali. Interpretirajte ovu relaciju kombinatorno!
P , Nakon rijegenog zadatka iz prethodne napomene vidimo da smo umjesto prve relacije koja povezuje (an) i (bn) mogli pisati takoaer jednostavnu relaciju:

Obrazloiite je!
98. Brojimo one raspodjele koji niti prvo, niti drugo,. . . , niti d-to dijete ne ostavljaju bez voda. Rasporede u kojima neko dijete ostaje bez voda lako je izraEunati, pa koristimo formulu ukljueivanja-iskljueivanja oduzimajudi prvo one raspodjele u kojima barem jedno dijete ostaje bez voda.

Ukupan broj podjela voda je :

a broj podjela u kojima barem i djece ostaje bez voda je :

8. ZBRKA ZADATAKA

193

Moramo jog izabrati i koja C e djeca ostati bez voCa, pa je broj dobrih podjela (formula

UI) :

99. 0 kojih C e se 2n predmeta raditi nije bitno, ali C e biti zgodno uzeti neke predmete koje prirodno moiemo spariti, d a bismo kasnije mogli interpretirati i n . Neka to bude n pari ( 2n komada) cipela.

; ) ( ' je broj naEina izbora dvaju cipela iz te hrpe. Na desnoj strani imamo 2(;) broj naEina izbora dvaju lijevih, odnosno dvaju desnih cipela te n2 - broj naEina izbora jedne lijeve i jedne desne cipele.
100. ObiEna funkcija izvodnica glasi: a(x)
=

(~+x2+x4+x6+~~~)~(1+x+z2+x3+~..)
\

crvene

bijele

plave kuglice 1 1 -.1- x2 (1- x)2

Dakle,

Ovu sumu moZete izraEunati sluieti se raznim instant-formulama. Mi Cemo iznos sume pomnoiene s dva izraEunati direktno. Zamislite (k l)(k 2) kuglica poslaganih u (k 1) x (k 2) pravokutnik. Citajmo sada sumu od k = r prema k = 0 , dva po dva Elana. Nacrtajte skicu! Dobivamo:

...+

2 2,. 3

+ 2,

ako je r paran ako je r neparan

(k

/ Iz sume kojom su i z r a i e d , vidi se da vrijedi :

+ I)', +1 .

ako je T paran ako je r neparan

194 pa bi i tom izrazu bilo zgodno nadi kombinatornu interpretaciju.

8. ZBRKA ZADATAKA

9+ Rijegite ovaj zadatak i direktno, sortirajudi rje'senja u ovisnosti o broju crvenih kuglica. Na t a j naEin dobiva se a r u obliku sume. Nju se prvo moie izraEunati, a onda uvrstiti u izraz za funkciju izvodnicu ili se moie direktno iz nje, mnoienjem s o r , dobiti funkcija izvodnica. Isprobajte oba naEina!
101. Za obiEne olovke trebamo odrediti samo broj 'LH''i broj "B" olovaka, a za kemijske i konkretne olovke. Za fiksni broj "H" olovaka h i fiksni broj "B" olovaka b , broj izbora n olovaka iz zadanog skupa olovaka je . Stoga je ukupni broj izbora jednak:

(LFb)

tP

Primijetimo da se za b

<N

- 2n

-h

sumiraju nule.

OEito je da se ovaj izraz ne moie vige sredivati. No, isti rasporedi mogu se prebrojati i malo drukEije -kao broj rjegenja diofantske jednadibe xl+x2+. . ' + x N - ~ , + ~ = n uz uvjete 0 x i , 2 2 n , 0 2 3 , . . . ,XN-2,+2 1 , :to se rjegava formulom UI i dobiveni izraz treba jog malo srediti.

<

<

<

<

102. Uvjeti zadatka kaiu d a ako funkcija f proizvoljni element x E Nn preslika u y E N n , ( y # x ) , onda f element y preslika u x . Nacrtajte nekoliko primjera preslikavanja N, -+ Nn koja su sama sebi inverz i nekoliko primjera preslikavanja koja uz to jog i nemaju fiksnih toEaka. Ovdje je osobito zgodno domenu i kodomenu prikazati istim skupom (istom slikom).
Svaka funkcija f sa svojstvima kao u zadatku odgovara po jednoj podjeli skupa N, u parove, tako d a je sa svakim elementom sparen onaj element koji je njegova slika (a i praslika) u odnosu na funkciju f . Dakle, nakon ;to je zadana funkcija, imamo odmah particiju skupa Nn u parove. Da dobijemo traienu uredenu particiju tog skupa trebamo iz svakog para izabrati po jedan element i uvrstiti ga u prvi skup particije ( A ) . Ostali elementi ulaze u drugi skup particije. Prvi 8 a n particije moie se izabrati na 2,12 naEina, a to je onda i ukupni broj naEina izbora particije.

tP Broj particija skupa N, dvostruko manji.

s odgovarajudim svojstvom (prirodniji zadatak!) je

103. Lako je odluEiti se za naEin dokazivanje danog identiteta: potrebno je uzeti r terokut kojem se nikoje tri dijagonale ne sijeku u jednoj toEki, oduzeti (otpiliti) jedan vrh sa susjednim stranama i prouEiti u kakvom su odnosu brojevi podrucja u polaznom r-terokutu i dobivenom ( r - 1) -terokutu. I u dobivenom ( r - 1) -terokutu se nikoje tri dijagonale ne sijeku i istoj toEki, pa njegov broj podrueja moiemo oznaEiti s a,--1 i operacija "smanjivanja" poligona ima smisla.
Pribrojnik r - 2 je dogao od prebrojavanja podruzja koje se nalaze uz otpiljeni vrh ( r - 3 dijagonale i dvije stranice novog r -terokuta Eine r - 2 nova podrueja - nacrtajte skicu!). Do prvog sumanda moie se do& na dva naEina: Prvi naEzn - prema predloienoj formuli. Treba dokazati d a dijagonale koje idu iz podrucja manjeg poligona. Taj broj je jednak broju otpiljenog vrha prolaze kroz (Ti1) trokuta koji imaju vrhove u vrhovima ( r - 1)- terolcuta .

8. ZBRKA ZADATAKA

Nati bijekciju izmedu skupa traienih podruEja i skupa trokuta!

Drugi nacin - direktno prebrojavanje. Numerirajmo vrhovek-1) -terokuta tako da novi vrh bude spojen s vrhovima broj 1i r - 1. Tada dijagonala 1 k ima ( 1 - l ) ( r - 1 - 1 ) presjeka sa starim dijagonalam~ odnosno stranicom 1( r - 1 ) . Stoga je broj novonastalih podruiija zbog dijagonale 1 k jednak ( k - l ) ( r - 1 - k) , a ukupni broj novonastalih podruEja:
r-2

x ( k - l)(r- 1
k=2

- k).

Ovaj izraz mogao bi se zbrojiti sredivanjem, te korigtenjem formula za sumu prvih nekoliko prirodnih brojeva, odnosno njihovih kvadrata, ali je jednostavnije pogledati njegovo kombinatorno znaEenje. Naime, on je konvolucijskog tipa, pa ideja dolazi sama . . .Radi se o broju izbora trozlanog podskupa (T - 1) -Elanog skupa, sortiranih po srednjem elementu. Poznato je da svih troElanih podskupova ima ( ' i l ) .

Racunanje ar , zajednitko za oba rjes'enja. Zbrajanjem jednakosti:

dobivamo:

104. To je isto kao da prebrojavamo permutacije 6-8anog skupa bez fiksnih toiiaka
- svakom pridruBimo bilo koji posao, samo ne onaj koji o n ne voli. Za postupak vidi

poglavlje o formuli UI. Ima ih 265.

105. Umjesto rasporedivanja kugala, moZemo rjegavati jednadzbu:

196
uz uvjete : 3)rl , 31r2, 3 l r 3 , 3 lr4 . F'unkcija izvodnica glasi: f(~) =

8. ZBRKA ZADATAKA

...)2.(x+x2+x4+x5+x7+...)2

(1+x3+x6+...)2.

.[(1 + x + x 2 + x 3 + x 4 + . . ) - ( 1 + x 3 + x 6 + . .

Tada je: (xT)f (1.1

x~(I+ x ) ~ - x2 2~~ 24 (1 - x3)4 (1 - x3)4


1

'

(xTW2)

- x3)4

I
2j

+ 2 ( ~ ' - ~ ) (1 - x3)4 + (x'-~) (1 1 x3)4 ako 3 / r - 1 ako 31r

2(((,134)13) (--L)('-~)/~, ako 3 ( r ((T1$13)(-1)(T-4)13, (T13)(-l)'23,

-2

2 ( 2 i k ) , ako je r = 3k, k E No ako j e r = 3 k + l , k E No

(2ik),
(3:k),

ako je r = 3 k + 2 , k E No.

106. U sluEaju d a je k 2 j zadatak ne postavlja nikakva ograniEenja, pa je ukupni broj puteva jednak . Ostaje nam jog sluEaj k < j .

(2)

Svaki put moie doCi na najvige jednu toEku spomenutog segmenta. Ta toEka moie imatiapscisu j - k , j - k + l , j - k + 2 , . . . , j + k iordinatu y = n - a : . DotoLke ( x , n - x ) moie se od ishodigta doCi na naEina; na isti broj naEina moie se doCi od (x, n - x) do (n, n) i t a dva dijela puta su nezavisna.

(2)

Stoga rezultat glasi:

tP U

sluEaju k

dobili bismo ono Sto smo odmah zakljuEili: suma je jednaka

(") .
107. Traiimo li maksimalni broj dana bez kige, svaki drugi dan treba znaEiti 2.,4., itd. dan, svaki treCi 3.,6., itd, jer su npr. moguCnosti da ved prvi dan pada kiSa ili d a se veC od drugog dana broji svaki treCi dan oEito loSije. Postoje i druge realizacije najrjecteg padanja kige (npr. brojeCi odostraga, a za neke n i druge), ali sve su one ekvivalentne u smislu broja kignih dana.
Dakle, zadatak se svodi na problem raEunanja koliko je prirodnih brojeva manjih ili jednakih n koji nisu djeljivi ni s 2 ni s 3 ni s 4 ni s 5. Djeljivost s Eetiri je oEito redundantna, pa moiemo promatrati samo dane Eiji redni brojevi nisu djeljivi niti s jednim od brojeva 2,3,5. Time se u ovom zadatku pojavljuje najtipiEnija situacija koja je rjeSava

8. ZBRKA ZADATAKA

197

formulom UI i rezultat glasi:

Specijalno, za n = 40 rezultat glasi 10.

tP Traieni brojevi ne smiju biti djeljivi s prva tri prosta broja. SljedeCi prosti broj je 7, pa buduCi d a je vet 7 . 7 = 49 > 40, vidimo da smo trebali prebrojati sve proste brojeve manje ili jednake 40 zajedno s jedinicom!
108. Dokazujemo d a je broj dvoElanih podskupova n-Elanog skupa jednak:
Zaista, postoji jedan dvoElani podskup s minimalnim elementom n - 1, dva dvoElana podskupa s minimalnim elementom n - 2 , itd., n dvoElanih podskupova s minimalnim elementom 1. Bududi da su n - 1,n - 2 , . . . ,1 jedine moguCnosti za izbor minimalnog elementa, tvrdnja je dokazana.

109. Nakon malo razmigljanja moie se vidjeti da nema smisla zbrajati Fibonaccijeve brojeve u eksplicitnom obliku i onda kontradikcijom dokazivati d a rezultat ne moie biti Fibonaccijev broj.
Pokugajmo onda zbojiti Fibonaccijeve brojeve koristeCi rekurzivnu relaciju koja mebu njima vrijedi. RaspisujuCi primjere za male rn 3 vidimo da su odgovarajute sume malo vebe od Fm+k+l , ali ne dovoljno da bi dostigle F m f k + 2 . Prvi dio ove slutnje lako se dokaie:

>

Drugi dio slutnje,

Fk

+. . . + Fk+,-i + Fk+,

< Fk+m+z.

moZe se dokazati npr. primjenom matematiEke indukcije i time je tvrdnja provjerena.

+f Spretniji rjeSavaEi, kao i oni ltoji se sjeCaju formule za zbroj prvih n Fibonaccijevih brojeva (i/ili njenog izvoda), mogli bi postupiti na sljedeCi naEin:
Fk

+ Fk+l+
= = =

+ Fk+2 + . . . + Fktm-i + Fk+m = Abracadabra! Fk+l + 2Fk+2 + Fk+3 +. . . + Fk+m-1 + Fk+m - Fk+1 Fk+2 + 2Fk+3 + Fk+4 +. .. + Fk+m-1 + Fk+m - Fk+1
Fk+2

Fk+2

+ ... + Fk+m-I+

Fk+m

Znamo d a Fibonaccijevi brojevi strogo rastu, a broj koji prethodi broju FkSm+2 je za Fk+, manji od njega. Ta razlika je sigurno veCa od Fk+l , pa se izraEunata suma za sve k,m E N , m 3 uvijek nalazi izmedu dva Fibonaccijeva broja.

>

110.

U zadatku je prepriEana sljedeCa rekurzivna relacija:

8. ZBRKA ZADATAKA

To je najjednostavnije rijeiiti pomoku funkcija izvodnica. Pomnoiimo jednadibu s xn i zbrojimo sve jednadibe za n 2 1. Time dobivamo (uz z0 = 0 ) : 1 x . z(x). - c(2) - co = z(x) - 20 1-x 1- 22 z(x) . - = c(2) - co 1-2 1-x z(x) = -. (.(XI - co). 1 - 22

Za zadani niz cn = 1,n E N , co = 0 dobivamo: x c(x) = 1-x'

ito se moie provjeriti i direktno.

111. Odakle poEeti? Pogledamo li sliku, vidimo d a imamo najvi'se izbora za prvo i zadnje slovo, manje za drugo i predzadnje, itd., a za srednje slovo "0"imam0 samo jednu moguknost izbora. Za lakie snalaienje u slici moiete svaku vrstu slova obojati jednom bojom.
Moramo upotrijebiti srednji "0".Dolazak i odlazak do tog slova su meitusobno nezavisni, a zbog simetrije slike ima ih jednak broj. Stoga raEunamo samo broj Eitanja od sredi'snjeg "0" do "?" (ne brojimo ulaske u "0"jer je prirodnije Eitati unaprijed nego unazad). Da bismo dobili traieni rezultat, trebamo dobiveni broj kvadrirati. Ako iz srednjeg "0" izademo prema gore, dalje se moiemo kretati samo u smjeru gore ili desno. Pomak u svakom koraku biramo proizvoljno (9 komada), pa imamo 2' moguknosti izbora. Ako iz srednjeg ''0" izaitemo desno, u sljedekem koraku moiemo bira:i tri "B"-a: broj zavriavanja retenice iz gornjeg, odnosno donjeg "B"-a je, analogno kao u prethodnom sluEaju jednak 28 . Za srednji "B" moSemo opet birati tri razna "In-a itd, sve dok ne doitemo do "?" iz kojeg moZemo nastaviti put samo na jedan naEin. Dakle, broj Eitanja reEenice od "0"nadalje je jednak: 2 . 28 2.27 2.2O + 1 = 2 10 +2'-1. 2'+
gore dolje "B"

- +

+...+

gore dolje "I"

gore dolje "7"

Rezultat je onda: (2''

+ 2' - I ) ~ .

Lako se moie pronaki i rekurzivna relacija za broj Eitanja polovice tablice rje'senja sortiramo po broju koraka, n (u na'sem sluEaju je n = 10 ). Naime, kad dodamo jog jedno slovo, svako staro Eitanje generira dva nova Eitanja, osim horizontalnog Eitanja koje generira tri nova Eitanja. Stoga je:

tP

Pn = (Pn-1 - 1) ' 2 + 3.
Zbog poEetnog uvjeta PI = 2 , koji se lako provjeri, dobivamo:
Pn

= 2"

+ 2n-1

- 1.

Programi
Zadaci vezani uz prebrojavanje privlaEe na rjeSavanje pomoCu raEunala. Zaista, ako treba prebrojati elemente nekog konkretnog skupa, mofemo napisati program koji to radi umjesto nas. Premda je za pisanje programa takoder potrebna vjegtina, ono ne moBe zamijeniti pravo dokazivanje. Idealno je pokuSati oboje. S druge strane, u nizu zadataka treba dokazati neku zadanu ili nasluCenu tvrdnju koja je npr. istinita za svaki prirodni broj n . BuduCi da prirodnih brojeva ima beskonaEno mnogo, raEunalo mote provjeriti tvrdnju za neke n , ali ne i dokazati je. U ovakvim sluEajevima je primjena raEunala zgodna kao motivirajubi faktor ("vidi, pa to stvarno vrijedi za male n!"). U zadacima tipa "za koje n vrijedi. . . " ili "dokaBi ili opovrgni. . . " moBemo pustiti raEunalo da umjesto nas provjerava za koje vrijednosti parametra n tvrdnja vrijedi. IspiSe li se rezultat u obliku nekakve zgodne tablice, to moBe biti polazigte za razne generalizacije koje potom treba dokazati. Na kraju spomenimo jog jednu korist od pisanja programa. GenerirajuCi neku strukturu, od pitanja "postoji li" i "kako generirati", preko pisanja programa, te pitanja "zaSto to radi", moZemo Eesto igrajuCi se doCi do npr. dokaza egzistencije. Rad na raEunalu s jedne strane, te klasiEno raEunanje i dokazivanje tvrdnji s druge, dovoljno su razliEite aktivnosti da mogu poslutiti i kao odmor jedna od druge. Uz to one se medusobno nadopunjuju i obogaCuju.

U ovom poglavlju pogledat Cemo neke algoritme koji slute za generiranje osnovnih kombinatornih struktura (podskupovi, uredene n -torke s i bez ponavljanja elemenat a). Za pojedini zadatak koji se Beli rijegiti pomoCu raEunala mogu se tada primijeniti predloieni algoritmi, ali je ipak zgodno razmisliti mote li se za konkretnu situaciju algoritam pojednostaviti ili mu se skratiti vrijeme izvodenja. Npr. mote se dogoditi da trebamo provjeriti neku tvrdnju o podskupovima parne veliEine. Tada moBemo koristiti program koji generira sve podskupove, zatim za svaki pojedini podskup provjeriti je li paran ili ne, te, ako jest, provjeriti zadanu tvrd-

nju. No vrijedi malo razmisliti i napisati program za generiranje podskupova parne veliEine. S tim u vezi potrebno je istaknuti jedno upozorenje. Brojevi elemenata u raznim kombinatornim konfiguracijama rastu vrlo brzo - najEeSCe eksponencijalno, odnosno s faktorijelom. To odmah povlaEi vrlo brzi rast vremena izvodenja s poveCavanjem vrijednosti paramet(a)ra. U praksi se toj neugodnoj pojavi doskaEe tako, da se prvo nekim jednostavnim dokazom eliminiraju neke konfiguracije. Zatim se napige program koji generira preostale konfiguracije (ili eventualno negto malo viSe, ako bi generiranje samih konfiguracija koje dolaze u obzir bilo preslokeno), te se na njima provjerava tvrdnja ili se u tom podskupu broje pokeljne konfiguracije. S jednim jednostavnim primjerom takve strategije susreCu se djeca veC u osnovnoj Skoli. cim se upoznaju prosti brojevi, zadatak ispisivanja poEetnog dijela niza svih prostih brojeva rjeSava se tako, da se prvo napige broj 2, a dalje se radi samo s neparnim brojevima. Druga je moguCnost podjela skupa svih dobrih konfiguracija na klase koje se daju parametrizirati tako, da je generiranje svake od njih brzo. Probleme tipa "postoji li. . . " na jako velikim skupovima mokemo raEunalom ispitivati na jog jedan naEin. Na sluEajan naEin izabiremo konfiguracije iz zadanog skupa i na njima provjeravamo tvrdnju. Pustimo li raCunalo da dovoljno dugo radi, moHda C e pronaCi neki element (ili vise njih) koji zadovoljava zadanu tvrdnju. Nakalost, neCe redi jesu li to jedini elementi, odnosno kako izgleda skup rjeSenja i zagto je on upravo takav. No, ispisana lista elemenata koji zadovoljavaju tvrdnju moke biti polazigte za razne zakljuEke o skupu rjegenja. S druge strane, moHe se dogoditi da raEunalo nakon duljeg vremena ne ispiSe niti jedan objekt za koji vrijedi tvrdnja koju ispitujemo. Tada, ako zaista mislimo da ne postoji niti jedan objekt s traZenim svojstvom, to moramo dokazati. Skice algoritama zapisivat Cemo tekstom, a konaEne razrade u programskom jeziku Pascal - zbog jasnoCe izragavanja i relativno Sirokog kruga ljudi koji se njime sluHe ili su ga barem u stanju Eitati. Zapis detaljnih razrada u obliku algoritama izostavljen je zbog toga jer bi u tom sluEaju trebalo definirati upotrijebljene algoritamske strukture. 0 tome u ovom trenutku ne postoji dostupna literatura (standardizacija), a opisivanje na ovom mjestu bi samo produljilo ovaj kratki pregled. Pretpostavke za razumijevanje ovog poglavlja su osnovno poznavanje kombinatornih struktura o kojima se radi, kao i nekoliko uspjelih pokuSaja samostalnog programiranja ne sasvim trivijalnih programa. Pri izboru programskog jezika u kojem Cete raditi zgodno je prvo provjeriti ima li taj programski jezik mogudnost rekurzivnog pozivanja potprograma. To je bitno - pogotovo ako kelite pisati jednostavne programe za koje vam nije bitno koliko traje izvodenje. Zgodno je pokugati i s nekim od neproceduralnih jezika. Zbog preglednosti, svi su programi bez dodatnih kontrola ulaznih podataka. Iz istog razloga su i ispisi najjednostavniji moguCi. Takoder, pretpostavlja se da se logiEki izrazi raEunaju tako, da se pri A A B ( A i B ) u sluEaju da je A

la6, B ne raEuna, te isto tako u sluEaju da je A istina u izrazu A V B ( A ili B ) . U Turbo Pascalu to je opcija Boolean evalutation - short circuit. Na kraju poglavlja nalazi se jednostavna biblioteka - za Turbo Pascal (ver. 5.5) pripremljen u n i t s imenom komb u kojem se nalaze procedure i tipovi potrebni za rjegavanje problema iz ovog poglavlja.

Podskupovi ~ E l a n o g skupa
Popisivanje elemenata kombinatornih struktura treba provoditi sistematski. Pri ruEnom popisivanju sistematsko popisivanje sluZi tome da se ne ispusti neki element. Osim toga mo6e slu6iti i tome da se na temelju sistema primijenjenog pri ruCnom popisivanju napige algoritam. Sistema, jasno, ima za svaki problem vige. U cijelom ovom poglavlju bit C e navedeni po jedan do dva algoritma po problemu. To ne bi trebalo obeshrabriti Eitatelje da ustraju na svojim idejama, koriste ih u ruEnom popisivanju i napigu programe koji ih realiziraju. PotraZimo sada algoritam koji generira sve podskupove n-Elanog skupa. I<ao polazni n-Elani skup uzmimo skup {I, . . . ,n ) . Svaki od elemenata tog skupa moZe uCi u podskup ili ne. Tako bi nekom tko traZi sve podskupove 5-Elanog skupa moglo pasti na pamet da napige sljedeCi program:
program podskupovi~5~clanog~skupa; var il,i2,i3,i4,i5 : boolean;

C il ce oznacavati ulazi li 1 u skup, 3 C i2 ... 2 . . . , 3 C itd. 1


begin for il := false to true do for i2 := false to true do for i3 := false to true do for i4 := false to true do for i5 := false to true do begin write ('elementi podskupa su if il then write (1:3); if i2 then write (2:3); if i3 then write (3:3); if i4 then write (4:3); if i5 then write (5:3); writeln end

: I ) ;

Premda bi ovaj program uspjeSno obavio svoj zadatak, on je daleko od dobrog programa za nalaEenje svih podskupova nekog skupa. Cak niti neka oEigledno moguCa sitna poboljSanja (npr. uvodenje niza indeksa umjesto pet varijabli, Sto bi pojednostavnilo ispis) bez zahvata u logiku programa ne bi donijela neSto bitno bolje. Naime, ovaj program traEi podskupove skupa toEno odredene veliEine. Uz to, pokugajte zamisliti kako bi izgledao program koji bi traEio sve podskupove 20-8anog skupa (osim naSih estetskih primjedbi na taj program, zbog previSe ugnijeEdenih petlji svoju bi primjedbu izrekla i veCina prevodilaca). PokuBat Cemo izmisliti algoritam kojem Cemo kao ulazni parametar moCi dati veliEinu skupa kojem traiimo podskupove. ProuEimo prvo jedan sistem koji koristimo pri popisivanju podskupova. Neka je zadana proizvoljna veliEina skupa Eije podskupove traiimo - oznaEimo je s n . PopiSimo prvo sve podskupove koji ne sadrze broj 1. Kad to zavrsimo, ostaje nam popisati sve podskupove koji sadrEe 1. Unutar skupine podskupova koji sadrEe 1 popiBimo prvo one podskupove koji ne sadrze broj 2, a zatim one koji sadrZe 2. SliEno napravimo i unutar skupine poskupova koji ne sadr6e 1. Time smo dobili Eetiri skupine podskupova.

Nakon Bto smo podijelili podskupove u skupine u ovisnosti o tome sadrEe li broj 2 ili ne, nastavimo unutar svake od tih skupina dijeliti podskupove na one koji sadrBe 3 i ne sadrEe 3, zatim 4, 5 itd. do n . Na primjer za n = 4 imamo:

Lako se vidi da se ovaj postupak moHe izraziti rekurzivnim algoritmom: biraj podskup koji sadrHi ranije izabmne elemente, te proizvoljne od preostalih elemenata: ako jog ima preostalih elemenata onda biraj podskup koji sadrZi izabrane elemente, te proizvoljne od preostalih elemenata bez prvog biraj podskup koji sadrHi izabmne elemente s prikljuEenim prvom elementom od preostalih, te proizvoljne od preostalih bez prvog inaEe (za sve elemente je odluEeno da li su u skupu ili ne) ispigi izabmne elemente Krede se od praznog izabranog podskupa i skupa preostalih elemenata jednakog cijelom skupu. Skup preostalih elemenata je uvijek oblika {i,i 1,.. . ,n ) za neki i koji se pri svakom ulasku u dubinu (rekurzivnom pozivu potprograma) povedava za 1. Zato se t a j broj ne mora svaki put traHiti medu preostalima, nego se moHe prenositi kao parametar.

program svi-podskupovi-n-skupa; uses komb; var n


: elementi;

procedure b i r a j ( izabrani : skup; na-redu, n : elementi ) ; begin i f na-redu > n then pisi-skup (izabrani'n) e l s e begin b i r a j (izabrani , na-redu+l , n) ; b i r a j (izabrani+ [na-redul , na-redu+l end end; { b i r a j 1 begin writeln; write ('Molim broj izmedju 1 i ',max,' > ' ) ; readln (n); biraj (Cl.l,n> end.

, n) ;

Umjesto ispisivanja svih podskupova, u program moHemo ugraditi npr. ispis svih podskupova koji zadovoljavaju neko svojstvo ili samo brojanje takvih podskupova. TehniEki, to moHemo napraviti ili na mjestu potprograma p i s i skup ili

prije njegovog poziva. Takvo rjegenje bi bilo dosta nepregledno, pa bismo mogli zaieljeti novi algoritam za generiranje svih podskupova zadanog skupa, tako da glavni program izgleda ovako:
begin writeln; write ('Molim broj izmedju 1 i ',max,' > '1; readln (n); prvi-skup (n,skup,kraj); { logicka varijabla kraj je ovdje true 1 { jer uvijek postoji barem jedan podskup 1 while not kraj do begin
{ {

uvdje ugraditi provjeru s ispisom dobrih podskupova ) ili s brojanjem 1

sljedeci-skup (n,skup,kraj) { varijabla kraj kaze da li je 1 { generirani skup u redu ili je ) { prethodni skup vec bio zadnji ) end

C ispisati zakljucak 1
end.

Primjer - listu svih podskupova EetveroElanog skupa -ranije smo promatrali po skupinama, u ovisnosti o tome da li je prvi element ukljuCen ili ne, zatim da li je drugi element ukljuEen ili ne itd. Promatrajmo sada te poskupove ne po gruparna, kao Sto je to bilo tarno predloieno, nego redom. Moiemo vidjeti da je prvi podskup prazan, te da se sljedbenik proizvoljnog podskupa moBe dobiti na sljedeCi naEin: nadi sljedbenika zadanog skupa: ako zadnji element nije izabran, onda ga ukljuEimo u skup inaEe nadimo prvi element koji nije izabran traieCi od zadnjeg elementa prema prvom; ako takav element postoji onda ga ukljuEimo u skup, a iz skupa izbacimo sve elemente veCe od njega, inaEe je naden zadnji skup Vidimo da se do kraja doSlo ako nije naden niti jedan element koji nije izabran (imamo "cijeli" skup). U svakom drugom sluEaju postoji joS podskupova koje treba pregledati.
.procedure sljedeci-skup (var s : skup; n : elti;

var kraj : boolean); var i : e l t i ; begin i := n; { broj elemenata u polaznom skupu 1 while i i n s do i := j, 1 ; i f i > O then begin s :=s+[i]-[i+l..n]; kraj := f a l s e end else kraj := true end; { sljedeci-skup

Zadaci.

1 . Napigite program prema ovom algoritmu i usporedite vremena izvodenja. Analizirajte rezultate za razne veliEine skupa.
ProuEimo drugi algoritam. UoEite pravilo kojim se izbacuju elementi. Koji element C e trebati izbaciti u k -tom koraku? Koliko puta C e element i biti izbaEen/ubaEen? MoEete li na temelju ovih razmatranja ubrzati program? MoEete li nekom drugom organizacijom podataka ubrzati program?
2.

NapiSite program koji generira sve podskupove zadanog skupa parne veliEiUputa: sjetite se kako smo brojali sve podskupove parne veliEine.

ne.

3. Napigite program koji generira podskupove toEno odredene veliEine (veliCinu skupa uCitati) .

9+ Uputa za analogon prvom programu: pratite veliEinu skupa koji generirate i zaustavite se kad dostignete ieljenu veliEinu. Razmislite i o programu nalik na drugi program za generiranje svih podskupova.
+P Moiete, ako s poEetka skupa nije izabrano dovoljno elemenata da bi se s ostalima napravio skup zadane veliEine, odustati od daljnjeg t r s e n j a podskupa s tim poEetkom.
Moiete traiiti i samo podskupove veliEine manje od polovine veliEine polaznog skupa, a ostale dobiti komplementiranjem skupova prve skupine.

4. Napigite program koji generira sve podskupove zadanog skupa tako da za svaka dva uzastopna skupa A i B vrijedi [AABl = 1. PoEnite s praznim skupom.

9+ Ispravni redoslijed podskupova dobit Cete ako prvo promatrate podskupove nekog malog skupa (veliEine n E {1,2,3)), te uoEite Zito se dogada pri prijelazu na sljedeCi
n.

+f' Prikaiemo li ovako generirani niz skupova u obliku karakteristiznih funkcija, dobivamo tzv. Grayev kod.

9-t RijeLite ovaj zadatak rekurzivnim potprogramom i potprogramom koji pronalazi sljedeCi podskup. 9-t Cini li vam se da je pisanje novih potprograma za pronalaienje sljedeCeg podskupa iiivljavanje u trenutku kad veC imamo jedan koji obavlja t u istu funkciju, evo protuargumenta za ovaj primjer: pretpostavimo da za generirane podkupove trebamo neLto izraEunati ili provjeriti, Lto ovisi o svakom pojedinom elementu. Tada, nakon Lto dobijemo novi podskup, moramo samo raEunati razliku u odnosu na prethodni podskup, Sto moie biti znatno jednostavnije i brie, nego raEunati sve ispoEetka. Jasno, za to treba iz potprograma prenijeti element koji se promijenio.
5. Napisite potprogram koji generira podskupove zadanog skupa leksikografskim urebajem.

Da je niz nizova znakova ureden lelcsikogmfski, znaEi da je niz a ispred niza b ako i samo ako je prvi znak slijeva koji se nalazi u a i razliEit je od znaka na odgovarajuCoj poziciji niza b po abecedi ispred tog znaka u b . Ako je neki od nizova prekratak da bismo ih mogli usporediti na ovaj naEin, nadopunimo ga prazninom za koju smatramo da se nalazi ispred svakog drugog znaka. Leksikografski uredaj susreCemo u rjeEnicima i telefonskim imepicima. Primjer: MIS je iza MINOTAURUSa, a MARTIN ispred MARTINOVICa. Urediti podskupove leksikografski znaEi napisati njihove elemente uzlaznim redoslijedom kao konaEne nizove i usporedivati ih na opisani naEin. Primjer: skup {1,3,5) je ispred skupa {1,5).
6. NapiEite potprogram koji odabire slucajni podskup zadanog skupa, tako da za svaki element na sluEajni naEin odredi da li je u skupu ili nije.

7. Razmislite kako .biste odabrali sluEajni podskup zadanog skupa toEno odredene veliEine.
+ I ' Vjerojatnost pojavljivanja svakog od podskupova ieljene veliEine mora biti jednaka. U praksi to znaEi d a se, Hto se program dulje vrti, brojevi pojavljivanja pojedinih skupova sve viSe izjednaEavaju (omjeri brojeva pojavljivanja su sve bliii jedinici). Ovaj zadatak nije jednostavan i lako se izmisle algoritmi koji neke podskupove generiraju EeLCe nego druge. Provjerite da li se to dogada i vaSem programu. Vodite u programu evidenciju o pojavljivanju skupova i ispiLite je nakon npr. svakih stotinjak iteracija.

Citatelji upoznati s osnovama teorije vjerojatnosti mogu pokuLati izraEunati i vjerojatnosti pojavljivanja skupova.

8. PomoCu nekog od algoritama za generiranje podskupova n-Elanog skupa napigite algoritam za generiranje uredenih particija zadanog broja. PomoCu nekog od algoritama za generiranje podskupova odrebene veliEine generirajte uredene particije brojeva s ograniEenim brojem dijelova.

% Uputa: sjetite se izvoda za broj uredenih particija broja.

Uredene mtorke s elementima iz zadanog skupa

Promatramo li podskupove zadanog skupa kroz njihove karakteristicne funkcije, vidimo da smo upravo na dva naEina rijegili problem ispisivanja svih uredenih n -torki s elementima iz skupa {O,1) . Problem rada s n -torkama sastavljenih od elemenata nekog veCeg skupa moie se rijegiti sliEnim postupcima. Pretpostavimo da trafimo n-torke s elementima iz k-Elanog skupa N k. U algoritmu koji bi bio nalik prvom algoritmu za traBenje svih podskupova poziv potprograma bi trebalo ponoviti k puta - za svaki element koji moie doCi na mjesto koje se u tom trenutku popunjava po jednom. Osim toga, vige ne mofemo spremati izabrane elemente u skup, nego moramo koristiti niz u koji Cemo u svaki element upisivati odgovarajuki element n-torke. Neka su poz sve moguCe pozicije u n-torci, e l t i svi moguCi elementi, te n i z = array [poz] of e l t i . Ako potprogram p i s i n i z ispisuje niz, onda je jedan potprogram za izbor svih uredenih n-torki elemenata iz N k:
procedure biraj-k-torku ( izabrani : n i z ; na-redu, n : poz; k : e l t i ) ; var i : e l t i ; begin i f na-redu > n then p i s i - n i z (izabrani,n) e l s e f o r i := 1 t o k do begin izabrani [na-redu] := i ; biraj (izabrani, na-redu+l, n) end end; { biraj-k-torku 1

I ako felimo napisati algoritam nalik drugom algoritmu, ponovno trebamo prvo na isti naEin promijeniti strukturu podataka. Pri traienju sljedeCe n-torke trebali bismo prvo pokugati poveCati za jedan zadnju koordinatu. Ako to nije mogute, tada treba poEeti traiiti odostraga prvi element koji se moie povebati, a nakon toga sve elemente koji stoje iza njega postaviti na minimalnu vrijednost.
Zadaci. 1 . Napigite do kraja programe prema oba algoritma.
2. Multiskup moiemo zadati kao niz kratnosti njegovih elemenata. Kako biste generirali sve njegove pod(multi)skupove? Napigite programe bez i s ograniEenjem na veliEinu podskupa, analogne prvom i drugom programu.

3. NapiSite program koji uCitava prirodni broj n i broji od 1 do n u oktalnom sustavu (sustavu s bazom 8). Napis'ite sliEni program koji broji u heksadecimalnom brojevnom sustavu (baza je 16, a znamenke su 0,1,2, . . . ,9, A, B, . . . ,F ). MoZete li napisati i opCeniti program koji bi uEitavao broj manji ili jednak veliEini skupa { O , l , . . . ,9) U { A , B, . . . , 2 ) i brojao u sustavu s bazom jednakom uEitanom broju.
4. Cantorov slcup dobiva se na sljedeCi naEin: uzme se segment [O, 1 1 i iz njega se izbaci interval (113,213) (srednju treCinu). Nakon toga iz svakog od dva preostala segmenta ponovno izbacimo srednju treCinu (to su (1/9,2/9) i (719,819) ). Postupak nasta~rimo na Eetiri preostala segmenta, zatim na osam preostalih itd, beskonaEno.

cesto se govori da interval po kojem pada kiSa pokrivamo tako da pokrijemo prvo srednju njegovu treCinu, zatim pokrijemo srednje treCine svake od preostalih treCina itd. Zanimljiv je i opis Cantorovog skupa, tj. skupa svih toCaka koje ostaju "mokre" . NapiSite program koji prvo nacrta jednu duZinu koja predstavlja polazni interval, a zatim pokriva sredine prema opisanom pravilu. Kako C e izgledati krovovi izaberite sami - to mogu biti horizontalne linije, trokuti iznad odgovarajuCih segmenata ili sliCno. Pokrivajte sredine onoliko dugo koliko vam omogudava rezolucija ekrana na kojem radite, tj. dok se krovovi koji crtate jog vide.

9+ Ako radite s rnedusobno sliEnim trokutima, usporedite dobivenu sliku s elementirna skupa koji se izbacuju pri prvom algoritmu koji pronalazi sljededi podskup.
5. Procijenite: ima li za zadane n i k medu svim k -torkama skupa N, vis'e onih koje imaju paran ili viSe onih koje imaju neparan broj jedinica?

~aiiSite program koji usporeduje ta dva broja za male n i k (osim "tko je veB", moZda je zgodno promatrati njihovu razliku ili omjer). UoEite pravilnost, formulirajte teorem i dokaBite ga.

Permutacije s elementima iz zadanog skupa


NapiSemo li sve permutacije n -Elanog skupa leksikografski uredene dolazimo do sljedeCeg zapaganja:

Sve permutacije dobivaju se tako da se na prvom mjestu izmjenjuju elementi 1,2,. . . , n , te se za svaki od prvih elemenata trebaju generirati jog sve permutacije preostalih elemenata. Dakle, za npr. n = 4 , na prvo mjesto prvo moHe doCi broj 1, a iza njega bilo koja permutacija skupa {2,3,4), zatim na prvo mjesto moBe doCi 2, a iza njega bilo koja permutacija skupa {1,3,4), zatim 3 na prvom mjestu a iza njega neka permutacija skupa {1,2,4), te na kraju 4 na prvom mjestu, a iza njega neka permutacija skupa {1,2,3). Dok traiiimo sve permutacije zadanog skupa koje poEinju elementom 1, trebamo dakle naCi sve permutacije skupa {2,3,4) . Njih traBimo analognim postupkom: prvo sve one koje poEinju elementom 2, zatim 3, i na kraju one koje poEinju elementom 4. Da dobijemo sve permutacije skupa {2,3,4) koje poEinju elementom 2, treba iza njega izredati sve permutacije skupa {3,4). Tako u svakom koraku imamo sve vise fiksiranih elemenata na

poEetku permutacije, a sve manji je skup kojeg jog treba permutirati. Kad tako dodemo do praznog skupa, imamo cijelu permutaciju. Algoritam dakle glasi: permutiraj skup prwstalih elemenata sa zadanim poZetkom permutaczje: ako je skup preostalih elernenata neprazan onda za svaki element x iz skupa preostalih elemenata: permutiraj prwstalz skup elemenata bez x , uz zadani poEetak koji je jednak starom poZetku s nalijepljenim x -om zdesna inaEe (permutacija je gotova i jednaka poeetku permutacije) ispiSi je Radit Cemo ponovno na skupu N, . Tako se poEinje sa skupom elemenata koje treba permutirati jednakim { 1 , 2 , . . . ,n ) i praznim poEetkom permutacije. Permutacije Cemo spremati u niz, a neuvrstene elemente Cemo drzati u skupu. Iz tehniEkih razloga se u realizaciji potprograma pojavljuju jos neki parametri koji se u algoritmu ne vide.
program sve-n-permutacije; uses komb; var n : elti; p : niz; procedure permutiraj
(

ostali : skup; n, spremi : elti; perm : niz ) ;

var i : elti; begin if ostali = [I the? pisi-niz (perm,n) else for i := 1 to n do if i in ostali then begin perm[spremil := i; permutiraj (ostali- [ i ] ,n,spremi+l,perm> end end; { permutiraj 1 begin writeln; write ('Molim broj izmedju 1 i

' 'max,' > '1;

readln (n); permutiraj (Cl. .nl ,n,l,p) end.

Zadaci.
1. Kako bi glasio uvjet za zaustavljanje (ispis) u proceduri permutiraj koji bi u sebi imao varijablu spremi umjesto ostali =

n?

2.

Modificirajte potprogram tako da nalazi sve k-permutacije n-Elanog sku-

Pa*
3. Prebrojite za razne n pomoCu raEunala, zatim generalizirajte i na kraju dokaiite svoju slutnju: koliko postoji permutacija n-Elanog skupa kojima elementi prvo rastu, a onda padaju? Rastu, padaju, pa jog jednom rastu, padaju?
4. Sjetite se dokaza rekurzivne relacije za broj permutacija bez fiksnih toEaka: D, = (n - l)(D,-l D , - z ) . Iskoristite tu ideju za pisanje programa koji nalazi sve permutacije zadanog skupa bez fiksnih toEaka.

Kao i u sluEaju skupova, moiemo zaHeljeti napisati algoritam koji C e iz proizvoljne permutacije nalaziti sljededu, tako da nam prolaienje kroz sve permutacije nalikuje na brojanje. Uzmimo kao primjer permutaciju 136425 i pokuSajmo odrediti njenog sljedbenika. PromatrajuCi niz leksikografski uredenih permutacija s poEetka ovog poglavlja vidi se da se permutacije, kao i razni brojeanici, "vrte" tako, da se najbrBe vrte elementi na desnom kraju. ToEnije, za svaki proizvoljni poEetak permutacije preostali elementi se u nizu uzastopnih permutacija pojavljuju u svih svojim permutacijama. Dakle, strategija za nalaienje sljedeCe permutacije bi mogla biti sljedeka: promatramo kraj permutacije, Sto je mogude kradi a da postupak bude provediv, i odredimo mu sljedeCu permutaciju (dok poeetak ostaje fiksan). Jasno je da taj kraj mora imati barem dva elementa. U primjeru moHemo uzeti kraj 2 5 i zamijeniti ga njegovom sljedeCom (jedinom moguCom) permutacijom 5 2. Tako dobivamo: 136452 U sljededem koraku iz kraja 5 2 ne moHemo napraviti niti jednu leksikografski "vedu" permutaciju, pa uzimamo kraj 4 5 2. Da bismo dobili sljedeCu permutaciju, moramo promijeniti element na prvom mjestu (inaEe bismo mogli povedati kraCu permutaciju s kraja). Prvi vedi element je 5, pa uzimamo minimalnu permutaciju koja poEinje tim elementom : 5 2 4. Lijepljenjem na fiksni

poEetak dobivamo permutaciju kojoj lako izraEunamo sljedbenika, pa su sljedeCe dvije permutacije 136524 i 136542. Sada je pomoCna permutacija kojoj treba naCi sljedbenika 3 6 5 4 2 , a njen sljedbenik je 4 2 3 5 6 . Dobivamo: 142356 Dokagite da se ovim postupkom dobivaju u n! koraka sve permutacije (jedna moguCa metoda je indukcijom po n ) . Sada mo2emo malo drugaEije opisati izabranu strategiju. KreCemo od rastuC e permutacije ( 1 2 3 . . . n) , a prelazak na sljedeCu permutaciju glasi: nadi sljedbenika zadane permutacije: uoEi "padajuCi rep" na kraju p e m t a c i j e - neka je prvi element tog repa na poziciji i 1 ako je i = 0 onda smo doSli do zadnje permutacije inaEe u padajuCem nizu naCi prvi element veCi od elementa ai na poziciji i - neka je to element a j na mjestu j staviti a j na mjesto i staviti a,, . . . ,aj+l, aj-1,. . . , ai kao minimalnu permutaciju na kraj

Jedna moguCa varijanta ovog algoritma (u svrhu olakSavanja programiranja) glasi:


'

nadi sljedbenika zadane perrnutacije: nadi poziciju gdje prestaje padajuCi rep - neka je to i okretati rep (tako da postane uzlazan) i pritom tragiti element veCi od ai - neka je to a j na poziciji j zamijeniti ai s a j pastaviti okretati rep do kraja!

Zadaci.
1. Da li mogete predvidjeti gdje poEinje padajuCi rep permutacije?
2.

NapiSite potprogram koji C e nalaziti sljedeCu Ic-permutaciju.

3. Permutacija skupa {1,2, . . . ,n) je parna ako se paran broj elemenata i nalazi iza pozicije i , za svaki i E N, . Ostale permutacije su neparne. Npr. permutacija 54132 je parna, a 54123 neparna.

Koliko ima parnih permutacija n-Elanog skupa? Izbrojite pomoCu racunala parne permutacije za nekoliko malih n , formulirajte tvrdnju i dokaHite je. Razmislite o generiranju parnih permutacija. MoHete u literaturi potraziti i njihova svojstva!
4. NapiSite potprogram koji generira sluEajnu permutaciju na sljedeti naEin: prvo izabere prvi element ureaene n-torke, zatim od preostalih drugi, pa od preostalih treCi itd.

9 Uputa: krenite od proizvoljne urectene n-torke, izaberite jedan element i zamijenite ga s prvim elementom. Tada je biranje L'jednogod preostalih elemenata" u stvari biranje jednog od elemenata s pozicija 2 , 3 , . . . ,n . Zamijenite njega s drugim elementom i nastavite postupak.
Generirajte zatim i k -permutacije.

RaEunanje Elanova niza pomoeu rekurzivnih relacija


Problem koji se Eesto javlja je sljedeti: ne moZemo direktno prebrojati neke objekte, nego napigemo rekurzivnu relaciju za njihov broj. Rekurzivnu relaciju ne moramo moCi rijeSiti. To se moHe dogoditi ako ima nekonstantne koeficijente. Jedna mogutnost za dolazak do rjegenja u zatvorenom obliku je pomoCu rekurzivne relacije generirati rjegenja za prvih nekoliko vrijednosti paramet(a)ra i iz toga pokugati naslutiti rjesenje. Nakon ostaje jog samo dokaz tog rjegenja, Sto je jednostavnije nego rjegenje izvesti. Napigimo funkcijski potprogram koji raEuna vrijednost n -tog Fibonaccijevog broja. Kad gledamo rekurzivnu relaciju za Fibonaccijeve brojeve f n = fn-1 fn-2 i poEetne uvjete fo = f l = 1 , prva ideja je deklarirati niz i u njega spremati Fibonaccijeve brojeve onako kako su indeksirani u formuli. Prva mana ovog pristupa je u tome, da ne znamo unaprijed koji Cemo Elan niza trebati izraEunati i koliko taj niz treba biti velik. Osim toga, u nizu imamo spremljene vrijednosti koje nam vise ne trebaju, jer se novi Fibonaccijev broj uvijek raEuna s dva u tom trenutku najsvjeZija Fibonaccijeva broja. Tu primjedbu moZemo shvatiti ili kao estetsku primjedbu ili kao primjedbu zbog rasipanja memorije. Ideja ovog algoritma je u svakom koraku imati spremljena dva zadnja Fibonaccijeva broja. Iz njih se izraEuna novi, te se ponovno pripreme dva zadnja Fibonaccijeva broja u odgovarajute varijable za sljedetu iteraciju. Zbog jednostavnosti algoritma piSemo odmah potprogram. Pretpostavljamo da je ulazni parametar (indeks Fibonaccijevog broja kojeg treba izraEunati) veCi ili jednak nuli.

function fibonacci (nmax:integer) : longint; var f-n-1, f-n-2, : longint; f-nevi n : integer; begin if (nmax = 0 ) or (nmax = 1) then fibonacci := 1 else begin 0 ) f-n-2 := 1; Cff-n-1 := 1; f-l) for n := 2 to nmax do begin f-novi := f-n-1 + f-n-2; f-n-2 := f-n-1; f-n-1 := f-novi end ; fibonacci := f-novi end end; I fibonacci

Zadaci.

1. NapiSite rekurzivni potprogram koji raEuna Fibonaccijeve brojeve. IzraEunajte broj zbrajanja za raEunanje n-tog Fibonaccijevog broja tim potprogramom i predloienim iterativnim programom.
2. Zadajte jednu rekurzivnu relaciju reda k i napi'iite potprogram koji raEuna vrijednosti jednog niza odredenog njome. Za spremanje starih vrijednosti koristiti k -Elani niz umjesto dvaju varijabli.

3. Preuredite sve ove potprograme tako, da ih moiete koristiti za razne rekurzije bez promjene:

neka rekurzivna relacija bude zadana funkcijskim potprogramom koji se poziva u potprogramu za raEunanje vrijednosti niza potprogram treba raditi za razne redove rekurzivnih relacija - red prenijeti u potprogram kao parametar i poEetne uvjete definirajte funkcijskim potprogramom.

Biblioteka

unit komb ; interface const max = 20; type elti elti0 skup niz
= l..max;
= O..max; = set of elti;

= array [elti] of elti;

( s : skup; medja : elti ) ; procedure pisi-skup procedure pisi-skup-c ( s : skup; medja : elti ) ;

procedure pisi-niz

p : niz; br-elta : elti ) ;

implementation uses dos; procedure pisi-skup ( s : skup; medja : elti ) ; var i : elti; begin write('--> '1; for i := 1 to medja do if i in s then write (i:3) else write ( ' ' :3); writeln end; { pisi-skup 3 procedure pisi-skup-c ( s : skup; medja : elti ) ; var i : elti; begin write('--> '1; for i := 1 to medja do if i in s then write (i:3); writeln end; { pisi-skup 3 p kao permutacija); procedure pisi-niz ( p : niz { br-elta : elti ) ;

var i : elti; begin write ('-> '1; for i := 1 to br-elta do write (pCil:3) ; writeln end; { pisi-niz 3 end.

ZakljuEni zadaci i napomene


Upoznali smo se s osnovnim alogritmima za generiranje osnovnih kombinatornih struktura. Njihovo poznavanje i razumijevanje dovoljno je da se generiraju mnogi od objekata brojanih u ovoj zbirci. Od bitnih kombinatornih struktura ostale su jog dvije, pa neka to budu prva dva zadatka.

Zadaci.
1 . NapiSite program koji generira sve particije uEitanog broja (neuredene particije).
2.

NapiSite program koji uEitava broj n i generira sve particije skupa N, .

Formirajte svoju biblioteku s korisnim potprogramima. Ako budete na vidljivom mjestu drfali popis potprograma (s parametrima) koji se u njoj nalaze, sigurno Cete te potprograme i koristiti (iskustvo pokazuje da se, ako ne postoji pregledni popis potprograma, biblioteke ne koriste). KoriStenje gotovih biblioteka bitno ubrzava rjeSavanje zadataka pomoCu raEunala. Ako budete koristili raEunalo pri rjeSavanju kombinatornih problema, sigurno C e vam se pojaviti i mnogi potprogrami koje mi ovdje nemamo: na primjer racunanje binomnih koeficijenata, faktorijela (oprez s velikim vrijednostima!), itd. Dobro dodu i potprogrami za raEunanje s razlomcima i s polinomima, te za rjeSavanje sustava linearnih jednadf bi.

Zadatak.

1. Usporedujte vremena izvodenja raznih programa koji obavljaju istu funkciju.


2. Prebrojite koliko se puta izvodi koja naredba u vaSem programu u ovisnosti o vrijednostima ulaznih parametara. VjeStina brojanja steEena u ovoj zbirci trebala bi za to biti dovoljna.

3. Za rekurzivne potprograme izraEunajte koliko puta potprogram poziva sam sebe (kolika je dubina posljednjeg potprograma). Ako znamo da svaki novi potprogram svoje parametre i varijable stavlja u slofaj (stack), broj rekurzivnih poziva govori o optereCenju memorije.

Na kraju spomenimo jog i komercijalne programske pakete namijenjene matematiEarima. Na primjer Mathematica je pametni kalkulator Eije su funkcije prilagodene potrebama matematiEara. Postoje verzije za PC-kompatibilna raEunala (potreban je koprocesor i najmanje 4MB radne memorije), Macintosh i mnoga velika raEunala. Mathematica podrf ava raEunanje s konkretnim vrijednostima i sa simboliEkim izrazima, zatim rjegavanja raznih jednadfbi, sumiranja, deriviranja, integriranja, 2D i 3D crtanja, razne funkcije korisne u kombinatorici i jog gtogta (unatoE tome Sto je vrlo jednostavna za korigtenje, priruEnik ima skoro 1000 strana!). Time se omogudava rjegavanje tehniEkih problema, provjera slutnji i prikaz rezultata. Postavlja se pitanje z d t o mi trebamo pisati programe, kad za to postoje gotovi pametni paketi? Kao prvo, u gotovom paketu sigurno neCe biti zastupljene sve funkcije koje nam mogu zatrebati. U tom sluEaju morat Cemo ih programirati sami, bilo u njemu ili u nekom drugom programskom jeziku. Osim toga, steEeno iskustvo u programiranju, a pogotovo osjeCaj za vrijeme izvodenja potrebno za rjegavanje nekog problema, su bitni za razumijevanje granica gotovih programskih paketa. Posljednji razlog mofda izgleda Eudnim, ali spomenimo ga ipak: treba poEeti vjeZbati s jednostavnim algoritmima, jer C e doCi vrijeme kad C e trebati poboljsati postojeke pakete ili koncipirati nove. A netko mora i to napraviti. . .

Posljednji zadatak
Prodite joS jednom svim zadacima i zadajte ih jog jednom sami, sa svojim brojevima. Pokugajte ih zadati s opCim brojevima, naznaEujuCi samo relacije medu njima. Pogledajte Sto se dogada kad zadacima ispustite pojedine uvjete, dodajte zadacima svoje, nove uvjete! Osim toga, pokugajte naCi nove interpretacije za zadatke i formule iz ove zbirke!

Definicije pojmova
blok Element particije. Ponekad se koristi i u opCenitijem znaEenju: podskup. cjelobrojna toEka ToEka u koordinatnom sustavu s cjelobrojnim koordinatama fiksne toEke permutacije Promatramo li permutacije kao bijekcije s nekog skupa na samog sebe, fiksne toEke su one toEke koje se preslikaju same u sebe. graf U ovoj knjizi graf se nikada ne pojavljuje u frazi "graf funkcije". Nama grafovi predstavljaju dijagrame sastavljene od toEaka povezanih crtama ili strelicama. Pritom nije bitno kako su veze nacrtane nego samo koje sve toEke i na koji naEin ulaze u spojeve. Prema ovoj ideji postoji i preciznija definicija i ozbiljna teorija ( t e o ~ j grafova). a karakteristizna funkcija podskupa Ako je A B , onda je karakteristicna funkcija skupa A u odnosu na B funkcija X A : B + {0,1) definirana pravilom x A ( x ) := 1 ako i samo ako je x E A . Cesto se koristi bijekcija izmedu svih karakteristienih funkcija na nekom skupu i njegovih podskupova. kombinacija Pojam kombinacija ovdje se koristi u znaEenju koje mu intuitivno pridjeljujemo u obienom Zivotu. Kombinacija tako obuhvaCa izbor elemenata promatranog skupa uz neki redoslijed. mod Operator mod daje ostatak pri dijeljenju navedenim brojem. Primjer: 14 mod 4 = 2 . Oznaka mod pojavljuje se i pri isticanju da dva broja imaju iste ostatke pri dijeljenju navedenim brojem (da su kongruentni modulo taj broj). Primjer: 14 = 2 (mod 4). multiskup Skupom smatramo svaku istaknutu skupinu razliEitih objekata. Multiskup dozvoljava i ponavljanje elemenata. Kratnost elementa nekog multiskupa je broj njegovih ponavljanja u tom multiskupu. Primjer: {1,1,1,4, A,*, !, !, !, !) . Element 1 ima u ovom multiskupu kratnost 3, elementi * i 4 po 1,a ! 4.

220

10. DEFINICIJE POJMOVA

najvede cijelo NajveCe cijelo nekog realnog broja je najveCi cijeli broj manji ili jednak tom broju. Oznaka: [XI. Bitno svojstvo: [x + k] = [XI + k , za cijeli broj Ic . VaIni primjeri: [xlk] - za pozitivni broj x broj brojeva manjih ili jednakih x djeljivih s k . [logkx] - najveCa potencija broja k manja ili jednaka niz Niz na skupu A (ili s elementima iz A ) je svaka funkcija iz N u A . Iz estetskih razloga elementi niza a se zapisuju s a, umjesto s a ( n ) , a, zovemo opCim Elanom niza, a niz oznaEavamo jog i s (a,). BuduCi da elemente skupa N prirodno promatramo sloBene po veliEini, nizove Eesto zapisujemo navodeCi vrijednosti u toEki 1, toEki 2 itd.: alla2,a3,.. .. KonaEni niz duljine n je funkcija iz N, u A (potpuno odgovara uredenim n-torkama, ali malo drugaEije izgleda). opdi Elan niza Vidi niz. padajudi faktorijeli PadajuCi faktorijeli su za n E R i k E N brojevi (n)k := n(n - l ) ( n - 2) . . . . . (n - Ic + 1 ) . Za k = 0 definira se (n)o := 1. Posebni sluEaj: (n), := n! . Primjer: uredenih Ic-torki s elementima iz N, ima (n)k. parcijalni razlomci, rastav na parcijalne razlomke. Postoji teorem koji kaBe da se svaka racionalna funkcija p(x)/q(x) (p(x) i q(x) su polinomi) moIe prikazati kao suma razlomaka a ( ~ ) / b ( x,)gdje ~ polinom b ( ~ dijeli ) ~ q(x) , a polinom a ima stupanj manji nego polinom b. Za oblik rastava odluEujuCi je rastav polinoma q na ireducibilne faktore (faktore koji se vise ne mogu dalje rastavljati - ako radimo s realnim koeficijentima to su polinomi prvog ili drugog stupnja). Ako se pri rastavu polinoma q na faktore niti jedan faktor ne pojavljuje s potencijom veCom od 1, onda svakom faktoru odgovara jedan sumand opisanog oblika (parcijalni razlomak). Ako se neki faktor b pojavljuje s potencijom k , onda se zbog njega u rastavu na parcijalne razlomke pojavljuju razlomci s naeivnicima b, b2, b3,. . . , bk . Primjer: 2x - 1 B Cx+D - A (x - 1)2(x2 1) x - 1 (x - 1 ) 2 x2 1 Nepoznati koeficijenti A, B, C, D dobivaju se mnogenjem cijelog izraza sa zajedniEkim nazivnikom i izjednaEavanje!n koeficijenata uz odgovarajuke potencije od x . Umjesto izjednacavanja potencija, mogu se u jednakost uvrstiti i 4'vrijednosti varijable x i na taj naEin dobiti sustav jednad8l;i u A, B , C, D (za ovu metodu potrebno je lukavo izabrati vrijednosti x-eva tako da se 30manje treba raEunati: 0 , 1, -1 te nultoEke polinoma u nazivniku su idealne vrijednosti)

+-. +

10. DEFINICIJE POJMOVA

particija broja, uredena particija broja Uredena particija broja n u k dijelova je svaka uredena k -torka pozitivnih brojeva (xl , x2, . . . , xk) Eija je suma jednaka n : xl x2 . . . xk = n .

+ + +

Particija broja je multiskup s odgovarajudim svojstvom. BuduCi da je svejedno kojim redoslijedom navodimo elemente multiskupa, obiEno ih navodimo u padajuCem redoslijedu. . Primjer: 7 = 3 3 1, 7 = 3 1 3 , 7 = 2 2 3 su tri razlicite uredene particije broja 7. 7 = 3 3 1, 7 = 3 2 2 , 7 = 3 2 1 1 su tri razliEite particije broja 7.

+ +

++ + +

+ + + +

+ + +

permutacija Permutacijama nazivamo bijekcije sa skupa na samog sebe. BuduCi da radimo uvijek s konaEnim skupovima, njihove elemente mozemo uvijek poredati (reCi koji je element prvi, koji drugi itd.). U tom kontekstu permutacija znaEi mijeSanje - na prvo mjesto umjesto prvog elementa stavljamo njegovu sliku, na drugo sliku drugog elementa itd. Tako za skup veliEine n dobivamo uredenu n-torku njegovih elemenata. k-permutacija je, za k n , uredena k-torka razliEitih elemenata polaznog skupa. Prema toj definiciji n-permutacija je permutacija u smislu prethodne definicije. Ponekad trebamo i 0-permutaciju koja ne sadrzi niti jedan element. Svaki skup ima toEno jednu 0-permutaciju.

<

rastuCa funkcija Ako su A, B C R , onda je funkcija f : A -+ B rastuCa ako je f (xl) 2 f (x2) Eim je x1 > 2 2 . Funkcija je strogo rastuCa ako vrijedi i f( 4 > f( ~ 2 ) . Bitno svojstvo: Strogo rastuCa funkcija je injekcija. Srodni tip funkcija: padajuCe funkcije. Za xl > x2 treba vrijediti f (xl) 2 f (x2). Analogno se definiraju i strogo padajuCe funkcije. Analogno se difiniraju i rastuCi i padajuCi nizovi.

[ ] Vidi najvec'e cijelo.


(xn) f (x) Koeficijent u redu f (x) koji stoji uz potenciju xn (nakon eventualnog sredivanja izraza). Primjer: (x3)(x 2)4 = 8 .

:= Oznaka za definiciju, pridruzivanje objektu na lijevoj strani vrijednosti na-

pisane na desnoj strani. n := k 1 Eitamo "neka je n jednak k 1". n = k 1 Citali bismo kao " n je jednako k 1" i mogli bismo pozeljeti to dokazati ili potraziti dokaz.

C Oznaka za sumaciju. ~

f (i) = f (15)+f (16)+. . .++f (10). CiEA f (i) : za svaki element i skupa A pi5e se po jedan pribrojnik. CibO, Xi - suma po svim i veCim ili jednakim 0, po svim moguCim i .

; l ~ ~

10. DEFINICIJE POJMOVA

A Oznaka za simetriEnu diferenciju skupova. A A B = ( A\ B ) U ( B \ A ) .


lA, l ~ . . . ,Identiteta na skupu A , na skupu X

,. . .

N , Skup prvih n prirodnih brojeva ( n 2 1). No Skup prirodnih brojeva s nulom: No := N U (0). M ( a l , . . . ,a,) NajveCa zajedniEka mjera brojeva a l , . . .a, V ( a l, . . . ,a,) Najmanji zajedniEki vigekratnik brojeva a l , . . . a,

Literatura
[I] A. Ralston, Discmte Mathematics in the First Two Years, MAA Notes, 1989.
Tuda iskustva o zamjeni redoslijeda poduEavanja matematiEke analize i diskretne matematike i o nastavi diskretne matematike opCenito.

[2] Zadaci s vec' odrzanih m a t e d c 7 c i h natjecanja.


Zadaci s novih natjecanja mogu se naCi u MatematiEko-fiziEkom listu.

[3] C. L. Liu, Elements of Discrete Mathematics, McGraw-Hill, Singapore, 1987.


Savrgena knjiga !

[4] N. J . Vilenkin, Kombinatorika, Nauka, Moskva, 1969. Engleski prijevod Academic Press 1971.
Zabavna zbirka s mnoHtvom rije'senih i nerijegenih zadataka, tegko dostupna. KraCa varijanta iste knjige je "Popularnaja Kombinatorika".

[5] D. Veljan, Kombinatorika s teorijom grafova, Skolska knjiga, Zagreb, 1989.


Dobra knjiga koja moie zadovoljiti Eitaoce s razliEitim ambicijama.
I

[6] D. 0. Skljarski, N. N. eenstov, I. M. Jaglom, Selected Problems and Theorems i n Elementary Mathematics, Mir, Moskva 1979.
Izvrsna zbirka zadataka s raznolikim (sadriajno i matematiEki), relativno tegkim zadacima. Svakako ponijeti na pusti otok!

[7] M. Aigner, D i s k t e Mathematik, Vieweg, Braunschweig/Wiesbaden 1993.


Knjiga za poEetnike autora poznatog po relativno teLkoj "Kombinatoric2'. Zanimljivi, nerijegeni zadaci.

[8] G. Polya, Mathematical Discovery, John Willey & Sons, 1981.


Knjiga koja moie svakom profesoru dati novi elan za rad. Stetiti ne moIe niti studentima. Svakako ponijeti na dulji godiinji odmor (uz dovoljno papira za rjesavanje mnogtva dobro izabranih zadataka)! Posluiiti moie i knjiiica "How to Solve It" istog autora.

Zahvala

TeSko je reCi tko me je sve uputio u otkrivanje radosti Eeprkanja po matematici. Roditelji, prva uEiteljica, profesori matematike, profesorica latinskog? Prijatelji, kolege, uEenici? Svi su oni zasluZni za ovu zbirku u kojoj tu radost pokuiiavam podijeliti s ostalima i hvala im, Ipak, za izvlacenje ove zbirke s mog diska sigurno je najzasluzniji njen izdavaE, prof. dr. Neven ElezoviC. Posebna zahvala njemu!

Maja CvitkoviC

U Zagrebu, listopada 1994

You might also like